• Shuffle
    Toggle On
    Toggle Off
  • Alphabetize
    Toggle On
    Toggle Off
  • Front First
    Toggle On
    Toggle Off
  • Both Sides
    Toggle On
    Toggle Off
  • Read
    Toggle On
    Toggle Off
Reading...
Front

Card Range To Study

through

image

Play button

image

Play button

image

Progress

1/846

Click to flip

Use LEFT and RIGHT arrow keys to navigate between flashcards;

Use UP and DOWN arrow keys to flip the card;

H to show hint;

A reads text to speech;

846 Cards in this Set

  • Front
  • Back
Standard treatment for the unstable patient with acute pulmonary edema due to ventricular failure includes all of the following except:

A) Intravenous morphine
B) Intravenous furosemide
C) Supplemental oxygen
D) Cardioselective beta blockade
E) Dopamine
The correct answer is: D
Beta blockade (cardioselective or nonselective) may worsen acute pulmonary edema and is contraindicated in unstable heart failure. Morphine is beneficial in reducing anxiety and peripheral vasoconstriction. Furosemide reduces circulating blood volume and dilates peripheral veins, decreasing pulmonary edema. Supplemental oxygen improves hypoxia due to interstitial pulmonary edema. The positive chronotropic and inotropic action of dopamine is useful in many patients.
Ref: CV/ Harrison's 16, p.1374, 1614-1615/ RAL
Which statement is true concerning the acute coronary syndromes (unstable angina, non-Q-wave myocardial infarction, and Q-wave myocardial infarction)?

A) All are likely to be related to rupture of an unstable plaque in a coronary artery.
B) Spasm plays a major role in the etiology of non-Q-wave myocardial infarction.
C) Serum markers, such CPK isoenzymes are typically elevated in unstable angina.
D) Most cases of sudden cardiac death occur due to complete occlusion of a coronary artery.
E) Among patients with acute myocardial infarction, elderly patients are more likely to present with typical substernal chest pain than younger patients.
The correct answer is: A

All three syndromes are initiated by plaque rupture and subsequent clot formation. Spasm plays a minor role, except in cocaine users. Serum markers are elevated with infarction only. Most cases of sudden cardiac death do not have a completely obstructive lesion. Finally, the elderly, women, and diabetics are less likely than other patients to present with typical chest pain. Ref: CV/ ACLS: Principles and Practice 2003, p. 381-385./RAL
A 67-year-old male presents to the emergency room with a 30-minute history of substernal chest pain. 12-lead ECG shows ST elevations in leads V1 -V4. Thrombolytic therapy may be considered unless the patient:

A) underwent total hip replacement 6 weeks ago.
B) had an embolic stroke 18 months ago.
C) has a known, asymptomatic 5-centimeter abdominal aortic aneurysm.
D) had a hemorrhagic stroke 24 months ago.
E) underwent medical treatment for peptic ulcer disease 6 months ago.
The correct answer is: D

Hemorrhagic stroke at any time in the past is an absolute contraindication to thrombolytic therapy. Non-hemorrhagic stroke within the last year is also a contraindication. Major surgery over 4 weeks ago, inactive peptic ulcer disease, and asymptomatic abdominal aortic aneurysm are not contraindications to thrombolysis. Ref: CV/ ACLS: Principles and Practice 2003, p. 415/RAL
Appropriate immediate treatment for patients presenting with ischemic chest pain include administration of all of the following agents except:

A) Oxygen at four liters per minute
B) Sublingual nitroglycerin
C) Oral nifedipine
D) Intravenous morphine
E) Oral aspirin
The correct answer is: C

Nifedipine has not been shown to benefit patients with acute MI and may be harmful in some cases. The other agents are appropriate initial treatment in ischemic chest pain, provided that a systolic blood pressure greater than 90mmHg is maintained. Ref: CV/Harrison's 16, p. 1455; ACLS for Experienced Providers 2003, p. 10/RAL
Which of the following clinical scenarios best characterizes stable angina?

A) Substernal chest pain occurring at rest only, generally relieved by sublingual nitroglycerin
B) Poorly localized chest pain of 6 months duration occurring with exertion (walking several blocks) and relieved by sublingual nitroglycerin
C) A one-week history of generalized chest pain occurring with emotional stress, relieved by nitroglycerin
D) A 2-year history of gradually lengthening episodes of substernal chest pain occurring with walking about one block
E) A 1-year history of similar episodes of chest pain, relieved by nitroglycerin, at gradually diminishing levels of exertion and with increasing severity
The correct answer is: B

Pain at rest (Answer A) , onset within the last 2 weeks (Answer C) , increasing duration or frequency of chest pain over time (Answer D) , and decrease in the level of activity required to precipitate chest pain (Answer E) are all diagnostic of unstable angina. Ref: CV/ Harrison's 16, p1444/RAL
Which statement is true concerning ventricular fibrillation?

A) There is no ventricular activity.
B) The rhythm is regular, but too rapid for effective pumping activity of the heart.
C) ST segments are elevated in ventricular fibrillation due to ischemia.
D) T waves may be upright or inverted.
E) There is no cardiac output.
The correct answer is: E

Ventricular activity is present, but too disorganized and irregular to produce any cardiac output. There are no ST segments or T waves. Ref: CV/ ACLS: Principles and Practice 2003, p.57/RAL
Which statement is true concerning the treatment of ventricular fibrillation?

A) Defibrillation should be attempted as soon as epinephrine has been administered.
B) The recommended first shock energy for defibrillation is 360 J.
C) Electrical defibrillation should be initiated after a single precordial thump in a witnessed arrest.
D) When there is associated trauma, a second rescuer, when available, should steady the victim's head during defibrillation in order to prevent injury to the cervical spine.
E) If persistent ventricular fibrillation is noted after three shocks have been delivered, resume CPR, establish IV access, and perform endotracheal intubation.
The correct answer is: E

Electrical defibrillation, starting at 200 J, should be attempted before administration of any medications. A preliminary precordial thump is not recommended when a defibrillator is available. All rescuers should be clear of the patient when shocks are administered. Ref: CV/ACLS: Principles and Practice 2003, p.59-60,/RAL
Administration of antiarrhythmic drugs in ventricular fibrillation:

A) have been shown to improve long-term survival.
B) should be given prior to initiation of chest compressions.
C) should be given prior to establishing an airway.
D) should be given prior to electrical defibrillation for best effect.
E) should generally be employed only after three attempts at electrical defibrillation.
The correct answer is: E

Antiarrhythmic drugs have not been shown to improve survival to hospital discharge. When employed, they should be given only after basic CPR and three attempts at electrical defibrillation have been given. Ref: CV/ACLS Principles and Practice 2003, p. 60-62/ RAL
Common cause of Pulseless Electrical Activity (organized, narrow-complex electrical activity of the heart without detectable pulsE) include all of the following, except:

A) Hypovolemia
B) Cardiac tamponade
C) Tension pneumothorax
D) Pulmonary embolism
E) Asystole
The correct answer is: E

In contrast to pulseless electrical activity (PEA) , asystole entails the absence of electrical activity by ECG. The other choices are common causes of PEA. Ref: CV/ACLS: Principles and Practice 2003, p.72/RAL
Useful measures in ventricular asystole may include all of the following except:

A) Confirm diagnosis by checking two perpendicular leads.
B) Administer electrical defibrillation.
C) Give epinephrine.
D) Give atropine.
E) Establish immediate transcutaneous pacing.
The correct answer is: B

Electrical defibrillation is not indicated in asystole and may decrease the chance of re-establishing a treatable rhythm. Checking perpendicular leads may help differentiate asystole from fine ventricular fibrillation. Transcutaneous pacing, atropine, and epinephrine all may be useful in the resuscitation of patients in asystole. Ref: CV/ACLS: Principles and Practice 2003, p. 74-79/RAL
All of the following are true concerning torsades de pointes, except:

A) Overdrive pacing may be helpful.
B) It may be caused by hypokalemia.
C) It may be associated with shortening of the QT interval to less than 0.40 seconds.
D) It may be caused by hypomagnesemia.
E) It may be associated with a reaction to certain antiarrhythmics.
The correct answer is: C

Torsades de points is associated with a prolongation of the QT interval. Hypomagnesemia, hypokalemia, and certain types of antiarrhythmic agents have been implicated as causes. Overdrive pacing and electrolyte replacement may be helpful. Ref: CV/ACLS Harrison's 16, p.1353/RAL
Which statement is true regarding atrial fibrillation?

A) Synchronized cardioversion is the treatment of choice in the hemodynamically stable patient.
B) Ventricular rates of greater than 200 beats per minute are typical.
C) Chemical cardioversion with digoxin is the treatment of choice in the hemodynamically stable patient.
D) Congestive heart failure is a common cause of atrial fibrillation.
E) P waves bearing no regular relation to QRS complexes are often seen on ECG.
The correct answer is: D

Congestive heart failure with elevated left atrial pressure is a frequent cause of new-onset atrial fibrillation. Synchronized cardioversion is the treatment of choice in unstable patients. Digoxin is used for rate control, not cardioversion. Typical rates for rapid atrial fibrillation are in the 120-160 range. There are no P waves seen on ECG. Ref: CV/ACLS Principles and Practice 2003, p. 318-323/RAL
Which statement is true regarding a third-degree atrioventricular (AV) block?

A) Third-degree AV block may be caused by digoxin or propranolol overdose.
B) Typical ventricular rates are 60 to 100 beats per minute.
C) The rhythm is irregularly irregular.
D) P waves are absent.
E) When due to coronary ischemia, the right coronary artery is most often involved.
The correct answer is: A

Digitalis and propranolol toxicity may cause third-degree heart block. Ventricular rates, determined by the rate of the ventricular escape pacemaker, are typically from 20 to 55 beats per minute. P waves are present, and atrial and ventricular rhythms are generally regular. Coronary ischemia, usually involving the left anterior descending artery, is the most common cause. Ref: CV/ACLS: Principles and Practice 2003, p.299-300/RAL
All of the following are effects of propranolol, except:

A) Decreased heart rate
B) Bronchoconstriction
C) Increase in myocardial contractility
D) Decrease in blood pressure
E) Decrease in myocardial oxygen consumption
The correct answer is: C

Propranolol decreases myocardial contractility. It also decreases heart rate, blood pressure, and myocardial oxygen consumption. It may cause bronchospasm, and should be used cautiously, if at all, in asthmatics. Ref: CV/ ACLS: Principles and Practice 2003, p. 234-235/RAL
Which statement is true concerning abdominal aortic aneurysms?

A) Men and women are equally affected.
B) Dissection is the most frequent complication.
C) Mortality for elective surgical repair is about 2 percent.
D) Mortality for emergency repair of ruptured abdominal aortic aneurysms is about 10 percent.
E) Lateral film of the abdomen is the diagnostic procedure of choice.
The correct answer is: C

There is a strong male predominance. Rupture is the most frequent complication. Mortality for elective repair of nonruptured, asymptomatic aneurysms is 1 to 2 percent. Mortality for emergency repair of ruptured abdominal aortic aneurysm is about 50 percent. Abdominal ultrasound is the diagnostic procedure of choice when an asymptomatic abdominal aortic aneurysm is suspected; CT and MRI are alternatives. Ref: CV/Harrison's 16, p. 1482-1483/RAL
A 72-year-old man presents to the ED complaining of the sudden onset of severe, unremitting abdominal and back pain of two hours ago. His Vital signs are as follows: T37.2; P124; R24; BP 82/palp. Examination reveals a tender 6cm pulsatile mass in the epigastrium. The most appropriate immediate management of this patient is to:

A) obtain large-bore IV access, CBC, and lateral film of abdomen.
B) stat MRI to rule out abdominal aortic aneurysm.
C) obtain large-bore IV access, CBC, and upper endoscopy to rule out aortoenteric fistula.
D) obtain large-bore IV access, type and cross for 6 units PRBC, proceed to the OR for repair of abdominal aortic aneurysm.
E) obtain large-bore IV access, vigorous fluid resuscitation, and abdominal ultrasound when stable.
The correct answer is: D

A patient with a pulsatile midline mass, back pain, and hypotension should be assumed to have a ruptured abdominal aortic aneurysm. His best chance for survival is immediate surgery. Ref: CV/Rutherford 5, p. 1298./RAL
Regarding elective repair of abdominal aortic aneurysms, which statement is true?

A) The diameter of the aneurysm does not affect the risk of rupture.
B) Operative mortality is in the 10-20% range.
C) Elective repair is generally recommended for aneurysms greater than 5 cm in diameter.
D) Colon ischemia is the most common postoperative complication.
E) Renal failure is the most common postoperative complication.
The correct answer is: C

The risk of rupture increases with increasing diameter, and elective repair is generally indicated for aneurysms 5cm or more in diameter. Operative mortality is typically less than 5%. The most common complications are cardiac, with myocardial infarction the most likely single-system cause of early and late mortality. Ref: CV/ Rutherford 5, p 1253-1266/RAL
Which statement is false regarding popliteal artery aneurysms?

A) If untreated, rupture is the most likely complication.
B) They are bilateral about 50% of the time.
C) They are the most common peripheral aneurysm.
D) The usual presenting complaint is limb ischemia.
E) They are frequently associated with abdominal aortic and femoral artery aneurysms.
The correct answer is: A

Thrombosis and embolization are more common than rupture. Popliteal artery aneurysms are the most common peripheral aneurysm, and they are bilateral 50-70% of the time. They are highly correlated with abdominal aortic and femoral aneurysms, and ischemia is the most common presenting complaint. Ref: CV/Rutherford 5, p1251-1255/RAL
Which of the following modalities generally has no role in the management of isolated superficial phlebitis?

A) NSAIDS
B) Elastic support
C) The application of moist heat
D) Systemic anticoagulation
E) Elevation of the affected leg
The correct answer is: D

Anticoagulation is indicated in the treatment of deep vein thrombosis only. Elastic support, elevation, moist heat and NSAIDS may provide symptomatic relief. Ref: CV/Rutherford 5, p.1980/RAL
A 60-year-old man with no ongoing risk factors for thrombosis experiences a single episode of deep vein thrombosis (DVT) following an elective general surgery procedure. He is being discharged after a 5-day course of intravenous heparin. An appropriate course of outpatient oral anticoagulation with warfarin (Coumadin) should be maintained for how long?

A) 1 to 3 weeks
B) 3 to 6 weeks
C) 3 to 6 months
D) 1 year
E) Indefinitely
The correct answer is: C

The usual duration of oral anticoagulation to prevent recurrence after an initial episode of DVT is 3 to 6 months. Longer-term prophylaxis is indicated if there are ongoing risk factors. Ref: CV/Rutherford 5, p.1952/RAL
Which of the following is not a risk factor for deep venous thrombosis (DVT)?

A) History of alcohol abuse
B) History of previous DVT
C) Pregnancy
D) Recent elective neurosurgical procedure
E) Presence of a central venous catheter
The correct answer is: A

All the listed conditions, except alcohol abuse, are associated with an increased risk of DVT. Ref: CV/Rutherford 5, p.1922-1924/RAL
A 75-year-old man recovering from a total hip replacement complains of the sudden onset of dyspnea and vague chest pain on his first day out of bed postoperatively. On exam he is cyanotic, tachypneic, and has a blood pressure of 90/40. He has pitting edema of the left leg. Arterial blood gas determination reveals hypoxia and hypercarbia. EKG shows sinus tachycardia and a right ventricular "strain" pattern. The most likely diagnosis is which of the following?

A) Acute myocardial infarction
B) Congestive heart failure
C) Pulmonary embolism
D) Exacerbation of COPD
E) Aortic dissection
The correct answer is: C

Although all diagnoses are conceivable, in this scenario, the symptoms suggest acute cor pulmonale, due to massive pulmonary embolism, in a patient who is at high risk for deep venous thrombosis following a hip procedure and a period of bed rest. Ref: CV/ Harrison's 16, p.1314, 1561-1562/RAL
In a hemodynamically stable patient with pulmonary embolism and no evidence of right heart failure, which choice represents the best treatment?

A) Open surgical pulmonary artery embolectomy
B) Catheter-based pulmonary artery embolectomy
C) Placement of a venocaval filter
D) Thrombolysis with recombinant tissue plasminogen activator (tPA)
E) Immediate anticoagulation with intravenous heparin
The correct answer is: E

Embolectomy and thrombolysis are usually reserved for patients with hemodynamic instability or evidence of right heart failure. The usual treatment in the stable patient is immediate anticoagulation. Venocaval filter placement is useful to prevent recurrent pulmonary embolism in the patient in whom full anticoagulation is contraindicated. Ref: CV/Harrison's 1561-1564,/RAL
Regarding the etiology of deep venous thrombosis (DVT), which choice represents all three elements of Virchow's Triad?

A) Previous surgery, anticoagulation, and a family history
B) Venous stasis, vein wall damage, and a hypercoagulable state
C) Previous history of DVT, occult malignancy, and anticoagulation
D) Orthopedic surgery, vascular surgery, and general surgery
E) Bed rest, elevation of the legs, and dehydration
The correct answer is: B

Virchow's classic triad of pathological changes leading to venous thrombosis consists of venous stasis, vascular endothelial damage, and a hypercoagulable state. Ref: CV/Rutherford 5, p.1920/RAL
A 36-year-old woman in the seventh month of an otherwise uncomplicated pregnancy presents to the emergency room complaining of the recent onset of severe, mildly painful, left leg swelling. The best management of this patient is which of the following?

A) Begin oral anticoagulation with warfarin and arrange follow-up within one week with her gynecologist.
B) Begin subcutaneous unfractionated heparin injections, apply an elastic stocking, and arrange outpatient follow-up for home subcutaneous heparin injections.
C) Obtain an immediate venous duplex scan and anticoagulate with warfarin if positive for DVT.
D) Obtain immediate venous duplex scan and admit for IV heparin therapy if positive for DVT.
E) Prepare the patient for emergency iliofemoral thrombectomy.
The correct answer is: D

Warfarin is teratogenic and is contraindicated at all stages of pregnancy. Thromboembolism is the most common medical cause of mortality in pregnant women. When clinically suspected, it should be confirmed promptly by noninvasive means and, if present, treated aggressively with intravenous unfractionated heparin or subcutaneous low molecular weight heparin. Ref: CV/Harrison's 16, p. 34-35/RAL
Accepted indications for placement of a vena caval filter include all of the following except:

A) Documented deep venous thrombosis (DVT) with a contraindication to anticoagulation
B) A history of DVT or pulmonary embolism (PE) in a first-order relative
C) A significant bleeding episode while treated for DVT with anticoagulation
D) Recurrent DVT while on adequate anticoagulation
E) As prophylaxis following pulmonary embolectomy
The correct answer is: B

The other choices are all accepted indications for placement of a vena caval filter. Ref: CV/ Rutherford 5, p.1969/RAL
Which statement is true concerning primary varicose veins?

A) Valvular incompetence and reflux in the superficial system is a key etiologic factor.
B) The disease is generally caused by thrombosis in the deep system.
C) Massive leg edema is a common physical finding.
D) Superficial thrombophlebitis is a common cause of fatal pulmonary embolism.
E) Brawny edema and venous stasis ulceration are typical late-stage findings.
The correct answer is: A

Valvular incompetence and reflux in the superficial system is central to the pathophysiology of primary varicose veins. Secondary varicose veins are due to disease in the deep system. Brawny edema, stasis ulceration, and massive edema are more typical sequelae of deep vein thrombosis. Superficial thrombophlebitis rarely causes clinically significant pulmonary embolism by extension of the thrombus into the deep veins. Ref: CV/ Rutherford 5, p. 1979, 1994/RAL
The most likely underlying cause of peripheral arterial embolic disease is:

A) Endocarditis
B) Abdominal aortic aneurysm
C) Chronic peripheral arterial occlusive disease
D) Intravenous drug abuse with inadvertent intra-arterial injection
E) Atrial fibrillation
The correct answer is: E

Atrial fibrillation causes between two thirds and three quarters of all peripheral emboli. The other choices are all relatively infrequent causes of embolic episodes. Ref: CV/ Question NumberRutherford 5, p. 822/RAL
The most likely site of arterial embolization (from all sources) is the:

A) upper extremities.
B) lower extremities.
C) brain.
D) kidneys.
E) mesenteric circulation.
The correct answer is: B

Approximately 70% of peripheral emboli involve the extremities, and the emboli lodge in the lower extremities 5 time more frequently than in the upper extremities. Ref: CV/Rutherford 5, p. 824/RAL
A 63-year-old man with a history of chronic atrial fibrillation and no previous history of peripheral arterial disease, presents to the Emergency Room with a 2-hour history pain and paresthesias of the left lower extremity. The left femoral and pedal pulses are absent. The right pedal pulses are normal. With a working diagnosis of an arterial embolus, the most important immediate intervention is which of the following?

A) Obtain an arteriogram of the left lower extremity.
B) Apply moist heat to the left foot.
C) Full anticoagulation with intravenous heparin
D) Obtain a stat echocardiogram to determine the embolic source.
E) Elevate the left leg.
The correct answer is: C

Immediate heparinization prevents thrombosis of the arteries distal to the embolus. Never apply heat to an ischemic extremity. Elevation of the extremity will make the ischemia worse. The other measures may be useful, but are less urgent. Ref: CV/Rutherford 5, p.825/RAL
The earliest symptom of heart failure is:

A) Paroxysmal nocturnal dyspnea
B) Orthopnea
C) Exertional dyspnea
D) Syncope
E) Cheyne-Stokes respiration
The correct answer is: C

Decreasing exercise tolerance due to dyspnea on exertion is usually the first symptom observed in progressive heart failure. Ref: CV/Harrison's 16, p13701/RAL
Which physical finding is most typical of congestive heart failure?

A) Increased pulse pressure
B) Hyperresonance to percussion
C) Crepitant rales at lung bases
D) Unilateral pedal edema
E) Flat neck veins
The correct answer is: C

Moist rales at the bases is typical. The lung bases may be dull to percussion due to pulmonary effusion. The pulse pressure may be decreased due to decreased stoke volume. Pedal edema, when present is bilateral and symmetrical. The jugular veins are distended due to increased systemic venous pressure. Ref: CV/ Harrison's 16, p.1370-1371/RAL
Which statement is true regarding the use of digitalis in heart failure?

A) Sinus bradycardia is an early sign of toxicity.
B) Its negative inotropic effect may be beneficial.
C) The serum concentration of digoxin is lowered by concurrent administration of amiodarone.
D) It may elevate the heart rate in atrial fibrillation.
E) Intoxication may be related to renal insufficiency.
The correct answer is: E

Premature ventricular beats constitute the most frequently observed arrhythmia in digitalis toxicity, which may be precipitated by renal insufficiency or the concurrent use of amiodarone, both of which may raise digoxin levels. Digoxin is a positive inotrope, and it may lower the heart rate in rapid atrial fibrillation. Ref: CV/ Harrison's 16, p.1375/RAL
Which statement is true regarding mitral valve prolapse?

A) The most common physical finding is a mid- or late-systolic click.
B) The ECG will usually show prolongation of the PR interval.
C) The condition is more common in males.
D) It is usually caused by rheumatic heart disease.
E) Most patients will develop severe, symptomatic mitral regurgitation.
The correct answer is: A

The ECG: Is usually normal. It is more common in females. It may occur as a manifestation of rheumatic heart disease, but the cause is usually unknown. Most patients remain asymptomatic. The development of severe mitral regurgitation is uncommon. Ref: Ref: CV/Harrison's 16, p. 1395-1396/RAL
Which statement is true regarding severe aortic stenosis in the older adult?

A) ECG usually demonstrates atrial fibrillation.
B) The characteristic auscultatory finding is a harsh systolic murmur at the right second intercostal space.
C) It is commonly associated with severe systolic hypertension.
D) Aortic valve replacement should only be undertaken when severe left-sided heart failure develops.
E) Widened pulse pressure is a late finding.
The correct answer is: B

The ECG Usually reveals regular sinus rhythm Systemic pressure is usually normal. Late finding may include a low systemic pressure and narrow pulse pressure. In most cases, surgical treatment should be undertaken before severe left ventricular failure occurs Ref: CV/Harrison's 16, p. 1397-1398/RAL
Which ECG finding is most characteristic of acute transmural ischemia?

A) Q waves in the lateral chest leads
B) ST segment elevation
C) ST segment depression
D) Prolongation of the PR interval
E) Q waves in leads II, III, and AVF
The correct answer is: B

ST segment elevation is most characteristic of acute transmural ischemia. ST segment depression is usually an indication of subendocardial ischemia. PR interval prolongation suggests an atrioventricular block. Q-waves are indicative of a completed myocardial infarction. Ref: CV/Harrison's. 16, p. 1316/RAL
A 70-year-old man presents with complaints of gradually progressive intermittent chest pain, progressive exercise intolerance due to dyspnea, and episodes of exertional syncope. A harsh systolic murmur at the right second intercostal space is noted on examination. His blood pressure is 110/90. The most likely diagnosis is:

A) Aortic insufficiency
B) Acute myocardial infarction
C) Aortic stenosis
D) Pulmonary embolism
E) Tricuspid stenosis
The correct answer is: C

The triad of angina pectoris, dyspnea, and exertional syncope is characteristic of aortic stenosis, as is a systolic murmur and narrow pulse pressure. Aortic insufficiency generally presents with hypertension and a widened pulse pressure. Tricuspid stenosis has a characteristic diastolic murmur. Acute myocardial infarction and pulmonary embolism are acute events. Ref: CV/Harrison's 16, p.1397, 1400-1401/RAL
Which statement is true concerning serum cardiac biomarkers for myocardial injury?

A) Total creatine phosphokinase (CK) will rise within 1 hour of infarction and remain elevated for 7 to 10 days.
B) Cardiac -specific troponin T will rise to twice the reference level and return to normal in 24 hours.
C) The MB isoenzyme of creatine phosphokinase (CK) may be elevated by intramuscular injection.
D) Total creatine phosphokinase (CK) may be elevated by skeletal muscle trauma.
E) The MB isoenzyme of creatine phosphokinase (CK) is not affected by electrical cardioversion.
The correct answer is: D

Total CK may be elevated by skeletal muscle trauma, including intramuscular injection. In acute myocardial infarction, total CK increases in 4 to 8 hours and returns to normal in 48 to 72 hours. The MB isoenzyme of CK may be elevated by electrical cardioversion or cardiac surgery, but is not elevated by skeletal muscle trauma. Cardiac-specific troponin T is not detectable in the normal individual, may rise to more than 20 times the reference value in the patient with acute MI, and returns to normal in 7 to 10 days. Ref: CV/Harrison's 16, p. 1450-1451/RAL
The most common cause of in-hospital death following myocardial infarction is:

A) pulmonary embolism.
B) arrhythmia.
C) iatrogenic complications.
D) arterial embolism.
E) pump failure.
The correct answer is: E

Cardiogenic shock due to pump failure is now the most common cause of death in patients hospitalized for acute myocardial infarction. Ref: CV/Harrison's 16, p. 1455, 1612-1613/RAL
Which statement is false concerning coronary artery bypass grafting?

A) Mortality rate of less than 1% has been achieved in experienced centers.
B) Comorbid conditions and decreased ventricular function increase operative risk.
C) Approximately 90% of patients undergoing the procedures are free of angina or significantly improved postoperatively.
D) Survival is significantly increased in virtually all patients undergoing the procedure.
E) In 10-20% of patients undergoing CABG, at least one saphenous vein graft will be occluded one year after the procedure.
The correct answer is: D

Improvement in survival has been shown only in patients with left main disease and in those with 2- or 3-vessel coronary artery disease which includes proximal left anterior descending artery obstruction. Ref: CV/Harrison's 16, p.1443/RAL
Which is not a criterion for unstable angina?

A) Onset of angina within the past 4 to 6 weeks
B) A pattern if increasing frequency of episodes of angina
C) Angina at rest or with minimal exertion
D) A pattern of increasing severity of discomfort in a patient with a 5-year history of angina
E) Chronic episodes of angina on physical exertion or emotional stress, lasting 5-10 minutes, and resolving with sublingual nitroglycerin
The correct answer is: E

Stable angina pectoris is characterized by chest or arm discomfort with exertion or stress, relieved by rest or by nitroglycerin. All other choices are typical of unstable angina pectoris. Ref: CV/Harrison's 16, p.1444/RAL
Which reading represents the lowest blood pressure diagnostic of hypertension?

A) 118/78 mmHg
B) 124/84 mmHg
C) 142/84 mmHg
D) 160/88 mmHg
E) 158/92 mmHg
The correct answer is: C

Systolic blood pressure of 140 mmHg or greater, or diastolic blood pressure of 90 mmHg or greater, represent the minimal criteria for Stage 1 hypertension and the minimum criterion for antihypertensive drug treatment. Ref: CV/The Seventh Report of the Joint National Committee on Prevention, Detection, Evaluation and Treatment of High Blood Pressure (JAMA. 2003;289:2560-2572),p.2561/RAL
Which statement is true regarding the measurement of blood pressure in the outpatient environment?

A) At each visit, at least 2 measurements of blood pressure by the auscultatory method should be obtained.
B) Ambulatory self-measurement of blood pressure is unreliable and should not be considered in treatment decisions.
C) The bladder of the cuff should encircle at least 50% of the arm.
D) Blood pressure should be measured after at least 15 minutes of rest in the supine or seated position.
E) Pressure should always be measured with the patient in the supine position.
The correct answer is: A

Two measurements should be obtained by auscultation, in the seated position, after 5 minutes of rest. The bladder should encircle at least 80% of the arm. Ambulatory self-measurement of blood pressure may be of use in assessing white-coat hypertension and in the evaluation of apparent hypotensive symptoms. Ref: CV/The Seventh Report of the Joint National Committee on Prevention, Detection, Evaluation and Treatment of High Blood Pressure (JAMA. 2003;289:2560-2572),p.2562-2563/RAL
Which of the following clinical features suggests work-up for a treatable cause of hypertension?

A) Labile hypertension accompanied by headache and palpitations
B) Gradually worsening hypertension with advancing age
C) Any Stage 2 hypertension in an African American male
D) New onset of hypertension in a post-menopausal woman
E) A family history of hypertension with onset in either parent before the age of fifty
The correct answer is: A

This presentation suggests the possibility of pheocromocytoma. The other choices are not inconsistent with essential hypertension. Ref: CV/Harrison's 16, p. 1470/RAL
Which class of antihypertensive agent would generally be the first choice in a type 2 diabetic patient with the recent onset of Stage 1 (140-150/90-99mmHg) hypertension?

A) A beta blocker
B) A calcium channel blocker
C) An angiotensin-converting enzyme (ACE) inhibitor
D) A direct vasodilator
E) An alpha 1 blocker
The correct answer is: C

ACE inhibitors slow the development of diabetic nephropathy and are generally the agent of first choice in the type 2 diabetic. Ref: CV/Harrison's 16, p.1479/RAL
Which of the following lifestyle modifications have not been shown to be of significant benefit in the treatment or prevention of hypertension?

A) Limit alcohol consumption (no more than 2 drinks per day for men; no more than 1 drink per day for women) B) Avoid dietary caffeine
C) Reduce weight (in overweight patients) D) Reduce dietary salt
E) Participate in regular aerobic physical activity
The correct answer is: B

All choices, except avoiding dietary caffeine, have been shown to be useful in managing hypertension Ref: CV/The Seventh Report of the Joint National Committee on Prevention, Detection, Evaluation and Treatment of High Blood Pressure (JAMA. 2003;289:2560-2572),p.2564/RAL
Which class of drug would probably represent the best first choice when starting a male with recent myocardial infarction and normal systolic function on treatment for moderate (Stage 2) hypertension?

A) A beta-adrenergic blocker
B) A calcium channel blocker
C) An alpha 1 blocker
D) A central alpha 2 agonist
E) A direct vasodilator
The correct answer is: A

A beta-adrenergic blocker in this patient with coronary artery disease has the added benefit of its favorable impact on the probability of reinfarction. Ref: CV/Harrison's 16, p.1479/RAL
Which antihypertensive agent is contraindicated in pregnant women?

A) Metoprolol
B) Captopril
C) Methyldopa
D) Hydralazine
E) All of the above
The correct answer is: B

Captopril and other ACE inhibitors may cause birth defects and should not be used in hypertensive women or in women who may become pregnant Ref: CV/The Seventh Report of the Joint National Committee on Prevention, Detection, Evaluation and Treatment of High Blood Pressure (JAMA. 2003;289:2560-2572),p.2568/RAL
In which of the following conditions is the use of angiotensin-converting enzyme (ACE) inhibitors and angiotensin receptor antagonists (ARBs) contraindicated?

A) Chronic renal insufficiency due to hypertensive nephrosclerosis
B) Chronic renal insufficiency due to diabetic nephropathy
C) Chronic renal insufficiency due to bilateral renal artery stenosis
D) Chronic renal insufficiency due to polycystic kidney disease
E) Chronic renal insufficiency due to lithium toxicity
The correct answer is: C

ACE inhibitors and ARB's are contraindicated in patients with bilateral renal artery stenosis, but may be useful in patients with unilateral renal artery stenosis and normal function in the contralateral kidney. Ref: CV/Harrisonäó»s 16, p. 1479/RAL
Which statement is false concerning dilated cardiomyopathy?

A) Exertional, substernal chest pain with radiation to the left arm is typical.
B) Beta blockers may be beneficial.
C) Chronic oral anticoagulation may be useful to prevent systemic embolization.
D) It is usually fatal within three years.
E) Patients may benefit by treatment with angiotensin converting enzyme (ACE) inhibitors.
The correct answer is: A

Patients usually have normal coronary arteries and do not have typical angina pectoris. Beta blockade and ACE inhibitors (or ARB's) are useful, as is anticoagulation. The majority of patients die with 3 years of onset. Ref: CV/ Harrison's 16, p.1409/RAL
Which statement regarding hypertrophic cardiomyopathy is true?

A) It is characterized by symmetrical left ventricular hypertrophy (LVH).
B) Diastolic filling pressures are reduced.
C) A diastolic murmur is a usual finding on physical examination.
D) Regular, strenuous physical exercise may improve outcomes.
E) Diuretics and dehydration should be avoided.
The correct answer is: E

LVH is typically asymmetrical. Diastolic filling pressures are increased, and a systolic murmur is usually present. Strenuous physical exertion may precipitate sudden death. Diuretics and nitrates should be avoided. Ref: CV/ Harrison's 16, p.1410-1411/RAL
Which finding is most typical in restrictive cardiomyopathy?

A) Decreased left ventricular pressures with increased diastolic filling
B) Left ventricular dilatation
C) Normal cardiac output
D) Symmetrically thickened left ventricular wall
E) Calcified pericardium on chest X-ray
The correct answer is: D

The left ventricular wall is symmetrically thickened, not dilated, and rigid, causing increased diastolic filling pressures and decreased cardiac output. A thickened, calcified pericardium is more typical of restrictive pericarditis. Ref: CV/ Harrison's 16, p. 1412/RAL
In acute pericarditis:

A) the pericardial friction rub is best heard at the right second intercostal space.
B) widespread ST segment elevations are typically seen on electrocardiogram.
C) the pericardial friction rub is best heard with the patient in the supine position.
D) chest pain is usually absent, differentiating it from acute myocardial infarction.
E) chest pain, when present, is relieved by deep inspiration or coughing.
The correct answer is: B

The pericardial friction rub is best heard at the lower left sternal border, with the patient in the sitting position. Widespread ST segment elevations are typical, as is retrosternal chest pain, which may be aggravated by coughing or deep inspiration. Ref: CV/ Harrison's 16, p.1414/ RAL
Which of the following is not usually associated with acute pericarditis?

A) Retrosternal chest pain relieved by leaning forward
B) Pericardial friction rub
C) Q waves on electrocardiogram
D) Pericardial effusion on echocardiogram
E) Pleuritic chest pain
The correct answer is: C

ST segment elevation without QRS changes are common. Severe chest pain, which may have a pleuritic component, is common, as are pericardial friction rubs and pericardial effusion. Ref: CV/ Harrison's 16 p.1414/ RAL
Post-cardiac injury syndrome, a form of acute pericarditis:

A) by definition requires a history of penetrating chest trauma.
B) may be a complication of cardiac surgery.
C) is usually painless.
D) rarely produces fever, except with secondary bacterial infection.
E) always occurs within 7 days of injury.
The correct answer is: B

Cardiac surgery, as well as other penetrating and nonpenetrating injury to the myocardium, may cause the post-cardiac injury syndrome. Chest pain, fever to 40ŒÁ Celsius, and onset 1 to 4 weeks after the precipitating event are typical. Ref: CV/ Harrison's 16 p.1417/ RAL
Standard treatment for acute viral or idiopathic pericarditis may include all of the following except:

A) Anticoagulation
B) Bed rest
C) Aspirin
D) Indomethacin
E) Prednisone
The correct answer is: A

Bed rest, steroids, and nonsteroidal anti- inflammatory medications are the main components of treatment. Anticoagulation is not indicated Ref: CV/ Harrison's 16, p. 1098/ RAL
The most common benign tumor of the heart is which of the following?

A) Lipoma
B) Myxoma
C) Sarcoma
D) Rhabdomyoma
E) Lymphoma
The correct answer is: B

Benign myxomas account for about half of all tumors of the heart. Sarcoma is the most common malignant tumor of the heart. Ref: CV/ Harrison's 16, p.1420-1421/RAL
Cardiac myxomas:

A) are benign tumors that are best managed by observation.
B) occur most commonly in the left ventricle.
C) are more common in men.
D) may present with peripheral embolization.
E) most commonly involve the aortic valve.
The correct answer is: D

Myxomas are most commonly found in the left atrium and most often involve the mitral valve, producing either mitral stenosis or mitral regurgitation. They are somewhat more common in females. They may present with tumor embolization and acute arterial occlusion, and they are generally best managed by surgical excision under cardiopulmonary bypass. Ref: CV/ Harrison's 16, p.1420-1421/ RAL
Normal central venous pressure, estimated by observation of the pulsation of the right jugular vein, is:

A) about 6cm above the sternal angle.
B) less than 10cm above the sternal angle.
C) less than 3cm above the sternal angle.
D) at the sternal angle.
E) Jugular pulsation should not be observable in any examining position.
The correct answer is: C

The highest point of observed pulsation in the right jugular vein should be no more than 3cm above the sternal angle, regardless of examining position. Ref: CV/Harrison's 16, p.1306/RAL
Relative contraindications to cardiac catheterization include all of the following except:

A) Renal failure
B) History of severe allergy to radiographic contrast
C) Full oral anticoagulation
D) Current febrile illness
E) Recent MI (within the last 6 months)
The correct answer is: E

Recent MI is not a contraindication to cardiac catheterization. The other choices, though relative contraindications, can be managed to permit cardiac catheterization in patients for whom the procedure is required. Ref: CV/ Harrison's 16, p. 981/RAL
Agents that may be useful in treating or preventing allergic contrast reactions include all of the following except:

A) Cimetadine (Tagamet) B) Metoprolol (Lopressor) C) Prednisone
D) Diphenhydramine (Benedryl) E) Epinephrine
The correct answer is: B

Cimetadine, prednisone, and diphenhydramine all may prevent or lessen contrast reactions. Epinephrine is a key element in the treatment of reactions. Beta-blockers, including metaprolol, are not known to be helpful. Ref: CV/ Harrison's 16, p.1328/RAL
When measuring ventricular performance, the ratio of stroke volume to end diastolic volume is known as the:

A) force-velocity curve.
B) cardiac index.
C) ejection fraction.
D) cardiac output.
E) Starling curve.
The correct answer is: C

The proportion of the resting volume of the ventricle (end diastolic volumE) ejected with each contraction, or the ratio of stoke volume to end-diastolic volume, constitutes the ejection fraction. The normal value is 67 α 8%. Ref: CV/ Harrison's 16, p.1361-1362/ RAL
Clinical signs associated with acute rheumatic fever may include all of the following except:

A) Migratory polyarthritis
B) Pruritic maculopapular rash
C) Carditis
D) Erythema marginatum
E) Chorea
The correct answer is: B

Answers A, C, D, and E are all components of the Updated Jones Criteria, considered major clinical manifestations of rheumatic fever. Ref: CV/ Harrison's 16, p. / RAL
The main reason for long-term antibiotic prophylaxis in patients diagnosed with rheumatic fever is to prevent:

A) peritonsillar abscess.
B) streptococcal infection of the heart.
C) recurrent streptococcal pharyngitis and reactivation of rheumatic fever.
D) the development of septic arthritis.
E) Sydenham's chorea.
The correct answer is: C

By preventing recurrent episodes of pharyngitis, immunologic reactivation may be avoided. Ref: CV/ Harrison's16, p.1979 / RAL
Which of the following is not a goal of antibiotic treatment in acute streptococcal pharyngitis?

A) Prevention of peritonsilar abscess
B) Prevention of retropharyngeal abscess
C) Prevention of bacteremia
D) Prevention of poststreptococcal glomerulonephritis
E) Prevention of acute rheumatic fever
The correct answer is: D

Antibiotic treatment of Group A streptococcal pharyngitis prevents suppurative complications of the local infection (Answers A, B, and C) as well as acute rheumatic fever. Unfortunately, such treatment does not prevent poststreptoccal glomerulonephritis. Ref: CV/ Harrison's 16, p.825/RAL
The usual duration of treatment with oral penicillin for streptococcal pharyngitis is which of the following?

A) A single dose
B) Three days
C) Seven days
D) Ten days
E) Fourteen days
The correct answer is: D

Regardless of when the symptoms of pharyngitis resolve, oral penicillin treatment is continued for ten days to eradicate the organism from the pharynx and prevent the development of rheumatic fever. Ref: CV/ Harrison's 16, p. 825/ RAL
Which statement is true concerning mitral stenosis?

A) Ventricular arrhythmias are typical.
B) Bacterial endocarditis is the most frequent cause.
C) The characteristic murmur is harsh, systolic, and heard best at the right second interspace.
D) Chest pain is the most frequent presenting complaint.
E) Pulmonary hypertension is a late complication.
The correct answer is: E

Atrial arrhythmias are most common, particularly atrial fibrillation. Most cases are sequelae of rheumatic heart disease. The characteristic murmur is low-pitched, rumbling and diastolic, heard at the apex. Dyspnea is the most frequent complaint. Pulmonary hypertension, by several mechanisms, develops late in the disease. Ref: CV/Harrison's 16, p.1390-1392/RAL
What degree of stenosis in the coronary arteries (reduction in cross-sectional areA) is the level at which symptomatic ischemic heart disease becomes manifest?

A) 10%
B) 50%
C) 70%
D) 90%
E) 100% (complete occlusion)
The correct answer is: C

Exertional angina generally begins when the cross-sectional area of the coronary artery is reduced by 70%. Ref: CV/Harrison's 16, p. 1434/ RAL
You are conducting an electrocardiographic stress test. Which of the following observations would require the termination of the test?

A) The patient complains of mild dyspnea.
B) A rise in the systolic blood pressure of 20mmHg
C) A fall in the systolic blood pressure of 20mmHg
D) The development of sinus tachycardia
E) The patient complains of mild cramping pain in the left calf.
The correct answer is: C

A fall in systolic blood pressure of greater than 10mmHg may indicate severe global ischemia or valvular disease and requires the termination of the test. Ref: CV/ Harrison's 16, p. 1436-1437/ RAL
Common areas of radiation of anginal chest pain include all of the following except:

A) Teeth
B) Left arm
C) Jaw
D) Right arm
E) Back
The correct answer is: E

Anginal chest pain may radiate to the teeth, jaw, or either arms, but rarely radiates below the umbilicus or to the back. Ref: CV/ Harrison's 16, p. 77/ RAL
Non-cardiac chest pain may masquerade as angina pectoris. Some likely causes of non-cardiac chest pain include all of the following except:

A) Aortic dissection
B) Pulmonary embolus
C) Gastroesophageal reflux
D) Peptic ulcer disease
E) Abdominal aortic aneurysm
The correct answer is: E

The pain of a symptomatic abdominal aortic aneurysm typically occurs in the back, flank, and groins. Chest pain is rare. All other choices may produce chest pain that is difficult to distinguish from angina pectoris. Ref: CV/ Harrison's 16, p. 78-79/ RAL
All of the following are true of pleuritic chest pain except:

A) May be caused by pericarditis
B) May be aggravated by cough
C) May be aggravated by deep breathing
D) Is typically described as a steady, substernal discomfort
E) May be caused by pneumothorax
The correct answer is: D

Pericarditis may irritate the parietal pleura. Respiratory movement, including cough and deep breathing aggravates pleuritic pain. Pneumothorax may cause typical pleuritic chest pain. Steady, substernal chest pain from pleural inflammation due to any cause is rare. Ref: CV/Harrison's 16, p. 78/ RAL
A 62-year-old man presents with a history of aching anterior chest pain and tightness of 2 days duration. ECG shows normal sinus rhythm with no ischemic changes. The oxygen saturation on room air is 98%. Chest X-ray is unremarkable. Pressure just lateral to the sternum, on either side, reproduces the pain. The most likely diagnosis is:

A) Aortic dissection
B) Expanding thoracic aneurysm
C) Costochondritis
D) Acute anterior-wall myocardial infarction
E) Bilateral pneumonia
The correct answer is: C

Reproduction of the patient's symptoms by application of pressure to the costochondral junction is highly suggestive of costochondritis. The other choices are less likely, especially in the presence of a normal chest X-ray, oxygen saturation, and ECG. Ref: CV/ Harrison's 16, p. 78-79/ RAL
Which statement is true regarding palpitations?

A) They are often associated with anxiety or depression.
B) They are usually noted during exertion.
C) They are associated with decreased catecholamine levels.
D) They may be caused by hypothyoidism.
E) They may be caused by hyperglycemia.
The correct answer is: A

Patients generally notice palpitations, an awareness of their own heart beating, during rest. Palpitations are frequently associated with psychiatric disorders, including anxiety and depression, but may be a symptom of virtually any cardiac arrhythmia. Noncardiac causes of palpitations include hyperthyroidism, hypoglycemia, or increased catecholamine levels due to any cause. Ref: CV/ Harrison's 16, p. 81/ RAL
A 45-year-old man with no previous cardiac history is recovering from an inguinal herniorrhaphy. He is noted to have isolated, multifocal ventricular premature contractions (VPC's) while monitored in the recovery room. Twelve-lead EKG confirms this finding, but is otherwise normal. He is asymptomatic. His blood pressure is 132/80; pulse is 78; respiratory rate is 18; he is afebrile; and his oxygen saturation on room air is 99%. His serum electrolytes and CBC are normal. Appropriate action at this point consists of which of the following?

A) Begin oral flecainide immediately
B) Administer a loading dose of intravenous lidocaine
C) Begin humidified oxygen 40% by face mask
D) Administer intravenous bolus of metoprolol
E) No action other than reassurance is required at this time.
The correct answer is: E

In an asymptomatic patient without cardiac disease, VPC's do not require treatment. Ref: CV/ Harrison's 16, p. 1343-1344/ RAL
Following the endovascular repair of an abdominal aortic aneurysm, the patient with an "endoleak" has:

A) flow from the aorta into any abdominal organ.
B) a fistula from the abdominal aorta to the inferior mesenteric artery.
C) persistent flow within the aorta, outside the endoluminal graft.
D) migration of the aortic prosthesis.
E) injury to the wall of the aorta leaking into and contained within the retroperitoneum.
The correct answer is: C

An endoleak refers to persistent flow within the aortic aneurysm, outside the lumen of the aortic prosthesis. It can be caused by an incomplete seal between the prosthesis and the native aorta, or by retrograde flow through collaterals into the aneurysm sac. Ref: CV/ Rutherford 5, p.1287-1288/ RAL
The innermost layer of the arterial wall consists of:

A) Internal elastic lamina
B) Endothelial cells
C) External elastic lamina
D) Basal lamina
E) Adventitia
The correct answer is: B

Arranged from most internal to most external, the layers are: Endothelium Basal lamina Internal elastic lamina External elastic lamina Adventitia Ref: CV/ Rutherford 5, p. 313-316/ RAL
In the unstable or "vulnerable" intracoronary plaque, the lipid core of the plaque is separated from the lumen of the artery by:

A) a fibrous cap.
B) a fatty streak.
C) medial calcification.
D) normal media.
E) organized thrombus.
The correct answer is: A

In the "vulnerable" plaque, a thin fibrous cap, which may be weakened by the action of inflammatory cells, separates the lipid core of the plaque from the flow of blood in the lumen of the artery. Rupture of the fibrin cap, exposing the interior of the plaque to the flow of blood, may precipitate an acute coronary syndrome. Ref: CV/ ACLS: Principles and Practice 2003, p.380-381/RAL
Which statement regarding the epidemiology of cardiovascular disease is true?

A) Death rates for coronary artery disease in the United States have been steadily increasing since the 1950's.
B) Cardiovascular diseases are the second leading cause of death in the United States.
C) Death rates for coronary artery disease in the United States have been steadily decreasing since the 1960's.
D) The overall prevalence of cardiovascular disease in the United States is about 2%.
E) The worldwide incidence of cardiovascular disease is declining.
The correct answer is: C

Death rates for coronary artery disease have been decreasing since the 1960's, but overall cardiovascular disease remains the most common cause of death in the United States. The overall prevalence in the U.S. is currently about 1 in 5, or 20%. The worldwide prevalence of cardiovascular diseases is rising. Ref: CV/ Harrison's 5 p. 1301/ RAL
For an asymptomatic individual at no increased risk for coronary heart disease, the recommended goal for low density lipoprotein (LDL) cholesterol is :

A) less than 70mg/dL.
B) less than 100mg/dL.
C) less than 160mg/dL.
D) less than 180mg/dL.
E) less than 240mg/dL.
The correct answer is: C

For lower-risk patients, the target LDL is less than 160 mg/dL. Ref: CV/ NCEP 2004, p.3/RAL
The most appropriate interval for total cholesterol screening in asymptomatic adults is:

A) every 6 months.
B) every year.
C) every other year.
D) every 5 years.
E) Routine screening is not indicated.
The correct answer is: D

The National Cholesterol Education Program recommends all adults over twenty years old undergo serum total cholesterol screening every 5 years. Ref: CV/ NCEP 2004, p. 5/ RAL
For purposes of lipid management, which of the following is not considered a risk factor for coronary heart disease?

A) Obesity
B) Cigarette smoking
C) Hypertension
D) Advanced age (greater than 45 years for men; greater than 55 years for women) E) Family history of premature heart disease
The correct answer is: A

Obesity is not considered an major independent risk factor for coronary artery disease. Ref: CV/ NCEP 2004, p. 6-7/ RAL
Causes of secondary dyslipidemia include all of the following except:

A) Corticosteroid treatment
B) Chronic renal failure
C) Diabetes
D) Hypertension
E) Use of anabolic steroids
The correct answer is: D

All conditions except hypertension are known to cause secondary dyslipidemia. Ref: CV/ NCEP 2004, p. 10/ RAL
In patients with coronary heart disease, or other atherosclerotic diseases, the target for LDL-cholesterol reduction is:

A) less than 100mg/dL.
B) less than 120mg/dL.
C) less than 140mg/dL.
D) less than 160mg/dL.
E) less than 180mg/dL.
The correct answer is: A

Target LDL for patients with established coronary artery disease or an equivalent (for example, peripheral vascular diseasE) is less than 100 mg/dL. The most recent guidelines cite an LDL less than 70 mg/dL as a "therapeutic option", noting trial evidence of additional benefit of even more intense treatment in this high-risk population. Ref: CV/ NCEP 2004, p. 4/RAL
A 45-year-old man with no evidence of coronary heart disease or other atherosclerotic risk factors undergoes routine screening. LDL cholesterol is 195mg/dL. The most appropriate next step in his management would be to:

A) begin treatment with atorvastatin.
B) begin treatment with controlled-release niacin.
C) counsel the patient in therapeutic lifestyle changes.
D) schedule the patient for a screening exercise tolerance test.
E) do nothing, but repeat the lipid panel in 1 year.
The correct answer is: C

In this low-risk patient, an attempt should me made to reach LDL goal (less than 160 mg/dL) through lifestyle modification (weight loss, dietary modification , exercisE) . Follow-up lipid panel should be obtained in three months, at which point consideration should be given to drug treatment if LDL goal has not been reached. Ref: CV/ NCEP 2004, p. 12/ RAL
Which is not a major clinical predictor of increased perioperative cardiovascular risk?

A) Unstable angina
B) Recent myocardial infarction (within 1 month) C) Atrial fibrillation with well-controlled ventricular response
D) Decompensated congestive heart failure
E) Acute myocardial infarction (within 7 days)
The correct answer is: C

Well-controlled atrial fibrillation is considered a "minor predictor" and doesn't significantly increase perioperative risk. The other choices are "major predictors". Ref: CV/ ACC/AHA Task Force 2002, p.5/ RAL
Which is not considered to be a high-risk surgical procedure?

A) Thoracic aortic reconstruction
B) Repair of an abdominal aortic reconstruction
C) Elective peripheral vascular reconstruction
D) Elective hip joint replacement
E) Emergency laparotomy for obstruction
The correct answer is: D

Elective orthopedic procedures are considered to be intermediate-risk. Aortic or other major vascular reconstructions, including elective peripheral reconstructions, are high-risk, as are all major emergency procedures. Ref: CV/ ACC/AHA Task Force 2002, p. 8/ RAL
A 63-year-old man presents for preoperative evaluation for an elective orthopedic procedure. He underwent CABG 3 years ago, and he denies cardiac symptoms. Cardiac and pulmonary exams are unremarkable. Appropriate cardiac testing for this patient would include:

A) Resting echocardiogram
B) Stress echocardiogram
C) Persantine thallium scan
D) Coronary arteriogram
E) None of the above
The correct answer is: E

A patient who has had coronary revascularization within the last 5 years, and who has no cardiac signs or symptoms, generally requires no further cardiac work-up. Ref: CV/ ACC/AHA Task Force 2002, p.10/ RAL
Which activity suggests a functional capacity consistent with good perioperative cardiac risk?

A) The ability to walk from room to room in the house
B) The ability to feed, dress, and wash oneself
C) The ability to walk one block at 2 mph on level ground
D) The ability to do light housework, such as dusting and washing dishes
E) The ability to participate actively in golf, tennis, or bowling
The correct answer is: E

Answers A through D represent activities requiring an energy expenditure of less than four mets (metabolic equivalents). The ability to tolerate activities requiring greater than four mets of energy expenditure, such as the recreational activities in Answer E, suggests a better perioperative risk. Ref: CV/ ACC/AHA Task Force 2002, p. 8/ RAL
Which procedure requires antibiotic prophylaxis in a patient with a prosthetic heart valve?

A) Dental extraction
B) Cystoscopy
C) Cardiac catheterization
D) Both A and B
E) All of the above
The correct answer is: D

In general, antibiotic prophylaxis is indicated in patients undergoing procedures or instrumentation of the GI, GU, or respiratory tracts. The American Heart Association does not recommend antibiotic prophylaxis for patients with prosthetic valves undergoing cardiac catheterization. Ref: CV/ Harrison's 16, p.739-740; ACC/AHA Guidelines 1998/ RAL
A 65-year old man with mild, stable angina pectoris is undergoing an elective transurethral prostate procedure. Your complete preoperative history and physical examination reveals that he has no previous symptoms of congestive heart failure and excellent functional exercise capacity. Appropriate further properative testing should include which of the following?

A) Resting transthoracic echocardiogram
B) Cardiac catheterization and percutaneous intervention as indicated
C) Persantine thallium scan
D) Exercise tolerance test
E) None of the above
The correct answer is: E

The patient with mild, stable angina, a good ejection fraction, and good exercise tolerance undergoing an intermediate risk surgical procedure (such as a transurethral prostate resection) does not require further invasive or noninvasive testing. Ref: CV/ ACC/AHA Task Force 2002, p. 9/ RAL
A 75- year-old diabetic man is about to undergo elective surgery. Which of the following statements concerning his perioperative risks is true?

A) He will be more likely to suffer a myocardial infarction compared with nondiabetic patients of the same age and gender.
B) He will be more likely to complain of retrosternal chest pain compared with nondiabetic patients experiencing myocardial ischemia.
C) If his serum glucose is carefully monitored, he is less likely than nondiabetic patients to experience congestive heart failure.
D) Regardless of his glycemic control, he will be more likely to complain of left arm and jaw pain compared with nondiabetic patients experiencing myocardial ischemia.
E) Regardless of glycemic control, he will be less likely to suffer a surgical wound infection compared with nondiabetic patients of the same age and gender.
The correct answer is: A

The diabetic patient is more likely to have myocardial ischemia during or following surgery. He is, however, less likely to report the pain of ischemia or infarction; he runs a higher risk of infectious complications, although the risk may be minimized by careful glycemic control; and he is more likely to develop heart failure. Ref: CV/ ACC/AHA Task Force 2002. p.6/ RAL
Which statement concerning the clinical characteristics of coronary artery disease (CAD) in women, as compared with their male counterparts, is true?

A) Women generally develop CAD at a younger age.
B) Women with CAD are more likely to complain of typical substernal anginal symptoms.
C) The initial presentation of CAD in women is more likely to be myocardial infarction.
D) Women under 50 presenting with myocardial infarction are more likely to die.
E) All of the above are true.
The correct answer is: D

Women develop CAD at an older age; they are more likely to have atypical angina; they are more likely to present with angina pectoris; and young women are more likely to die than their male counterparts of the same age. Ref: CV/ Harrison's 16 p.29/RAL
A 50-year-old male smoker with a history of coronary artery disease is undergoing an EKG exercise stress test as part of a preoperative evaluation. Which of the following findings would be most likely to cause delay or cancellation of the surgical procedure?

A) Ischemic changes consisting of horizontal, down-sloping ST depressions at 4 mets of exercise, at 70% of his predicted heart rate, or at a heart rate of less than 100/minute
B) The occurrence of typical exercise-induced angina
C) Ischemic EKG changes at 7 mets of activity, or at a heart rate greater than 130 beats/ minute, or at 85% of the patient's predicted heart rate
D) A and B
E) All of the above
The correct answer is: D

Only the findings in Answers A and B would suggest that the case be delayed pending further work-up, or simply canceled as too high-risk. The ischemic changes in Answer C occur at a much higher level of exercise and are, therefore, indicative of a lesser degree of CAD and a lower level of surgical risk. Ref: CV/ ACC/AHA Task Force 2002, p.21-22/RAL
Which statement is true regarding the perioperative management of patients undergoing elective procedures?

A) Beta-blocking agents should be discontinued 7 days prior to elective surgery.
B) To diagnose hypertension in a preoperative patient, the clinician must note elevated pressure on at least two occasions in a stressful environment.
C) A preoperative patient with severe hypertension, poorly controlled on his present medical regimen should probably be delayed for further evaluation.
D) In a patient with mild-to-moderate essential hypertension, unassociated with other cardiovascular or metabolic abnormalities, the surgery should probably take place as soon as end-organ damage has been ruled out by renal ultrasound.
E) All of the above
The correct answer is: C

Beta blockers should be continued through the perioperative period. The diagnosis of hypertension should rest on at least two blood pressure determinations in a non-stressful environment. Patients with poorly controlled severe hypertension (systolic pressure greater than 180mmHg; diastolic pressure greater than 110mmHg) should probably be postponed pending further work-up and better control. In a patient with mild-to-moderate hypertension, work-up for end-organ damage need not delay surgery. Ref: CV/ ACC/AH Task Force 2002, p.13/RAL
A 45-year-old man with no evidence of coronary heart disease or other atherosclerotic risk factors undergoes routine screening. Total cholesterol is 242mg/dL. This patient undergoes lipoprotein analysis, revealing an LDL-cholesterol of 155mg/dL. Repeat analysis confirms this finding. The best management of this patient would include which of the following?

A) Counseling to reduce total dietary saturated fat to no more than 7% of caloric intake
B) Counseling to reduce dietary cholesterol to less than 200mg per day
C) Beginning treatment with nicotinic acid
D) A and B
E) A, B, and C
The correct answer is: D

Appropriate intervention for this patient includes initiation of "therapeutic lifestyle changes," including the dietary changes indicated. Drug treatment is not indicated in this relatively low-risk patient with LDL cholesterol already below the goal of 160 mg/dL. Ref: CV/ NCEP 2004, p. 11-13/ RAL
Common causes of low-output heart failure include all of the following except:

A) Ischemic heart disease
B) Hypertension
C) Anemia
D) Dilated cardiomyopathy
E) Aortic stenosis
The correct answer is: C

In heart failure due to anemia, the cardiac output is typically high. In heart failure due to ischemic heart disease, hypertensive disease, dilated cardiomyopathy and valvular disease, the cardiac output is low. Ref: CV/ Harrison's 16, p. 1369/ RAL
What is the usual duration of antibiotic prophylaxis after a single episode of rheumatic fever?

A) Ten days
B) Six weeks
C) Until age 25, or at least 5 years, whichever is longer
D) Until age 40, or at least 10 years, whichever is longer
E) Lifelong
The correct answer is: C

After the initial episode of rheumatic fever, antibiotic prophylaxis should be continued for 5 years, or until age 25, whichever is longer. After the second episode, antibiotic prophylaxis should be continued for 10 years, or until age 40. Ref: CV/Goroll 4, p. 91/RAL
Congenital atrial septal defect:

A) usually presents with cyanosis in infancy.
B) usually produces a right-to-left shunt in childhood.
C) most typically produces a harsh, 3/6 diastolic murmur.
D) results in increased pulmonary blood flow.
E) is incompatible with life beyond infancy, unless repaired.
The correct answer is: D

Atrial septal defect is a noncyanotic congenital defect producing a left-to-right shunt, a systolic murmur, and increased pulmonary blood flow. It is often asymptomatic in childhood. Later in life, pulmonary hypertension may result in a reversal of the direction of the shunt. Ref: CV/ Schwartz 8, p. 612; Hay 17, p. 572-573/ RAL
Congenital ventricular septal defects (VSD's):

A) Generally close spontaneously
B) When large (6-10mm in diameter) should be repaired in late adolescence, as soon as the patient reaches his predicted adult stature
C) When large, a harsh diastolic murmur is present
D) If progressive pulmonary hypertension develops, surgery should be delayed and vigorous medical management is indicated.
E) Surgical repair should never be attempted in the presence of congestive heart failure.
The correct answer is: A

80-85% of VSD's are small and will close spontaneously, usually before the age of 6. Large VSD's produce a harsh systolic murmur. They may produce progressive pulmonary hypertension and congestive heart failure, either of which is an indication for surgery, usually before the age of 2. Ref: CV/ Hay 17, p. 574-575/ RAL
A 10-day-old infant is noted to have a machinery-like murmur at the second-intercostal space, left sternal border. The murmur is also easily audible in the left infraclavicular space. There is no cyanosis, and peripheral pulses in all four extremities are bounding. Which is the best initial diagnostic test to evaluate these findings?

A) Cardiac catheterization
B) Echocardiogram
C) PA and lateral chest X-ray
D) Arterial blood gas determination.
E) MRI of the chest.
The correct answer is: B

The findings are typical of patent ductus arteriosus. Echocardiogram is the best noninvasive test to confirm the diagnosis, estimate the size of the shunt, and detect associated cardiac abnormalities. Ref: CV/ Hay 17, p. 577-578; Schwartz 8, p. 617-618/ RAL
The ductus arteriosus:

A) normally closes on the first day of life.
B) when persistently patent, is a benign anomaly requiring no specific treatment.
C) normally closes in utero, during the third trimester of gestation.
D) is stimulated to contract by decreases in the oxygen saturation of arterial blood.
E) when persistently patent, should be surgically ligated at age 6 to 10 years.
The correct answer is: A

The ductus arteriosus is patent in utero and usually closes during the first day of extra-uterine life, partly due to increases in the oxygenation or arterial blood after delivery. Patent ductus arteriosus (PDA) may cause congestive heart failure or endocarditis. Closure of the PDA should be accomplished, surgically or by means of a catheter-based technique, during the first year of life. Ref: CV/ Hay 17, p. 577-578; Schwartz 8, p. 617-618/ RAL
An infant is noted to have absent femoral pulses and a systolic murmur audible in the left axilla. What is the most likely diagnosis?

A) Ventricular septal defect
B) Atrial septal defect
C) Coarctation of the aorta
D) Pulmonary atresia
E) Transposition of the great arteries
The correct answer is: C

Absent femoral pulses and systolic murmur in the left axilla or the left back, along with arterial hypertension, strongly suggest a diagnosis of coartation of the aorta. Ref: CV/Hay 17, p. 582/ RAL
Which of the following cardiac anomalies is NOT a component of the tetralogy of Fallot (TOF)?

A) Left ventricular hypertrophy
B) Dextroposition of the aorta
C) Ventricular septal defect
D) Right ventricular outflow obstruction
The correct answer is: A

Right ventricular hypertrophy occurs with TOF. The ventricular septal defect and the obstruction of the right ventricular outflow result in a right-to-left shunt. This is the most common cyanotic congenital heart disease. Ref: CV/ Hay 17, p. 588-589; Schwartz 8, p. 636/ RAL
An elderly woman of Scandinavian descent presents with a complaint of fever, headache, malaise and arthralgia. On examination, you note tenderness on palpation of the right temporal area. What is the most appropriate next step in establishing the diagnosis?

A) Carotid duplex scan
B) Erythrocyte sedimentation rate (ESR) C) Arch and carotid arteriography
D) Random serum glucose
E) CT scan of the head
The correct answer is: B

Giant cell arteritis (or temporal arteritis) presents with the symptoms described. Tenderness may be noted on palpation of the temporal artery. The most useful laboratory test is the ESR, which will be elevated if giant cell arteritis is present. Ref: CV/ Harrison's 16, p. 2009/ RAL
Definitive treatment of temporal arteritris consists of which of the following?

A) Excisional biopsy of the temporal artery
B) A course of oral prednisone
C) A course of methotrexate
D) A course of oral anticoagulation with warfarin
E) Temporal artery bypass graft
The correct answer is: B

Temporal artery biopsy establishes the diagnosis. Treatment of temporal (or giant cell) arteritis consists of a course of oral prednisone 40-60mg per day for one month, then tapered over a period of about 2 years. Ref: CV/ Harrison's 16, p. 2008-2009/ RAL
A 45-year-old woman complains of dilated tortuous superficial veins in both legs. She reports that she initially noticed the venous dilatation after the birth of her first child, 20 years ago, and complains that the veins are painful after long periods of standing. She reports a family history of similar symptoms in her mother and several aunts. She has never been treated with anticoagulants for blood clots in her legs. What is the most likely diagnosis?

A) Deep vein thrombosis (DVT) B) Postphlebitic syndrome
C) Primary varicose veins
D) Superficial thrombophlebitis
E) Venous stasis dermatitis
The correct answer is: C

Primary varicose veins result from inherited intrinsic pathology in the superficial veins and venous valves of the lower extremities. Deep venous thrombosis typically presents as unilateral leg swelling with calf tenderness in a patient with risk factors for thrombosis (malignancy, immobilization, etc.). Postphlebitic syndrome, including stasis dermatitis, follow an episode of DVT. Superficial thrombophlebitis results from the thrombosis of varicose vein and presents with localized pain and inflammation. Ref: CV/ Schwartz 8, p. 823/ RAL
The usual treatment for symptomatic primary varicose veins is:

A) elastic compression stockings.
B) oral anticoagulation with warfarin.
C) anticoagulation with subcutaneous, low-molecular-weight heparin.
D) surgical stripping and ligation of the veins.
E) ablation of the veins with sclerosing injections.
The correct answer is: A

Medium- or heavy-weight elastic compression stockings usually provide symptomatic relief for patients complaining of leg pain due to primary varicose veins. Anticoagulation is not indicated. Various techniques for ablation of the veins

including surgical stripping, sclerosing injections, and laser or radiofrequency catheter-based techniques

are available for individuals who fail elastic compression. Ref: CV/ Schwartz 8, p. 823/ RAL
An elderly diabetic patient presents with a complaint of lightheadedness on standing. You suspect orthostatic hypotension and measure blood pressure in the supine and standing position. What is the minimum change in systolic blood pressure consistent with the diagnosis of orthostatic hypotension?

A) 10 mmHg
B) 20 mm Hg
C) 30 mmHg
D) 40 mmHg
E) 50 mmHg
The correct answer is: B

Orthostatic hypotension is defined as a sustained drop in systolic blood pressure of at least 20 mmHg. Alternatively, a drop in diastolic blood pressure of 10 mmHg would meet accepted criteria. Ref: CV/ Harrison's 16, p. 2430/ RAL
Which of the following agents are used to treat orthostatic hypotension?

A) Nifedipine
B) Furosemide
C) Midodrine
D) Chlorpromazine
E) Phenobarbital
The correct answer is: C

Midodrine is an alpha 1 agonist used to treat orthostatic hypotension. All other choices are agents (a calcium-channel blocker, a diuretic, a phenothiazine, and a barbiturate, respectively) that may cause or aggravate orthostatic hypotension. Ref: CV/ Harrison's 16, p. 2430-2434/ RAL
A patient with an automatic implantable cardiac defibrillator (AICD) is scheduled to undergo an elective carpal tunnel release. He presents preoperatively for advice about endocarditis prophylaxis. The most appropriate treatment is which of the following?

A) Amoxicillin 2g po taken 1 hour before the procedure
B) Ampicillin 2g IV given 30 minutes before the procedure
C) Cephalexin 2g po taken 30 minutes before the procedure
D) Clindamycin 600mg po 30 minutes before the procedure
E) Advise the patient that he will not need prophylactic antibiotics.
The correct answer is: E

Patients with implantable pacemakers and defibrillators do not require antibiotics for endocarditis prophylaxis regardless of the proposed procedure. Patients with conditions that place them at high or moderate risk for endocarditis (eg., prosthetic heart valves) require prophylactic antibiotics when undergoing procedures likely to cause transient bacteremia

in general, those procedures involving instrumentation of the GI or GU or respiratory tracts. Examples are: dental extractions, rigid bronchoscopy, cystoscopy, or dilatation of an esophageal stricture. Ref: CV/ Harrison's 16, p. 739-740/ RAL
Which EKG abnormality is associated with hyperkalemia?

A) Peaking of the T waves
B) Prominent U waves
C) Narrowing of the QRS complex
D) T wave inversions
E) ST depressions
The correct answer is: A

Hyperkalemia causes peaked T waves, QRS widening, and eventually cardiac arrest. Prominent U waves are seen in hypokalemia. T wave inversions and ST segment depressions are signs of myocardial ischemia. Ref: CV/ Harrison's 16, p. 1318-1319/ RAL
CARDIOVASCULAR I

This agent lowers LDL-cholesterol and triglycerides while raising HDL-cholesterol but its use may be limited by bothersome vasoactive side effects. A) Nicotinic Acid
B) Cholestyramine
C) Estrogen replacement
D) HMG CoA reductase inhibitors ("statins")
E) Gemfibozil
The correct answer is: A

Nicotinic acid has a beneficial effect on all lipoprotein fractions, but may not be tolerated due to effects related to vasodilatation. Ref: CV/NCEP 2004, p.13/RAL
This is the most potent agent in lowering LDL-cholesterol, but may cause myopathy.

A) Nicotinic Acid
B) Cholestyramine
C) Estrogen replacement
D) HMG CoA reductase inhibitors ("statins") E) Gemfibozil
The correct answer is: D

The "statins" are the most potent agents for lowering LDL-cholesterol but may produce myalgias and CPK elevations in a small minority (about 0.1%) of patients. Ref: CV/NCEP 2004, p. 13/RAL
This agent lowers LDL-cholesterol without reducing triglycerides: it is not absorbed from the GI tract.

A) Nicotinic Acid
B) Cholestyramine
C) Estrogen replacement
D) HMG CoA reductase inhibitors ("statins")
E) Gemfibozil
The correct answer is: B

Cholestyramine and colestipol act within the lumen of the intestine and bind bile acids, increasing the conversion of cholesterol to bile acids in the liver. Ref: CV/NCEP 2004, p.13/RAL
This agent produces a marked decrease in triglycerides with modest effect on LDL-cholesterol; it may increase the risk of gallstone disease.

A) Nicotinic Acid
B) Cholestyramine
C) Estrogen replacement
D) HMG CoA reductase inhibitors ("statins")
E) Gemfibozil
The correct answer is: E

Fibric acid derivatives, such as genfibrozil, are effective in lowering triglycerides, but may accelerate gallstone formation. Ref: CV/NCEP 2004, p. 13/RAL
This agent may increase HDL cholesterol and have a beneficial effect on osteoporosis.

A) Nicotinic Acid
B) Cholestyramine
C) Estrogen replacement
D) HMG CoA reductase inhibitors ("statins")
E) Gemfibozil
The correct answer is: C

Estrogen replacement therapy raises HDL in women, but may actually increase the incidence of coronary heart disease (CHD) . It is no longer recommended for prevention of CHD, despite its beneficial effect on bone mineral density. Ref: Rader DJ. Effects of nonstatin lipid drug therapy on high-density lipoprotein metabolism. Am J Cardiol 2003; 91(suppl): 18E-23E. Ref: U.S. Preventive Services Task Force. Postmenopausal hormone replacement therapy for primary prevention of chronic conditions: recommendations and rationale. Ann Intern Med 2002; 137:834-839.
This type of edema usually has a hemosiderin deposition with "brawny edema; feet spared; usually asymmetrical.

A) Venous stasis disease
B) Lymphedema
C) Cardiac orthostatic edema
The correct answer is: A

Chronic venous hypertension in venous stasis disease causes the extravasation of red cells and the characteristic "brawny edema". Ref: CV/Rutherford 5, p. 10/RAL
This type of spongy edema is not relieved by elevation.

A) Venous stasis disease
B) Lymphedema
C) Cardiac orthostatic edema
The correct answer is: B

In chronic lymphedema, the skin is hypertrophied, without abnormal pigmentation or atrophic changes, and although the swelling yields to finger pressure, it does not "pit", but rebounds quickly. Ref: CV/Rutherford 5, p. 10/RAL
This type of "pitting" edema is completely relieved by elevation and is always bilateral.

A) Venous stasis disease
B) Lymphedema
C) Cardiac orthostatic edema
The correct answer is: C

Cardiac edema is always bilateral, painless, and dramatically relieved by elevation of the legs. Finger pressure produces a characteristic persistent pitting. Ref: CV/Rutherford 5, p. 10/RAL
A 74-year old diabetic man, who notes a one-year history of pain in both calves on walking, promptly relieved by rest, presents complaining of the onset, two weeks ago, of right foot pain waking him from sleep at night. For the last week he has obtained some relief by sleeping in a reclining chair, with his feet dependent, but for the last 48 hours the pain has been nearly constant. Examination of the lung fields, heart, and abdomen are normal. There are normal femoral and absent popliteal and pedal pulses bilaterally. There is +1 bilateral ankle edema. Although both feet are pale on your initial examination, with the patient in the supine position, when he sits up on the examining table, his right foot becomes bright red. What is the most appropriate immediate measure in the management of this patient?

A) Venous duplex scan
B) Chronic anticoagulation with warfarin
C) Arteriography
D) Initiation of antiplatelet therapy
E) Echocardiogram
The correct answer is: C

The immediate management of this patient with progression from intermittent claudication to critical limb ischemia would most likely involve arteriography followed by endovascular or surgical intervention, if feasible. Neither venous disease nor embolus from a cardiac source are most likely in this scenario. Chronic anticoagulation or antiplatelet therapy, if indicated, would most likely play a role after intervention.
A 74-year old diabetic man, who notes a one-year history of pain in both calves on walking, promptly relieved by rest, presents complaining of the onset, two weeks ago, of right foot pain waking him from sleep at night. For the last week he has obtained some relief by sleeping in a reclining chair, with his feet dependent, but for the last 48 hours the pain has been nearly constant. Examination of the lung fields, heart, and abdomen are normal. There are normal femoral and absent popliteal and pedal pulses bilaterally. There is +1 bilateral ankle edema. Although both feet are pale on your initial examination, with the patient in the supine position, when he sits up on the examining table, his right foot becomes bright red. Question: The patient's long-standing leg pain with walking most likely represents:

Question: A) chronic intermittent claudication due to arterial occlusive disease.
B) bilateral posterior compartment syndrome.
C) bilateral popliteal artery aneurysms.
D) pseudoclaudication due to spinal stenosis.
E) venous stasis disease.
The correct answer is: A

Absent distal pulses, pain occurring after walking a consistent distance, and prompt relief with rest suggests intermittent claudication.
A 74-year old diabetic man, who notes a one-year history of pain in both calves on walking, promptly relieved by rest, presents complaining of the onset, two weeks ago, of right foot pain waking him from sleep at night. For the last week he has obtained some relief by sleeping in a reclining chair, with his feet dependent, but for the last 48 hours the pain has been nearly constant. Examination of the lung fields, heart, and abdomen are normal. There are normal femoral and absent popliteal and pedal pulses bilaterally. There is +1 bilateral ankle edema. Although both feet are pale on your initial examination, with the patient in the supine position, when he sits up on the examining table, his right foot becomes bright red. Question: His recent development of foot pain at night most likely represents:

A) venous thrombosis with proximal propagation.
B) ischemic rest pain due to progression of arterial occlusive disease.
C) diabetic neuropathy.
D) expanding popliteal artery aneurysm on the right.
E) embolization from a proximal abdominal aortic or femoral artery aneurysm.
The correct answer is: B

The development of right foot pain at night, when decreased cardiac output further reduces distal perfusion, and the relief of pain with dependency, suggest ischemic rest pain due to the progression of atherosclerosis. Your observation of dependent rubor of the right foot also favors this diagnosis. Slight edema is probably due to sleeping with the legs in a dependent position. Normal abdominal exam and normal right femoral pulse make embolization from a proximal aneurysm unlikely.
A 50-year old man is noted to have a 1.5cm nontender left supraclavicular lymph node on routine physical examination. Which of the following is the most appropriate next step in establishing the diagnosis?

A) Plant PPD
B) Obtain serology for heterophil antibody
C) MRI of neck and upper thorax
D) Biopsy of suspect lymph node
E) CT scan of the abdomen
The correct answer is: D

Supraclavicular lymphadenopathy is highly suggestive of malignancy. A left supraclavicular lymph node, sometime called Virchow's node, suggests metastasis from a gastrointestinal tumor. Expeditious biopsy is recommended. Ref: Resp/Harrison's 16, p. 344/ RAL
The most common cause of pneumonia is:

A) inhalation of aerosolized infectious particles.
B) aspiration of infectious particles colonizing the oropharynx.
C) direct hematogenous spread from a cardiac or vascular focus of infection.
D) instrumentation, e.g., tracheal intubation of the respiratory tract.
E) trauma.
The correct answer is: B

Approximately half of all normal adults aspirate material from the oropharynx while sleeping. Most common pulmonary pathogens colonize the oropharynx, at least transiently. The other methods of inoculating the lower respiratory tract are significant in some settings, or for some specific organisms, but are less common. Ref: Resp/ Harrison's 16, p. 1529/ RAL
The most common organism responsible for hospital-acquired (nosocomial) pneumonia is:

A) Pseudomonas aeruginosa
B) Staphylococcus aureus
C) Streptococcus pneumoniae
D) Mycoplasma pneumoniae
E) Haemophilus influenzae
The correct answer is: B

S. aureus is the most frequently isolated species in cases of nosocomial pneumonia in the United States, although 61% of patients will have some gram-negative bacillus isolated, including P. aeruginosa, Acinetobacter species, Enterobacter species, and Klebsiella pneumoniae. Ref: Resp/Harrison's 16, p. 1539/ RAL
Which is not an indication for hospitalization of outpatients presenting with pneumonia?

A) Systolic blood pressure less than 90 mmHg
B) Fever greater than 100.5ŒÁ F
C) Alteration in mental status
D) Arterial PO2 less than 60mmHg, or oxygen saturation less than 90%, while on room air
E) Respiratory rate greater than 28/minute
The correct answer is: B

Fever is not a contraindication to the outpatient management of pneumonia. Multilobar involvement and certain types of pleural effusion may also indicate admission for inpatient treatment. Ref: Resp/Harrison's 16, p. 1536/ RAL
A good indicator of upper airway contamination of expectorated sputum is:

A) the presence of Gram negative organisms in the specimen.
B) the pH of the specimen.
C) the presence of anaerobic organisms in the specimen.
D) the presence of epithelial cells in the specimen.
E) the color of the specimen under low power magnification.
The correct answer is: D

Sputum specimens with fewer than 10 epithelial cells and more than 25 polymorphonuclear leukocytes per low power field are likely to represent lower airway secretions with minimal upper airway contamination. Ref: Resp/ Harrison's 16, p. 1533/ RAL
A 50-year old otherwise healthy man presents to an emergency department with complaints of fever, cough, and purulent sputum. Chest X-rays confirm the presence of a pulmonary infiltrate. He meets clinical criteria for inpatient treatment and is admitted. Which is the pathogen most likely to be causing his infection?

A) Mycoplasma pneumoniae
B) Streptococcus pneumoniae
C) Chlamydia pneumoniae
D) Legionella species
E) Haemophilus influenzae
The correct answer is: B

Among patients admitted for the treatment of community-acquired pneumonia, about 50% are infected with S. pneumoniae. The other organisms are also frequently implicated in community-acquired pneumonia, but are less likely. Ref: Resp/ Harrison's 5, p. 1530/ RAL
Which antibiotic represents the best empirical choice in an otherwise-healthy 60-year-old female, who does not meet clinical criteria for hospital admission, and who does not have risk factors for drug-resistant Streptococcus pneumoniae (DRSP) infection?

A) Penicillin
B) Amoxicillin
C) Azithromycin
D) Trimethoprim-sulfamethoxazole
E) Metronidazole
The correct answer is: C

Macrolides, such as azithromycin, have excellent activity against Streptococcus pneumoniae, as well as against "atypical" organisms causing community-acquired pneumonia, including Mycoplasma pneumoniae, Chlamydia pneumoniae, and Legionella species. Ref: Resp/Harrison's 16, p. 1535/RAL
You are evaluating a previously healthy 40-year-old-male outpatient with fever and a productive cough of two days duration. He specifically denies any previous history of cardiac or pulmonary disease. While considering a diagnosis of pneumonia, which of the following is the most useful clinical sign?

A) Egophony on auscultation of the lung fields
B) A history of generalized arthralgias
C) Systolic blood pressure greater than 160mmHg
D) Respiratory rate of greater than 30/minute
E) Dullness on percussion of the lung fields
The correct answer is: D

While all clinical findings may be present in a patient with pneumonia, the most useful indicator of the severity of the disease is tachypnea greater than 30/minute. Ref: Resp/Harrison's 16, p. 1532/ RAL
The most common stimulus initiating an acute exacerbation of asthma is:

A) aspirin.
B) food colorings.
C) viral upper respiratory tract infections.
D) psychological stress.
E) environmental irritants.
The correct answer is: C

While all choices can initiate an asthmatic attack, viral URI's are the most common. Ref: Resp/ Harrison's 16, p. 1510/ RAL
Pathophysiologic changes typical of asthma include which of the following?

A) Bronchial wall edema
B) Increased respiratory secretions
C) Contraction of smooth muscle in the bronchial wall
D) Both B and C
E) All of the above
The correct answer is: E

All pathophysiologic findings listed play a part in the acute asthmatic exacerbation. Ref: Resp/ Harrison's 16, p. 1511/ RAL
The most worrisome finding(s) on arterial blood gas analysis in a patient suffering an acute asthmatic exacerbation is (arE) :

A) Hypoxemia
B) Hypocapnia
C) Metabolic acidosis
D) Respiratory alkalosis
E) All are equally worrisome
The correct answer is: C

Metabolic acidosis is a late finding associated with severe airway obstruction. Hypoxia, hypocapnia, and respiratory alkalosis, on the other hand, are typical of most asthmatic attacks. Ref: Resp/ Harrison's 16, p. 1511/RAL
Once the diagnosis of asthma has been established, response to therapy can be judged by which of the following?

A) Chest X-ray
B) FEV1 (forced expiratory volume in one seconD) C) PEFR (peak expiratory flow ratE) D) Both A and B
E) Both B and C
The correct answer is: E

The course of the disease and effectiveness of treatment can be ascertained by FEV1 and PEFR. Chest X-ray is generally not helpful. Ref: Resp/ Harrison's, 16 p. 1515/ RAL
Which agent or class of agents, used in the treatment of asthma, is a selective beta 2 agonist?

A) Epinephrine
B) Theophylline (Theo-Dur, Slo-bid, etc.) C) Inhaled steroids (Azmacort, Flovent, Vanceril, etc.) D) Albuterol
E) Ipratropium bromide (Atrovent, Combivent)
The correct answer is: D

Epinephrine is a nonselective beta agonist. Albuterol is a longer-acting selective beta 2 agonist. Ref: Resp/ Harrison's 16, p. 1512/RAL
Which agent or class of agents, used in the treatment of asthma is anticholinergic, with an onset of action in 15 minutes and a peak effect in 60 to 90 minutes?

A) Epinephrine B) Theophylline (Theo-Dur, Slo-bid, etc.)
C) Inhaled steroids (Azmacort, Flovent, Vanceril, etc.)
D) Chromolyn (Intal)
E) Ipratropium bromide (Atrovent, Combivent)
The correct answer is: E

The anticholinergic effect of ipratropium bromide begins in 15 minutes and peaks at 60 to 90 minutes. Ref: Resp/ Harrison's 16, p. 1513/RAL
Which agent or class of agents, used in the treatment of asthma is a methylxanthine; may be given orally in divided doses; and should be followed with serum drug levels?

A) Epinephrine
B) Theophylline (Theo-Dur, Slo-bid, etc.)
C) Inhaled steroids (Azmacort, Flovent, Vanceril, etc.)
D) Chromolyn (Intal)
E) Ipratropium bromide (Atrovent, Combivent)
The correct answer is: B

Theophylline, a methylxanthine, may be given orally. Absorption and clearance can be determined by following serum theophylline levels. Troublesome sleep disruption associated with use of this agent has largely relegated it to "second-line" status. Ref: Resp/ Harrison's 16, p. 1513/RAL While it is true that theophylline is no longer a mainstay in the treatment of asthma, the National Asthma Education and Prevention Program Expert Panel Report: Guidelines for the Diagnosis and Management of Asthma Update on Selected Topics

2002 (J Allergy and Clin Immunol 2002 Nov; 110(5 Suppl):S141-219.) continues to recommend it as a useful agent in some situations. The report is available online: http://www.nhlbi.nih.gov/guidelines/asthma/index.htm.
Which agent or class of agents, used in the treatment of asthma reduces airway reactivity and may cause thrush?

A) Epinephrine
B) Theophylline (Theo-Dur, Slo-bid, etc.)
C) Inhaled steroids (Azmacort, Flovent, Vanceril, etc.)
Chromolyn (Intal)
E) Ipratropium bromide (Atrovent, Combivent)
The correct answer is: C

Inhaled steroids may cause oral candidiasis or thrush. Ref: Resp/ Harrison's 16, p. 1514/RAL
Which agent or class of agents, used in the treatment of asthma may block airway response to a participating antigenic stimulus if inhaled 15 to 20 minutes before contact?

A) Epinephrine
B) Theophylline (Theo-Dur, Slo-bid, etc.)
C) Inhaled steroids (Azmacort, Flovent, Vanceril, etc.)
D) Chromolyn (Intal)
E) Ipratropium bromide (Atrovent, Combivent)
The correct answer is: D

Chromolyn, a mast cell stabilizing agent, blocks the response of these cells to environmental antigens. Ref: Resp/ Harrison's 16, p. 1514/ RAL
Which of the following is not a physical sign of COPD?

A) Clubbing of the digits
B) Pursed-lip breathing
C) Pulmonary hyperinflation with barrel chest
D) Decreased breath sounds bilaterally
E) A prolonged expiratory phase of respiration
The correct answer is: A

Clubbing of the digits is not seen in COPD. The other physical signs are typical of this disease. Ref: Resp/ Harrison's 16, p. 1551/ RAL
In pulmonary function testing, the total lung capacity (TLC) less the residual volume (RV) is known as the:

A) Spirogram
B) Vital capacity (VC)
C) Forced vital capacity (FVC)
D) Forced expiratory flow (FEF)
E) Functional residual capacity (FRC)
The correct answer is: B

The volume in the lungs after maximal inhalation (TLC) , less the volume of gas in the lungs after maximal expiration (RV), is the maximum volume of gas that can be voluntarily expelled from the lungs, or vital capacity (VC) . Ref: Resp/ Harrison's 16, p. 1498-1499/RAL
A patient with reduced 1-second forced expiratory volume (FEV1) noted on pulmonary function testing, which then returns to near-normal with introduction of inhaled bronchodilators, most likely has:

A) Emphysema
B) Chronic bronchitis
C) Asthma
D) Pneumonia
E) Pulmonary embolism
The correct answer is: C

Reversibility of airways obstruction with inhaled bronchodilators is typical of asthma. Ref: Resp/ Harrison's 16, p. 1549/RAL
Pa02 and oxygen saturation of hemoglobin are typically normal or near normal in patients with this disease.

A) Emphysema
B) Chronic bronchitis
C) Pulmonary embolism
D) Congestive heart failure
E) Acute asthmatic attack
The correct answer is: A

Normal hemoglobin saturation and the characteristic tachypnea with exhalation through pursed lips, has led to the term "pink puffers" to describe patients with emphysema. Ref: Resp/ Harrison's 16, p. 1551/RAL
Which of the following values decreases in severe COPD?

A) Lung volume
B) Total lung capacity
C) Functional residual capacity
D) Residual volume
E) 1-second forced expiratory volume
The correct answer is: E

1-second forced expiratory volume (FEV1) typically decreases in COPD. All other values listed typically increase. Ref: Resp/ Harrison's 16, p. 1551/ RAL
Which of the following is true in patients with COPD who experience a superimposed acute respiratory infection?

A) Sputum production increases and becomes mucopurulent in character.
B) Haemophilus influenzae, Streptococcus pneumoniae, and Moraxella catarrhalis are the most common bacterial pathogens.
C) A 7 to 10 day course of broad-spectrum antibiotics should be prescribed on an empirical basis.
D) Both A and B
E) All of the above
The correct answer is: E

In the presence of toxic symptoms (fever, chills), pleuritic chest pain, or failure to respond to empirical therapy, sputum Gram stain and culture should be obtained to guide treatment. Ref: Resp/ Harrison's 16, p. 1553/ RAL
A 75-year old man with a long history of COPD presents with the acute onset of worsening dyspnea, increased productive cough, and marked agitation. His PaO2, usually in the mid 60's, has dropped to 50mmHg. His PaCO2, likewise, is around 50mmHg. His arterial pH is 7.35. In the immediate management of this patient:

A) Supplemental oxygen administration should be avoided, due to the threat of worsening hypercarbia.
B) Antibiotic therapy should be avoided in the absence of infection proven by sputum culture.
C) Administration of a rapid-acting inhaled beta agonist is probably indicated.
D) Oral steroids should be avoided, due to threat of immunocompromise and worsening infection.
E) All of the above
The correct answer is: C

The hypoxic patient should be supported with supplemental oxygen and bronchodilators. Empirical use of antibiotics and oral steroids is common and should be considered. Ref: Resp/ Harrison's 16, p. 1553; Goroll 4, p. 302/ RAL
A 75-year-old man with a long history of COPD presents with the acute onset of worsening dyspnea, increased productive cough, and marked agitation. His PaO2, usually in the mid 60's, has dropped to 50mmHg. His PaCO2, likewise, is around 50mmHg. His arterial pH is 7.35. This patient is later noted to develop a deteriorating mental status, becoming lethargic and finally, obtunded. Although his hypoxemia has improved, his hypercapnia has worsened. Appropriate management at this point includes which of the following?

A) Oxygen supplementation with a 100% non-rebreather mask
B) Discontinue all supplemental oxygen and observe
C) Endotracheal intubation and mechanical ventilation
D) Emergency tracheostomy to reduce "dead space"
E) None of the above
The correct answer is: C

Intubation and mechanical ventilation to reduce hypercapnia is the best intervention. Improving FIO2 by mask will not improve hypercapnia. Removing all supplemental oxygen in a patient whose respiratory drive is depressed by CO2 retention will not increase respiratory rate, even after hypoxia develops. Ref: Resp/ Harrison's 16, p. 1554/ RAL
Carcinoma of the lung is the leading cause of cancer death:

A) among men.
B) among women.
C) among smokers.
D) Both A and C
E) All of the above
The correct answer is: E

Primary carcinoma is the leading cause of cancer death among men and recently surpassed breast cancer as the leading cause of cancer death among women. The incidence of lung cancer is 13 times higher among current smokers, compared with nonsmokers. Ref: Resp/Harrison's 16, p. 506/RAL
The histologic type of lung cancer associated with the worst prognosis is:

A) Epidermoid carcinoma
B) Small cell carcinoma
C) Adenocarcinoma
D) Large cell carcinoma
E) The 5-year survival is approximately equal for all cell types.
The correct answer is: B

Small cell carcinoma is usually unresectable at the time of diagnosis and has the worst prognosis. The 5-year survival rate is about 5%. Ref: Resp/ Harrison's 16, p. 506/ RAL
Screening for lung cancer with annual chest X-rays:

A) Improves 5-year survival among smokers only
B) Improves survival only when combined with annual screening sputum cytology
C) Is indicated in male patients over 45 years old
D) Improves survival when positive tests are followed-up with spiral computed tomography
E) None of the above
The correct answer is: E

Annual chest X-rays, even among high-risk populations, (e.g., smokers over 45 years olD) , has not been shown to improve survival. Ref: Resp/ Harrison's 16, p. 509/ RAL
A 34-year old non-smoking male is noted to have a solitary 1cm pulmonary nodule on routine chest X-ray. You obtain old films, which demonstrate no change in the nodule over a 2-year period. Accordingly, you recommend:

A) fine-needle aspiration of the nodule under CT guidance.
B) referral for wedge resection of the suspect nodule.
C) bone scan to rule out metastases.
D) pulmonary function studies in preparation for pulmonary lobectomy.
E) None of the above
The correct answer is: E

Lack of change in the nodule, as determined by chest X-rays over 2-year intervals, is a reliable indicator that the nodule is benign, especially in this very low-risk (young, nonsmoking) individual. Ref: Resp/Goroll 4, p. 285/ RAL
In a patient suspected of having obstructive sleep apnea, the most useful diagnostic procedure is:

A) MRI of the head and neck
B) 24-hour Holter monitor
C) Polysomnography
D) Sleep-deprived EEG
E) Flexible fiberoptic nasopharyngoscopy
The correct answer is: C

Polysomnography is a sleep study that monitors oxygen saturation, respirations, level of sleep, and EEG. It is the definitive test for sleep apnea. Ref: Resp/Harrison's 16, p. 1574/ RAL
Adult respiratory distress syndrome (ARDS):

A) is most commonly associated with sepsis.
B) requires aggressive hydration with higher than normal left atrial filling pressures.
C) rarely requires positive end-expiratory pressure (PEEP) ventilation.
D) is characterized by decreased permeability of the alveocapillary membrane.
E) generally does not produce hypoxemia or hypercapnia.
The correct answer is: A

Pulmonary edema in ARDS results from an increase in the alveocapillary membrane permeability. Sepsis is the most common underlying cause. Both hypercapnia and hypoxemia are typical. Treatment generally requires mechanical ventilation with PEEP and reduction of left atrial filling pressures, to the extent that maintenance of hemodynamics will permit. Ref: Resp/ Harrison's 16, p. 1592-1594/ RAL
The characteristic X-ray findings in ARDS are:

A) small to moderate bilateral pulmonary effusions.
B) clear lungs with elevated hemidiaphragm.
C) diffuse bilateral interstitial and alveolar infiltrates.
D) a patchy infiltrate in a lobar distribution.
E) cardiomegaly with hilar fullness.
The correct answer is: C

The characteristic infiltrates are bilateral and diffuse, without pulmonary effusions, cardiomegaly, or vascular redistribution. Ref: Resp/ Harrison's 16, p. 1592/ RAL
Which of the following unilateral findings suggest a diagnosis of closed pneumothorax?

A) Tracheal deviation toward the affected side
B) Dullness to percussion
C) Decreased breath sounds on the affected side
D) Refractory hypertension
E) All of the above
The correct answer is: C

Tracheal deviation away from the affected side, hyperresonance to percussion, decreased breath sounds on the affected side, and hypotension are typical. Ref: Resp/Schwartz 8, p. 131/ RAL
Which statement is true concerning flail chest?

A) Respiratory failure due to flail chest requires intubation and mechanical ventilation..
B) Fracture of a single rib may produce flail chest.
C) Underlying lung contusion is rare.
D) The initial chest X-ray tends to overestimate the degree of pulmonary contusion.
E) All of the above are true.
The correct answer is: A

Respiratory failure, whether due to paradoxical movement of the chest wall or to underlying chest contusion (as is common) should be treated by intubation and mechanical ventilation. Generally, fracture of a least 4 ribs is necessary to produce flail chest. The initial chest X-ray tends to underestimate the degree of pulmonary contusion. Ref: Resp/ Schwartz 8, p. 131/ RAL
Tension pneumothorax results when a one-way valve effect allows air to enter the pleural space on inspiration, but prevents its release during expiration. Physiologic or anatomic changes associated with this condition include which of the following?

A) Sudden, severe hypertension
B) Negative intrapleural pressure on the affected side
C) Deviation of the trachea away from the affected side
D) A sudden decrease in the central venous pressure
E) None of the above
The correct answer is: C

Additional typical findings include: systemic hypotension; positive intrapleural pressure on the affected side; and increased central venous pressure with distended neck veins. Ref: Resp/ Schwartz 8, p. 131/ RAL
In tube thoracostomy for tension pneumothorax, the preferred site for insertion of the tube is which of the following?

A) Second intercostal space in the anterior axillary line, contralateral to the injury
B) Second intercostal space in the midclavicular line, ipsilateral to the injury
C) Fourth or fifth intercostal space in the anterior axillary line, ipsilateral to injury
D) Fourth or fifth intercostal space in the midaxillary line, contralateral to the injury
E) Seventh intercostal space in the anterior axillary line, ipsilateral to the injury
The correct answer is: C

The tube should be inserted at the fourth or fifth intercostal space, in the anterior axillary line, on the side of the injury. Ref: Resp/ Schwartz 8, p. 131/ RAL
The most common cause of transudative pleural effusion is:

A) Pulmonary tuberculosis
B) Bacterial pneumonia
C) Primary pulmonary malignancy
D) Viral pneumonia
E) Left-sided heart failure
The correct answer is: E

Transudative pleural effusions are caused by systemic factors, the most common of which is left-sided heart failure. The other choices cause accumulation of pleural fluid due to local factors, or exudative pleural effusions. Ref: Resp/Harrison's 16, p. 1566/ RAL
Which of the following characteristics of aspirated pleural fluid are consistent with an exudative pleural effusion?

A) Pleural fluid protein greater than one half the serum protein
B) Ratio of pleural fluid LDH to serum LDH greater than 0.6
C) Pleural fluid creatinine greater than serum creatinine
D) Both A and B
E) All of the above
The correct answer is: D

Either Answers A or B constitute a diagnostic criterion for a pleural exudate. A pleural LDH greater than two-thirds the upper limit of normal for serum is also an adequate diagnostic criterion. Pleural fluid that fails to meet any of these criteria is a transudate. Ref: Resp/ Harrison's 16, p. 1566/ RAL
Which regimen represents an appropriate use of purified protein derivative (PPD) in the diagnosis of tuberculosis?

A) 25 tuberculin units (TU) injected subcutaneously; reaction read in 24 hours
B) 5 tuberculin units (TU) injected intradermally; reaction read in 48 to 72 hours
C) 10 tuberculin units (TU) injected subcutaneously; reaction read in 48 to 72 hours
D) 5 tuberculin units (TU) injected subcutaneously; reaction read in 24 hours
E) 5 tuberculin units (TU) injected intradermally; reaction read in 24 hours
The correct answer is: B

5 TU PPD is injected intradermally. Reaction is read in 48-72 hours as the transverse diameter of the area of induration. Ref: Resp/ Harrison's 16, p. 964/ RAL
What is the minimum diameter of induration, in response to a PPD test, considered positive in an HIV-infected individual?

A) 15mm
B) 10mm
C) 5mm
D) 2mm
E) Any reaction in an HIV-infected individual is positive.
The correct answer is: C

For high-risk individuals, including patients with HIV infection, an area of induration 5mm in diameter or greater is considered positive. Ref: Resp/ Harrison's 16, p. 965/ RAL
The definitive diagnosis of pulmonary tuberculosis is established by which of the following findings?

A) A positive PPD skin test
B) Isolation of Mycobacterium tuberculosis from a sputum specimen
C) A chest X-ray showing bilateral upper lobe infiltrates
D) Demonstration of acid-fast bacilli on three consecutive expectorated sputum specimens
E) Any of the above
The correct answer is: B

Definitive diagnosis requires a culture positive for M. tuberculosis. Ref: Resp/ Harrison's 16, p. 960/ RAL
The most common significant side-effect of multi-drug regimens for the treatment of tuberculosis is:

A) Drug-induced hepatitis
B) Thrombocytopenia
C) Renal failure
D) Optic neuritis
E) Gouty arthritis
The correct answer is: A

Hepatitis is the most common side-effect of treatment causing interruption of therapy. Pyrazinamide also causes gouty arthritis; rifampin may cause thrombocytopenia; and ethambutol may cause optic neuritis. Ref: Resp/ Harrison's 16, p. 963/ RAL
Which statement(s) is (arE) true concerning the prevalence of sarcoidosis in the United States?

A) In the United States, sarcoidosis is most common among African Americans.
B) Sarcoidosis usually presents in childhood.
C) There is a 2:1 male predominance.
D) Cigarette smoking is a risk factor.
E) All of the above
The correct answer is: A

Sarcoidosis is an inflammatory disorder of unknown etiology. The ratio of African-Americans to whites (in the United States) is at least 10:1, and there is a slight female predominance. It usually presents between the ages of 20 and 40, and is more common among nonsmokers. Ref: Resp/ Harrison's 16, p. 2017-2018/ RAL
Which is not a common manifestation of sarcoidosis?

A) Productive cough with purulent sputum
B) Lymphadenopathy
C) Fatigue
D) Dyspnea
E) Abnormal chest X-ray
The correct answer is: A

Patients with sarcoidosis typically have dyspnea and a dry cough. Peripheral lymphadenopathy (cervical, axillary, inguinal, epitrochlear) is common, as are constitutional complaints, such as fever, anorexia, and weight loss. The classical finding on chest X-ray is bilateral hilar adenopathy. Ref: Resp/Harrison's 16, p. 2019-2020/ RAL
The most common underlying cause of bronchiectasis is which of the following?

A) Lower respiratory tract infections
B) Exposure to inhaled toxic chemical agents
C) Chronic aspiration
D) Cystic fibrosis
E) Primary disorders of mucociliary clearance
The correct answer is: A

Bronchiectasis results when infection or inflammation of the bronchial wall results in permanent dilatation of the bronchi, eventually impairing the clearance of secretions and causing obstruction of smaller airways. Although all choices may be implicated in the pathophysiology of the disease, viral and bacterial lower respiratory tract infections are the most common. Ref: Resp/Harrison's 16, p. 1541-1542/ RAL
Treatment of bronchiectasis commonly includes:

A) Oral antibiotics
B) Bronchodilators
C) Pulmonary lobectomy
D) Both A and B
E) All of the above
The correct answer is: D

Surgical treatment is rarely helpful. Chronic antibiotic therapy or intermittent treatment to control acute exacerbations are both used. Bronchodilators are useful in patients with reversible airway obstruction. Ref: Resp/ Harrison's 16, p. 1543/ RAL
A 65-year old man develops constitutional symptoms, sore throat, and cough during a winter influenza outbreak. He is treated symptomatically with acetaminophen and rest, and is nearly well again in 7 days. Four days later he returns with recurrent fever, myalgias, and a productive cough with purulent sputum. The most likely diagnosis is which of the following?

A) Post-influenza asthenia
B) Viral pneumonia
C) Secondary bacterial pneumonia
D) Recurrent influenza (a second viral typE) E) Underlying chronic bronchitis
The correct answer is: C

Secondary bacterial pneumonia is the most common complication of influenza. The most common causative agents are Streptococcus pneumoniae, Staphylococcus aureus, and Haemophilus influenzae. Appropriate antibiotic treatment is indicated. Ref: Resp/ Harrison's 16, p. 1068/ RAL
Inactivated influenza vaccines are recommended for:

A) Diabetics
B) Healthy individuals over 65 years old
C) Immunocompromised individuals
D) Patients in renal failure
E) All of the above
The correct answer is: E

Influenza vaccines are inactivated and can safely be given to immunocompromised patients. They are also recommended for patients in chronic renal failure, diabetics, and persons 65 years old or older. Ref: Resp/ Harrison's 16, p. 1070/ RAL
Which of the following is the most common malignancy associated with asbestos exposure?

A) Carcinoma of the lung
B) Pleural mesothelioma
C) Peritoneal mesothelioma
D) Gastric carcinoma
E) Squamous cell carcinoma of the oropharynx
The correct answer is: A

Carcinoma of the lung, including squamous cell and adenocarcinoma, are the most frequent associated malignancies. Pleural mesothelioma and, less frequently, peritoneal mesothelioma are also causally related to asbestos exposure. Ref: Resp/ Harrison's 16, p. 1522-1523/ RAL
Routine annual surveillance chest X-rays:

A) improve survival in the general adult population.
B) improve survival among male smokers.
C) improve survival among male smokers when combined with annual sputum cytology.
D) improve survival among male and female smokers, but are prohibitively expensive.
E) have no measurable impact on survival.
The correct answer is: E

Studies have failed to show that routine chest X-rays, with or without sputum sampling for cytology, provide any benefit in terms of survival. Ref: Resp/ Goroll 4, p. 258-259/ RAL
Which neonatal disorder may be an early presenting symptom of cystic fibrosis?

A) Gastroschisis
B) Duodenal atresia
C) Meconium ileus
D) Conjunctivitis
E) Anemia
The correct answer is: C

The newborn with meconium ileus is presumed to have cystic fibrosis until ruled out by further testing. 18% of children with cystic fibrosis will present this way. Ref: Resp/ Hay 17, p. 47; Harrison's 16, p. 1545/ RAL
Which is the most appropriate test to establish the diagnosis of cystic fibrosis (CF)?

A) Serum protein electrophoresis
B) Sweat chloride level
C) Hemoglobin electrophoresis
D) Bronchoscopy with bronchoalveolar lavage
E) Contrast-enhanced CT scan of the chest
The correct answer is: B

Sweat testing, with a sweat chloride level greater than 60 mEq/L, confirms the diagnosis of CF. Ref: Resp/ Harrison's 16, p. 1545; Hay 17, p. 527/ RAL
A patient with excessive morning and daytime sleepiness is referred for polysomnography, during which recurrent apneas without respiratory effort are observed. The most likely diagnosis at this point is:

A) Obstructive sleep apnea
B) Central sleep apnea
C) Narcolepsy
D) Sedative abuse
E) Seizure disorder
The correct answer is: B

Apnea without respiratory effort is the key finding in polysomnography supporting the diagnosis of central sleep apnea. Ref: Resp/ Harrison's 16, p. 1576/ RAL
The first phase of acute respiratory distress syndrome (ARDS) is the:

A) proliferative phase.
B) exudative phase.
C) fibrotic phase.
D) hypoxic phase.
E) consolidation phase.
The correct answer is: B

During the initial exudative phase of ARDS, damage to the alveolar epithelium and capillary endothelium lead to accumulation of protein-rich fluid in the alveoli and interstitium, as well as to formation of microthrombi in the pulmonary vasculature. Ref: Resp/ Harrison's 16, p. 1592/ RAL
In managing the mechanical ventilation of a patient with acute respiratory distress syndrome (ARDS), what FI02 should be maintained in order to avoid oxygen toxicity?

A) Less than 1.0
B) Less than 0.8
C) Less than 0.6
D) Less than 0.4
E) No more than 0.21
The correct answer is: C

Other ventilator parameters (PEEP, rate, tidal volumE) should be adjusted to maintain adequate oxygenation with an FIO2 of less than 0.6. Ref: Resp/ Harrison's 16, p. 1594/ RAL
In a patient on a mechanical ventilator, the primary purpose of the pressure support ventilation (PSV) mode is:

A) stabilizing the patient with acute respiratory failure.
B) weaning a patient from mechanical ventilation.
C) avoiding barotrauma in chronically ventilator-dependent patient.
D) treating acute respiratory distress syndrome (ARDS).
E) ventilating a patient with chronic restrictive lung disease during an exacerbation.
The correct answer is: B

PSV is a patient-triggered ventilator mode primarily used during the weaning process. Ref: Resp/ Harrison's 16, p. 1598-1599/ RAL
A previously-healthy 30-year-old male smoker with no known pulmonary disease and no history of trauma presents in the Emergency Department with the sudden onset of pleuritic chest pain and dyspnea. He is hemodynamically stable, has now distention of his neck veins, and has decreased breath sounds on the right. Chest X-ray reveals the absence of peripheral lung markings. The most likely diagnosis is:

A) Primary spontaneous pneumothorax
B) Secondary spontaneous pneumothorax
C) Tension pneumothorax
D) Pulmonary embolism
E) Aspiration pneumonia
The correct answer is: A

The history, physical findings, and X-ray are typical of pneumothorax. A patient with pneumothorax and no pre-existing lung disease or history of trauma has primary spontaneous pneumothorax. A patient with spontaneous pneumothorax and a history of prior lung disease (usually COPD) has secondary spontaneous pneumothorax. Ref: Resp/ Harrison's 16, p. 1568/ RAL
The emergency treatment of an unstable patient with tension pneumothorax includes the insertion of a large-bore needle into the pleural space at the:

A) 5th intercostal space in the posterior axillary line.
B) 7th intercostal space in the anterior axillary line.
C) 7th intercostal space in the midaxillary line.
D) 2nd intercostal space in the posterior axillary line.
E) 2nd intercostal space in the midclavicular line.
The correct answer is: E

The intrathoracic pressure can be relieved by the insertion of a large-bore needle at the 2nd intercostal space in the midclavicular line. When the patient is stabilized, a thoracostomy tube and closed water-seal drainage are instituted. Ref: Resp/ Harrison's 16, p. 1568/ RAL
Which is the most appropriate treatment for a patient with recurrent spontaneous pneumothorax?

A) Pulmonary lobectomy
B) Pulmonary segmentectomy
C) Thoracoscopic pleural abrasion
D) Thoracoscopic instillation of glucocorticoid solution
E) Rigid bronchoscopy with re-expansion of the collapsed lobe
The correct answer is: C

Pleural abrasion by thoracoscopy is effective in preventing recurrent spontaneous pneumothorax. Ref: Resp/ Harrison's 16, p. 1568/ RAL
A patient with a chest tube in the pleural space is placed on water-seal drainage. When he is asked to cough, bubbles pass through the water seal chamber. What is the most likely significance of this observation?

A) There is a break in the tubing between the patient and the water seal.
B) There is an air leak into the pleural space.
C) There is a tension pneumothorax on the opposite side.
D) The chest tube has been inadvertently placed in the lung parenchyma.
E) There has been a failure of the suction apparatus sometime in the previous hours.
The correct answer is: B

Bubbles passing through the chamber during voluntary cough suggest an air leak into the pleural space. Ref: Resp/ Schwartz 8, p. 555/ RAL
Cancer of which organ causes the greatest mortality among women?

A) Ovary
B) Breast
C) Colon
D) Lung
E) Uterus
The correct answer is: D

Lung cancer causes 25% of all cancer mortality among women. Breast cancer causes 15%, and colorectal cancer causes 11%. Ovarian and uterine cancer cause 5% and 3%, respectively. Ref: Resp/ Schwartz 8, p. 557/ RAL
Which symptom would most likely be related to tumor invasion of the recurrent laryngeal nerve?

A) Neck pain
B) Hoarseness
C) Facial paresthesias
D) Facial asymmetry
E) Dyspnea
The correct answer is: B

Damage to the recurrent laryngeal nerve causes hoarseness, or weakness of the voice, due to vocal cord paralysis. Ref: Resp/ Schwartz 8, p. 562/ RAL
Invasion of the phrenic nerve by a lung tumor may cause paralysis of the diaphragm. When this is suspected, what would be the most appropriate X-ray study to confirm the diagnosis?

A) CT scan of the chest without contrast
B) Contrast-enhanced CT scan of the chest
C) Cross-table lateral film of the chest
D) Supine and upright films of the abdomen
E) Chest fluoroscopy
The correct answer is: E

Chest fluoroscopy while the patient inhales and exhales ("sniff test") demonstrates the motion or paralysis of the diaphragm. Ref: Resp/ Schwartz 8, p. 562/ RAL
Tumor invasion or compression of the superior vena cava (SVC) may result in its occlusion, leading to the superior vena cava syndrome. What symptoms would be the most likely?

A) Transient monocular loss of vision
B) Permanent bilateral loss of vision
C) Swelling of the face
D) Hypotension
E) Upper airway obstruction
The correct answer is: C

SVC syndrome causes swelling of the head, neck and arms. Ref: Resp/ Schwartz 8, p. 562/ RAL
Which is the most common cause of lung abscess?

A) Tuberculosis
B) Community-acquired pneumonia
C) Complications of percutaneous biopsy procedures
D) Aspiration
E) Hematogenous spread from nonpulmonary infections
The correct answer is: D

Most primary lung abscesses are caused by aspiration. Because the geometry of the bronchial tree and gravity tend to direct aspirated secretions to the right lung, most abscesses occur on the right side. Ref: Resp/ Schwartz 8, p. 573/ RAL
Which is the most frequent tumor the anterior mediastinum?

A) Neurofibroma
B) Gangloineuroma
C) Thymoma
D) Malignant schwannoma
E) Neuroblastoma
The correct answer is: C

The thymus is in the anterior mediastinum, and thymomas are the most frequent neoplasm in this compartment. Neurogenic tumors, represented by the other choices, tend to be in the posterior compartment. Ref: Resp/ Schwartz 8, p. 591/ RAL
Which drug is approved for treatment of intractable hiccups?

A) Diazepam (Valium)
B) Ranitadine (ZantaC)
C) Chlorpromazine (ThorazinE)
D) Calcium gluconate
E) Carbamazepine (Tagamet)
The correct answer is: C

Chlorpromazine by intravenous injection is FDA approved for the treatment of intractable hiccups. Hypotension and extrapyramidal symptoms are potential side effects.
Which is a noninvasive intervention that will speed the reabsorption of a small, stable, spontaneous pneumothorax?

A) Place the patient in the left lateral decubitus position.
B) Administer nasal oxygen.
C) Administer intravenous sodium bicarbonate to alkalinize the urine.
D) Administer a beta agonist by inhalation.
E) Have the patient exhale into a paper bag.
The correct answer is: B

Nasal oxygen at 3-4 liters per minute increases the rate at which a small pneumothorax will be reabsorbed. Ref: Resp/ Tintinalli 6, p. 463/ RAL
Which physical finding is NOT consistent with an acute asthmatic attack?

A) Hyperresonance to percussion
B) Decreased intensity of breath sounds
C) Prolongation of the expiratory phase of respiration
D) Wheezing
E) Dependent rales
The correct answer is: E

Dependent rales are more consistent with interstitial pulmonary edema, due to cardiogenic or other causes. The other choices are physical findings typical of an acute asthmatic episode. Ref: Resp/ Tintinalli 6, p. 468/ RAL
In a three-year-old child with stridor, fever, a sore throat, and drooling, what is the most appropriate diagnostic X-ray study?

A) PA and lateral chest X-ray
B) Lateral neck films with soft-tissue technique
C) CT scan of the chest without contrast
D) Barium swallow
E) Lateral decubitus X-ray of the chest
The correct answer is: B

This is a presentation highly suggestive of epiglottitis. A soft-tissue lateral film of the neck may confirm the diagnosis. Ref: Resp/ Tintinalli 6, p. 849-850/ RAL
An 18-month-old child presents with a 4-day history of a worsening cold with low-grade fever, followed by the development of a barking cough and stridor on both inspiration and expiration. The most likely diagnosis is:

A) Epiglottitis
B) Bacterial pneumonia
C) Foreign body obstructing the airway
D) Croup
E) Asthma
The correct answer is: D

Croup (viral laryngotracheitis) commonly presents with a 1-5 day URI-like prodrome, low-grade fever, and characteristic "barking" cough. It is most common between the ages of 1 and 2 years, and uncommon after the age of 6 years. Ref: Resp/ Tintinalli 6, p. 851-852/ RAL
Which agent is standard treatment for moderate to severe croup?

A) Oral acyclovir
B) Nebulized epinephrine
C) Intravenous cefuroxime
D) Intravenous vancomycin
E) Nebulized albuterol
The correct answer is: B

Nebulized racemic epinephrine is a mainstay of treatment in croup. It may provide clinical improvement in as little as 10 minutes. Oral steroids, typically dexamethasone, are also standard treatment. Ref: Resp/ Tintinalli 6, p. 852-853/ RAL
A 2-year-old child is brought to the Emergency Department with a history of a choking episode observed after his well-meaning older brother was seen feeding him peanuts. Although noted to have stridor on presentation, this rapidly and spontaneously clears. Auscultation of the chest reveals only occasional wheezing. The child is awake and alert and maintaining adequate oxygen saturation on room air. Chest X-ray is unremarkable. What is the best management of the patient at this point?

A) Prepare the patient for bronchoscopy.
B) Administer intravenous antibiotics and observe.
C) Administer oral antibiotics and discharge to home after appropriate counseling.
D) Notify the appropriate authorities and admit the child pending legal proceedings.
E) Obtain an MRI of the neck and chest.
The correct answer is: A

Stridor and wheezing can improve or worsen with movement of the foreign body, or the patient may be asymptomatic, with a normal physical examination, at the time of presentation. Chest X-ray may be nondiagnostic. If there is strong clinical suspicion, as in this scenario, aspiration of a foreign body should be ruled out by bronchoscopy. Ref: Resp/ Tintinalli 6, p. 855/ RAL
Which of the following agents, or classes of agents, are contraindicated in the treatment of pregnant women with asthma exacerbations?

A) Humidified oxygen by mask
B) Beta 2- agonists
C) Inhaled corticosteroids
D) Oral steroids
E) None of the above
The correct answer is: E

Oxygen, corticosteroids, and beta 2 -agonists are generally considered safe. Ref: Resp/ Tintinalli 6, p. 473/ RAL
Indications for mechanical ventilation in the COPD patient suffering an acute exacerbation include which of the following?

A) Oxygen saturation less than 93% on room air
B) The production of excessive, purulent sputum
C) Severe acidosis (pH less than 7.25) D) Frequent premature ventricular contractions on EKG
E) History of recent myocardial infarction (within the last 6 months)
The correct answer is: C

Acidosis is an indication for mechanical ventilation. Other indications are: a respiratory rate greater than 35 per minute; impaired mental status; hypoxemia (PaO2 less than 50 mmHg); and respiratory arrest. Ref: Resp/ Tintinalli 6, p. 479/ RAL
Which of these radiographic studies is the most appropriate in a patient with suspected pulmonary embolism?

A) Stat portable chest X-ray
B) Thoracic ultrasound
C) Helical CT scan without contrast
D) Helical CT scan with contrast
E) Apical lordotic chest X-ray
The correct answer is: D

The contrast-enhanced helical CT scan of the chest is an accurate means of detecting pulmonary embolism, currently in use in many centers. Alternatives are the ventilation-perfusion scan and the gold-standard (but more invasivE) pulmonary angiogram. Ref: Resp/ Tintinalli 6, p. 485/ RAL
Which of the following conditions generally produce an exudative pleural effusion?

Left -sided heart failure
B) Right-sided heart failure
C) Pulmonary embolization
D) Nephrotic syndrome
The correct answer is: A

A) Pulmonary and pleural malignancies, most commonly bronchogenic carcinoma, produce exudative pleural effusions. Likewise, infections, like pneumonia and empyema, cause exudates. Transudates are generally caused by congestive heart failure (right- or left-sideD) , pulmonary emboli, and the nephritic syndrome. Pleural fluid is tested for protein and LDH to determine whether it is exudative or transudative. Ref: Resp/ Goroll 4, p. 280/ RAL
In the chronic (compensateD) phase both pH and PCO2 are decreased.

A) Respiratory acidosis
B) Respiratory alkalosis
C) Metabolic acidosis
D) Metabolic alkalosis
The correct answer is: C

Compensation of metabolic acidosis results in tachypnea and increased respiratory losses of CO2, representing partial respiratory compensation of a metabolic acidosis. Ref: Resp/Harrison's 16, p. 263-265/RAL
This condition is usually caused by inadequate ventilation, as in airway obstruction, respiratory depression, or intrinsic lung disease, etc.

A) Respiratory acidosis
B) Respiratory alkalosis
C) Metabolic acidosis
D) Metabolic alkalosis
The correct answer is: A

In addition to respiratory acidosis, atelectasis, pneumonia, and pleural effusion are other causes. Ref: Resp/Harrison's 16, p. 269/RAL
In the acute stage of this condition, the PCO2 is normal and the serum bicarbonate is elevated; in the chronic stage the PCO2 may be elevated.

A) Respiratory acidosis
B) Respiratory alkalosis
C) Metabolic acidosis
D) Metabolic alkalosis
The correct answer is: D

Late compensation of metabolic alkalosis involves incomplete compensation by the retention of CO2. Ref: Resp/Harrison's 16, p. 267/RAL
This condition may be due to pain, anxiety, or excessive mechanical ventilation; it causes hypokalemia.

A) Respiratory acidosis
B) Respiratory alkalosis
C) Metabolic acidosis
D) Metabolic alkalosis
The correct answer is: B

Excessive respiration due to any of these causes results in respiratory alkalosis by "blowing off" CO2. As potassium ions move intracellularly to replace the hydrogen ions released to correct the pH, relatively minor hypokalemia may develop. Ref: Resp/Harrison's 16, p. 270, 270/RAL
In the chronic (compensateD) phase of this condition, the pH is high and the serum bicarbonate is low.

A) Respiratory acidosis
B) Respiratory alkalosis
C) Metabolic acidosis
D) Metabolic alkalosis
The correct answer is: B

Full metabolic compensation of respiratory alkalosis takes several days and involves the increased renal excretion of bicarbonate. Ref: Resp/Harrison's 16, p. 270/RAL
Decreased tissue perfusion (as in hemorrhagic shock), renal failure, or severe diarrhea may cause this condition.

A) Respiratory acidosis
B) Respiratory alkalosis
C) Metabolic acidosis
D) Metabolic alkalosis
The correct answer is: C

Failure to excrete hydrogen ions, as in renal failure; the accumulation of lactic acid, caused by tissue hypoxia in shock; and bicarbonate losses, as in diarrhea, may cause metabolic acidosis. Ref: Resp/Harrison's 16, p. 265/RAL
This condition may result from protracted vomiting or nasogastric suction.

A) Respiratory acidosis
B) Respiratory alkalosis
C) Metabolic acidosis
D) Metabolic alkalosis
The correct answer is: D

Elevated arterial pH due to loss of acidic gastric secretions, as in nasogastric aspiration or chronic vomiting, may cause metabolic alkalosis. Ref: Resp/Harrison's 16, p. 267-268/RAL
Which antacid produces a laxative effect and must be avoided in patients in renal failure?

A) Calcium carbonate
B) Aluminum hydroxide
C) Magnesium hydroxide
The correct answer is: C

Magnesium hydroxide may be absorbed systemically, producing hypermagnesemia in patients with renal insufficiency. All patients may experience loose stools. Ref: GI/ Harrison's 16, p. 1753/RAL
Relaxation of the lower gastroesophageal sphincter, allowing reflux of gastric contents into the distal esophagus, may be caused by which of the following?

Calcium channel blocking drugs (CCB's)
A) CCBs
B) Smoking
C) Coffee
D) Both A and C
E) All of the above
The correct answer is: E

Smooth muscle relaxing drugs (such as CCB's), nicotine, and xanthene-containing beverages (coffee and teA) may cause relaxation of the lower esophageal sphincter. Ref: GI/ Harrison's 16, p. 1742/ RAL
A disorder in which the lower esophageal sphincter fails to relax on swallowing is:

A) Odynophagia
B) Scleroderma
C) Achalasia
D) Boerhaave's syndrome
E) Barrett's esophagus
The correct answer is: C

Achalasia is a motor disturbance of the lower esophageal sphincter in which it fails to relax on swallowing, causing chest pain and regurgitation.
A 50-year old obese male smoker presents with a complaint of "heartburn" following large meals, when in a recumbent position, or when wearing tight-fitting clothing. He obtains transient relief from antacids. He denies dysphagia. An appropriate next step in this patient's management is:

A) Upper GI endoscopy and biopsy
B) Bernstein test
C) Barium swallow
D) Both A and C
E) None of the above
The correct answer is: E

In an unambiguous case of uncomplicated gastroesophageal reflux disease, a trial of lifestyle modification (weight loss, smoking cessation, avoidance of caffeine and alcohol) and drug therapy, usually an H-2 blocking agent or a proton pump inhibitor (PPI), is a reasonable first step. Ref: GI/ Harrison's 16, p. 1743/ RAL
A 50-year-old male smoker presents with a complaint of "heartburn" following large meals, when in a recumbent position, or when wearing tight-fitting clothing. He obtains transient relief from antacids. He also notes progressive dysphagia of three months duration with an unintended 25-pound weight loss. An appropriate next step in the management of this patient is:

A) A trial of metoclopramide (Reglan) B) Prescribe a mechanical soft solid diet
C) Measurement of the pH of an aspirated gastric sample
D) Esophagoscopy and biopsy
E) None of the above
The correct answer is: D

Dysphagia and weight loss are unusual in uncomplicated gastroesophageal reflux disease. Esophagoscopy and biopsy of any suspicious lesion is a reasonable next step. Ref: GI/ Harrison's 16, p. 1743/ RAL
A 60-year old man with known peptic ulcer disease (PUD) , presents with abdominal pain, board-like abdominal rigidity, and rebound tenderness. Which complication of PUD is most likely?

A) Posterior penetration
B) Free peritoneal perforation
C) Upper gastrointestinal hemorrhage
D) Gastric outlet obstruction
E) Gastrocolic fistula
The correct answer is: B

Anterior perforation produces peritonitis, with typical peritoneal signs on physical exam. Ref: GI/Harrison's 16, p. 1752; Schwartz 8, p. 959/RAL
A 60-year old man with known peptic ulcer disease (PUD) , presents with abdominal pain, protracted vomiting, and metabolic alkalosisis. On examination, the abdomen is mildly tender without peritoneal signs. Which complication of PUD is most likely?

A) Posterior penetration
B) Free peritoneal perforation
C) Upper gastrointestinal hemorrhage
D) Gastric outlet obstruction
E) Gastrocolic fistula
The correct answer is: D

Patients with complete gastric outlet obstruction typically present with hypochloremia, hypokalemia, and metabolic alkalosis, and nonbilious vomiting. Ref: GI/Harrison's 16, p. 1752; Schwartz, p. 960/RAL
What percent of patients who present with upper gastrointestinal hemorrhage due to peptic ulcer disease will stop bleeding spontaneously if maintained NPO (non per os) and treated with parenteral medication to suppress gastric acid secretion?

A) 100%
B) 95%
C) 90%
D) 75%
E) 50%
The correct answer is: D

About 75% will stop bleeding. The remaining 25% will require endoscopic or surgical treatment. Ref: GI/ Schwartz 8, p. 959/ RAL
In an otherwise-healthy 60-year old man with known peptic ulcer disease develops upper gastrointestinal hemorrhage. Which of the following represents an indication for surgical or endoscopic treatment?

A) The patient develops hematemesis.
B) The patient requires fluid resuscitation with crystalloid IV solution.
C) The patient requires more than 4 units of packed red blood cells i(PRBC) in 24 hours.
D) The patient develops melena.
E) None of the above
The correct answer is: C

Requirement of more than 4 units of PRBC in 24 hours is generally an indication for surgical or endoscopic intervention. Ref: GI/ Schwartz 8, p. 967/ RAL
Antacids are frequently used in the management of peptic ulcer disease. Which antacid may produce constipation or phosphate depletion?

A) Calcium carbonate
B) Aluminum hydroxide
C) Magnesium hydroxide
D) Sodium bicarbonate
The correct answer is: B

Aluminum binds phosphate in the GI tract and may produce hypophosphatemia. Ref: GI/ Harrison's 16, p. 1753/RAL
Which of the following antacids has the potential to cause systemic alkalosis?

A) Calcium carbonate
B) Aluminum hydroxide
C) Magnesium hydroxide
D) Sodium bicarbonate
The correct answer is: D

Oral sodium bicarbonate has the potential to be absorbed, causing systemic metabolic alkalosis. Ref: GI/ Harrison's 16, p. 1753/ RAL
Which of the following anti-ulcer medications is contraindicated in women who may become pregnant?

A) Ranitidine (ZantaC) B) Omeprazole (PriloseC) C) Famotidine (PepciD) D) Misoprostal (CytoteC) E) Bismuth compounds (Pepto-Bismol)
The correct answer is: D

Misoprostal may induce uterine contractions, and is contraindicated in pregnancy. Ref: GI/ Harrison's 16, p. 1754/ RAL
What is the recommended duration of acid-suppression therapy in patients with peptic ulcer disease (PUD) who test positive for H. pylori?

A) 4-6 weeks
B) 8-12 weeks
C) 6 months
D) 1 year
E) They should be treated for life.
The correct answer is: A

Acid-suppression therapy should be continued for 4-6 weeks in this group of patients. They should also be treated for 14 days for H. pylori. Ref: GI/ Harrison's 16, p. 1754-1755/ RAL
Which of the following approaches DIFFERENTIATES the treatment of gastric ulcers (GU) from the treatment of duodenal ulcers (DU)?

A) Patients with GU should be tested for H. pylori.
B) Patients with GU require repeat endoscopy at 8-12 weeks.
C) Patients with NSAID-related DU, who cannot discontinue NSAIDS, should be treated with a proton pump inhibitor (PPI).
D) 90% of patients with DU will heal their ulcer with standard medical therapy.
E) All statements are true for both DU and GU.
The correct answer is: B

Due to the higher potential for malignancy with GU, patients with gastric ulcers require biopsy at the time of initial diagnosis and repeat endoscopy and biopsy at 8-12 weeks. Other statements are true for both DU and GU. Ref: GI/ Harrison's 16, 1756/ RAL
Which agent(s) are of benefit in the prevention of peptic ulcer disease (PUD) in patients on long-term nonsteroidal anti-inflammatory drug (NSAID) therapy?

A) Omeprazole (PriloseC)
B) Misoprostol (CytoteC)
C) Sucralfate (CarafatE) D) A and B are effective
E) All are effective
The correct answer is: D

Misoprostol and proton pump inhibitors (PPI's), such as omeprazole, have shown benefit in preventing gastric ulceration in patients on NSAID therapy. Ref: GI/ Harrison's 16, p. 1755/ RAL
When vagotomy and antrectomy are performed for duodenal ulcer disease, continuity of the gastrointestinal tract is maintained by constructing:

A) a gastroduodenostomy.
B) a gastrojejunostomy.
C) the interposition of a loop of colon.
D) Both A and B
E) None of the above
The correct answer is: D

Antrectomy and vagotomy may entail either a Billroth I (gastric remnant to duodenum) or a Billroth II (gastric remnant to jejunum) anastomosis. Ref: GI/ Schwartz 8, p. 962/ RAL
A patient who has undergone truncal vagotomy and Billroth II reconstruction returns postoperatively, complaining of light-headedness, palpitations, and diaphoresis, occurring about 30 minutes after a substantial meal. The most likely diagnosis is which of the following?

A) Dumping syndrome
B) Afferent loop syndrome
C) Retained antrum
D) Bile reflux gastritis
E) Hypoglycemia
The correct answer is: A

Dumping syndrome occurs when hyperosmolar stomach contents are rapidly passed into the small intestine, resulting in fluid shifts, the release of vasoactive hormones, and the symptoms noted. Ref: GI/Schwartz 8, p. 985/ RAL
A patient presents with symptoms determined to be due to anemia several years following subtotal gastric resection. What possible causes of his anemia should be considered and ruled out?

A) Iron-deficiency
B) Bleeding from a recurrent peptic ulcer
C) Pernicious anemia
D) Both B and C
E) All of the above
The correct answer is: E

Iron-deficiency, probably due to malabsorption, is the most common cause of anemia in patients who have undergone peptic ulcer surgery. Folate deficiency is also common. Patients who have undergone total or near-total gastrectomy, and who have lost all or most of their parietal cell mass may develop pernicious anemia due to lack of intrinsic factor. Finally, chronic blood loss from a recurrent peptic ulcer should be ruled out. Ref: GI/ Schwartz 8, p.990/ RAL
A patient with a recurrent duodenal ulcer refractory to the usual medical treatment is found to have markedly high levels of basal acid output on aspiration of gastric contents. You suspect Zollinger-Ellison syndrome. The best confirmatory test is which of the following?

A) Endoscopic biopsy of the ulcer
B) CT of the abdomen with oral contrast
C) Ultrasound of the abdomen
D) Secretin stimulation test (serum gastrin levels) E) Endoscopic retrograde cholangiopancreatography (ERCP)
The correct answer is: D

The confirmatory diagnostic test is serum gastrin level determination before and after stimulation by intravenous injection of the hormone secretin. Ref: GI/ Schwartz 8, p. 970/ RAL
What is the cause of gastric stress ulceration following burn or other major injury?

A) Acid hypersecretion in the stomach
B) Mucosal injury from indwelling nasogastric tubes
C) Unopposed gastrin secretion
D) Gastric mucosal ischemia
E) Transpyloric reflux of bile
The correct answer is: D

Gastric stress ulcers are likely due to mucosal ischemia, not gastric acid over-secretion. The best management, however, is prevention with the many antisecretory or cytoprotective agents currently available. Ref: GI/Schwartz 8, p. 971/ RAL
Which agent(s) has (havE) been shown to be effective in Helicobacter pylori-related peptic ulcer disease (PUD) ?

A) Metronidazole (Flagyl)
B) Bismuth compounds
C) Omeprazole (PriloseC)
D) Both A and B
The correct answer is: E

All agents listed have been successfully employed in medical regimens for gastritis related to H. pylori colonization. In addition, amoxicillin, tetracycline, and clarithromycin (Biaxin) are included in some regimens. Ref: GI/ Harrison's 16, p. 1754/ RAL
This type of inflammatory bowel disease is nearly always limited to the rectum and colon.

A) Ulcerative colitis
B) Crohn's Disease
The correct answer is: A

Ulcerative colitis almost always involves the rectum, and is generally limited to the rectum and colon, although inflammation of the distal ileum, so-called "backwash ileitus", does occur. Ref: GI/ Harrison's 16, p. 1778/RAL
This inflammatory bowel disease involves the full-thickness of the bowel wall and adjacent mesentery.

A) Ulcerative colitis
B) Crohn's Disease
The correct answer is: B

While ulcerative colitis is primarily a disease of the colonic mucosa and submucosa, Crohn's disease involves all layers of the bowel wall. Ref: GI/ Schwartz 8, p. 1077/RAL
This inflammatory bowel disease has a tendency toward fistula and abscess formation.

A) Ulcerative colitis
B) Crohn's Disease
The correct answer is: B

Crohn's disease, due to its tendency toward transmural involvement, may be complicated by perforation, abscess formation, fistulization to bladder, vagina, or skin, and intestinal obstruction. Ref: GI/ Harrison's 16, p. 1780/ RAL
Skip areas of normal bowel may be interposed between areas of disease with this inflammatory bowel disease.

A) Ulcerative colitis
B) Crohn's Disease
The correct answer is: B

The area of involvement in ulcerative colitis is continuous, while in Crohn's disease there may be several discontinuous areas of disease. Ref: GI/ Schwartz 8, p. 1077/ RAL
Which agent or class of agents is not generally useful in the management of ulcerative colitis (UC) ?

A) Sulfasalazine (AzulfidinE)
B) Broad spectrum antibiotics
C) Glucocorticoids
D) Mesalamine (Asacol, PentasA)
E) Azathoiprine
The correct answer is: B

All responses represent agents effective against the inflammatory process of UC, except antibiotics, which generally have no role in treatment. Ref: GI/ Harrison's 16, p. 1784-1786/ RAL
In a patient at average risk for colon cancer, fecal occult blood testing (FOBT) should begin at:

A) age 35 and be repeated at 5-year intervals.
B) age 35 and be repeated annually.
C) age 50 and be repeated at 2-year intervals.
D) age 50 and be repeated annually.
E) age 65 and be repeated every 5 years.
The correct answer is: D

Annual FOBT, beginning at age 50, is one of several accepted screening strategies for colorectal cancer. Ref: GI/ Schwartz 8, p. 1090/ RAL
Which statement is true regarding Meckel's diverticulum?

A) Hemorrhage is usually related to acid-producing ectopic gastric mucosa.
B) It is present in about 10% of adults.
C) Treatment of obstruction due to Meckel's diverticulum involves a trial of IV steroids.
D) It is usually found on the mesenteric border of the jejunum.
E) The diagnosis is established by upper gastrointestinal contrast study with small bowel follow-through.
The correct answer is: A

Hemorrhage is usually related to ulceration of normal small bowel mucosa by acid from ectopic gastric mucosa lining the diverticulum. Meckel's diverticulum is present in about 2% of adults; it is found on the anti-mesenteric border of the distal ileum. The diagnostic procedure of choice is the technetium scan. The usual treatment for obstruction, inflammation, or hemorrhage, most commonly seen in children and adolescents, is surgical resection. Ref: GI/ Schwartz 8, p. 1043-1045/ RAL
Which is NOT characteristic of acute diverticulitis?

A) Left lower quadrant pain
B) Hematochezia
C) Rebound tenderness
D) Leukocytosis
E) Anorexia
The correct answer is: B

Although diverticulosis is the most common cause of hematochezia in individuals over 60 years of age, diverticuitis does not present with frank blood per rectum. Diverticula and diverticulitis are most common in the sigmoid colon. Peritoneal signs, such as rebound tenderness, are frequently noted in acute diverticulitis, as are anorexia and leukocytosis. Ref: GI/ Harrison's 16, p. 1796/ RAL
The best confirmatory study in suspected diverticulitis is which of the following?

A) Single-contrast barium enema
B) Double-contrast barium enema
C) CT scan of the abdomen
D) Rigid sigmoidoscopy
E) Fiberoptic colonoscopy
The correct answer is: C

Diverticular inflammation and abscess can be reliably (and non-invasively) detected by CT scan. Colonoscopy, sigmoidoscopy, and contrast enema with barium or water-soluble medium all risk possible worsening of peridiverticular abscess and conversion to generalized peritonitis. Ref: GI/ Schwartz 8, p. 1082; Harrison's 16, p. 1796/ RAL
Which of the following measures are included in the outpatient management of acute diverticulitis?

A) Adequate oral narcotics for analgesia
B) High-fiber diet
C) Broad-spectrum oral antibiotics for 7 to 10 days
D) Both A and C
E) All patients with suspected diverticulitis should be treated on an inpatient basis.
The correct answer is: C

Narcotics may mask worsening of the infection and should, therefore, be avoided. A clear liquid diet and broad-spectrum antibiotics are prescribed during the acute phase. Ref: GI/ Goroll 4, p. 495; Harrison's 16, p. 1796/ RAL
Which of the following is NOT a common characteristic of irritable bowel syndrome?

A) Diarrhea waking the patient from sleep at night
B) Alternating diarrhea and constipation
C) Abdominal distention
D) Crampy, episodic abdominal pain
E) Worsening of symptoms with emotional stress
The correct answer is: A

Diarrhea waking the patient from sleep is rare. The other choices are typical of irritable bowel syndrome. Ref: GI/ Harrison's 16, p. 1789-1791/ RAL
The most common cause of mechanical small bowel obstruction is:

A) Adhesions
B) Diverticulitis
C) Crohn's disease
D) Carcinoma
E) Intussusception
The correct answer is: A

About 75% of small bowel obstructions are due to adhesions related to prior surgical procedures. Ref: GI/ Schwartz 8, p. 1027/ RAL
Common historical findings in small bowel obstruction include which of the following?

A) Crampy abdominal pain
B) Bilious vomiting
C) Feculent vomiting
D) Both A and C
E) All of the above
The correct answer is: E

Crampy, intermittent abdominal pain and vomiting are typical. In high small bowel obstruction the character of the vomitus tends to be bilious; in more distal small bowel obstruction, it may become darker-colored and foul-smelling ("feculent") due to bacterial overgrowth in the obstructed bowel. Ref: GI/ Harrison's 16, p. 1803-1804/ RAL
Common physical findings in mechanical small bowel obstruction include which of the following?

A) Distended abdomen
B) High-pitched bowel sounds
C) Absent bowel sounds
D) Both A and B
E) All of the above
The correct answer is: E

Distention is common, although less prominent in high small bowel obstruction. Bowel sounds are hyperactive and high-pitched early and absent later in the course. Ref: GI/ Harrison's16, p. 1804/ RAL
Radiographic findings typical of complete mechanical small bowel obstruction include which of the following?

A) Air-fluid levels in the small bowel
B) Distended cecum
C) Minimal gas in the colon
D) Both A and C
E) All of the above
The correct answer is: D

Air-fluid levels with a "stepladder" appearance and absent gas in the large bowel (including the cecum) strongly suggest small bowel obstruction. Distention of the cecum is characteristic of large bowel obstruction. Ref: GI/ Harrison's 16, p. 1804/ RAL
A 65 year old woman with a history of previous abdominal surgery (cholecystectomy and appendectomy) in the remote past, presents to the Emergency Department at 4 P.M. with a 48-hour history of crampy, mid-abdominal pain, bilious vomiting, and abdominal distention. Initially she had a loose bowel movement, but has not passed stool or gas per rectum since early in the course of her present illness. On exam she has a distended, diffusely tender abdomen without peritoneal signs, and occasional high-pitched bowel sounds. You obtain supine and upright films of the abdomen and chest, which demonstrate air-fluid levels in the small bowel and nearly absent gas in the colon and rectum. Her blood pressure is 96/50; pulse is 118; temperature is 37.8ŒÁC; respirations are 24/min. The most appropriate course at this point is:

A) Admit; pass a Cantor tube (long tubE) ; begin TPN; and repeat abdominal films in morning
B) Admit; maintain NPO; start broad-spectrum antibiotics; and schedule barium enema for the next morning
C) Admit; pass a rectal tube; attempt to stimulate bowel with a mild cathartic; begin a trial of clear liquids in the morning
D) Admit; insert nasogastric tube and maintain continuous suction; begin vigorous fluid resuscitation and electrolyte replacement; plan for laparotomy in 6 to 8 hours
E) Admit; maintain complete bowel rest for 48 hours; obtain surgical consultation if pain and distention fail to resolve
The correct answer is: D

The patient has the classical clinical finding associated with small bowel obstruction. She should undergo a brief course (6 to 8 hours) of nasogastric suction and fluid resuscitation, followed by laparotomy for small bowel obstruction. Ref: GI/ Harrison's 16, p. 1805/ RAL
The most common site of intestinal volvulus is the:

A) third portion of the duodenum.
B) sigmoid colon.
C) transverse colon.
D) cecum.
E) mid-jejunum.
The correct answer is: B

90% of cases involve the sigmoid colon. Ref: GI/ Schwartz 8, p. 1097/ RAL
The most common cause of colon obstruction is which of the following?

A) Volvulus
B) Carcinoma
C) Diverticulitis
D) Crohn's colitis
E) Intussusception
The correct answer is: B

The most common cause of colon obstruction is carcinoma, followed by diverticulitis and volvulus. Ref: GI/ Harrison's 16. p. 1803/ RAL
The most likely site of perforation in a patient with an obstructing lesion in the sigmoid colon and a competent ileocecal valve is the:

A) cecum.
B) splenic flexure.
C) mid transverse colon.
D) hepatic flexure.
E) distal right colon.
The correct answer is: A

In a closed-loop obstruction of the colon (competent ileocecal valvE) the greatest tension on the wall of the colon occurs at the point of greatest radius, as described by the Law of Laplace. Thus, the most likely site of perforation or rupture is the area of greatest diameter, the cecum. Ref: GI/ Harrison's 16, p. 1803/ RAL
Which of the following is NOT a risk factor for the development of malignant gastric neoplasm?

A) Helicobacter pylori infection
B) Chronic alcohol abuse
C) High intake of smoked or pickled food
D) History of gastric ulcer
E) Chronic atrophic gastritis
The correct answer is: B

All are risk factors except alcohol abuse, which seems not to effect rates of gastric malignancy. Regular use of aspirin, diets high in fruits and vegetables, high intake of vitamins C and E, and a history of duodenal ulcer lower the risk of developing malignant gastric disease. Ref: GI/ Schwartz 8, p. 972-975/ RAL
Which statement regarding the Zollonger-Ellison (Z-E) syndrome is TRUE?

A) It is caused by an autonomous insulin-secreting tumor.
B) It is caused by a benign pancreatic lesion.
C) Work-up generally includes basal gastric acid secretion levels obtained by nasogastric aspiration.
D) Proton pump inhibitors (PPI's) are generally ineffective on controlling gastric acid secretion.
E) None of the above
The correct answer is: C

The Zollinger-Ellison syndrome is caused by an autonomous gastrin-secreting tumor. 60% are malignant at the time of diagnosis and a significant percentage are extrapancreatic. Work-up includes measurement of basal acid secretion via nasogastric aspiration, serum gastrin levels, and a variety of imaging techniques, including CT and MRI, endoscopic ultrasound, and nuclear imaging of gastrinomas ("octreoscan"). High-dose PPI's are effective in reducing gastric acid secretion. When possible, surgery to excise the gastrin-secreting tumor(s) is indicated. Ref: GI/ Harrison's p. 1758-1760/ RAL
Which statement regarding carcinoid syndrome is FALSE?

A) The most common symptoms are flushing and diarrhea.
B) Aortic stenosis is the most common cardiac manifestation.
C) Symptoms may be precipitated by the intake of alcohol.
D) Bronchospasm is the most common pulmonary manifestation.
E) Symptoms typically develop after the carcinoid tumor metastasizes to the liver.
The correct answer is: B

Fibrosis of the pulmonic and tricuspid valves are the most common structural lesion in the heart. In 80% of patients with cardiac involvement, heart failure eventually develops. Flushing and diarrhea, sometimes precipitated by stress, alcohol, and certain foods, are the most common clinical feature. In one study, 91% of patients with the carcinoid syndrome had liver metastases. Asthma-like symptoms may also occur. Ref: GI/ Harrison's 16, p. 2224-2225/ RAL
Which underlying conditions may be associated with constipation?

A) Hypocalcemia
B) Hypokalemia
C) Hypothyroidism
D) B and C
E) All of the above
The correct answer is: D

Hypercalcemia may also cause constipation. Ref: GI/ Goroll 4, p. 421/ RAL
A 75-year-old man presents with a history of progressive constipation over a six-month period. He denies melena or hematochezia, but admits an unintended 10-pound weight loss over that period. Abdominal and rectal exam are unremarkable. His stool is negative for occult blood. His hematocrit is 27% with microcytic, hypochromic indices. The best management of this patient entails which of the following?

A) A 4 week trial of a bulk-forming laxative
B) Screen for hypothyroidism and return for follow-up in 8 weeks.
C) Refer for colonoscopy.
D) Transfuse 2 units of packed red blood cells and start ferrous sulfate 325mg po tid.
E) Provide materials for outpatient testing of stool for occult blood and advise a more active lifestyle.
The correct answer is: C

Weight loss and anemia, as well as the relatively recent and progressive nature of the symptoms, suggest the possibility of colon cancer; work-up to exclude this possibility is indicated. Ref: GI/ Goroll 4, p. 422/ RAL
Internal hemorrhoids typically:

A) Are above the dentate line
B) May produce bright red blood per rectum
C) Are exquisitely painful
D) Both A and B
E) All of the above
The correct answer is: D

Internal hemorrhoids occur above the dentate line; may produce bleeding; and are usually painless. Ref: GI/ Schwartz 8, p. 1296/ RAL
Initial therapy for bleeding internal hemorrhoids includes all of the following measures except:

A) Increase dietary fiber intake
B) Elastic ligation
C) Stool softeners
D) Increase intake of water
E) Avoid straining to defecate
The correct answer is: B

Elastic ligation and other surgical approaches are usually reserved for failure of the more conservative measures represented by the other choices. Ref: GI/ Schwartz 8, p. 1101/ RAL
Which statement is FALSE regarding anal fissures?

A) 90% are found anteriorly, at the dentate line.
B) They typically present with pain and bleeding on defecation.
C) They are associated with spasm of the internal sphincter muscle.
D) Most acute fissures will heal with medical treatment.
E) The surgical treatment of choice is lateral internal anal sphincterotomy.
The correct answer is: A

About 90% of anal fissures are found posteriorly; 10% are anterior. More than 99% are in the midline. Ref: GI/ Schwartz 4 p. 1104/ RAL
Typical symptoms of chronic intestinal ischemia include:

A) Gnawing abdominal pain when the patient is hungry
B) Unintended weight gain despite moderate dietary intake
C) Chronic constipation
D) Postprandial abdominal pain
E) All of the above
The correct answer is: D

Abdominal pain after eating and unintended weight loss are the most common symptoms. Chronic diarrhea may be reported. Ref: GI/ Harrison's 16, p. 1799/ RAL
Which statement regarding acute intestinal ischemia is TRUE?

A) Elevated white blood cell count is an early finding.
B) Striking peritoneal signs are usually present early.
C) Emboli from the heart cause most acute arterial occlusions.
D) Intermittent abdominal pain of insidious onset is typical.
E) Mesenteric venous occlusions cause 50% of acute intestinal ischemia.
The correct answer is: C

Most cases are due to embolic occlusion of the superior mesenteric artery; cardiac sources of emboli are usual. Typically, pain is sudden in onset, severe, and continuous, but peritoneal signs are absent until late in the course, when infarcted bowel perforates. Leukocytosis is a late sign. Mesenteric venous thrombosis, usually involving the superior mesenteric vein, accounts for 10-15% of cases. Ref: GI/ Harrison's 16, p. 1798; Schwartz 8, p. 1047/ RAL
Which of the following interventions would generally NOT be appropriate in treating a patient with nonocclusive mesenteric ischemia?

A) Broad-spectrum antibiotics
B) Fluid resuscitation
C) Intra-arterial papavarine
D) Central intravenous norepinephrine
E) All of the above may be appropriate
The correct answer is: D

Vigorous fluid resuscitation, antibiotics to cover bowel flora, and vasodilating agents, such as papaverine, directly infused via cannulation of the superior mesenteric artery, may be helpful. Vasopressors, such as norepinephrine, given by any route, would likely make mesenteric vasospasm worse, and would be contraindicated. Ref: GI/ Harrison's 16, p. 1799; Schwartz 8, p. 1049/ RAL
A 30-year-old male presenting with complaints of pain and swelling in the postsacral area with intermittent purulent drainage most likely has which of the following?

A) Chancroid
B) Pilonidal abscess
C) Perianal abscess
D) Supralevator abscess
E) Herpes proctitis
The correct answer is: B

Pilonidal abscesses typically occur in the postsacral area about 5cm from the anus. Perianal abscess usually appears as a tender, inflamed mass at the anal verge.
Which statement is true concerning the guaiac test for occult blood?

A) Recent ingestion of rare red meat and some vegetables may cause false positive results.
B) Sigmoidoscopy provides adequate follow-up for patients with a positive test.
C) A single specimen annually provides adequate screening for colorectal cancer.
D) No benefit, in terms of mortality, has ever been shown for fecal occult blood testing programs.
E) All of the above
The correct answer is: A

Vegetables that may cause a false positive test include horseradish, broccoli, turnips and cauliflower. Patients with a positive test require colonoscopy or sigmoidoscopy plus air-contrast barium enema. Due to the intermittent nature of bleeding from colorectal tumors, fecal occult blood testing (FOBT) should be performed twice a day for three days. A large study has shown some benefit in terms of mortality. Ref: GI/Goroll, p.360-361 RAL
An adenocarcinoma of the colon involving all layers of the bowel wall, but confined to pericolic fat, without spread to regional lymph nodes or distant metastases, would be classified as:

A) Stage I
B) Stage II
C) Stage III
D) Stage IV
E) Cannot be classified with the information given
The correct answer is: B

Tumors spreading to the pericolic fat, without nodal involvement or distant metastases, are classified in the TNM system as T3, N0, M0, or Stage II. The 5-year survival for disease at this stage is 54-65%. Ref: GI/ Schwartz 8, p. 1090-1091/ RAL
The physiologic function of the appendix is:

A) Secretion of immunoglobulins
B) Serves as a reservoir for mucosal secretions
C) Absorbs water from colon contents
D) Contains baroreceptors regulating colonic peristalsis
E) It has no known function.
The correct answer is: A

The appendix is now known to secrete immunoglobulins, although appendectomy does not appear to be associated with any clinically significant compromise of the immune system. Ref: GI/ Schwartz 8, p. 1119/ RAL
Which is most likely to be the first symptom of acute appendicitis?

A) Anorexia
B) Change in bowel habits
C) Vomiting
D) Diffuse abdominal pain
E) Localized abdominal pain
The correct answer is: A

95% of patients with acute appendicitis report that anorexia was their first symptom. Ref: GI/ Schwartz 8, p. 1122/ RAL
Rovsing's sign, seen in acute appendicitis, consists of:

A) pain on the passive internal rotation of the right thigh.
B) pain on the passive extension of the right thigh.
C) localized pain on palpation of the abdomen at a point one third the distance between the anterior iliac spine and the umbilicus.
D) pain in the right lower quadrant on palpation of the left lower quadrant.
E) None of the above
The correct answer is: D

Rovsing's sign, indirect tenderness, possibly due to distention of the right colon with compression of the left colon, is classically described in acute appendicitis. Direct tenderness at McBurney's point, as described in Answer C, results from local peritoneal irritation. The obturator sign and the psoas sign, represented by Answers A and B respectively, are useful, particularly in the case of a retrocecal appendix. Ref: GI/ Schwartz 8, p. 1122/ RAL
In patients undergoing a surgical procedure with a preoperative diagnosis of acute appendicitis, what percent have a postoperative diagnosis of acute appendicitis?

A) 99%
B) 95%
C) 90%
D) 85%
E) 80%
The correct answer is: D

The preoperative diagnosis is correct about 85% of the time. This reflects the optimum threshold for surgical intervention given the limitations of diagnostic methods available and the consequences of both delayed and unnecessary operation. Ref: GI/ Schwartz 8, p. 1126/ RAL
Appropriate treatment for a 55-year-old woman incidentally noted to have asymptomatic gallstones on abdominal ultrasound consists of which of the following?

A) Open cholecystectomy should be scheduled electively.
B) Laparoscopic cholecystectomy should be scheduled within 6 months.
C) Follow-up ultrasound should be obtained in one year.
D) Cholesterol-lowering therapy should be initiated.
E) None of the above
The correct answer is: E

In general, no treatment or follow-up is required. Ref: GI/ Schwartz 8, p. 1194-1195/ RAL
A 45-year-old woman presents with a history of right upper quadrant pain and protracted vomiting following large meals consisting mainly of fried food approximately 8 hours previously. The pain is radiating to her back in the region of the right scapula. On examination, she has rebound tenderness at the right costal margin. Her WBC is 16,000/mm3 and temperature is 38.3ŒÁC. Despite a normal supine and upright film of the abdomen, you strongly suspect acute cholecystitis. The most appropriate diagnostic study at this point is which of the following?

A) CT scan of the abdomen
B) Right upper quadrant ultrasound
C) Oral cholecystogram
D) Endoscopic Retrograde Cholangiopancreatography (ERCP) E) Cross-table lateral film of the abdomen
The correct answer is: B

Ultrasound is highly accurate in the detection of gallstones, as well as in noting pathologic changes in the gall bladder wall and ductal dilatation. It is more cost effective and sensitive than CT scanning, and has largely replaced oral cholecystograms. The patient here, in any case, would be unlikely to absorb oral contrast due to severe vomiting. ERCP is invasive and inappropriate at this point. Additional films of the abdomen would not likely be helpful. HIDA scan, a nuclear medicine procedure, though not offered as a choice here, is also an accurate means of establishing or excluding the diagnosis of acute cholecystitis. Ref: GI/ Schwartz 8, p. 1199/ RAL
A 50-year-old diabetic woman presents with a 6-month history of intermittent right upper quadrant pain radiating to the intrascapular region, usually precipitated by a fatty meal. She is asymptomatic at present, afebrile, and has a normal WBC. Serum total bilirubin and amylase are normal. You obtain a right upper quadrant ultrasound, which confirms the presence of gallstones. Appropriate treatment, at this point, consists of which of the following?

A) Referral for laparoscopic cholecystectomy in the near future
B) Counsel the patient to maintain a low-fat diet and return if symptoms persist.
C) Obtain follow-up ultrasound in one year.
D) Admit patient, start broad-spectrum IV antibiotics, and refer for cholecystectomy within 48 hours.
E) Obtain HIDA scan to confirm diagnosis and refer for surgery if positive.
The correct answer is: A

The treatment of choice for chronic cholecystitis is laparoscopic cholecystectomy. Early operation is particularly important in diabetics, who have a higher incidence of complications of cholecystitis, although admission and emergency operation is not appropriate. No further confirmatory studies would generally be required at this point. Ref: GI/ Schwartz 8, p. 1199 / RAL
The most common cause of acute pancreatitis in the United States is:

A) Gallstones
B) Alcoholism
C) Endoscopic retrograde cholangiopancreatography (ERCP) D) Hypertriglyceridemia
E) Drug toxicity
The correct answer is: A

Gallstones cause 30-60% of acute pancreatitis. Alcoholism, ERCP, hypertriglyceridemia, and drug side effects are less common causes. Ref: GI/ Harrison's 16, p. 1896/ RAL
Typical clinical, laboratory and radiologic findings seen in acute pancreatitis, or in an acute exacerbation of chronic pancreatitis, include all of the following except:

A) Serum amylase greater than 3 times normal
B) "Sentinel loop" or "colon cutoff sign" on plain abdominal film
C) Vomiting and epigastric pain radiating to the back
D) Hyperactive bowel sounds
E) Left pleural effusion
The correct answer is: D

Bowel sounds are usually diminished or absent due to ileus. Local irritation of adjacent bowel may produce either a single dilated loop of colon or small bowel or spasm in the adjacent transverse colon producing a characteristic "cutoff" in the large bowel gas pattern. There may be a "sympathetic" left pleural effusion. Epigastric pain with radiation to the back is usual. Ref: GI/ Harrison's 16, p. 1893, 1897/ RAL
Medical management of uncomplicated, mild pancreatitis includes all of the following except:

A) NPO until abdominal pain and ileus have resolved
B) Broad spectrum antibiotics
C) Aggressive fluid resuscitation
D) Parenteral pain medication
E) Nasogastric suction
The correct answer is: B

Broad spectrum antibiotics do not improve the outcome in uncomplicated pancreatitis. Complete bowel rest (NPO, nasogastric suction) should be maintained until symptoms have resolved. Third-space fluid loss may be severe and should be rapidly replaced. Parenteral pain medication is indicated, but morphine should be avoided, as it may cause spasm of the sphincter of Oddi. Ref: GI/ Harrison's 16, p. 1899; Schwartz 8, p. 1238/ RAL
Which of the following is the best treatment for a patient with gallstone panceatitis, who fails to pass the obstructing stone after 24 hours of observation?

A) Continue supportive medical management
B) Urgent cholecystectomy
C) Endoscopic retrograde cholangiopancreatography (ERCP) with endoscopic sphincterotomy and stone extraction
D) Surgical T-tube placement to drain the common bile duct
E) Pancreaticojejunostomy (Puestow procedurE) to drain the common pancreatic duct
The correct answer is: C

Although most gallstones will pass into the small bowel in a matter of hours, ERCP and stone extraction is indicated if the obstructing stone fails to pass after 24 hours of observation and support. Ref: GI/ Schwartz 8, p. 1240/ RAL
Which statement(s) is (arE) true concerning carcinoma of the pancreas?

A) Most patients present with jaundice, weight loss, and abdominal pain.
B) Abdominal CT scan is the diagnostic procedure of choice when pancreatic carcinoma is suspected.
C) 85-90% of patients will have unresectable disease at presentation.
D) Both A and B
E) All of the above
The correct answer is: E

Most patients present with abdominal pain, jaundice, and weight loss. CT scan is the best initial diagnostic procedure. The tendency of the disease to present at an advanced stage results in a grim prognosis, with the majority of patients unable to survive for one year from the time of diagnosis. Ref: GI/ Schwartz 8, p. 1279-1281/ RAL
Which statement is true concerning anorexia nervosa?

A) The disease has a fatal outcome in about 5% of patients.
B) Most female patients report dysmenorrhea.
C) Most patients appear listless and complain of somnolence.
D) Recovery to normal weight is rare.
E) None of the above
The correct answer is: A

Amenorrhea is usual. Patients are usually active and complain of insomnia. About 75% of patients recover. Ref: GI/ Goroll 4, p. 1206/ RAL
Medical consequences of bulimia nervosa include all of the following except:

A) Hypochloremic metabolic alkalosis
B) Hypokalemia
C) Decalcification and erosion of dental enamel
D) Upper gastrointestinal hemorrhage
E) Cardiac arrhythmias
The correct answer is: D

Metabolic derangements, as listed in Answers A and B are common, sometimes resulting in cardiac arrhythmias and sudden death. Chronic exposure of the teeth to acidic stomach contents results in dental disease. Upper GI hemorrhage is uncommon. Ref: GI/ Goroll 4, p. 1207 RAL
What is the approximate number of kcal per day needed to maintain the weight of a moderately active female weighing 280 pounds?

A) 1400 kcal
B) 1800 kcal
C) 2200 kcal
D) 2800 kcal
E) 3600 kcal
The correct answer is: D

The number of kcal required to maintain weight is estimated by multiplying present weight by 10 (for active women), or by 8 (for sedentary women). The multipliers for active and sedentary obese men are 12 and 10 respectively. A daily caloric intake of 500 kcal less than the maintenance requirement will result in about a one-pound weight loss per week. Ref: GI/ Goroll 4, 1194/ RAL
Benefits of combining an exercise program with dietary modification in a weight-loss program include:

A) Preservation of lean body mass
B) Increase rate of weight loss and loss of body fat
C) Increase psychological well-being of patient by increasing feelings of self control and stress reduction
D) Both A and C
E) All of the above
The correct answer is: E

In addition to the benefits listed, regular exercise may improve the cardiovascular status of the patient. Ref: GI/ Goroll 4 , p. 1195/ RAL
The target heart rate to be achieved during exercise by an obese, sedentary 40-year-old man embarking on a weight-loss program is about:

A) 160/minute
B) 140/minute
C) 120/minute
D) 100/minute
E) 80/minute
The correct answer is: D

The target heart rate is calculated as .6 to .8 times the predicted maximum heart rate for the individual patient. Sedentary patients should begin at the lower end of the scale. The predicted maximum heart rate is calculated as 200 minus the patients age.
Which statement(s) is (arE) true concerning very low calorie diets (400 to 800 kcal per day)?

A) They are indicated for insulin-dependent diabetic patients who are 20% over their ideal body weight.
B) They are based on a high complex carbohydrate diet.
C) They may be safely undertaken by otherwise healthy children who are at least 40% over their ideal body weight.
D) Patients require an initial cardiac evaluation.
E) All of the above
The correct answer is: D

Very low calorie diets (VLCD's) are contraindicated in insulin-dependent diabetics, children, the elderly, and in patients with severe renal or hepatic disease. They are based on a high-protein diet. They are indicated for patients who are at least 30-40% over their ideal body weight. They must be undertaken after a cardiac evaluation and under close medical supervision. Ref: GI/ Goroll 4, p. 1202/ RAL
Which statement is true concerning the surgical treatment of morbid obesity?

A) It should be offered only to patients who are either 100 pounds overweight or greater than 200% of their ideal body weight.
B) It should be considered only after maximum medical therapy has failed.
C) Jejunoileal bypass is the procedure of choice.
D) Both A and B
E) All of the above
The correct answer is: D

Jejunoileal bypass has been largely abandoned due to unacceptable complications. Gastric stapling or gastric bypass are the procedures generally offered. They should be restricted to massively overweight patients who have failed other therapies. Ref: GI/ Goroll 4, p. 1202/ RAL
A hernia is said to be strangulated when:

A) it does not spontaneously reduce with the patient in the supine position.
B) it cannot be reduced with gentle pressure by the examiner on the hernia sac.
C) there is compromise of the blood supply to the portion of abdominal contents within the hernia.
D) there is necrosis or ulceration of the overlying skin.
E) the patient presents in septic shock.
The correct answer is: C

A hernia that cannot be reduced by the examiner is said to be incarcerated. A strangulated hernia generally contains a loop of bowel whose circulation is compromised, usually by compression at the neck of the hernia. There is the potential for necrosis of the bowel and sepsis. Prevention of this complication is the most important reason for elective surgical repair. Patients with strangulated hernias require emergency surgical repair. Ref: GI/ Schwartz 8, p. 1320-1321, 1356/RAL
Which statement is true concerning umbilical hernias?

A) They are always congenital.
B) Adults with asymptomatic umbilical hernias should be referred for surgical repair.
C) Congenital umbilical hernias usually close spontaneously.
D) Both A and C
E) All of the above
The correct answer is: C

Umbilical hernias may be congenital or acquired. When congenital, they usually close spontaneously by the age of 5 years. Small asymptomatic umbilical hernias in adults generally do not require treatment. Ref: GI/ Schwartz 8, p.1321/ RAL
Which statement about the physical examination of hernias is true?

A) Femoral hernias are generally palpable above the inguinal ligament.
B) Patients with inguinal hernia are generally examined in the right or left lateral decubitus position (depending on the location of the suspected herniA) .
C) To detect ventral hernias, patients should be asked to fully abduct their arms.
D) A hernia protruding into the scrotum is likely an indirect inguinal hernia.
E) The examiner should never attempt to reduce a hernia.
The correct answer is: D

Femoral hernias are palpable below the inguinal ligament. Patients should be examined standing and supine. Ventral hernias are detected by asking the supine patient to raise his head. Protrusion into the scrotum generally indicates an indirect inguinal hernia. A gentle attempt to reduce the hernia is a normal part of the exam. Ref: GI/ Goroll 4, p. 432-433/ RAL
Which statement about femoral hernias is true?

A) They are always palpated below the inguinal ligament.
B) They have the highest rate of strangulation of all hernias.
C) They require prompt surgical repair.
D) Both B and C
E) All of the above
The correct answer is: E

Femoral hernias are the most likely to cause strangulation and should be repaired promptly. They appear inferior to the inguinal ligament, passing through the femoral canal. They are more common in women and, like inguinal hernias, are more common on the right than on the left side. Ref: GI/ Goroll 4, p. 433; Schwartz 8, p.1357/ RAL
A 72-year-old diabetic, hypertensive man with a history of an uncomplicated myocardial infarction 3 months previously presents with a 24-hour history of a progressively painful mass in the right groin. He is known to have had a right inguinal hernia for several years, but has declined elective repair. He has vomited several times in the previous 12 hours. On examination, the inguinal mass is exquisitely tender with erythema of the overlying skin. The patient is tachycardic and has an oral temperature of 39ŒÁC; his blood pressure is 98/60. The best immediate care of this patient would include:

A) Attempted reduction of the mass by firm digital pressure
B) Fluid resuscitation and broad spectrum IV antibiotics
C) Emergency surgical exploration
D) Both B and C
E) All of the above
The correct answer is: D

The picture of a strangulated inguinal hernia with sepsis contraindicates attempted manual reduction (sometimes called "taxis"), for fear of returning gangrenous bowel into the abdominal cavity. The patient requires fluid, antibiotics, and emergency exploration. Ref: GI/ Schwartz 8, p. 1355-1356/ RAL
Which statement is true concerning hepatitis A?

A) The incubation period is about 30 days.
B) The incidence of hepatitis A in the United States has been increasing since the 1970's.
C) 40% of patients remain in a carrier state following acute infection.
D) It is usually transmitted by a percutaneous route.
E) Virtually all infections become symptomatic within 45 days.
The correct answer is: A

The incubation period varies from 15-45 days, with a mean of 30 days. The incidence of infection in the United States has been declining since the 1970's. There is no carrier state. The mode of transmission is the fecal-oral route, frequently related to contaminated water, food, poor hygiene, and crowded institutional conditions. Many, if not most infections detectable by laboratory means, never become symptomatic. Ref: GI/ Harrison's 16 , p. 1829-1831/ RAL
Which statement is true concerning the serum markers of hepatitis B infection?

A) The first serum marker detectable is hepatitis B surface antigen (HBsAg).
B) HBsAg is detectable in serum indefinitely following hepatitis B infection.
C) The antibody to hepatitis B core antigen (anti-HBcAg) may be present in serum for years following clinical infection.
D) Both A and C
E) All of the above
The correct answer is: D

HBsAg is the first detectable serum marker; it appears at about 4 weeks after infection and becomes undetectable at about 24 weeks. Anti-HBcAg appears in serum at about 6 weeks and may persist for years; isolated anti-HBcAg (in the absence of other serum markers) probably indicates hepatitis B infection in the remote past. Ref: GI/ Harrison's 16, p. 1823/ RAL
In general, the care of patients with acute hepatitis A includes:

A) Glucocorticoids
B) Isolation
C) Strict enteric precautions
D) Hospitalization until clinical improvement is evident
E) Avoidance of medications metabolized in the liver
The correct answer is: E

Glucocorticoids have not been shown to be useful. Isolation and special enteric precautions are not necessary. Hospitalization and complete bed rest are generally not required. Medications metabolized in the liver should be avoided. Ref: GI/ Harrison's 16, p. 1835/ RAL
Patients at high risk for hepatitis B include all of the following except:

A) Health care workers exposed to blood
B) Developmentally handicapped patients at custodial institutions
C) Intravenous drug users
D) Prison inmates
E) All of the above
The correct answer is: E

At-risk individuals also include household contacts of HBsAg carriers, sexual contacts, and patients requiring ongoing treatment with blood products. Ref: GI/ Harrison's 16, p. 1836-1837/ RAL
For a healthcare worker who sustains a needlestick injury contaminated by secretions from an HBsAg-positive patient, prophylaxis should include:

A) A single dose of hepatitis B immune globulin
B) Three doses of hepatitis B vaccine
C) A 3-week course of lamivudine
D) Both A and B
E) All of the above
The correct answer is: D

Immune globulin and vaccine only are indicated for prophylaxis. Ref: GI/ Harrison's 16/ p. 1837/ RAL
The most common malignant tumor of the liver is:

A) Hepatocellular carcinoma
B) Metastatic disease from other primary tumors
C) Cholangiocarcinoma
D) Hepatoblastoma
E) Hemangioma
The correct answer is: A

Hepatocellular carcinoma is the most common primary tumor of the liver. Cholangiocarcinoma and hepatoblastoma are less common. Hemangiomas are the most common benign tumors. Ref: GI/Harrison's 16, p. 533-534 / RAL
Which of the conditions listed does NOT cause cirrhosis?

A) Right-sided heart failure
B) Chronic alcoholism
C) Hepatitis A
D) Hepatitis B
E) Hepatitis C
The correct answer is: C

Chronic passive congestion of the liver due to right-sided heart failure may cause cirrhosis. Chronic alcoholism is the most common cause of cirrhosis in the United States. Cirrhosis may result from previous hepatitis B or C infection, but not from hepatitis A. Ref: GI/ Harrison's 16, p. 1858-1863/ RAL
Typical findings in alcoholic cirrhosis include all of the following except:

A) A history of excessive alcohol intake for ten or more years
B) Obesity
C) Bleeding due to esophageal varices
D) Encephalopathy
E) Ascites
The correct answer is: B

Patients usually present with anorexia and loss of skeletal muscle. Cirrhosis generally develops after ten or more years of chronic alcohol abuse. Ascites, gastroesophageal varices, and encephalopathy are common late findings. Ref: GI/ Harrison's 16, p. 1858-1859/ RAL
Which is a risk factor for carcinoma of the pancreas?

A) Smoking
B) Drinking more than two cups of coffee daily
C) Consuming more than two alcoholic drinks daily
D) A and C
E) None of the above
The correct answer is: A

Smoking doubles the risk of developing pancreatic carcinoma. Coffee and alcohol are not risk factors. Ref: GI/ Schwartz 8, p. 1277/ RAL
Which is the earliest radiographic finding in hematogenous osteomyelitis?

A) Periosteal reaction
B) Lytic changes
C) Increased bone density at the site of infection
D) Gas in the soft tissue surrounding the site of infection
E) No radiographic changes are visible on plain radiographs
The correct answer is: A

The earliest radiographic sign is periosteal reaction at the site of infection, which may be seen within 10 days. Lytic changes are evident in 4 to 6 weeks. Ref: MS/ Harrison's 16, p. 745-746/ PS-RAL
Which is the most common pathogen in hematogenous osteomyelitis?

A) Escherichia coli
B) Candida species
C) Group B streptococci
D) Pseudomonas aeruginosa
E) Staphylococcus aureus
The correct answer is: E

S. aureus is cultured in about 50% of cases. Ref: MS/ Harrison's 16, p. 746/ PS-RAL
A 35-year-old African American woman presents with a complaint of symmetric swelling and pain of her wrists. In addition to soft tissue swelling and tenderness of the involved joints, you note a malar rash and consider the diagnosis of systemic lupus erythematosis (SLE) . What is the best single laboratory test to rule out or confirm this diagnosis?

A) CBC with differential
B) Antinuclear antibodies (ANA) C) Antiplatelet antibodies
D) Antineuronal antibodies
E) Antiphosphoplipid antibodies
The correct answer is: B

ANA is the best screening test and will be positive in 98% of patients with SLE. A repeated negative test makes the diagnosis of SLE highly unlikely. Ref: MS/ Harrison's 16, p. 1960-1962; Goroll 4, p. 827-828/ RAL
In spina bifida, which of the following represents a fluid filled sac, caused by the protrusion of the meninges through the defective neural arch of a vertebra?

A) Meningocele
B) Myelomeningocele
C) Pilonidal cyst
D) Sacrococcygeal sinus
E) Spina bifida occulta
The correct answer is: A

Failure in fusion of the neural arch of a vertebra is spina bifida. In spina bifida occulta, there is no protrusion of the meninges. When the meninges form a sac protruding through the defective arch, a meningocele is said to occur. The sac is covered by intact skin and is filled with spinal fluid. When the sac contains spinal cord or cauda equina, it is termed a myelomeningocele. Occasionally, a sinus tract, termed a sacrococcygeal sinus, leads from a spina bifida occulta to the skin of the sacral region, occasionally mistaken for a pilonidal cyst. Ref: MS/ Harrison's 16, p. 96/ PS-RAL
Which of the following crystals cause acute gouty arthritis?

A) Monosodium urate (MSU)
B) Calcium pyrophosphate dihydrate (CPPD)
C) Calcium hydroxyapetite (HA)
D) Calcium oxalate (CaOx)
E) None of the above
The correct answer is: A

Monosodium urate, seen in gouty effusions, appear as long, needle-shaped, negatively birefringent, usually intracellular crystals. Calcium pyrophosphate dihydrate is usually appears as short, rhomboid-shaped, positive birefringent crystals, and is the agent causing pseudogout. Deposition of HA and CaOx in and around joints may also cause clinical disease. Ref: MS/ Harrison's 16, p. 2046-2050, PS-RAL
Localized swelling on the dorsum of the wrist associated with myxomatous degeneration of the joint capsule suggests:

A) Ganglion
B) Sebaceous cyst
C) Synovitis
D) Lipoma
E) Neuroma
The correct answer is: A

A ganglion cyst represents myxomatous degeneration of the capsule of the wrist joint, most frequently occurring on the dorsum of the naviculolunate joint. They may also be found on the volar surface of the wrist. Most require no treatment. Ref: MS/Goroll 4, p. 866/ PS-RAL
Which represents a condition in which the skeletal muscle is damaged by a nonsuppurative inflammatory process, often associated with a skin rash of the eyelids, nose, and cheeks?

A) Lupus
B) Myositis
C) Rheumatoid arthritis
D) Scleroderma
E) Malignancy
The correct answer is: B

Dermatomyositis and polymyositis are conditions of unknown etiology in which the skeletal muscle is damaged by a nonsuppurative inflammatory process dominated by lymphocytic infiltration. The term Dermatomyositis is used when a rash accompanies the muscle damage. The rash may typically involve the eyelids, bridge of the nose, and cheeks. Ref: MS/ Harrison's 16, p. 315-316/ PS-RAL
Muscle pain in the absence of weakness, as often occurs in viral illnesses, particularly influenza, is best described as:

A) Myositis
B) Myalgia
C) Myasthenia
D) Myotonia
E) Tetany
The correct answer is: B

Whether related to an acute viral illness, polyalgia rheumatica, fibromyalgia, or some other cause, myalgia refers specifically to muscle pain. Myositis and myasthenia are accompanied by weakness. Tetany and myotonia refer to involuntary contraction of muscles. Ref: MS/ Harrison's 16, p. 138/ RAL
Asymmetric polyarticular synovitis involving any joint, especially the distal IP joints, spine, and sacroiliac joints, combined with a chronic inflammatory skin disorder and characteristic nail changes, describes which of the following disorders?

A) Rheumatoid arthritis
B) Ankylosing spondylitis
C) Osteoarthritis
D) Psoriatic Arthritis
E) Systemic Lupus Erythematosus
The correct answer is: D

Rheumatoid arthritis tends to be symmetrical and frequently effects the proximal, not distal IP's. Though synovitis is possible in osteoarthritis, it is generally a late finding, and skin changes include the formation of Bouchard's and Heberden's nodes. Patients with SLE may develop arthritis, but it is generally symmetric and most frequently effects the proximal IP joints. Asymmetric polyarticular synovitis involving any joint, especially the distal IP joints, combined with the chronic inflammatory skin disorder psoriasis, suggests psoriatic arthritis. Ref: MS/ Harrison's 16, p. 1998/ PS-RAL
What is the treatment of suppurative tenosynovitis?

A) Moist heat
B) Physical therapy
C) Incision and drainage
D) Glucocorticoids
E) NSAID's
The correct answer is: C

Suppurative tenosynovitis, usually a complication of a puncture wound, is a very serious hand infection in which early diagnosis and treatment are mandatory to prevent permanent disability from destruction of the tendon or its sheath. The symptoms are generalized swelling of the digit, exquisite tenderness over the flexor tendon sheath, flexion of the fingers, and excruciating pain with passive extension of the digit. Immediate incision of the sheath is indicated, to prevent not only tendon damage but also proximal extension of the process into the hand or forearm. Intravenous antibiotic treatment should accompany surgery. Ref: MS/ Schwartz 8, p. 1762/ PS-RAL
Gamekeeper's thumb refers to which of the following?

A) Disruption of the radial collateral ligament of the thumb
B) Disruption of the ulnar collateral ligament of the thumb
C) Acute ischemia due to digital artery embolization
D) Open dislocation of the first interphalangeal joint
E) None of the above
The correct answer is: B

Gamekeeper's thumb refers to disruption of the ulnar collateral ligament , usually caused by jamming the thumb. Ref: MS/ Schwartz 8, p. 1740/ RAL
A 70-year-old woman, with no history of trauma, complains of right shoulder pain, particularly during overhead activities and with internal rotation of the joint. You note pain over the deltoid region and weakness of shoulder elevation and of external rotation. At this point, the most likely diagnosis is:

A) Reflex sympathetic dystrophy
B) Degenerative disease of the acromioclavicular joint
C) Thoracic outlet syndrome
D) Rotator cuff tear
E) Glenohumeral joint dislocation
The correct answer is: D

The signs and symptoms given are typical of rotator cuff tear, the most common cause of shoulder pain among older patients. Ref: MS/ Gorrol 4, p. 850/ RAL
De Quervain's tenosynovitis is exacerbated by:

A) overhead extension of the arms.
B) extension of the neck.
C) the use of the thumb.
D) repetitive upper body exercises.
E) exposure to cold weather.
The correct answer is: C

De Quervain's tenosynovitis is a nonspecific inflammation of the abductor pollicis longus and the extensor pollicis brevis. It is exacerbated by the use of the thumb. Ref: MS/ Goroll 4, p. 866/ RAL
A 25-year old runner complains of a sudden inversion injury to the right ankle two hours previously. He complains of pain, swelling, and discoloration of the ankle. X-rays show no evidence of fracture. On examination you note a positive anterior drawer sign and obtain stress X-rays, which reveal an unstable joint. The most likely diagnosis is which of the following?

A) First degree sprain of the ankle
B) Second degree sprain of the ankle
C) Third degree sprain of the ankle
D) An occult bimalleolar fracture
E) Cannot be determined from the information given
The correct answer is: C

Complete ligamentous disruption, or third degree ankle sprain, is likely with instability of the joint, as noted in this scenario. Ref: MS/ Goroll 4, p. 871-873/ RAL
A 25-year old runner complains of a sudden inversion injury to the right ankle two hours previously. He complains of pain, swelling, and discoloration of the ankle. X-rays show no evidence of fracture. On examination you note a positive anterior drawer sign and obtain stress X-rays, which reveal an unstable joint. What treatment plan is appropriate at this point?

A) Wrap the ankle with an elastic bandage and schedule follow-up in 2 weeks.
B) Immobilization the ankle in a cast for 4 to 8 weeks.
C) Tape the ankle to prevent inversion and teach the patient partial weight-bearing with crutches.
D) Wrap the ankle with an elastic bandage and begin physical therapy when the swelling subsides.
E) Apply ice packs for 24 hours, wrap the ankle in an elastic bandage, and resume running in 2 to 4 weeks.
The correct answer is: B

A third degree ankle sprain, with marked joint instability, requires cast immobilization. Ref: MS/ Goroll 4, p. 871-873/RAL
In the straight leg raising (SLR) test:

A) the patient should be placed in the lateral decubitus position.
B) a positive test indicates lumbar disk disease.
C) the knee is flexed 30 degrees.
D) the leg is actively raised by the patient with the knee extended.
E) the leg must be raised beyond 90 degrees.
The correct answer is: B

SLR is a test for lumbar disk disease. The leg is passively moved between 30 and 70 degrees, in the supine position, with the knee straight. Ref: MS/ Goroll 4, p. 838/ RAL
Which symptom is a common feature of cauda equina syndrome?

A) Urinary retention
B) Loss of anal sphincter tone
C) Loss of sensation over the perineum
D) Bilateral lower extremity sensory deficits
E) All of the above
The correct answer is: E

The spinal cord ends at the L1 level. Compression of the nerve roots the extend caudad beyond that level (the cauda equinA) , as by a herniated disk, may cause any of the findings listed. Ref: MS/ Gorell 4, p. 837/ RAL
What is the most common cause of sciatica?

A) Structural scoliosis
B) Herniation of a lumbar disk
C) Osteoarthritis of the facet joints
D) Vertebral compression fracture
E) Metastatic carcinoma
The correct answer is: B

95% of sciatica is caused by lumbar disk disease.
Which is a common characteristic of vertebral compression fractures?

A) Local pain begins gradually.
B) They are most likely to occur in the lumbar spine.
C) Pain radiates to the lower extremities, usually bilaterally.
D) They are most common in the cervical spine.
E) None of the above
The correct answer is: E

Compression fractures are most common in elderly patients with osteoporosis, metastatic neoplasms, or who are receiving long-term treatment with steroids. The onset of pain is usually abrupt and generally does not radiate to the lower extremities. The thoracic spine is the most common location. Ref: MS/ Goroll 4, p. 836/ RAL
The term for anterior slippage of a vertebral body, frequently causing low back pain, is:

A) Spondylosis
B) Spondylolisthesis
C) Spinal stenosis
D) Sprain
E) Strain
The correct answer is: B

Spondylolithesis is anterior movement of the vertebral body, most often L4, and may be asymptomatic or cause low back pain. Ref: MS/ Harrison's 16, p. 96/ RAL
Heberden's and Bouchard's nodes are associated with what chronic condition?

A) Rheumatoid Arthritis (RA) B) Osteoarthritis (OA) C) Systemic lupus erythrmatosis (SLE) D) Fibromyalgia
E) Sarcoidosis
The correct answer is: B

Heberden's nodes, bony enlargement of the DIP, and Bouchard's nodes, enlargement of the PIP, are associated with osteoarthritis. Ref: MS/ Harrison's 16, p. 2040/PS-RAL
The main feature of which joint disease is loss of articular cartilage?

A) Gout
B) Rheumatoid Arthritis (RA) C) Osteoarthritis (OA) D) Bursitis
E) Osteomyelitis
The correct answer is: C

In addition to loss of cartilage from weight-bearing surfaces, osteoarthritis is characterized by hypertrophy of bone, thickening of the joint capsule, and muscle wasting. Ref: MS/ Harrison's 16/ p. 2038/ RAL
Which is the most common organism causing septic arthritis in adults?

A) Neisseria gonorrhoeae
B) Staphylococcus aureus
C) Borrelia burgdorferi
D) Streptococcus pneumoniae
E) Treponema pallidum
The correct answer is: A

N. gonorrhoeae is the most common organism causing septic arthritis in adults, usually following a recognized or unrecognized disseminated gonococcal infection. S. aureus causes most nongonococcal joint infections. B. burgdorferi joint infections are a common complication of Lyme disease. Pneumococcal arthritis is often seen in immunocompromised hosts, and syphilitic arthritis occur at different stages of the disease. Ref: MS/ Harrison's 16, p. 2050-2053/ RAL
A fracture of the neck of the fifth metacarpal is a commonly called:

A) Green stick fracture
B) Comminuted fracture
C) Boxer's fracture
D) Compound fracture
E) None of the above
The correct answer is: C

This type of fracture is commonly called a boxer's fracture, and is caused by a dorsal force applied directly to the metacarpal head. Ref: MS/ Sabiston/ PS-RAL
In the treatment of a boxer's fracture, how much palmar angulation of the 4th metacarpal is acceptable?

A) No angulation is acceptable
B) Less than 30 degrees
C) 30-50 degrees
D) 50-70 degrees
E) Any angulation is acceptable, so long as apposition of the fragments is maintained.
The correct answer is: C

30-50 degrees of palmar angulation of the 4th or 5th metacarpal is acceptable. Ref: MS/ Schwartz 8, p. 1738/ PS-RAL
The greatest frequency of shoulder dislocations occur in which direction?

A) Inferior
B) Anterior
C) Posterior
D) Multidirectional
E) Superior
The correct answer is: B

In 95% of glenohumeral dislocations, the humeral head is displaced anteriorly. Ref: MS/ Schwartz 8, p. 1694/ PS-RAL
Typically, the physical examination of a patient with a femoral neck fracture reveals the:

A) proximal fragment is posteriorly displaced.
B) affected limb is shortened and externally rotated.
C) affected limb is shortened and internally rotated.
D) distal fragment is posteriorly displaced.
E) affected limb appears elongated and internally rotated.
The correct answer is: B

The affected limb appears shortened and externally rotated. Rotational movement of the limb causes severe pain. Ref: MS/ Schwartz 8, p. 1685/ RAL
Spinal fractures resulting from axial compression of the vertebral column are commonly known as what type of fracture?

A) Flexion distraction
B) Fracture dislocation
C) Burst fractures
D) Compound fractures
E) Simple fractures
The correct answer is: C

Burst fractures are caused by axial compression forces applied to the vertebra, as a vertical force is applied directly to the top of the head in a diving injury. Displacement of the vertebral bodies into the spinal canal, and resultant neurological injury, is common. Ref: MS/ Schwartz 8, p. 1702/ SP-RAL
Which statement about spinal cord injuries is TRUE?

A) All patients with high-energy injuries should be stabilized with a rigid backboard and a cervical collar at the trauma site.
B) Even with a stable spinal injury, some neurological deficit may be seen.
C) A displaced fracture of the odontoid process is known as a "hangman's fracture."
D) Presence of active flexation of the great toe indicates preservation of spinal cord function at the lumbar level.
E) None of the above
The correct answer is: A

All high-energy trauma victims should be presumed to have spinal injuries and stabilized accordingly. Any neurological defect observed is evidence of instability. A "hangman's fracture" occurs at the C2 level. Flexion of the great toe (and intact rectal tonE) indicates function at the sacral level. Ref: MS/ Schwartz 8, p. 1700-1702/ RAL
Fracture of the distal radius with dorsal displacement of the radial fragment is known as the:

A) Colles-Ponteau fracture.
B) Smith-Goyrand fracture.
C) The Chauffeur's fracture.
D) Hutchinson's fracture.
E) None of the above
The correct answer is: A

The Colles-Ponteau fracture occurs at the dital radius with dorsal angulation of the distal fragment. If the distal fragment is displaced toward the volar surface, it is a Smith-Goyrand fracture. A fracture of the radial styloid has been termed either a Chauffeur's or a Hutchinson's fracture. Ref: MS/ Schwartz 8, p. 1699/ RAL
Proper care of an open fracture generally includes which of the following?

A) Intravenous antibiotics
B) Palpation of pulses distal to the injury
C) Operative debridement
D) Immobilization
E) All of the above
The correct answer is: E

Initial assessment includes distal neurologic and vascular examination. IV antibiotics, debridement in the operating room, and some means of immobilization are standard. Ref: MS/ Schwartz 8, p. 1680-1681/ PS- RAL
Which statement concerning compartment syndrome is TRUE?

A) Passive movement of the involved extremity is usually painless.
B) A measured compartment pressure of less than 60mmHg or 50% of the systemic systolic pressure generally rules out compartment syndrome.
C) The posterior compartment is the most common location of lower leg compartment syndrome.
D) Foot drop and renal failure are known complications.
E) None of the above
The correct answer is: D

Pain on passive movement of the extremity and compartment pressures greater than 20 to 25mmHg are consistent with compartment syndrome. The anterior compartment is the most common location in the lower leg, and foot drop may result from an untreated compartment syndrome in this location. Breakdown products of necrotic muscle from any location may cause myoglobinurea and renal failure. Ref: MS/ Schwartz 8, p. 349/ PS-RAL
Which statement about rheumatoid arthritis is FALSE?

A) Involvement is usually symmetrical.
B) Both large and small joints of the upper and lower extremity are involved.
C) Ulnar deviation is characteristic of hand deformities.
D) Narrowing of the joint space is noted on plain X-ray films.
E) Permanent erosion of the joint space is rare.
The correct answer is: E

After inflammation of an individual joint for a year or more, permanent destruction of cartilage and bone take place. Ref: MS/ Goroll 4, p. 876-877/ RAL
Which statement about osteoarthritis is TRUE?

A) Multiple serum markers have been identified.
B) Physical therapy may be helpful.
C) Inflammation in the joint space plays a major role.
D) Pulmonary effusions are a common extraarticular manifestation.
E) The disease spares the large, weight-bearing joints.
The correct answer is: B

Unlike rheumatoid arthritis, there are no serum markers or extraarticular manifestations, and inflammation of the synovium plays a minimal role. The knees and hips are commonly involved. Physical therapy is helpful in both rheumatoid arthritis and osteoarthritis. Ref: MS/ Goroll 4, p. 878, 888-890/ RAL
Proximal fractures of the humerus generally result from which of the following?

A) Falls on the outstretched hand
B) Hyperabduction injuries
C) Hyperextension injuries
D) Dislocations of the shoulder
E) Pathological fractures due to metastatic malignancies
The correct answer is: A

Falling on the outstretched hand is the usual mechanism of injury, seen most commonly in the elderly. Ref: MS/ Schwartz 8, p. 1696/ RAL
Fibrosis of the skin, blood vessels, and viscera, caused by the overproduction of collagen and other proteins is typical of:

A) Behcet's Syndrome.
B) Ankylosing Spondylitis.
C) Scleroderma.
D) Sjogrens's Syndrome.
E) None of the above
The correct answer is: C

The overproduction of collagen is a key feature of scleroderma. Ref: MS/ Harrison's 16, p. 1981/ PS-RAL
Which of the following statements concerning polymyalgia rheumatica is true?

A) It is a post-viral syndrome.
B) The erythrocyte sedimentation rate (ESR) is usually normal.
C) It is associated with temporal arteritis.
D) Muscle wasting is a predominant feature.
E) There is a 2:1 male predominance.
The correct answer is: C

Polymyalgia rheumatica is an inflammatory process of unknown etiology presenting with shoulder, neck and hip pain and diffuse swelling of the hands. The ESR is elevated; the disease is more common in females; muscle weakness is not a prominent feature; and it is associated with temporal arteritis. Ref: MS/ Goroll 4, p. 911-912/ RAL
Which of the following are risk factors for osteoporosis?

A) Daily calcium intake less than 400 grams
B) Chronic treatment with glucocorticoids
C) Chronic alcoholism
D) A and C
E) All of the above
The correct answer is: E

A life-long deficiency in dietary calcium may cause osteoporosis in later life. Alcoholism, and chronic treatment with glucocorticoids, cyclosporine and excessive exogenous thyroid hormone may contribute to the disease. Ref: MS/ Harrison's 16, p. 2268-2271/ RAL
A 32-year-old amateur athlete complains of elbow pain after a weekend practicing his favorite racquet sport. Examination of the elbow reveals no masses or deformities, but typical pain is reproduced during resisted extension of the wrist or elbow with the forearm in pronation. Plain X-ray films are normal. What is the most likely diagnosis?

A) Carpal tunnel syndrome
B) Olecranon bursitis
C) Medial epicondylitis
D) De Quervain's tenosynovitis
E) Lateral epicondylitis
The correct answer is: E

Lateral epicondylitis ("tennis elbow") presents with the symptoms and findings noted, as well a point tenderness over the lateral epicondyle. The tenderness is medial in medial epicondylitis ("golfers elbow"). In olecranon bursitis there is a tender fluctuant mass posteriorly, over the elbow. De Quervain's tenosynovitis and carpal tunnel syndrome present with symptoms in the hand. Ref: MS/ Goroll 4, p. 864-866/ RAL
Patients with multiple myeloma have decreased immunity and suffer frequent bacterial infections. The most frequent types of infection are which of the following?

A) Pneumonia and pyelonephritis
B) Soft tissue infections and osteomyelitis
C) Conjunctivitis and sinusitis
D) Septicemia and meningitis
E) Upper respiratory infections and sexually transmitted diseases (STD's)
The correct answer is: A

Pneumonia and pyelonephritis are the most common bacterial infections in multiple myeloma, a monoclonal neoplasm affecting B lymphocytes. The most frequent symptom in patients with multiple myeloma is bone pain. Ref: MS/ Harrison's 16, p. 656-658/ PS-RAL
What anatomical structures define the carpal tunnel?

A) Thenar eminence and the ulnar nerve
B) Carpal bones and the transverse carpal ligament
C) Deep palmar arch and the radial nerve
D) Median nerve and extensor tendons
E) Flexor tendons and median nerve
The correct answer is: B

The carpal tunnel is a relatively narrow space bordered by the carpal bones and the transverse carpal ligament. Ref: MS/ Harrison's 16, p. 1776-1767/ PS-RAL
Which of the following structures are contained in the carpal tunnel?

A) Flexor tendons and ulnar nerve
B) Ulnar artery and radial nerve
C) Median nerve and extensor tendons
D) Flexor tendons and median nerve
E) Transverse carpal ligament and carpal bones
The correct answer is: D

The median nerve and the flexor tendons travel through a relatively narrow space bordered by the carpal bones and the transverse carpal ligament. Ref: MS/ Schwartz 8, p. 1766-1769/ PS-RAL
When positive, Tinel's sign, a clinical test for carpal tunnel syndrome, elicits which of the following symptoms?

A) Pain on the ulnar aspect of the hand
B) Pain in the thenar eminence
C) Paresthesias in the 1st through the 4th digits
D) Weakness in the forearm
E) Loss of the ulnar pulse
The correct answer is: C

Tapping over the median nerve on the volar surface of the wrist typically produces paresthesias in the sensory distribution of the median nerve, the first 3 digits and the radial surface of the 4th digit. Ref: MS/ Goroll 4, p. 866; Schwartz 8, p. 1767/RAL
Asymmetrical arthritis of the lower extremities in a 35-year-old man, associated with nongonococcal urethritius and iritis or conjunctivitis, suggests which of the following cause of joint pain?

A) Pseudogout
B) Lyme disease
C) Sarcoidosis
D) Reiter's syndrome
E) Acute rheumatic fever
The correct answer is: D

Reiter's syndrome presents with nongonococcal urethritis, arthritis, and ocular symptoms in men, although Chlamydia trachomatis is cultured from the urethra in 70% of cases. Symptoms may represent an immune response to C. trachomatis, although the etiology remains obscure. Ref: MS/ Harrison's 16, p. 1012; Goroll 4, p. 829/ RAL
This arthritis presents, in adolescence or early adulthood, with symptoms predominantly in the lower back, although pain in other joints is sometimes present. There is a 3:1 male predominance. MRA may demonstrate sacroiliitis.

A) Lyme disease
B) Sarcoidosis
C) Ankylosing spondylitis
D) Reiter's syndrome
E) Psoriatic arthritis
The correct answer is: C

Ankylosing spondylitis is an immune-mediated disease of unknown etiology occurring mainly (but not exclusively) in patients who are HLA-B27 positive. NSAIDS, and more recently, the immunosuppressive drug infliximab, have been useful in treatment. Ref: MS/ Harrison's 16, p. 1993-1995 /RAL
Which of the following structures is NOT a component of the rotator cuff?

A) Infraspinatus
B) Teres minor
C) Supraspinatus
D) Subscapularis
E) Deltoid
The correct answer is: E

The first four muscles and their tendons insert onto the head of the humerus, forming the rotator cuff. The deltoid is superficial to these, inserting onto the clavicle. Ref: MS/ Schwartz 8, p. 1706/ PS-RAL
Which statement regarding temporomandibular joint (TMJ) dysfunction is true?

A) It is usually bilateral.
B) It is usually caused by degenerative changes within the TMJ.
C) It is usually the consequence of bruxism, or nocturnal jaw clenching.
D) The radiographic study of choice is the barium swallow.
E) Arthroscopic surgery is the initial therapy of choice.
The correct answer is: C

Bruxism has been implicated as the cause of most temporomandibular joint pain. TMJ dysfunction usually presents with unilateral symptoms, which are most commonly due to muscle spasm rather than internal derangement of the joint. The diagnosis study of choice is MRI. Surgical options are usually reserved for patients who fail more conservative therapies. Ref: MS/ Goroll 4, p.1145/RAL
Which is the most common primary malignant tumor of bone?

A) Osteoma
B) Osteosarcoma
C) Osteoblastoma
D) Osteoid osteoma
E) Fibrosarcoma
The correct answer is: B

Osteosarcoma appears most commonly in the femur, tibia, and humerus

in that order

usually presenting between 10 and 25 years of age. Osteoma, osteoblastoma, and osteoid osteoma are benign. Fibrosarcoma is malignant, but relatively rare. Ref: MS/ Schwartz 8, p. 1664-1668/ RAL
In slipped capital femoral epiphysis:

A) patients usually present at age 3 to 5 years.
B) 75% of cases are bilateral.
C) the leg is typically held in internal rotation.
D) most patients are below the 50% percentile for weight and height.
E) None of the above
The correct answer is: E

Patients present in adolescence with the leg held in external rotation and limited internal rotation on exam. About one-third of cares are bilateral. Most patients are overweight. Ref: MS/ Schwartz 8, p. 1716/ RAL
Which of the following is a risk factor for osteoporosis in women?

A) Cigarette smoking
B) Obesity
C) Late menopause
D) High-impact aerobic exercise
E) African-American race
The correct answer is: A

Current cigarette smoking is a risk factor. Others include:
Body mass index below the 25th percentile Early menopause Sedentary lifestyle Caucasian race Ref: MS/ Goroll 4, p. 821/ RAL
Septic arthritis most commonly arises from:

A) direct penetrating trauma to the joint.
B) lymphatic spread from distant sites of infection.
C) hematogenous spread from other sites of infection.
D) erosion into the joint space from adjacent sites of infection.
E) a spontaneous bacterial infection without a detectable source.
The correct answer is: C

Most episodes of septic arthritis arise from hematogenous seeding of the joint. Ref: MS/ Goroll 4, p. 823/ RAL
Salter fractures:

A) always involve the shaft of a long bone.
B) involve disruption of the joint space in adults.
C) imply a communication between the fracture and the skin.
D) only occur in children.
E) involve no displacement of the fragments.
The correct answer is: D

Salter fractures involve the epiphyseal plate at the end of a long bone in a growing child. By definition, they cannot occur in adults. Ref: MS/ Tintinalli 6, p. 1656/ RAL
A fracture in a child, involving a compression load to the proximal end of the tibia, which compresses, but does not displace the epiphysis of the bone, will result in which type of Salter fracture?

A) Salter I
B) Salter II
C) Salter III
D) Salter IV
E) Salter V
The correct answer is: E

Compression injury, without displacement of the epiphysis, results in a Salter V type fracture. The injury may not be evident until growth disturbances occur, but should be suspected based on the mechanism of injury and the location of tenderness. Ref: MS/ Tintinalli 6, p. 1657/ RAL
The innervation of the palmar surface of the thumb is provided by the:

A) radial nerve.
B) ulnar nerve.
C) median nerve.
D) None of the above
The correct answer is: C

The sensory innervation of the palmar surfaces of the thumb, the 2nd, 3rd, and the radial aspect of the 4th fingers is provided by the median nerve. Ref: MS/ Tintinalli 6, p. 1670/ RAL
A patient presents with a dislocation of the proximal interphalangeal (PIP) joint of the 4th finger. After digital block, the dislocation is reduced by longitudinal traction. At the end of the procedure, active motion and strength are intact. Which of the following is the most appropriate next step in the management of this patient?

A) Splint the joint in 30 degrees of flexion for 3 weeks.
B) Splint the joint in 90 degrees of flexion for 1 week.
C) Splint the joint in slight extension for 3 weeks.
D) No immobilization is required.
E) Apply a short arm ulnar gutter splint.
The correct answer is: A

The joint should be immobilized in 30 degrees of flexion for 3 weeks. Operative repair is required if the dislocation cannot be reduced by traction, or if there is evidence of complete disruption of the ligaments. Ref: MS/ Tintinalli 6, p 1672/ RAL
Gamekeeper's thumb (or skier's thumB) results from which of the following mechanisms of injury?

A) Direct longitudinal compression
B) Twisting or shearing motion of the thumb
C) Direct trauma to the interphanlangeal joint
D) Forced abduction of the thumb
E) Forced adduction of the thumb
The correct answer is: D

Rupture of the ulnar collateral ligament (gamekeeper's thumb or skier's thumB) results from forced abduction of the thumb. Ref: MS/ Tintinalli 6, p. 1673/ RAL
A "boxer's fracture" refers to which of the following?

A) Fracture of the 2nd metacarpal head
B) Fracture-dislocation of the first metacarpophalangeal (MCP) joint
C) Fracture of the distal phalanx of the 3rd finger
D) Undisplaced compression fracture of the 4th proximal phalanx
E) Fracture of the 5th metacarpal neck
The correct answer is: E

Fracture of the neck of the 5th metacarpal is known as a "boxer's fracture". Ref: MS/ Tintinalli 6, p. 1673/ RAL
What is the maximum amount of volar angulation acceptable in a fracture of the 5th metacarpal neck ("boxers fracture")?

A) No angulation is acceptable
B) 10 degrees
C) 20 degrees
D) 30 degrees
E) 40 degrees
The correct answer is: E

40 degrees of volar angulation is the greatest amount consistent with a good functional result. Greater degrees of angulation will result in disability. The boxer's fracture should be splinted with the wrist in 20 degrees of extension and the metacarpothalangeal (MP) joint splinted in 90 degrees of flexion. Ref: MS/ Tintinalli 6, p. 1673/ RAL
After a fall on the outstretched hand, a 60-year-old woman presents in the Emergency Department with a painful wrist. On examination you note tenderness in the "anatomical snuffbox". What is the most likely injury at this point?

A) Colles fracture
B) Smith fracture
C) Ulnar styloid fracture
D) Scaphoid fracture
E) Triquetrum fracture
The correct answer is: D

Tenderness in the "anatomical snuffbox", the space on the dorsum of the wrist bounded by the radial styloid, the extensor pollicis brevis tendon and the extensor pollicis longus tendon, is associated with fracture of the scaphoid bone. Ref: MS/ Tintinalli 6, p. 1679/ RAL
A fracture of the distal radius with volar angulation of the fragment, a "reverse Colles fracture" is known as a:

A) Smith fracture.
B) trapezium fracture.
C) hamate fracture.
D) boutonniere fracture.
E) swan-neck fracture.
The correct answer is: A

The "reverse Colles fracture", with volar angulation of the distal fragment, is known as a Smith fracture. The trapezium and hamate are wrist bones, and boutonniere and swan-neck refer to finger deformities related to injury. Ref: MS/ Tintinalli 6, p. 1683/ RAL
Volkmann's ischemic contracture is a complication of which of the following injuries?

A) Sternoclavicular dislocation
B) Triceps rupture
C) Biceps rupture
D) Clavicle fracture
E) Supracondylar fracture of the humerus
The correct answer is: E

Impending Volkmann's ischemic contracture should be suspected in supracondylar humeral fracture, when there is pain on passive extension of the fingers or forearm tenderness. Ref: MS/ Tintinalli 6, p. 1687/ RAL
A patient with a forearm injury is unable to extend the wrist against resistance and has lost sensation on the dorsum of the hand, in the web space between the thumb and index finger. What nerve is likely to have been injured?

A) Radial nerve
B) Ulnar nerve
C) Median nerve
The correct answer is: A

Extension of the wrist and sensation on the dorsum of the hand, as described, are tests of the motor and sensory function of the radial nerve. Ref: MS/ Tintinalli 6, p. 1691/ RAL
A patient with an injury to the forearm is unable to make the "OK sign" (thumb to index finger) and has lost sensation on the radial aspect of his palm. What nerve is likely to have been injured?

A) Radial nerve
B) Ulnar nerve
C) Median nerve
The correct answer is: C

The median nerve provides motor innervation to the thenar muscles of the thumb and sensory innervation to the radial side of the palm. The maneuvers described are a quick test of its function. Ref: MS/ Tintinalli 6, p. 1691/ RAL
A patient with a forearm injury is unable to spread the fingers of the hand against resistance. He has lost 2-point discrimination in the distal aspect of the 5th finger. What nerve is most likely to have been injured?

A) Radial nerve
B) Ulnar nerve
C) Median nerve
The correct answer is: B

The strength of the intrinsic muscles of the hand, as assessed by active abduction of the 5th finger, and sensation in the distal 5th finger are standard tests of the motor and sensory function of the ulnar nerve. Ref: MS/ Tintinalli 6, p. 1691/ RAL
An elderly patient presents after hearing a sharp "pop" while moving boxes in his basement, followed by acute pain in the anterior aspect of his shoulder. In addition to swelling and tenderness of the shoulder, a ball-like mass is noted in the mid upper arm with active flexion. What is the most likely diagnosis?

A) Stenoclavicular dislocation
B) Supracondylar humerus fracture
C) Biceps tendon fracture
D) Acute upper extremity DVT
E) Brachial artery dissection
The correct answer is: C

The mechanism and presentation are typical of biceps tendon rupture. Immediate treatment consists of immobilization in a sling, the application of ice, and analgesics as needed. Active patients may eventually require surgical repair, although function is generally well-maintained by other muscles groups. Ref: MS/ Tintinalli 6, p. 1693-1694/ RAL
The most common glenohumeral (shoulder) dislocation is:

A) Anterior dislocation
B) Posterior dislocation
C) Inferior dislocation
D) Superior dislocation
The correct answer is: A

Anterior dislocations comprise well over 90% of all shoulder dislocations. The Hippocratic, Stimson, and Milch techniques are some of the many maneuvers described for reduction. Ref: MS/ Tintinalli 6, p. 1697/ RAL
Which injury is most likely to be produced by extreme hyperextension of the neck?

A) Jefferson burst fracture of the atlas
B) Hangman's fracture (traumatic spondylolisthesis) C) Clay-shoveler's fracture
D) Flexion teardrop fracture
E) Bilateral interfacetal dislocation
The correct answer is: B

The Hangman's fracture is characterized by anterior displacement of C2 on C3, and may be seen in judicial hangings or high-speed motor vehicle accidents causing severe hyperextension of the neck. Bilateral interfacetal dislocation, flexion teardrop fracture, and clay-shoveler's fracture are caused by flexion. Jefferson fracture of the atlas is caused by axial loading, as in a blow to the top of the head. Ref: MS/ Tintinalli 6, p. 1705-1708/ RAL
The best treatment for a displaced femoral neck fracture in a 60-year-old woman who has fallen consists of which of the following? (Assume stable medical condition and no other injuries)

A) Bedrest with skeletal traction
B) Bedrest with skin traction
C) Splinting, ice, and analgesics
D) Immediate open reduction and internal fixation (ORIF) E) Skeletal traction for 2 weeks, followed by total hip arthroplasty
The correct answer is: D

Skeletal traction is contraindicated, due to the increased risk of avascular necrosis of the femoral head. Most patients who are well enough to tolerate surgery should undergo emergency ORIF. Ref: MS/ Tintinalli 6, p. 1722/ RAL
Following an acute arterial occlusion and subsequent femoral embolectomty, with restoration of arterial inflow to the leg, a patient begins to complain of worsening pain. On examination, the lower leg is swollen and tense, and there is exquisite tenderness anteriorly. The patient is unable to extend his toes and has lost sensation in the web space between the 1st and 2nd toes. Which is the best test to establish the diagnosis?

A) AP and lateral films of the leg
B) CT scan with contrast of the lower leg
C) Femoral arteriography
D) Measure compartment pressures
E) Nerve conduction studies
The correct answer is: D

The presentation is typical for anterior compartment syndrome, frequently seen after revascularization of an acutely ischemic extremity. The diagnosis is confirmed by inserting a needle into the compartment (in this case the findings refer to the anterior compartment of the lower leg) and measuring the pressure. If the pressure is elevated, surgical fasciotomy to relieve the swelling in the closed compartment is indicated. Ref: MS/ Tintinalli 6, p. 1748/ RAL
A multiple trauma patient with extensive contusions of the muscles of the upper legs and back develops dark, "Coca-cola colored" urine 12 hours after his injury. His hourly urine output is noted to be decreasing. Review of his most recent blood work reveals that the CPK is 10 times the upper limit of normal, although the CK-MB fraction is not elevated. What is the most likely diagnosis at this point?

A) Injury to one or both kidneys
B) Rupture of the bladder
C) Hemolysis
D) Rhabdomyolysis
E) Retroperitoneal hematoma
The correct answer is: D

Rhabdomyolysis can result from severe muscle trauma, and other causes, and typically produces dark-colored urine and serum CPK's at least 5 times the upper limit of normal. The main complication is renal failure. Treatment includes vigorous hydration, alkalinizing of the urine (with intravenous sodium bicarbonatE) and diuresis with mannitol. Ref: MS/ Tintinalli 6, p. 1749-1751/ RAL
A trauma patient with extensive pelvic fractures is exsanguinating due to retroperitoneal hemorrhage, despite normalized coagulation profile and core temperature and massive resuscitation with crystalloid and blood products. What additional treatment should be considered?

A) Angiography with embolization of bleeding vessels
B) Placement of military antishock (MAST) trousers
C) Hypothermia
D) Surgical exploration of the retroperitoneum
E) Ligation of both hypogastric arteries
The correct answer is: A

Arteriography and embolization, although not routinely required in pelvic fractures, may represent the best option in this desperate situation. Ref: MS/ Tintinalli 6, p. 1716/ RAL
A front seat passenger in a motor-vehicle accident presents with hip pain. On presentation the right leg is shortened, with the hip in adduction and internal rotation. AP and lateral films of the hip confirm posterior dislocation. There are no fractures and no other serious injuries. What is the best immediate management of this patient?

A) Open reduction and internal fixation (ORIF) B) Closed reduction under conscious sedation or general anesthesia
C) Skeletal traction until soft tissue swelling has subsided
D) Skin traction until spontaneous reduction takes place
E) Intravenous benzodiazopines for 24 hours, to reduce muscle spasm
The correct answer is: B

Closed reduction with appropriate anesthesia or sedation should take place within 6 hours, to minimize the risk of avascular necrosis of the femoral head. Ref: MS/ Tintinalli 6, p. 1724/ RAL
A college football player is running for a pass. He turns sharply to the left, immediately notes a loud "pop" from his right knee, and is immobilized by severe knee pain. There is no contact involved. Two hours later, he has developed a swollen, painful right knee. What is the most likely diagnosis?

A) Anterior cruciate ligament injury
B) Posterior cruciate ligament injury
C) Knee dislocation
D) Fracture of the patella
E) Fracture of the tibial plateau
The correct answer is: A

The pathognomonic "pop" and the non-contact nature of the injury favor an anterior cruciate ligament disruption, the most common ligamentous injury of the knee. Ref: MS, Tintinalli 6, p. 1728-1729/ RAL
A patient is being examined for a suspected knee injury. The examiner places the hip in 90 degrees of flexion and the knee in 45 degrees of flexion. He attempts to displace the tibia forward with respect to the femur and notes about 10mm of foreword movement. What is the name of the test described?

A) Lachman's test
B) Lateral pivot shift
C) McMurray test
D) The anterior drawer sign
E) The posterior drawer sign
The correct answer is: D

The anterior drawer sign is elicited in suspected anterior cruciate ligament injury. It is considered positive if forward displacement of the tibia of more than 6mm is possible. Ref: MS/ Tintinalli 6, p. 1729/ RAL
A patient with a stable lateral ankle sprain should be treated with which of the following?

A) Open reduction and internal fixation
B) Immobilization in plaster, with the ankle in eversion, for 6 weeks
C) Immobilization in plaster, with the ankle in inversion, for 6 weeks
D) Immobilization in plaster for 2 weeks, followed by delayed surgery
E) Rest, ice, compression with an elastic bandage and elevation of the leg for 24-72 hours
The correct answer is: E

Rest, ice, compression and elevation (RICE) is appropriate treatment for most stable, lateral ankle sprains. Ref: MS/ Tintinalli 6, p. 1738/ RAL
A 75-year-old man complains of pain in the plantar surface of the foot on the first step out of bed in the morning. The pain often improves, only to recur later in the day. On examination, forced dorsifelion of the toes reproduces the pain. What is the most likely diagnosis?

A) Peroneal nerve injury
B) Intermittent claudication
C) Plantar faciitis
D) Heel spurs
E) Morton's neuroma
The correct answer is: C

The presentation and physical findings are typical for plantar fasciitis. Treatment may include rest, ice, Achiles tendon stretches, and local steroid injection. Ref: MS/ Goroll 4, p. 870/ RAL
A patient on chronic glucocorticoid therapy for COPD notes the gradual onset of hip pain. Examination is unremarkable, except for range of motion of the joint limited by pain. X-ray shows segmental collapse of the femoral head with wedge-shaped densities. What is the most likely diagnosis?

A) Hematogenous osteomyelitis
B) Osteoarthritis
C) Reiter's syndrome
D) Avascular necrosis
E) Polymyalgia rheumatica
The correct answer is: D

Patients on chronic glucocorticoid therapy are subject to steroid-induced avascular necrosis of the head of the femur. The pathophysiology involves changes in tissue perfusion leading to necrosis. Ref: MS/ Goroll 4, p. 858/ RAL
A 50-year-old woman complains of burning pain in the right 3rd and 4th toes. Compression of the forefoot reproduces the pain. X-ray is normal. What is the most likely diagnosis?

A) Morton's neuroma
B) Plantar fasciitis
C) Peroneal nerve injury
D) Ischemic rest pain
E) Sesamoid fracture
The correct answer is: A

Morton's neuroma is a nerve compression syndrome involving branches of the medial and lateral plantar nerves. Treatment consists of wearing wider shoes, various orthotics, NSAID's, and local steroid injections. Ref: MS/ Goroll 4, p. 870/ RAL
The most common causative organism in acute, diffuse otitis externa (swimmer's ear) is:

A) Pseudomonas species
B) Staphylococcus aureus
C) Staphylococcus epidermitis
D) Haemophilus influenzae
E) Aspergillus species
The correct answer is: A

Pseudomonas aeruginosa is the most common pathogen. Ref: ENT/ Harrison's 16, p. 188; Goroll 4, p. 1126/ RAL
An elderly diabetic man presents to the emergency department at 10 P.M. with a complaint of intense otalgia and purulent drainage. Examination of the symptomatic external canal reveals exuberant granulation tissue and greenish purulent drainage. The tympanic membrane appears normal. The most likely diagnosis is which of the following?

A) Simple otitis externa
B) Malignant (or necrotizing) otitis externa
C) Serous otitis media
D) Squamous cell carcinoma of the external ear canal
E) Basal cell carcinoma of the external ear canal
The correct answer is: B

Malignant otitis externa (or "invasive otitis externa") presents with ear pain, purulence, and characteristically, impressive granulation tissue in the external canal. Elderly diabetics or immunocompromised individuals are the most frequent victims. The offending organism is usually pseudomonas aeruginosa. Ref: ENT/ Harrison's 16, p. 189; Goroll 4, p. 1126/ RAL
An elderly diabetic man presents to the emergency department at 10 P.M. with a complaint of intense otalgia and purulent drainage. Examination of the symptomatic external canal reveals exuberant granulation tissue and greenish purulent drainage. The tympanic membrane appears normal. The most appropriate immediate management of this patient is:

A) Refer to an otolaryngologist within 24 hours.
B) Prescribe neomycin, polymyxin B and hydrocortisone (Cortisporin) otic solution to be instilled three times per day; follow-up with primary care provider in one week.
C) Insert a gauze wick in the external canal and instill gentamicin otic solution; patient to return to the emergency department for follow-up the next morning.
D) Prescribe oral ciprofloxacin; follow-up with an otolaryngologist within 24 to 48 hours.
E) Admit the patient to the hospital and begin intravenous anti-pseudomonal antibiotic coverage.
The correct answer is: E

Malignant otitis externa is a dangerous soft tissue infection that poses a substantial risk of widespread sepsis and may be life threatening. Management includes intravenous antibiotics and occasionally debridement of the external canal. Ref: ENT/ Harrison's 16, p. 189; Goroll 4, p. 1126/ RAL
The typical presentation of acute purulent (bacterial) otitis media includes all of the following except:

A) Hearing loss
B) Fever
C) Ear pain
D) Retracted tympanic membrane
E) Otorrhea
The correct answer is: D

The tympanic membrane is typically bulging and erythematous, with loss of the bony landmarks. The triad of pain, diminished hearing, and fever are typical. Purulent drainage, or otorrhea, may occur with perforation. Retraction of the tympanic membrane, with preservation of the bony landmarks, is more often seen in serous otitis media. Ref: ENT/ Goroll 4, p. 1126/ RAL
The most common causative organism in acute purulent otitis media is:

A) Streptococcus pneumoniae
B) Moraxella catarrhalis
C) Haemophilus influenzae
D) Neisseria catarrhalis
E) Staphylococcus epidermidis
The correct answer is: A

S. pneumoniae, cultured from 35% of aspirates, is the most common causative organism. Ref: ENT/ Harrison's 16, p. 189/ RAL
What is the antibiotic of choice in acute otitis media?

A) Amoxicillin
B) Ciprofloxacin
C) Trimethoprim-sulfa (Bactrim) D) Amoxacillin-clavulanate (Augmentin) E) Metronidazole
The correct answer is: A

Amoxicillin is the antibiotic of first choice, according to the Centers for Disease Control (CDC) . As some organisms may be penicillin-resistant, a change to Augmentin may be indicated if clinical improvement is not seen in 3 days. Bactrim or erythromycin are alternatives in penicillin-allergic patients. Ref: ENT/ Harrison's 16, p. 189; Goroll 4, p. 1126/ RAL
A middle-aged man presents to your clinic complaining of fever, purulent nasal drainage, facial pain, and toothache. The most likely diagnosis is:

A) Ethmoid sinusitis
B) Sphenoid sinusitis
C) Frontal sinusitis
D) Maxillary sinusitis
E) None of the above
The correct answer is: D

Acute purulent bacterial sinusitis localized to the maxillary sinus produces pain over the cheeks, radiating to the teeth, with purulent drainage from the middle meatus of the nasal turbinates. Ref: ENT/ Goroll, p. 1127/ RAL
A patient previously seen for a diagnosis of acute bacterial ethmoid sinusitis returns to the clinic 2 days later with complaints of pain and swelling in the left eye. His visual acuity in the affected eye is intact, as are the extraocular movements. His mental status is intact. You note periorbital edema and a low-grade temperature. The most likely diagnosis is:

A) Orbital cellulitis
B) Cavernous sinus thrombophlebitis
C) Epidural abscess
D) Subdural empyema
E) Bacterial meningitis
The correct answer is: A

The most likely diagnosis, given these findings, is simple orbital cellulitis produced by extension of the bacterial infection in the ethmoid sinus. The other, even more serious infections listed are usually associated with loss of visual acuity, cranial nerve palsies, mental status changes, and a more toxic picture. Nevertheless, to prevent more serious sequellae, the patient should be treated with broad spectrum intravenous antibiotics and should be studied with CT or MRI to rule out abscess requiring surgical drainage. Ref: ENT/ Goroll 4, p. 1130; Harrison's 16, p. 173/ RAL
Which statement concerning allergic rhinitis is false?

A) Symptoms are usually worse early in the day.
B) In northern climates, virtually all patients will report resolution of their symptoms after the first frost.
C) Symptoms include a thin, watery nasal discharge and injection of the conjunctivae.
D) The nasal mucosa typically appears pale and boggy.
E) Patients allergic to tree pollen become symptomatic in March and April.
The correct answer is: B

Although 90% of patients with allergic rhinitis are sensitive to pollens and will improve with the first frost, the remaining ten percent are sensitive to dust, molds, and animal dander, and will remain symptomatic throughout the year. Ref: ENT/ Goroll 4, p. 1137/ RAL
An obese, hypertensive 60-year-old man with chronic stable angina presents to your clinic for treatment of his seasonal allergic rhinitis. Which agent(s) would probably not be an appropriate part of his management?

A) Loratadine (Claritin) orally
B) Beclomethasone dipropionate (Vancinase, BeconasE) nasal spray
C) Pseudoephedrine orally
D) Fluinsolide (NasalidE) nasal spray
E) Cromolyn nasal spray
The correct answer is: C

Sympathomimetics may worsen hypertension and coronary artery disease, and they should be avoided in this setting. Ref: ENT/ Goroll 4, p. 1141/ RAL
Which statement is true concerning epistaxis?

A) Local trauma or irritation is the cause of 90% of episodes.
B) Anterior epistaxis tends to be more severe.
C) Posterior epistaxis is more likely in children.
D) The patient is best treated in the supine position with the legs elevated.
E) Posterior nasal packing in ambulatory patients requires follow-up within 48 hours.
The correct answer is: A

Posterior epistaxis generally produces more brisk bleeding and is very uncommon in children. The patient is best treated sitting upright and leaning forward. Posterior nasal packing requires that the patient be admitted for IV hydration and management of his airway until the packing is removed. Ref: ENT/ Goroll 4, p. 112-1114/ RAL
Which of the following are risk factors for oral cancer?

A) Cigarette smoking
B) Pipe smoking
C) Use of "smokeless" tobacco
D) Chronic alcohol abuse
E) All of the above
The correct answer is: E

All forms of tobacco use increase risk of oral cancer; "smokeless" tobacco, in particular, increases the risk of oral malignancy by a factor of 50. Heavy alcohol users are also at increased risk. Ref: ENT/ Goroll 4, p. 1106/ RAL
During routine examination of a 65-year-old male smoker, you note a 1 x 2cm whitish plaque on the oral mucosa. The patient suspects that the lesion is due to irritation from his dentures, and he agrees not to wear them until they can be evaluated by his dentist. What is the most appropriate plan for management of this patient?

A) Instruct the patient to call your office if the lesion becomes painful or bleeds.
B) Obtain an immediate consultation with a head and neck surgeon.
C) Schedule follow-up in 2 weeks and refer the patient if erythroplasia has developed.
D) Schedule follow-up in 2 weeks and refer the patient for biopsy if the lesion is still present.
E) Schedule follow-up in 2 weeks with an MRI of the head and neck.
The correct answer is: D

The lesion should be biopsied if still present 2 weeks after the possible source of irritation has been eliminated. Ref: ENT/ Goroll 4, p. 1106-1107/ RAL
Which statement is true concerning epiglottitis?

A) It is most common in older children and young adults.
B) 70% of cases are viral.
C) Outpatient Indirect laryngoscopy may be attempted after adequate sedation.
D) The patient is typically febrile, toxic, drooling, and breathing with audible stridor.
E) The epiglottis should be examined carefully using a headlamp and a tongue blade.
The correct answer is: D

Epiglottitis is a bacterial infection and may occur at any age up to adulthood. Typically, the causitive organism is Haemophilus influenzae, although other organisms may cause the disease. The average patient is 2.5 years old, but the epidemiology is changing due to the availability of H. influenzae vaccine. If clinical suspicion of epiglottitis exists, based on the clinical picture summarized in Answer D, no attempt should be made to instrument or examine the oropharynx in the outpatient environment, as this may precipitate laryngospasm and complete airway obstruction. Ref: ENT/ Harrison's 16, p. 192-193/ RAL
Which of the following is an appropriate treatment for Haemophilus influenzae epiglotitis in a non-penicillin-allergic patient?

A) Ampicillin/sulbactam
B) Cefuroxime
C) Cefotaxime
D) Ceftriaxone
E) All of the above
The correct answer is: E

Many strains of H. influenzae are presently ampicillin-resistant, so ampicillin/sulbactam or a second- or third-generation cephalosporin should be used. Ref: ENT/ Harrison's 16, p. 193/ RAL
Hoarseness of what duration requires referral to an otolaryngologist for thorough evaluation?

A) 3 to 5 days
B) 3 weeks
C) 6 weeks
D) 3 months
E) 6 months
The correct answer is: B

Patients with hoarseness of three weeks duration and no obvious infectious process explaining the symptom should be referred. Ref: ENT/ Goroll 4, p. 1123/ RAL
The management of acute viral laryngitis generally includes which of the following measures?

A) Broad-spectrum antibiotics
B) Voice rest
C) Inhaled steroids
D) Systemic steroids
E) All of the above
The correct answer is: B

Voice rest, cough suppressants, and the inhalation of warm humidified air are beneficial. Antibiotics are reserved for acute bacterial infections and steroids are used only when an allergic etiology is suspected. Ref: ENT/ Goroll 4, p. 1123/ RAL
Which statement is true concerning malignant tumors of the head and neck?

A) Most are squamous cell carcinomas.
B) Immunosupression increases the risk.
C) Sunlight exposure is a risk factor.
D) Smoking and alcohol are synergistic risk factors.
E) All of the above
The correct answer is: E

Most are squamous cell. Renal transplant recipients (who are on chronic immunosuppressive drugs) have a 30-fold increase in risk of head and neck cancers. Combined alcohol and tobacco use increase the risk 35-fold. Ultraviolet light exposure increases the risk of cancer of the lip, predominantly squamous cell carcinoma, affecting the lower lip in 88 to 98% of cases. Ref: ENT, Schwartz 8, p. 515-518/ RAL
In the Weber test, the sound of the tuning fork is perceived to be louder:

A) in the ear with the conductive hearing loss.
B) in the ear with the sensorineural hearing loss.
C) by the examiner (with intact auditory acuity) than by the patient.
D) when placed on the mastoid process.
E) bilaterally when placed on either mastoid process.
The correct answer is: A

The Weber test lateralizes to the ear with the conductive hearing loss. The tuning is placed on the mastoid process in the Rinne test. The Schwabach test compares the examiners hearing with the patient's. Ref: ENT, Goroll 4, p. 1108, 1110/ RAL
An anxious 37-year-old woman complains of the insidious onset of unilateral, aching jaw pain exacerbated by chewing. On exam you note limited mobility of the mandible, tenderness of the muscles of mastication, and jaw asymmetry on movement. The most likely diagnosis at this point is:

A) Chronic otitis media with referred pain
B) Temporal arteritis
C) Temporomandibular joint dysfunction
D) Acute parotitis
E) Laryngeal carcinoma, until proven otherwise
The correct answer is: C

Temporomandibular joint dysfunction is usually related to a response to stress consisting of involuntary jaw clenching, particularly at night (bruxism). Ref: ENT/ Goroll 4, p. 1145-1146/ RAL
An anxious 37-year-old woman complains of the insidious onset of unilateral, aching jaw pain exacerbated by chewing. On exam you note limited mobility of the mandible, tenderness of the muscles of mastication, and jaw asymmetry on movement. Appropriate initial management of this patient would consist of all of the following except:

A) Advise the patient to cut food into small pieces and avoid chewing gum.
B) Refer to a physical therapist for local heat and massage to the muscles of the jaw.
C) Obtain an MRI of the head and neck to rule out structural lesions.
D) Nonsteroidal anti-inflammatory agents
E) A short course of a minor tranquilizer at bedtime
The correct answer is: C

MRI is generally indicated only after failure of conservative treatment. Ref: ENT/ Goroll 4, p. 1145/ RAL
Which of the following processes does NOT produce pigmented oral lesions?

A) Malignant melanoma
B) Hairy leukoplakia
C) Peutz-Jeghers syndrome
D) Addison's disease
E) Kaposi's sarcoma
The correct answer is: B

Hairy leukoplakia is a whitish lesion produced by the Epstein-Barr virus (EBV). Malignant melanoma, Peutz-Jeghers syndrome, and Addison's disease produced pigmented oral lesions. Kaposi's sarcoma may produce red or blue plaques. Ref: ENT/ Harrison's 16, p. 196-199/ RAL
Which statement is false regarding Herpes simplex virus (HSV) gingivostomatitis?

A) It is the most common manifestation of primary HSV-1 infection.
B) HSV-2 does not infect the oral mucosa.
C) Patients with primary herpes stomatitis generally present with fever, myalgias, and cervical lymphadenopathy.
D) Oral acyclovir may be of some benefit in initial infections.
E) Constitutional symptoms are generally self-limited within 3 to 14 days.
The correct answer is: B

HSV-2, although more commonly associated with genital lesions, may produce clinical HSV gingivostomatitis. Antiviral treatment with acyclovir, famciclovir, and valacyclovir may be of benefit. Ref: ENT/ Harrison's 16, p. 1037, 1040/ RAL
Which statement is TRUE concerning psesbycusis, the most common type of hearing loss in the elderly?

A) It is a conductive hearing loss.
B) It is generally abrupt in onset.
C) Early in the course of disease, low-frequency hearing is most affected.
D) It is usually unilateral.
E) It is a sensorineural hearing loss.
The correct answer is: E

Presbycusis is a sensorineural disorder of hearing in the elderly related to changes in the cochlea. It is gradual in onset, symmetrical, and begins with high-frequency hearing loss. Ref: ENT/ Goroll, p. 1108-1109/ RAL
Which condition does NOT produce a conductive hearing loss?

A) Otosclerosis
B) Exostoses
C) Acoustic trauma
D) Impacted cerumen
E) External otitis
The correct answer is: C

Acoustic trauma produces a sensorineural hearing loss. Ref: ENT/ Goroll 4, p. 1108-1109/ RAL
Which statement concerning acute labyrinthitis is TRUE?

A) It is often caused by ototoxic drugs.
B) It may develop following a viral upper respiratory infection (URI).
C) Symptoms generally resolve in 24 hours.
D) It may result in permanent hearing loss.
E) It is distinguished from Meniere's disease by the absence of tinnitus.
The correct answer is: B

Acute labyrinthitis develops following a viral URI. Vertigo, tinnitus, and temporary hearing loss are common and resolve completely in 3 to 6 week. Ref: ENT/ Goroll 4, p. 941/ RAL
Which statement correctly describes aphthous ulcers?

A) They are generally painless.
B) Topical steroids may be beneficial.
C) They are caused by the herpes simplex virus.
D) A and C are true.
E) All of the above
The correct answer is: B

Topical steroids may give symptomatic relief. They are painful and distinct from herpes labialis. Ref: ENT/ Harrison's 16, p. 195/ RAL
Which physical finding is generally present in allergic rhinitis?

A) Injected pharynx
B) Erythematous nasal mucosa
C) Purulent nasal discharge
D) A and C
E) None of the above
The correct answer is: E

The pharynx is normal; the nasal mucosa is pale and edematous; and the nasal discharge is watery and profuse. Ref: ENT/ Harrison's 16, p. 1954-1955/ RAL
Which statement concerning glaucoma is TRUE?

A) Manual visual field testing is a sensitive technique for screening.
B) It is rare in African Americans.
C) Increased cupping of the optic disc is a detectable fundoscopic sign of glaucoma.
D) Screening for glaucoma should begin at 65 years old and be repeated each year.
E) Intraocular pressure less than 20mmHg reliably rules out glaucoma.
The correct answer is: C

Visual-field testing is unreliable for this purpose. Glaucoma is the leading cause of blindness in African Americans. Pathologic "cupping" of the optic disc is a fundoscopic sign of glaucoma. Screening of asymptomatic individuals should begin at 20 years of age. Normal intraocular pressure does not rule out the disease ("low-tension glaucoma"). Ref: Eye/ Goroll 4, p. 1075-1076; Harrison's 16, p. 171/ RAL
Which of the following should generally not be used in the treatment of bacterial conjunctivitis?

A) Erythromycin ophthalmic ointment
B) Polymyxin/trimethoprim ointment
C) Topical steroid ophthalmic ointment
D) Bacitracin ophthalmic ointment
E) Sodium sulfacetamide ointment
The correct answer is: C

Steroid preparations may worsen some infections and cause corneal ulceration. Ref: Eye/ Goroll 4, p. 1080/ RAL
Viral conjunctivitis remains contagious for about how long?

A) 24 hours
B) 3 days
C) 1 week
D) 2 weeks
E) 1 month
The correct answer is: D

Virus particles are shed in the tears for about 2 weeks. The disease is self-limited and should resolve without treatment in 2-3 weeks. Ref: Eye/ Goroll 4, p. 1080/ RAL
The usual management for subconjunctival hemorrhage is which of the following?

A) Laser photocoagulation
B) Nonsteroidal anti-inflammatory ophthalmic solutions applied twice daily
C) Obtain stat coagulation studies and correct any abnormalities
D) Topical anesthetics and drainage of the hematoma with a 26-gauge needle
E) Cool and then warm compresses, lubricating ointment, reassurance
The correct answer is: E

The condition is generally self-limiting and the treatment is nonspecific and symptomatic. Patients on anticoagulant medications, however, should have coagulation studies checked. Ref: Eye/ Goroll 4, p. 1080/ RAL
An elderly male presents to your office noting several episodes over the past month of transient monocular visual impairment, "like a shade coming down over my eye". He now reports sudden and painless complete loss of vision in the same eye of about 4 hours duration. The most likely diagnosis is:

A) Stroke involving the occipital cortex
B) Central retinal artery occlusion
C) Rapidly progressive pituitary adenoma
D) Detached retina
E) Acute angle-closure glaucoma
The correct answer is: B

The history is most consistent with recurrent emboli to the central retinal artery, most likely to be from an atherosclerotic plaque at the carotid bifurcation, the most recent of which has completely occluded the central retinal artery, producing ischemia of the retina. Ref: Eye/ Goroll 4, p. 1082/ RAL
An elderly woman with a history of polymyalgia rheumatica presents with complaints of jaw pain on chewing her food, tenderness in the right temporal area, and sudden onset of decreased vision in the right eye. The most important therapeutic intervention in this case is:

A) Begin high-dose glucocorticoids
B) Begin pilocarpine 2% to both eyes
C) Refer for emergency laser photocoagulation
D) Refer for emergency carotid endarterectomy
E) Begin broad-spectrum IV antibiotics
The correct answer is: A

Temporal or giant-cell arteritis is an inflammatory process often seen in patients with polymyalgia rheumatica. Involvement of the central retinal artery can cause blindness. The diagnosis can be confirmed by temporal artery biopsy, but this need not take place before the initiation of definitive therapy with oral prednisone. Ref: Eye/ Goroll 4, p. 1084/ RAL
A middle-aged woman presents with complaints of throbbing, unilateral eye pain with blurred vision. She also notes nausea and vomiting. On examination the eye is red, the pupil is fixed in midposition, and slight opacification of the cornea is noted. The most likely diagnosis is:

A) Retinal detachment
B) Acute bacterial conjunctivitis
C) Acute angle-closure glaucoma
D) Cytomegalovirus infection
E) Optic neuritis
The correct answer is: C

The presentation is typical of acute angle closure glaucoma. Elevated pressure in the uveal tract produces pain and ciliary spasm, and blurred vision is due to corneal edema. Vagal stimulation may produce nausea and vomiting. Ref: Eye/ Goroll 4, p. 1079/ RAL
A middle-aged woman presents with complaints of throbbing, unilateral eye pain with blurred vision. She also notes nausea and vomiting. On examination the eye is red, the pupil is fixed in midposition, and slight opacification of the cornea is noted. The immediate management of this patient should include:

A) Topical pilocarpine
B) Pain medication
C) Acetazolamide (Diamox) orally or intravenously
D) Both A and B
E) All of the above
The correct answer is: E

Pilocarpine is a parasympathomimetic agent that causes the ciliary body to contract, facilitating drainage of the aqueous humor through the trabecular meshwork. Acetazolamide is a carbonic acid inhibitor that reduces the rate of production of the aqueous humor. Ref: Eye/ Goroll 4, p. 1081/ RAL
The most common presentation of open-angle glaucoma involves:

A) a complaint of loss of peripheral vision.
B) a visual field cut noted on physical exam.
C) eye pain radiating to the temporal area.
D) an asymptomatic patient noted to have increased intraocular pressure on screening exam.
E) ophthalmologic investigation for unexplained visual deterioration.
The correct answer is: D

The disease is asymptomatic in the early stages and can only be detected by routine screening, which should begin at age 20 in asymptomatic patients. Increased cupping of the optic disk may be noted on fundoscopy. Visual field cuts are a late sign. Ref: Eye/ Goroll 4, p. 1075-1076/ RAL
Floaters, or opacities seen moving across the visual field, may be associated with which of the following condition(s)?

A) Retinal detachment
B) Retinal tear
C) Cytomegalovirus retinitis
D) Intraocular hemorrhage
E) All of the above
The correct answer is: E

Floaters that are multiple and chronic tend to be benign, related to myopia and the aging process. The acute onset of a floater, however, may be associated with one of the more serious conditions listed above. Ref: Eye/ Goroll 4 p. 1089-1091/ RAL
The visual phenomena most likely to be associated with digitalis toxicity is:

A) Halos
B) Visual hallucinations
C) Yellow discoloration
D) Both A and C
E) All of the above
The correct answer is: D

Digitalis toxicity may cause yellow discoloration or halos. Acute glaucoma may also produce colored halos. Ref: Eye/ Goroll 4, p. 1090/ RAL
An elderly man presents with complaints of gradually decreasing central vision. On fundoscopy hard, yellow-white lesions are seen in the region of the macula. There is some neovascularization in the region. The most likely diagnosis is:

A) Age-related macular degeneration
B) Hypertensive retinopathy
C) Diabetic retinopathy
D) Early cataract
E) Cytomegalovirus infection
The correct answer is: A

Loss of central vision and the accumulation of debris or drusen in the area of the macula are typical. When neovascularization is present, patients may benefit from laser photocoagulation. Ref: Eye/ Goroll 4, p. 1082/ RAL
An elderly man presents with complaints of gradually decreasing central vision. On fundoscopy hard, yellow-white lesions are seen in the region of the macula. There is some neovascularization in the region. The patient returns some weeks later reporting sudden flashes of light, followed by a number of floaters, and a curtain-like defect in the visual field of one eye. The most likely diagnosis at this point is:

A) Retinal detachment
B) Hysterical conversion reaction
C) Keratitis
D) Uveitis
E) Acute angle-closure glaucoma
The correct answer is: A

Flashes of light, a shower of floaters, and a visual field defect are all seen in an acute retinal detachment. Ref: Eye/ Goroll 4, p. 1082/ RAL
What is (arE) the principal lesion(s) in proliferative diabetic retinopathy?

A) Microaneurysms
B) Neovascularization
C) Flame-shaped hemorrhages
D) Cotton wool infarcts
E) Hard exudates
The correct answer is: B

The formation of new vessels from areas of retinal ischemia into the vitreous cavity is the characteristic pathological change seen in late proliferative retinopathy. The other findings are characteristic of earlier, nonproliferative retinopathy. Ref: Eye/ Goroll 4, p. 1101; Harrison's 16, p. 2163-2164/ RAL
What is the recommended interval for ophthalmological screening for Type II diabetics?

A) 6 months
B) 1 year
C) 2 years
D) 5 years
E) Only as indicated by the development of new visual symptoms
The correct answer is: B

Current standards call for an annual screening by an ophthalmologist for Type II diabetics. Ref: Eye/ Goroll 4, p. 1101/ RAL
A patient presents after an industrial accident which has resulted in a small retained foreign body in the eye. Appropriate treatment by the primary care provider includes:

A) Irrigation with normal saline
B) Attempt to remove the foreign body with a dry sterile cotton-tipped applicator
C) Attempt to remove foreign body with a sterile 22-gauge needle
D) Measurement of intraocular pressure
E) None of the above
The correct answer is: A

An attempt to remove a foreign body by flushing with sterile saline solution is appropriate. If this is not successful, referral to an ophthalmologist is indicated. A dry cotton-tipped applicator will remove normal corneal epithelium. Intraocular pressure should never be measured in the presence or suspicion of external or penetrating foreign bodies. Ref: Eye/ Goroll 4, p. 1081, 1087/ RAL
In Type 1 diabetics, ophthalmological consultation should be obtained:

A) at the time of diagnosis.
B) at the onset of puberty.
C) five years after diagnosis of the disease.
D) only if visual symptoms are noted.
E) only when abnormal fundoscopic findings are noted.
The correct answer is: C

Type 1 diabetics should be referred to an ophthalmologist for evaluation five years after the onset of the disease. Ref: Eye/ Goroll 4, p. 1084/ RAL
Appropriate treatment in a patient with corneal abrasion includes all of the following EXCEPT:

A) A topical anesthetic may be used to facilitate examination.
B) A cycloplegic (eg., cyclopentolate 1%) should be instilled every 8 hours for pain relief.
C) Oral analgesics may be prescribed for symptomatic management.
D) A topical antibiotic ointment should be prescribed.
E) A topical anesthetic should be used every 8 hours as needed for pain relief.
The correct answer is: E

Repeated use of topical anesthetics may be toxic to the cornea. Ref: Eye/ Tintinalli 6, p. 1455/ RAL
Appropriate eye examination in a patient with a suspected ocular foreign body includes:

A) Visual acuity testing
B) Eversion of the eyelids
C) Direct fundoscopy
D) Both A and C
E) All of the above
The correct answer is: E

In addition, examination after fluorescein staining and slit-lamp examination should routinely be undertaken. If no foreign body is seen, plain X-rays and CT scanning may be used. MRI is contraindicated if a metallic foreign body is suspected. Ref: Eye, GR Schwartz, p. 144-147/ RAL
A left-sided lesion in the optic tract posterior to the optic chiasm produces:

A) total blindness in the right eye.
B) total blindness in the left eye.
C) a right temporal field cut.
D) left homonymous hemianopsia.
E) bitemporal hemianopsia.
The correct answer is: C

Lesions posterior to the chiasm produce homonymous hemianopsia of the contralateral visual field. Lesions anterior to the chiasm produce blindness in the ipsilateral eye. Lesions at the chiasm usually produce bitemporal hemianopsia. Ref: Eye/ Harrison's 16, p. 164-165/ RAL
Which is a common feature of hypertensive retinopathy?

A) Splinter hemorrhages
B) Drusen
C) Hollenhorst plaque
D) Cupping of the optic disc
E) All of the above
The correct answer is: A

Splinter hemorrhages occur in hypertensive retinopathy. Drusen are calcifications. Hollenhorst plaques are cholesterol emboli. Progressive cupping of the optic disc is a sign of glaucoma. Ref: Eye/ Harrison's 16, p. 168-171/ RAL
In assessing a patient with a history of blunt trauma to the eye, a flat anterior chamber signifies:

A) well-controlled intraocular pressure.
B) elevated intraocular pressure.
C) bleeding in the anterior chamber (hyphemA) .
D) a ruptured globe.
E) dislocation of the ciliary body.
The correct answer is: D

In the presence of a flat anterior chamber, the globe is ruptured. A metal shield should be placed over the eye and consultation with an ophthalmologist obtained. Ref: Eye/ Tintinelli 6, p. 1457/ RAL
Which statement is true regarding diabetic retinopathy?

A) It is the leading cause of blindness in the United States among persons under 65 years old.
B) Its progression can be slowed by tight glycemic control in Type I diabetics.
C) Its progression can be slowed by daily low-dose aspirin therapy.
D) Both A and B
E) All of the above
The correct answer is: D

Aspirin has not been shown to be beneficial. Ref: Eye/ Goroll 4, p. 1102/ RAL
Enlarged tonsil and anterior pharyngeal pillar are characteristic of this suppurative complication of streptococcal pharynitis

A) Vincent's angina
B) Ludwig angina
C) Retropharyngeal abscess
D) Peritonsillar abscess
The correct answer is: D

Typical physical findings in peritonsillar abscess include enlarged tonsil and anterior pharyngeal pillar as well as anterior cervical lymphadenopathy. The uvula is typically deviated away from the involved tonsil. Ref: ENT/ GR Schwartz, 926-927; Harrison's 16, p. 191/ RAL
This condition usually arises from dental abscess in a lower molar and has the potential to cause fatal airway obstruction. Patients may present with dysarthria and drooling.

A) Vincent's angina
B) Ludwig angina
C) Retropharyngeal abscess
D) Peritonsillar abscess
The correct answer is: B

Ludwig's angina is a life-threatening infection of the floor of the mouth causing a tender neck and swollen tongue. The immediate risk to life arises from the chance of sudden airway obstruction. Ref: ENT/ GR Schwartz, 926-927; Harrison's 16, p. 192-193/ RAL
This condition involves infection in a deep space that runs to the superior mediastinum. Patients may present with a bulge in the posterior pharynx and neck pain

A) Vincent's angina
B) Ludwig angina
C) Retropharyngeal abscess
D) Peritonsillar abscess
The correct answer is: C

Treatment of retropharyngeal abscess involves surgical drainage, after the airway is secured, and IV antibiotics. Ref: ENT/ GR Schwartz, 926-927; Harrison's 16, p. 193/ RAL
A fusiform bacillus of the Borrelia genus causes this condition. The uvula is deviated away from the lesion.

A) Vincent's angina
B) Ludwig angina
C) Retropharyngeal abscess
D) Peritonsillar abscess
The correct answer is: A

Vincent's angina, or trench mouth, is a severe gingival infection caused by Borrelia vicenti. It responds to oral penicillin. Ref: ENT/ GR Schwartz, 926-927; Harrison's 16, p. 192/ RAL
This condition produces a characteristic "dendritic" pattern on staining with fluorescein dye.

A) Uveitis
B) Viral conjunctivitis
C) Bacterial conjunctivitis
D) Herpes simplex keratitis
The correct answer is: D

Ocular HSV infection produces pain, lacrimation, and photophobia. A characteristic branching dendritic pattern is seen on fluorescein staining. Treatment is with topical or systemic antiviral medications. Ref: Eye/ Goroll 4, p. 1081; Tintinalli 6, p. 1453; Harrison's 16, p. 1038/ RAL
This ophthalmological condition commonly presents with a mucopurulent discharge.

A) Uveitis
B) Viral conjunctivitis
C) Bacterial conjunctivitis
D) Herpes simplex keratitis
The correct answer is: C

In bacterial conjunctivitis, the discharge is thick and purulent, and may cause the eyelids to stick together, most commonly in the morning. Both bacterial and viral conjunctivitis is highly contagious. Ref: Eye/ Goroll 4, p. 1077; Tintinalli 6, p. 1452/ RAL
Patients with this condition have a watery discharge and preauricular lymphadenopathy is common.

A) Uveitis
B) Viral conjunctivitis
C) Bacterial conjunctivitis
D) Herpes simplex keratitis
The correct answer is: B

In viral conjunctivitis, the discharge is typically thin, and enlargement of an ipsilateral preauricular node is often seen. Ref: Eye/ Goroll 4, p. 1077; Tintinalli 6, p. 1452/ RAL
This condition is often associated with autoimmune diseases. The patient may experience pain, photophobia, and ciliary flush.

A) Uveitis
B) Viral conjunctivitis
C) Bacterial conjunctivitis
D) Herpes simplex
The correct answer is: A

Ciliary flush (dilated vessels around the iris) and associated autoimmune disease, such as ankylosing spondylitis, sarcoidosis, or rheumatoid arthritis, may lead to the diagnosis of uveitis or iritis. Ref: Eye/ Goroll 4, p. 1078 RAL
What percent of uterine leiomyomas will eventually become malignant?

A) 25%
B) 10%
C) 5%
D) 1%
E) 0%
The correct answer is: D

Although approximately 50% of women will develop a leiomyoma during their lifetime, the risk of malignant degeneration is less than 1%. Although benign, leiomyomas may cause pain, bleeding or infertility. Ref: Repro/ Schwartz 8, p. 1578/ RAL
Which statement is TRUE concerning endometrial cancer?

A) Radical hysterectomy is the treatment of choice for this disease of any stage.
B) It is generally preferable to leave at least one ovary intact in premenopausal women.
C) Tumors are not generally sensitive to radiation therapy.
D) It is the most common malignant tumor of the female genitalia.
E) Late menarche is a risk factor.
The correct answer is: D

Radical hysterectomy does not improve survival in comparison with less radical procedures (simple, extra-fascial, or complete abdominal hysterectomy) and radiotherapy. Bilateral salpingo-oophorectomy is usually performed, as 5% of ovaries will harbor metastatic disease. Early menarche and late menopause are risk factors, along with diabetes, hypertension, low parity, obesity, and advanced age. Ref: Repro/ Schwartz 8, p. 1585-1588/ RAL
The most common symptom of endometrial cancer is which of the following?

A) Abdominal pain
B) Postmenopausal bleeding
C) Urinary frequency
D) Purulent vaginal discharge
E) Uterine prolapse
The correct answer is: B

Postmenopausal bleeding may be the only early symptom. In some cases a mass may be palpable on routine pelvic exam. Ref: Repro/ Goroll 4, p. 735/ RAL
Which statement is true concerning ovarian carcinoma?

A) It tends to present with symptoms early in the course of the disease, facilitating treatment.
B) Effective screening programs have reduced mortality due to ovarian carcinoma.
C) Vaginal bleeding is a frequent presenting symptom.
D) Epithelial ovarian cancer is most common in the 75 to 80-year old range.
E) In premenopausal woman, most palpable adnexal masses represent malignant disease.
The correct answer is: D

Ovarian tumors do not generally produce specific symptoms and so tend to present late in their course. There are currently no effective technologies for screening. Vaginal bleeding or discharge are rare. Most tumors (85%) are of the epithelial type, with the peak incidence in the eighth decade. In premenopausal women, most palpated adnexal masses are benign functional cysts.. Ref: Repro/ Harrison's 16, 553-554/ RAL
Which of the following is not a risk factor for breast cancer?

A) A family history of breast cancer
B) Late menopause
C) A history of benign breast disease
D) Early menarche
E) The use of oral contraceptives
The correct answer is: E

Oral contraceptives do not appear to increase the risk of breast cancer. Family history, late menopause, early menarche, and benign breast disease are all risk factors. Ref: Repro/ Goroll 4, p. 667-668; Harrison's 16, p. 517/ RAL
Current, generally accepted recommendations for screening mammography offer the test to:

A) all adult women on an annual basis.
B) women over 30 with 2 or more risk factors, to be given every 1 to 2 years.
C) any woman over 30, when insurance coverage is available.
D) all woman 50-74 years old, at 1 to 2 year intervals.
E) woman over 60 with large, pendulous breasts that are difficult to examine.
The correct answer is: D

It has been demonstrated that annual mammography improves outcomes in women in this age range. Screening choices may be individualized in other situations. Ref: Repro/ Goroll 4, p. 668-670/ RAL
The daughters of women exposed to diethylstilbestrol (DES) during pregnancy are more likely to develop:

A) breast cancer.
B) ovarian cancer.
C) vaginal cancer.
D) invasive cervical cancer.
E) None of the above
The correct answer is: C

DES exposure increases risk of clear cell carcinoma of the vagina, but so far, no increased risk of any other malignancy has been observed. Ref: Repro/ Goroll 4, p. 678-680/ RAL
Among women experiencing otherwise normal menstrual cycles, the most common reason for abnormal uterine bleeding is:

A) the presence of an intrauterine device (IUD) .
B) submucosal fibroids.
C) underlying bleeding diathesis.
D) carcinoma of the cervix.
E) cervical erosions.
The correct answer is: B

Although all are potential causes of abnormal vaginal bleeding in women with otherwise normal cycles, leiomyomas, or uterine fibroids, are the most common cause. Ref: Repro/ Goroll 4, p. 681/ RAL
Prolonged bleeding occurring at irregular intervals is known as:

A) Metrorrhagia
B) Menorrhagia
C) Breakthrough bleeding
D) Mittelschmerz
E) Bleeding diathesis
The correct answer is: A

Metrorrhagia occurs at irregular intervals and is a symptom of disturbance in the hyothalamic-pituitary-ovarian axis. Menorrhagia simply refers to abnormally heavy menstrual bleeding. Breakthrough bleeding is vaginal bleeding between menses caused by inadequate estrogens in a particular formulation of oral contraceptive

or by failure of compliance with the medication. Mittelschmerz refers to small amounts of vaginal bleeding and pelvic pain at the time of ovulation. A bleeding diathesis is any abnormality of the platelets or coagulation system that causes unusual bleeding. Ref: Repro/ Goroll 4, p. 681-682/ RAL
Risk factors for ectopic pregnancy include:

A) History of previous pelvic inflammatory disease (PID) B) History of use of an intrauterine device (IUD) for contraception
C) History of previous tubal surgery to increase fertility
D) History of use of drugs to enhance ovulation
E) All of the above
The correct answer is: E

All choices represent significant risk factors. In addition, previous pelvic surgery and a past history of ectopic pregnancy are risk factors. The incidence of ectopic pregnancy overall is increasing, while the rate of fatality is decreasing, probably due to better diagnostic tools. Ref: {Repro/ Goroll 4, p. 699/ RAL
A 28-year old woman, with a history of PID 6 years ago, presents to the Emergency Department with a 12-hour history of progressive abdominal pain. Her last menstrual period was 8 weeks ago, but she has had intermittent, scant dark brown vaginal discharge for several weeks. The patient is hemodynamically stable. She is afebrile. On abdominal exam you discover diffuse tenderness with peritoneal signs. Pelvic exam reveals tenderness to motion of the cervix and a vague adnexal mass on the right. The patient assures you that she could not be pregnant. The most important diagnostic tests at this point are:

A) Supine and upright abdominal films and serum electrolytes
B) Abdominal CT scan and liver function studies
C) Serum electrolytes and culdocentesis
D) Serum human chorionic gonadotropin (hCG) and pelvic ultrasound
E) Serum hCG and CT scan of the abdomen and pelvis
The correct answer is: D

The presentation is suggestive of an ectopic pregnancy. The patient's denial of pregnancy cannot be accepted as definitive. A normal hCG rules out the possibility of ectopic pregnancy. If the hCG is positive, ultrasound very likely would be helpful in locating an ectopic pregnancy or ruling in an intrauterine pregnancy. Ref: Repro/ Tintinalli 6, p. 494-495/ RAL
The most important physical finding in the diagnosis of carcinoma of the breast is:

A) a dominant breast mass.
B) axillary lymphadenopathy.
C) breast tenderness.
D) mobility of the breast mass.
E) irregularity of the border of the breast mass.
The correct answer is: A

The single most important finding is the presence of a dominant nodule, regardless of its mobility, regular or irregular contour, or the presence of axillary lymph nodes. Tenderness, if no associated mass is present, is not a useful physical finding in terms of the diagnosis or suspicion of breast cancer. Ref: Repro/ Goroll 4, p. 692/ RAL
Which of the following antihypertensive agents are appropriate for use in a woman who develops hypertension during pregnancy?

A) Hydralazine
B) Metoprolol
C) Propranolol
D) Lisinopril
E) All of the above
The correct answer is: A

Hydralazine and methyldopa are regarded as safe at any stage of pregnancy. Beta-blockers (such as propranolol and metoprolol) should be avoided in the first trimester, due to possible growth retardation of the fetus. ACE-inhibitors (such as lisinopril) and angiotensin-receptor blockers (ARB's) are contraindicated due to a possible adverse impact on placental blood flow. Ref: Repro/ Gorill 4, p. 158/ RAL
In a pregnant woman at 20 or greater weeks of gestation, which of the following findings suggests severe preeclampsia?

A) Blood pressure greater than 160/110
B) Oliguria
C) Headache
D) Visual disturbances
E) All of the above
The correct answer is: E

In addition, symptoms suggestive of severe preeclampsia include generalized edema, elevated liver function studies, thrombocytopenia, or hemoglobinuria. Ref: Repro/ Harrison's 16. p. 32-33/ RAL
Risk factors for preeclampsia include all of the following EXCEPT:

A) Multiparity
B) Pre-existing chronic essential hypertension
C) Diabetes mellitus (DM) D) Maternal age greater than 35 years
E) Maternal age less than 15 years
The correct answer is: A

Nulliparas are at increased risk. Chronic hypertension, DM, and both extremes of maternal age are also risk factors. Ref: Repro/ Harrison's 16, p. 32-33/ RAL
The agent(s) of choice for the prevention or control of seizures in hypertensive disease of pregnancy (preeclampsia or eclampsiA) is (arE) :

A) Diazepam
B) Phenytoin
C) Magnesium sulfate
D) A or C are equally effective
E) All are equally effective
The correct answer is: C

Large clinical trials have established magnesium sulfate as the agent of choice in preeclampsia or eclampsia of pregnancy. Ref: Repro/ Harrison's 16, p. 33/ RAL
Immediate treatment of hypertension is indicated in preeclampsia/eclampsia when the diastolic pressure is equal to or greater than 110mmHg. The agent of choice is which of the following?

A) Bedrest and salt restriction are usually adequate
B) Oral captopril (Capoten) C) Intravenous hydralazine (ApressolinE) D) Intravenous furosemide (Lasix) E) Intravenous mannitol
The correct answer is: C

Hydralazine is the most frequent choice in this setting. Intravenous beta-blockers (for example, labatalol) may also be used. Diuretics and salt-restriction should be avoided, as extravascular volume is already contracted. Ref: Repro/ Harrison's 16, p. 33; Goroll 4, p. 158/ RAL
A patient early in the third trimester of pregnancy, who has had a prior cesarean section, presents with an episode of painless, bright-red vaginal bleeding. You estimate that her total blood loss has been no more than 100ml, and she is hemodynamically stable. The best next step in her diagnostic work-up would be:

A) Immediate emergency cesarean section
B) Bimanual pelvic examination
C) Visual examination of the cervix using a plastic speculum
D) Culture of the cervical os
E) Emergency abdominal ultrasound
The correct answer is: E

In placenta previa, the placenta overlies the cervical os. It is most common in nullipara or women who have had a previous cesarean section. Pelvic examination can precipitate exsanguinating hemorrhage in a previously stable patient and should be avoided. The diagnosis can be accurately ascertained or ruled out by ultrasound. Ref: Repro/ Tintinalli 6, p. 680/ RAL
Which statement is TRUE regarding abruptio placentae?

A) Patients present with hyperactive uterine contractions.
B) All patients will have some degree of vaginal bleeding.
C) It is usually precipitated by trauma.
D) In cases of hypovolemic shock, the mother should be stabilized and vaginal delivery attempted.
E) None of the above
The correct answer is: A

Abruptio placentae is premature separation of the placenta from the uterine wall. Hemorrhage may be confined within the uterus or "concealed". Trauma is implicated in a minority of cases. When there is no sign of fetal distress or maternal instability, vaginal delivery can be attempted. Vaginal delivery may also be attempted when the fetus is nonviable and the mother is stable. In cases of fetal distress or maternal instability, cesarean section is the treatment of choice. Ref: Repro/ DeCherney 9, p. 357-362; Tintinalli 6, p. 679/ RAL
The first stage of labor:

A) begins with full dilatation of the cervix and ends with delivery of the placenta.
B) increases in length with multiparity.
C) begins with the onset of true labor and ends with full dilatation of the cervix.
D) lasts about 2 hours in the primipara.
E) ends with delivery of the infant.
The correct answer is: C

The second stage extends from full dilatation to birth. The third stage is the period from birth to delivery of the placenta. The first stage lasts about 8 hours in the primipara and is more brief in the multipara. Ref: Repro/ DeCherney 9, p. 213-214 / RAL
The most common cause of spontaneous abortion is:

A) Fetal abnormality
B) Maternal diabetes mellitus
C) Trauma
D) Maternal psychological stress
E) Abnormalities of the placenta
The correct answer is: A

Genetic abnormalities account for 50-60% of all spontaneous abortions. Ref: Repro/ Tintinalli 6, p. / RAL
Which is the most common cause of vaginal bleeding in late pregnancy?

A) Extrusion of cervical mucus
B) Uterine rupture
C) Placenta previa
D) Abruptio placentae
E) Disseminated intravascular coagulopathy (DIC)
The correct answer is: A

Extrusion of cervical mucus ("bloody show") rarely requires treatment and is the most common. The other choices, less common causes of bleeding, may be life-threatening emergencies. Ref: Repro/ DeCherney 9, p. 354-355/ RAL
The most common procedure used in induced abortions in the United States is which of the following?

A) Suction curettage
B) Surgical curettage
C) Intra-amniotic instillation of hypertonic saline
D) Intra-amniotic instillation of hyperosmolar urea
E) Dilatation and evacuation
The correct answer is: A

90% of abortions in the United States are performed by suction curettage. Ref: Repro/ DeCherney 9, p. 646/ RAL
Torsion of the testis usually presents as:

A) chronic bilateral aching testicular pain.
B) acute urinary retention.
C) acute unilateral testicular pain.
D) a consideration in an infertility evaluation.
E) a complaint following a history of multiple trauma.
The correct answer is: C

The pain in testicular torsion is acute and unilateral, and it may be continuous or intermittent. Doppler ultrasound or nuclear imaging may be helpful in establishing the diagnosis. Torsion is an ischemic event, and delay in treatment may result in necrosis and infertility. Ref: Repro/ Tintinalli 6, p. 616-617; Goroll, p. 759/ RAL
A 70-year-old man presents with a complaint of the gradual onset of right-sided testicular pain over several days. He notes that the pain is somewhat relieved in the supine position. On examination you note scrotal and testicular tenderness and, on rectal examination, tenderness of the prostate. The most likely diagnosis at this point is:

A) Torsion of the testes
B) Referred pain from a ureteral stone
C) Epididymitis
D) Varicocele
E) Primary testicular cancer
The correct answer is: C

Infection of the epididymis from involved adjacent structures, including the prostate, is common. Torsion presents with a more acute onset of pain. The testes are not tender to palpation when pain is referred from the ureters. Varicocele and testicular tumors are not generally tender, although acute hemorrhage into a testicular tumor may present as acute testicular pain. Ref: Repro/ Tintinalli 6, p. 617-617; Goroll 4, p. 759-760/ RAL
The most common malignancy in men between the ages of 15 and 35 years old is:

A) Osteosarcoma
B) Malignant melanoma
C) Basal cell carcinoma
D) Germ cell tumor of the testis
E) Renal cell carcinoma
The correct answer is: D

Germ cell tumors of the testes are the most common malignant tumors in this age group. They typically present as a painless scrotal mass and are treated with orchiectomy, radiation, and sometimes chemotherapy. Cure rates for early-stage disease approach 100% and are above 90% even for late-stage disease. Ref: Repro/ Goroll 4, p. 816-817; Schwartz 8, p. 1538/ RAL
Which is typical of endometriosis?

A) Tender nodules in the posterior vaginal fornix
B) Dyspareunia in between periods
C) Pain relieved with the onset of menstrual flow
D) Adnexal tenderness or mass
E) All of the above
The correct answer is: E

All findings suggest endometriosis. Ref: Repro/ DeCherney 9, p. 770/ RAL
Which combination poses the greatest risk to the fetus of Rh isoimmunization?

A) Rh-negative mother, heterozygous Rh-positive father
B) Rh-positive mother, heterozygous Rh-positive father
C) Rh-positive mother, homozygous Rh-positive father
D) Rh-negative mother, homozygous Rh-positive father
E) Rh-negative mother, Rh-negative father
The correct answer is: D

The risk of isoimmunization occurs only when the mother is Rh-negative and the father is Rh-positive. If the father is homozygous for D, the major Rh antigen, the fetus will certainly be positive; if the father is heterozygous, there is a 50% chance that the fetus will be negative, and Rh incompatibility will not be problematical. Ref: Repro/ DeCherney 9, p. 296/ RAL
The major risk of hormone replacement therapy (HRT) in postmenopausal women is:

A) Hot flashes
B) Endometrial cancer
C) Osteoporosis
D) Alzheimer's disease
E) Gallbladder disease
The correct answer is: B

The risk of endometrial cancer is tripled in short-term HRT and may be increased by a factor of ten in long-term therapy. Hot flashes and other vasomotor symptoms are improved by HRT. Osteoporosis is improved. The effect on Alzheimer's disease is uncertain. Gallbladder disease may be worsened. Other major risks of HRT are breast cancer and venous thromboembolism. Ref: Repro/ Goroll 4, p. 709; Harrison's 16, p. 2210-2213/ RAL
The most common cause of maternal death in the United States is which of the following?

A) Septic abortion
B) Thromboembolic disease
C) Hypertensive diseases
D) Abruptio placentae
E) Placenta previa
The correct answer is: B

Thromboembolic disease is the most common cause. Hypertensive disorders are in second place. Ref: Repro/ Tintinalli 6, p. 678/ RAL
This organism causes a frothy, foul-smelling vaginal discharge, vulvar edema, and petechiae is often seen. Occasionally "strawberry cervix" is noted. This is a sexually transmitted disease. The pathogen may be seen on wet-mount microscopic exam.

A) Chlamydia trachomatis
B) Trichomonas vaginalis
C) Candida albicans
D) Gardnerella vaginalis and other organisms
The correct answer is: B

The motile flagellated protozoan Trichomonas vaginalis can be seen on a slide prepped with normal saline. Ref: Repro/ Goroll 4, p. 703/ RAL
This organism produces a thick, white, "cottage-cheese" discharge; pathogen may be seen on KOH prep

A) Chlamydia trachomatis
B) Trichomonas vaginalis
C) Candida albicans
D) Gardnerella vaginalis and other organisms
The correct answer is: C

The "cheesy" discharge, examined after exposure to 10% KOH reveals hyphae and budding yeast, typical of Candida albicans. Ref: Repro/ Goroll 4, p. 703-704/ RAL
This organism is the cause of a thick, yellow-white discharge seen at the cervical os; Gram's stain gives indirect evidence of this pathogen.

A) Chlamydia trachomatis
B) Trichomonas vaginalis
C) Candida albicans
D) Gardnerella vaginalis and other organisms
The correct answer is: A

The discharge is seen exuding from the cervical os. Friability and ulceration of the cervix may be seen. Gram's stain with more than 10 leukocytes per high-power field suggests the diagnosis. Ref: Repro/ Goroll 4, p. 703/ RAL
This organism produces a thin white discharge; "clue cells" can be seen on wet-mount exam

A) Chlamydia trachomatis
B) Trichomonas vaginalis
C) Candida albicans
D) Gardnerella vaginalis and other organisms
The correct answer is: D

On wet-prep motile bacilli can be seen adherent to vaginal epithelial cells-so called "clue cells". Ref: Repro/ Goroll 4, p. 703-704/ RAL
Which statement is TRUE concerning Type 2 diabetes mellitus?

A) Type 2 DM usually presents in children and young adults.
B) It is about as frequent in the adult population as Type 1 DM.
C) Patients with type 2 DM are prone to ketoacidosis if oral hypoglycemics are withheld.
D) It involves some degree of insulin resistance.
E) It develops from autoimmune beta cell destruction.
The correct answer is: D

Type 2 DM usually develops in older adults, but may present in obese adolescents. The prevalence of type 2 DM is greater and is rising faster than type 1. Type 2 patients are unlikely to develop ketoacidosis. The pathophysiology always involves some degree of insulin resistance, and unlike type 1, does not primarily involve the destruction of pancreatic beta cells. Ref: Endo/ Harrison's 16, p. 2152-2155/ RAL
A 55-year-old male type 2 diabetic fails attempts at adequate glycemic control with diet alone or with diet and oral agents. Concerning general guidelines to be observed in initiating treatment with subcutaneous insulin, which statement is TRUE?

A) 10-15 units of intermediate -acting insulin is a reasonable starting regimen.
B) The patient should be hospitalized for the initiation of therapy.
C) All oral agents should be discontinued when subcutaneous insulin administration begins.
D) Obese patients are less likely to be resistant to insulin.
E) Counsel the patient that insulin must not be stored at room temperature for more than 2 hours.
The correct answer is: A

Most patients can be managed on an outpatient basis. A modest dose of NPH insulin can be added to their oral hypoglycemic regimen. Obese patients are more likely to experience insulin resistance. Insulin can be stored at room temperature for up to 12 hours. Ref: Endo/ Goroll 4, p. 638-639/ RAL
Which of the following clinical and laboratory criteria are adequate to establish the diagnosis of diabetes mellitus?

A) Random plasma glucose of 200 mg/dL or greater in a patient with symptomatic hyperglycemia (polydipsia, polyuria, polyphagiA) B) Fasting plasma glucose of at least 126mg/dL
C) Serum glucose of at least 200mg/dL 2 hours following a 75g oral glucose load
D) None of the above
E) All of the above
The correct answer is: E

A random glucose greater than 200mg/dL in a symptomatic patient is diagnostic. Fasting glucose greater than 126mg/dL is diagnostic, but should be repeated in a different day unless "unequivocal hyperglycemia" is present. The diagnosis may be established by an oral glucose tolerance test in which the 2-hour plasma glucose is 200mg/dL, although the National Diabetes Data Group and the World Health Organization no longer recommend this method of testing. Ref: Endo/Harrison's 16, p. 2153/ RAL
In general, when is the best time to give regular insulin?

A) 2 hours before a meal
B) 30-45 minutes before a meal
C) At meal time
D) 30-45 minutes after a meal
E) 2 hours after a meal
The correct answer is: B

Regular insulin is usually dosed about 30-45 minutes before meals. Ref: Endo/ Harrison's 16, p. 2173/ RAL
Which oral agent should be avoided in patients with significant renal insufficiency?

A) Chlorpropamide (DiabinesE) B) Metformin (GlucophagE) C) Glyburide (DiaBeta, GlynasE) D) Acarbose
E) All of the above
The correct answer is: E

Chlorpropamide and glyburide are subject, at least in part, to renal clearance. Metformin may cause lactic acidosis in patients with renal insufficiency and is contraindicated in such patients. Acarbose is contraindicated in patients with a serum creatinine greater than 2.0 mg/dL. Ref: Endo/ Harrison's 16, p. 2175-2176/ RAL
Which symptom is not generally related to diabetic autonomic neuropathy?

A) Impotence
B) Diarrhea
C) Nausea and vomiting
D) Urinary incontinence
E) Orthostatic hypotension
The correct answer is: D

Diabetic autonomic dysfunction produces urinary retention. Decreased gastrointestinal motility may produce overgrowth of colonic bacteria (diarrheA) and delayed emptying of the stomach (nausea and vomiting). Impotence is the most common symptom of autonomic dysfunction in diabetic men. Postural hypotension is also common. Ref: Endo/ Goroll 4, p. 641/ RAL
Measured hemoglobin A1C (HbA1C) :

A) is the best means of determining glycemic control over time.
B) correlates with glycemic control over the preceding 2 to 3 months.
C) is usually less than 8% if the serum glucose has generally been less than 200mg/dL.
D) Both A and B
E) All of the above
The correct answer is: E

HbA1C levels greater than 11-12% correlate with serum glucose levels generally over 300mg/dL and suggest inadequate glycemic control over the previous several months. Ref: Endo/ Goroll 4, p. 642/ RAL
Which statement is TRUE concerning diabetic nephropathy?

A) Angiotensin-converting enzyme (ACE) inhibitors should be avoided.
B) Annual selective renal arteriography may be helpful.
C) Progression of proteinuria to greater than 3g per day suggests poorer prognosis.
D) Both A and C
E) All of the above
The correct answer is: C

ACE inhibitors are helpful in preserving renal function. Contrast studies should be avoided in diabetics, if possible, as they may worsen renal insufficiency. Progression of proteinuria is an ominous prognostic sign. Ref: Endo/ Goroll 4, p. 640/ RAL
Potential causes of increasing hyperglycemia in an insulin-dependent diabetic include:

A) Occult infection
B) The Somogyi phenomenon
C) Insulin resistance
D) Both A and C
E) All of the above
The correct answer is: E

Occult infection, emotional stress, or coronary ischemia may raise the serum glucose. The Somogyi phenomenon results when patients experience rebound hyperglycemia following episodes of hypoglycemia. Insulin resistance is most usually due to obesity, but may represent a true immunologic reaction to a specific type of insulin. Ref: Endo/ Goroll 4, p. 639/ RAL
Which statement regarding exercise in the diabetic patient is true?

A) Exercise increases glucose production and raises serum glucose.
B) Exercise may increase insulin sensitivity.
C) Subcutaneous injection of insulin in the exercising limb may delay the absorption of insulin.
D) Exercise may increase insulin resistance.
E) All of the above are true.
The correct answer is: B

Subcutaneous injection into a limb that has been or will be involved in vigorous exercise may result in unusually rapid absorption of insulin, resulting in hypoglycemia. Exercise increases glucose absorption and lowers serum glucose. It improves insulin sensitivity and decreases insulin resistance. Ref: Endo/ Goroll 4, p. 633/ RAL
The agent of choice for controlling diabetes in pregnant women is:

A) Insulin
B) Glipizide
C) Metformin
D) Pioglitazone
E) Glyburide
The correct answer is: A

Oral agents have not been approved for use in pregnancy. Therapy in pregnant women consists of medical nutrition therapy and insulin administration. Ref: Endo/ Harrison's 16, p. 2179/ RAL
Which of the following is the MOST LIKELY pattern in diabetic peripheral neuropathy?

A) Peripheral mononeuropathy
B) Isolated cranial neuropathy
C) Distal symmetric polyneuropathy
D) Autonomic neuropathy
E) Polyradiculopathy
The correct answer is: C

All the listed neuropathic patterns occur, but distal symmetric neuropathy with pain, paresthesias, and eventually, sensory loss, is the most common pattern. Ref: Endo/ Harrison's 16, p. 2165-2166/ RAL
Which statement is TRUE regarding Grave's disease?

A) It is the most common cause of hyperthyroidism.
B) It is due to an acute viral infection.
C) Ophthalmopathy (true proptosis) is rare.
D) Typically the thyroid gland is tender to palpation.
E) Decreased uptake on the thyroid scan is typical.
The correct answer is: A

Grave's disease is an autoimmune condition not clearly associated with a viral etiology. About 40% of patients have ophthalmopathy. A painless, diffusely enlarged gland is apparent on examination. Thyroid scan shows diffusely increased uptake. Ref: Endo/ Goroll 4, p. 646-649/ RAL
Which statement is TRUE concerning lab testing for hyperthyroidism?

A) A normal serum TSH (thyroid stimulating hormonE) reliably rules out hyperthyroidism.
B) Both T3 and T4 should be obtained for screening.
C) Free T3 is the best measure of circulating thyroid hormone.
D) If TSH levels are undetectable, no further laboratory testing is indicated.
E) All of the above are true.
The correct answer is: A

TSH is usually low to undetectable in hyperthyroidism, and a normal TSH is a reliable indicator that hyperthyroidism is not present. TSH assay alone, when TSH is undetectable, establishes the diagnosis of hyperthyroidism, but confirmation by measurement of either free T4, or free thyroxine index (FT4I) is recommended. T3 is rarely necessary. Ref: Endo/ Harrisonäó»s 16, p. 2108; Goroll 4, p. 648/ RAL
Clinical findings typical in hyperthyroidism include all of the following EXCEPT:

A) Sinus tachycardia
B) Lid retraction and widened palpebral fissure
C) Fine tremor
D) Constipation
E) Palmar erythema
The correct answer is: D

Transit time in the gastrointestinal tract is decreased, often leading to diarrhea, not constipation. The other findings are typical. Ref: Endo/ Harrison's 16, p. 2114; RAL
Which manifestation(s) of Grave's disease would NOT be likely to improve with propranolol?

A) Tremor
B) Tachycardia
C) Atrial fibrillation
D) Heat intolerance
E) Ophthalmopathy
The correct answer is: E

The adrenergic manifestations of thyrotoxicosis (Answers A through D) would all tend to resolve; ophthalmopathy, which occurs by a different mechanism, would not likely benefit. Ref: Endo/ Harrison's 16, p. 2116; Goroll 4, p. 649-650/ RAL
The principal benefit of the antithyroid drugs, methimazole and propylthiouracil, lies in their ability to:

A) Blunt the response of tissues to the thyroid hormone
B) Block the release of TSH
C) Block the synthesis of T4
D) Both A and C
E) All of the above
The correct answer is: C

Both agents block the synthesis of the thyroid hormone. Propylthiouracil also interferes with the conversion of T4 to T3. Ref: Endo/Goroll 4, p. 650/ RAL
In a patient treated with antithyroid drugs for hyperthyroidism, the effect of treatment should be monitored by:

A) TSH
B) Unbound T4
C) Unbound T3
D) Clinical response only
E) Total T3
The correct answer is: B

Unbound T4 should be checked 3-4 weeks after initiating therapy. Ref: Endo/ Harrison's 16, p. 2116/ RAL
A patient with untreated hyperthyroidism requires an emergency peripheral vascular surgical procedure. Postoperatively he becomes irritable, disoriented, and combative. He develops a fever of 41ŒÁC, sinus tachycardia, and later, he becomes hypotensive. You recognize the possibility of thyroid storm. Which of the following agents are appropriate for immediate management of this patient?

A) Propylthiouracil
B) Aspirin
C) Propranalol
D) A and C
E) All of the above
The correct answer is: D

Propylthiouracil and propranalol, along with glucocorticoids, and iodine compounds (to suppress the release of thyroid hormonE) are the mainstays of treatment. Aspirin may result in an increase of free T4 and should be avoided. Ref: Endo/ Harrison's 16, p. 2117; Schwartz 8, p. 1410; Tintinalli 6, p. 1312-1313/ RAL
Which is the most common cause of hypothyroidism in the United States?

A) Hashimoto's thyroiditis
B) Lithium therapy
C) Postpartum state
D) Post surgical hypothyroidism following subtotal thyroidectomy
E) Iodine deficiency
The correct answer is: A

Primary hypothyroidism accounts for 95% of hypothyroidism, and Hashimoto's thyroiditis, an autoimmune disease, is the most common cause of primary hypothyroidism in populations in which dietary iodine is adequate. Secondary hypothyroidism implies a deficiency of TSH production, most commonly as a result of a pituitary adenoma. All choices are known causes of primary hypothyroidism. Worldwide, iodine deficiency is the most common cause of primary hypothyroidism. Ref: Endo/ Harrison's 16, p. 2109; Goroll 4, p. 655/ RAL
All of the following are symptoms of hypothyroidism except:

A) Cold intolerance
B) Dry skin
C) Tremor
D) Fatigue
E) Hoarseness
The correct answer is: C

Tremor is associated with hyperthyroidism; the other choices are typical of hypothyroid states. Ref: Endo/ Gortoll 4, p. 655/ RAL
A 35-year-old woman presents complaining of fatigue, hoarseness, amenorrhea, weight gain (despite decreased appetitE) and depression. You suspect hypothyroidism and obtain a TSH level, which is, however, low. On further evaluation, you obtain a free T4 level, which is also well below the lower limit of normal. An appropriate next step in the work-up is:

A) Total T3 level
B) CT scan of the abdomen and pelvis
C) Antimicrosomal antibodies
D) MRI of the head
E) Free thyroxine index
The correct answer is: D

Clinical hypothyroidism with a low TSH suggests secondary hypothyroidism, most likely to be due to a pituitary adenoma. An MRI of the head is a reasonable next step in the evaluation of this patient. Ref: Endo/ Goroll 4, p. 657/ RAL
A 60-year-old woman is treated for primary hypothyroidism with replacement therapy and is clinically and chemically euthyroid 6 months later. The most appropriate follow-up plan is which of the following?

A) Follow-up only as needed for recurrent symptoms.
B) Follow-up in one month for repeat T4 determination.
C) Follow up in 4 to 6 weeks for repeat TSH and T4 determination.
D) Follow-up in 6 months for repeat TSH only.
E) Follow-up in one year for repeat T3, T4, and TBG.
The correct answer is: D

Follow-up should occur after the replacement dose is established and thereafter consists of TSH determination every 6 to 12 months. Ref: Endo/ Goroll 4, p. 659/ RAL
Which is the most common primary malignant tumor of the thyroid?

A) Anaplastic carcinoma
B) Papillary carcinoma
C) Follicular carcinoma
D) Hurthle cell carcinoma
E) Medullary thyroid carcinoma
The correct answer is: B

Papillary carcinoma accounts for 70-90% of all thyroid malignancies. Medullary thyroid carcinomas account for about 5% of thyroid malignancy and arise from parafollicular cells. Anaplastic carcinoma, papillary carcinoma, and follicular carcinoma all arise from thyroid follicular epithelium. Hurthle cell tumors are an aggressive subtype of follicular carcinoma. Ref: Endo/ Harrison's 16, p. 2124-2126; Schwartz 8, p. 1416-1426 / RAL
In a patient presenting with a solitary thyroid nodule, which factors increase the likelihood that the patient has a thyroid malignancy?

A) Family history of thyroid cancer
B) A nodule that is hard to palpation
C) Previous history of radiation exposure
D) A nodule that is fixed to adjacent structures
E) All of the above
The correct answer is: E

A history or previous exposure to ionizing radiation increases the risk of malignancy. While about 15% of solitary nodules are malignant, this increases to between 30% and 45% if the nodule feels hard on palpation. Ref: Endo/ Schwartz 8, p. 1413-1415/ RAL
A 55-year-old woman presents for routine physical examination and is noted to have a single palpable thyroid nodule. Her TSH is found to be low, and she is referred for a thyroid scan. The scan shows increased uptake in the nodule and decreased function in the remainder of the gland. The most likely diagnosis at this point is:

A) Toxic adenoma
B) Pappilary thyroid cancer
C) Sporadic hyperparathyroidism
D) Follicular thyroid cancer
E) Anaplastic thyroid cancer
The correct answer is: A

A solitary nodule with focal uptake on radionuclide scan (a "hot" nodulE) establishes the diagnosis of toxic adenoma and rules out malignancy. Ref: Endo/ Harrison's 16, p. 2122/ RAL
Which is NOT a symptom of hypoglycemia?

A) Lethargy
B) Seizure
C) Transient monocular visual loss
D) Tremor
E) Diaphoresis
The correct answer is: C

Transient monocular visual loss is usually associated with retinal ischemia. Lethargy and seizure are symptoms associated with the effect of hypoglycemia on the CNS, while tremor and diaphoresis are among the symptoms of hypoglycemia related to adrenergic stimulation. Ref: Endo/ Harrison's 16, p. 2181/ RAL
Which of the following is the most common cause of spontaneous hypoglycemia?

A) Gastric surgery
B) Insulinoma
C) Alcohol abuse
D) Excessive intake of oral hypoglycemic agents or insulin
E) Excessive intake of concentrated sweets
The correct answer is: D

Excessive doses of glucose-lowering medications are the most common cause of hypoglycemia. The other choices are less common causes of spontaneous or reactive hypoglycemia. Ref: Endo/ Harrison, p. 218/ RAL
Which glucose determination is most useful in establishing a diagnosis of reactive hypoglycemia?

A) Serum glucose obtained while the patient is symptomatic
B) Fasting serum glucose
C) Random serum glucose
D) 2-hour postprandial serum glucose
E) 5-hour glucose tolerance test
The correct answer is: A

While there is no established glucose level associated with hypoglycemic symptoms, a normal serum glucose level in a symptomatic patient would suggest some cause other than hypoglycemia. Establishing the diagnosis of reactive hypoglycemia requires that typical symptoms be documented; that low plasma glucose be demonstrated during the symptoms; and that symptoms be relieved by raising the plasma glucose

the three components of Whipple's triad. Ref: Endo/ Harrison's 16, p. 2180-2183/ RAL
Which is most typical of acute gouty arthritis?

A) Insidious onset over days or weeks
B) Symmetrical polyarticular symptoms with morning stiffness
C) Intense pain, tenderness, and erythema of a single joint
D) Asymmetrical polyarticular symptoms with erythema and joint swelling
E) Any of the above
The correct answer is: C

Acute gouty arthritis is usually monoarticular and of a sudden onset. The involved joint is exquisitely tender and swollen, and the overlying skin is inflamed. Ref: Endo/ Harrison's 16, p. 2046/ RAL
Which is the cystal deposited in joints during an episode of gouty arthritis?

A) Monosodium urate (MSU) B) Calcium pyrophosphate (CPPD) C) Calcium hydroxyapatite (HA) D) Calcium oxalate (CaOx) E) None of the above
The correct answer is: A

MSU crystals are seen on polarized light microscopy of the aspirated fluid in acute gouty arthritis. CPPD causes pseudogout. HA and CaOx may also cause pain by deposition in joints. Ref: Endo/ Harrison's 16, p. 2046/ RAL
Which medication is given chronically for the prevention of acute gouty arthritis?

A) Indomethacin
B) Ibuprofen
C) Prednisone
D) ACTH
E) Colchicine
The correct answer is: E

All choices are used in the treatment of acute gouty arthritis; colchicine alone may also be used for prophylaxis Ref: Endo/ Harrison's 16, p. 2046-2047/ RAL
Which statement is TRUE concerning the treatment of acute gouty arthritis?

A) The use of colchicine may be limited by gastrointestinal side effects.
B) Colchicine should be administered every 8 hours until symptoms resolve.
C) Long-acting NSAIDS should be used to prevent recurrence.
D) Intravenous colchicine may be given every hour until the symptoms subside.
E) NSAID's, when used, should be discontinued after a 3-day course.
The correct answer is: A

Oral colchicine may produce nausea, abdominal pain, and diarrhea. It is normally administered as a 0.6mg dose every hour until symptoms resolve or 4 to 8 doses have been given. Intravenous colchicine is given every 6 hours, up to a maximum total dose of 4mg. If indomethacin or other NSAID is used, resolution of symptoms usually takes 5 to 7 days. Short-acting NSAIDS, such as indomethacin or ibuprofen, are most effective. Ref: Endo/ Harrison's 16, p. 2046-2047/ RAL
Which is the best estimate of the extracellular fluid volume in a 70kg man?

A) 3.5 liters
B) 10.5 liters
C) 14.0 liters
D) 20.5 liters
E) 28.0 liters
The correct answer is: C

Total extracellular fluid volume in an adult male is about 20% of total body weight, or about 14 liters in a 70kg man. Intravascular volume is about 5% of body weight (3.5 liters); the remaining extracellular volume, 10.5 liters, is interstitial fluid. Ref: Endo/ Schwartz 8, p. 43-44/ RAL
A 55-year-old woman presenting for her routine annual physical examination is noted to have a solitary thyroid nodule. The next step in the evaluation of this nodule should be:

A) Fine-needle aspiration
B) Ultrasound-guided biopsy
C) Serum TSH level
D) Excisional biopsy
E) Subtotal thyroidectomy
The correct answer is: C

If TSH is normal, fine-needle aspiration is indicated; if TSH is low, radionuclide scan is the next step. Ref: Ende/ Harrison's 16, p. 2126-2127/ RAL
This electrolyte imbalance may be a complication of parathyroidectomy. Deep tendon reflexes are hyperactive and a positive Chvostek's sign may be observed.

A) Hypocalcemia
B) Hyperkalemia
C) Hyponatremia
D) Hypokalemia
E) Hypercalcemia
The correct answer is: A

Hypocalcemia is a common complication of parathyroidectomy. Symptoms include hyperactive deep tendon reflexes, circumoral and digital paresthesias, muscle and abdominal cramps, and spasm when tapping over the facial nerve (Chvostek's sign). Ref: Endo/ Schwartz 8, p. 50/ RAL
This fluid or electrolyte abnormality may cause increased intracranial pressure.

A) Hypocalcemia
B) Hyperkalemia
C) Hyponatremia
D) Hypokalemia
E) Hypercalcemia
The correct answer is: C

Hyponatremia causes an increase in intracellular water, which may result in increased intracranial pressure. This is particularly dangerous in patients who have suffered closed head trauma. Ref: Endo/ Schwartz 8, p. 46/ RAL
In this fluid abnormality, ECG manifestations include peaked T waves and widened QRS complexes.

A) Hypocalcemia
B) Hyperkalemia
C) Hyponatremia
D) Hypokalemia
E) Hypercalcemia
The correct answer is: B

ECG signs of hyperkalemia include peaked T waves, prolonged PR interval, and widened QRS complexes. Cardiac manifestations include ventricular fibrillation. Ref: Endo/ Schwartz, p. 57-65/ RAL
This fluid abnormality may be caused by respiratory or metabolic alkalosis.

A) Hypocalcemia
B) Hyperkalemia
C) Hyponatremia
D) Hypokalemia
E) Hypercalcemia
The correct answer is: D

In the alkalotic patient, hydrogen ions are conserved by the increased excretion of potassium ions in the renal tubules. Change in serum potassium can be estimated by the formula: Potassium decreases 0.3mEq for every 0.1 increase in pH above normal. Ref: Endo/ Schwartz 8, p. 48/ RAL
This electrolyte disturbance occurs in patients with hyperparathroidism and bony metastases.

A) Hypocalcemia
B) Hyperkalemia
C) Hyponatremia
D) Hypokalemia
E) Hypercalcemia
The correct answer is: E

Hypercalcemia may result in confusion, muscle weakness, vomiting, coma, complete heart block and cardiac arrest. Treatment includes forced diuresis by the administration of intravenous saline and furosemide. Ref: Endo/ Schwartz 8, p. 49-5; Harrison's 16, p. 2261-2262/ RAL
The most common type of syncope is:

A) Vasovagal syncope
B) Cardiac syncope
C) Micturition syncope
D) Cough syncope
E) Hyperventilation syndrome
The correct answer is: A

The most common cause is a vasovagal reaction, resulting from increased vagal tone causing bradycardia and vasodilatation with sudden fall in systemic blood pressure and cerebral perfusion. Ref: Neuro/ Goroll 4, p. 133-135/DP-RAL
Which is the most common finding in brain abscess?

A) Nuchal rigidity
B) Transient monocular blindness
C) Headache
D) Seizure
E) Paraplegia
The correct answer is: C

Headache is present in 75% of cases. Other common symptoms are fever and focal neurologic deficits. Confusion may also be present. Ref: Neuro/ Harrison's 16, p. 2486/ DP-RAL
Which of the following symptoms is present in the most patients with bacterial meningitis?

A) Headache
B) Fever
C) Meningismus
D) Alteration in mental status
E) All of the above
The correct answer is: E

Headache, fever, and meningismus are each present in about 90% of cases. Alteration of mental status is present in 75% of cases. Ref: Neuro/ Harrison's 16, p. 2473/ DP-RAL
What laboratory test may be used to detect cerebrospinal fluid in the nasal drainage of a trauma patient?

A) Beta-transferrin level of the drainage
B) Glucose level of serum
C) BUN level of the drainage
D) BUN of the serum
E) Protein level of the drainage
The correct answer is: A

If mixed blood and CSF are suspected in the nasal drainage, beta-transferrin of the drainage will be positive only if CSF is present. Ref: Neuro/ Schwartz 8, p. 1617/ RAL
The classic syndrome of Parkinson's disease includes:

A) Resting tremor, bradykinesia, shuffling gait, rigidity, and stooped posture
B) Intention tremor, dysmetria, magnetic gait, and fasciculations
C) Resting tremor, dementia, weakness, flaccidity
D) Resting tremor, dysarthria, magnetic gait, dementia
E) Intention tremor, transient focal deficit, visual field cut
The correct answer is: A

The disease usually presents after the age of 50. Tremor is absent in 20% of cases. Ref: Neuro/ Goroll, p. 986/ DP-RAL
The most common primary site for metastatic brain tumors is where?

A) Skin
B) Lung
C) Breast
D) Colon
E) Testes
The correct answer is: B

The most frequent primary tumor identified in brain metastases is carcinoma of the lung. Ref: Neuro/ Schwartz 8, p. 1633/ DP-RAL
Which of the following is a diagnostic criterion for migraine?

A) Occipital or nuchal soreness
B) Scalp tenderness
C) Jaw claudication
D) Unilateral location
E) Lacrimation or nasal stuffiness ipsilateral to the headache
The correct answer is: D

Unilateral location is a diagnostic criterion for migraine. Scalp tenderness and jaw claudication are typical of giant cell arteritis. Nuchal soreness is seen in tension-type headache. Ipsilateral lacrimation and nasal congestion are seen in cluster headache. Ref: Neuro/ Goroll 4, p. 932-935/ DP-RAL
Which statement concerning migraine headache is true?

A) Migraines never occur during childhood.
B) Migraines are always accompanied by an aura.
C) Migraines are more common in women.
D) Women with migraine headaches are less likely to experience symptoms during menstruation.
E) Pregnancy tends to exacerbate symptoms of migraine.
The correct answer is: C

Migraines often begin during childhood; they need not be accompanied by an aura; in women, symptoms are exacerbated by menstruation and are often relieved by pregnancy. Ref: Neuro/ Goroll 4 , p. 932/ DP-RAL
Components of the aura associated with migraine headache may include:

A) Scintillating scotomata
B) Hemiplegia
C) Diplopia
D) Both A and C
E) All of the above
The correct answer is: E

In addition, the aura may entail: hemianopsia, vertigo, and aphasia. In acephalgic migraine, the aura may occur without subsequent headache. Ref: Neuro/ Goroll 4, p. 932/ RAL
Bell's palsy involves a deficit in the muscles innervated by which cranial nerve?

A) Third
B) Fourth
C) Fifth
D) Seventh
E) None of the above
The correct answer is: D

Bell's palsy presents with unilateral weakness of the upper and lower facial muscles innervated by the seventh cranial nerve. Ref: Neuro/ Goroll 4, p. 991/ RAL
Which of the following is NOT a clinical feature of diabetic peripheral neuropathy?

A) Loss of position sense in the toes
B) Loss of sensation to light touch
C) Hyperactive ankle reflexes
D) Hyperesthesia
E) Paresthesia
The correct answer is: C

Ankle reflexes are typically lost, as is position sense and sensation to light touch. Ref: Neuro/ Harrison's 16, p. 2165/ RAL
Which statement is FALSE concerning Reye's syndrome?

A) It is usually associated with a recent viral illness.
B) Patients are usually jaundiced.
C) All patients are less than 15 years old.
D) It typically presents with stupor, progressing to seizures and coma.
E) Central nervous system changes include cerebral edema.
The correct answer is: B

Although there is typically fatty infiltration of the liver, which is palpably enlarged, patients are not generally jaundiced. In addition to the clinical findings noted, there is an association between aspirin use in children and the incidence of Reye's syndrome. Ref: Neuro/ Harrison's 16, p. 1871/ RAL
By definition, a transient ischemic attack (TIA) lasts how long?

A) More than 15 minutes
B) More than 1 hour
C) More than 3 hours
D) Less than 3 hours
E) Less than 24 hours
The correct answer is: E

Focal neurological deficits lasting less than 24 hours are TIA's. Ref: Neuro/ Harrison's 16 p. 2372/ RAL
Which of the following is a risk factor for stroke?

A) Hypertension
B) Diabetes mellitus
C) Atrial fibrillation
D) Both A and C
E) All of the above
The correct answer is: E

In addition, smoking and hyperlipidemia are risk factors. Ref: Neuro/ Harrison's 16, p. 2377, 2380/ RAL
Which is the most common cause of transient ischemic attacks (TIA's)?

A) Hemorrhage
B) In situ thrombosis
C) Embolism
D) Transient hypotension
E) Seizure disorder
The correct answer is: C

Most TIA's are embolic, with emboli commonly arising from arterial plaques or from intracardiac thrombus. Ref: Neuro/ Gorell 4, p. 970/ RAL
The treatment of choice for a patient who has had a TIA, and who is subsequently shown to have a high-grade stenosis of the ipsilateral internal carotid artery is:

A) a short course of intravenous anticoagulation.
B) a six-month course of oral anticoagulation.
C) life-long oral anticoagulation.
D) life-long daily aspirin for stroke prophylaxis.
E) carotid endarterectomy.
The correct answer is: E

A number of randomized clinical trials have demonstrated that for more than 70% of patients with TIA and an ipsilateral carotid stenosis, the treatment of choice is carotid endarterectomy. Ref: Neurol/ Harrison's 16, p. 2377/ RAL
Alzheimer's disease is the most common cause of dementia in the United States. What is the second most common cause?

A) Head trauma
B) Vascular dementia
C) Chronic alcohol intoxication
D) Parkinson's disease
E) Drug or medication intoxication
The correct answer is: B

Alzheimer's disease is the most common cause of dementia; vascular dementia, including multi-infarct dementia and diffuse white matter disease, is second. The other listed choices are less common causes of dementia. Ref: Neuro/ Harrison's 16, p. 2395, 2401 RAL
Which of the following are risk factors for vascular dementia?

A) Known carotid artery disease
B) Poorly controlled hypertension
C) Diabetes mellitus
D) Both A and B
E) All of the above
The correct answer is: E

Multi-infarct dementia may be due to recurrent emboli, or to lacunar infarcts related to hypertensive small vessel disease. Diabetes mellitus, smoking, and atrial fibrillation are risk factors. Ref: Neurol/ Goroll 4, p. 957/ RAL
A patient has insomnia if:

A) he thinks he does.
B) she gets less than 6 hours of sleep per night.
C) her sleeplessness compromises her ability to function normally.
D) he takes more than one hour to fall asleep at night.
E) he awakes spontaneously at least 3 times per night.
The correct answer is: C

Insomnia is diagnosed when persistent sleeplessness affects normal functioning. Ref: Neuro/ Goroll 4, p. 1188/ RAL
Which of the following statements related to the pharmacotherapy of insomnia is true?

A) Benzodiazepines should not be prescribed for patients with a current or past history of drug or alcohol abuse.
B) A one or two week course of treatment may help restore normal sleep patterns.
C) Long-term treatment with an intermediate-acting benzodiazepine may safely be discontinued abruptly.
D) Both A and B
E) All of the above
The correct answer is: D

Abrupt discontinuation of long-term benzodiazepine therapy may precipitate rebound insomnia and other withdrawal symptoms. The medication should be tapered off over several weeks. Ref: Neuro/ Goroll 4, p. 1191/ RAL
Good sleep hygiene entails which of the following recommendations?

A) Establish a regular bedtime.
B) Once going to bed, remain in bed, even if unable to sleep.
C) Exercise vigorously before retiring.
D) Take frequent naps during the day to cope with nighttime insomnia.
E) Spend as much time as possible in bed during the day (e.g., snacking, reading, watching television) to learn to feel more comfortable there.
The correct answer is: A

Patients should go to bed only when ready for sleep, and should leave bed if unable to sleep. Vigorous exercise just before bed should be discouraged. Daytime napping should be avoided. Ref: Neuro/ Goroll 4, p. 1192/ RAL
Which statement best characterizes petit mal seizures?

A) They involve activity arising from a specific area of the motor cortex.
B) They usually entail loss of consciousness and respiratory arrest.
C) Typical postictal confusion may take several days to resolve.
D) They involve sudden brief lapses of consciousness without loss of postural control.
E) All of the above
The correct answer is: D

Petit mal seizures , or absence seizures, are brief changes in consciousness without loss of postural control or gross motor activity (although subtle motor activity, such as blinking, may be present). There is no postictal confusion. Ref: Neuro/ Harrison's 16, p. 2358/ RAL
Which statement is FALSE concerning seizures in the adult?

A) They are frequently the result of high fevers.
B) Alcoholic withdrawal seizures usually occur within 48 hours of the last drink.
C) Embolic strokes rarely present as a partial or generalized seizure.
D) In patients with an established seizure disorder, alcohol withdrawal is more likely to cause a seizure.
E) In the young adult population, drugs and alcohol are frequently implicated in the new onset of seizures.
The correct answer is: A

Febrile seizures are rare in adults. Alcohol-withdrawal seizures typically present within 48 hours. Up to 25% of patients with embolic strokes may eventually have a seizure, but this is rarely the initial presentation. Alcohol-related seizures are more likely in patients with underlying seizure disorder. When an underlying cause of seizures is discovered in adults between 18 and 45, it is most likely to be due to drugs or alcohol, followed by neoplasm and trauma. Ref: Neuro/ Goroll 4, p. 963-964/ RAL
A young woman who is treated with phenytoin (Dilantin) for a seizure disorder is contemplating pregnancy. She should be advised that:

A) Phenytoin is associated with fetal abnormalities.
B) Even off all anti-seizure medications, women with seizure disorders have an increased risk of having a child with birth defects.
C) Valparoate (DepakotE) and carbmazepine (Tegretol) are also associated with fetal abnormalities.
D) A seizure during pregnancy may injure the fetus through hypoxia or mechanical injury.
E) All of the above
The correct answer is: E

Even so, women on phenytoin for seizures have a 94% chance of having a normal baby. Ref: Neuro/ Goroll 4, p. 968/ RAL
Which statement is FALSE concerning multiple sclerosis?

A) The peak incidence is between the second and fourth decades of life.
B) Blurred vision due to optic neuritis is common and usually unilateral.
C) Cognitive function is invariably spared.
D) The most common motor symptom is weakness in one or more extremity.
E) Sensory symptoms commonly include paresthesias.
The correct answer is: C

The incidence peaks between 20 and 40 years old, with a 2:1 female predominance. Monocular and, less commonly, binocular blurred vision, sometimes progressing to severe visual impairment, is frequently seen. Cognitive function may be disturbed, with memory loss, impaired judgment, and inappropriate affect. Progressive weakness and paresthesias are common. Ref: Neuro/ Harrison's 16, p. 2461-2463/ RAL
Symptoms of human rabies infection include all of the following, EXCEPT:

A) Profuse sweating, lacrimation, and salivation
B) Migrating areas of sensory loss
C) High fever
D) Paresthesias or fasciculations in the area of the suspect animal bite wound
E) Agitation and confusion
The correct answer is: B

Hyperesthesia is common, with increased sensitivity to light, sound, and light touch. The other cited manifestations are common. Ref: Neuro/ Harrison's 16, p. 1157/ RAL
A 14-year-old boy is bitten by an aggressive stray dog at a playground. The animal cannot be captured for evaluation. The boy is seen in the ED with a bite wound on the anterior thigh involving skin and subcutaneous tissue. Appropriate management of this patient would entail:

A) Tetanus prophylaxis; vigorous cleansing and debridement of the wound; observe patient for 10 days and begin broad-spectrum antibiotics if he develops signs and symptoms of rabies
B) Tetanus prophylaxis; vigorous cleansing and debridement of the wound; begin a 10-day course of antibiotics immediately
C) Tetanus prophylaxis; vigorous cleansing and debridement of the wound; begin broad- spectrum antibiotics immediately; begin passive immunization with human rabies immune globulin (HRIG) D) Tetanus prophylaxis; vigorous cleansing and debridement of the wound; begin broad- spectrum antibiotics; active immunization with human rabies vaccine; and begin HRIG if signs and symptoms of rabies exist
E) Tetanus prophylaxis; vigorous cleansing and debridement of the wound; begin broad- spectrum antibiotics; passive immunization with HRIG; active immunization with human rabies vaccine immediately
The correct answer is: E

Passive and active immunization against this viral disease should be begun at once if the animal cannot be observed. The other less specific supportive measures are recommended whether or not the animal is captured and evaluated. Ref: Neuro/ Harrison's 16, p. 1159/ RAL
Status epilepticus refers to:

A) continuous or repetitive seizure activity.
B) a history of recurrent seizures.
C) a family history of seizures.
D) an impaired level of consciousness following a seizure.
E) a condition of lowered seizure threshold.
The correct answer is: A

Status epilepticus refers to continuous or repetitive seizure activity, without restoration of normal consciousness, for a period longer than 15 to 30 minutes. Ref: Neuro/ Harrison's 16, p. 2370/ RAL
Which is generally the first drug of choice in status epilepticus?

A) Propofol
B) Lorazepam
C) Pentobarbitol
D) Phenobarbital
E) Phenytoin
The correct answer is: B

Intravenous lorazepam (Ativan) is the initial drug of choice in controlling seizures in status epileptic. Ref: Neuro/ Harrison's 16, p. 2370-2371; Tintinalli 6, p. 1416/ RAL
Which is the most common movement disorder in the United States?

A) Huntington's disease
B) Parkinson's disease
C) Essential tremor
D) Tourette syndrome
E) Tardive dyskinesia
The correct answer is: C

All choices are movement disorders. Essential tremor is by far the most common; it is characterized by bilateral involvements of the upper extremities, with tremors of the head, leg, larynx, and trunk being less common. Ref: Neuro/ Harrison's 16, p. 2414/ RAL
The eidemiologic pattern of Huntingtonäó»s disease is:

A) Sporadic
B) Epidemic
C) Inherited
D) Environmentally activated
E) None of the above
The correct answer is: C

Huntington's disease is an inherited disease transmitted in an autosomal dominant pattern. Clinical features include: arrhythmic involuntary movements, gradually progressing to bradykinesia and rigidity. Various psychiatric manifestations are also common. Ref: Neuro/ Harrison's 16, p. 2415/ RAL
Match to the appropriate lumbosacral nerve root: Pain in the lateral calf and weakness in the dorsiflexors of the foot.

A) L2
B) L3
C) L4
D) L5
E) S1
The correct answer is: D

These symptoms are indicative of the 5th lumbosacral nerve root. Ref: Neuro/ Harrison's 16, p. 96
Match to the appropriate lumbosacral nerve root. Pain in the posterior buttocks, thigh, calf and plantar surface of the foot, gastrocnemius weakness.

A) L2
B) L3
C) L4
D) L5
E) S1
The correct answer is: E

These symptoms are indicative of S1. Ref: Neuro/ Harrison's 16, P. 96/ RAL
Match to the appropriate lumbosacral nerve root: Pain in the anterior thigh, weakness of the psoas muscle (weakness in hip flexion).

A) L2
B) L3
C) L4
D) L5
E) S1
The correct answer is: A

These symptoms are indicative of L2. Ref: Neuro/ Harrison's 16, p. 96/ RAL
This type of headache typically presents with chronic, unilateral, dull, or aching pain involving jaw, neck, or shoulders.

A) Cluster headache
B) Tension-type headache
C) Temporomandibular joint pain
D) Temporal (or giant cell) arteritis
The correct answer is: C

The pain of TMJ dysfunction is associated with chronic jaw-clenching and tooth-grinding during sleep (bruxism). Clicking of the temporomandibular joint and tenderness of the muscles of mastication are common physical findings. Ref: Neuro/ Goroll 4, p. 933
The following symptoms are typical of this type of headache: Intense, unilateral non-throbbing pain behind the eye. More common in men. Accompanied by ipsilateral lacrimation, ptosis and miosis . Tends to occur at night

A) Cluster headache
B) Tension-type headache
C) Temporomandibular joint pain
D) Temporal (or giant cell) arteritis
The correct answer is: A

Cluster headaches typically occur in middle-aged men, may continue nightly for weeks and then resolve completely

only to recur months later. Ref: Neuro/ Goroll 4, p. 933-934; Harrison's 16, p. 93/ RAL
In this type of headache, the pain is dull, bilateral, and steady and may last for days or weeks.

A) Cluster headache
B) Tension-type headache
C) Temporomandibular joint pain
D) Temporal (or giant cell) arteritis
The correct answer is: B

Tension headaches are often associated with anxiety or depression; are probably related to muscle tension; and are bilateral 90% of the time. Ref: Neuro/ Goroll 4, p. 933; Harrison's, p. 88/ RAL
With this type of headache, pain may be throbbing or dull, and is often associated with scalp tenderness or jaw claudication; ESR is elevated; may cause blindness.

A) Cluster headache
B) Tension-type headache
C) Temporomandibular joint pain
D) Temporal (or giant cell) arteritis
The correct answer is: D

Temporal arteritis is an inflammation of medium sized arteries often associated with polymyalgia rheumatica; local tenderness over the involved artery is common, as is pain on chewing (jaw claudication); blindness may be caused by involvement of the ophthalmic artery; the disease generally responds to steroids. Ref: Neuro/ Goroll 4, p. 933; Harrison's 16, p. 88-93/ RAL
A 79-year old woman complaining of vertigo in the recumbent position with the head turned to the right is most likely suffering from which of the following?

A) Meniere's disease
B) Benign positional vertigo
C) Acoustic neuroma
D) Acute labyrinthitis
The correct answer is: B

Patients with benign positional vertigo are typically elderly and complain of vertigo in the recumbent position, usually with the head turned to a particular side. Ref: Neuro/Goroll 4, p. 941; Harrison's 16, p. 132/ RAL
A 40-year old man complaining of vertigo, tinnitus, and unilateral hearing loss following an upper respiratory tract infection most likely has which of the following?

A) Meniere's disease
B) Benign positional vertigo
C) Acoustic neuroma
D) Acute labyrinthitis
The correct answer is: D

Patients with acute labyrinthitis give a history of a viral upper respiratory tract infection, followed by the onset of vertigo, tinnitus, and hearing loss. Symptoms usually resolve permanently in 3 to 6 weeks. Ref: Neuro/Goroll 4, p. 941; Harrison's 16, p. 131/ RAL
A 55-year old woman complaining of progressive hearing loss and tinnitus, and the recent onset of ipsilateral facial numbness and ataxia is probably suffering from which of the following?

A) Meniere's disease
B) Benign positional vertigo
C) Acoustic neuroma
D) Acute labyrinthitis
The correct answer is: C

Acoustic neuroma, or schwannomas of the acoustic nerve, typically presents with progressive symptoms in middle age. Compression of the facial nerve may cause ipsilateral facial numbness; compression of the brain stem may cause incoordination. Ref: Neuro/ Goroll 4, p. 941; Harrison's 16, p. 132/ RAL
A 42-year old man complaining of the sudden onset of vertigo, hearing loss, and tinnitus, which is relieved by bed rest, followed by complete resolution and then by recurrent episodes has a likely diagnosis of which of the following?

A) Meniere's disease
B) Benign positional vertigo
C) Acoustic neuroma
D) Acute labyrinthitis
The correct answer is: A

The onset of Meniere's disease is typically sudden, with vertigo, tinnitus, and hearing loss. Episodic recurrences may last minutes to hours. Hearing loss may eventually become permanent. Ref: Neuro/Goroll 4, p. 941; Harrison's 16, p. 182/ RAL
A 19-year old female presents with a tachycardia of 110 beats per minute, sweaty palms, and a feeling of impending doom. Her sweating and tachycardia resolve in the waiting room. What is the most likely diagnosis?

A) Depression
B) Panic disorder
C) Bipolar depression
D) Schizophrenia
E) Multiple personality disorder
The correct answer is: B

The somatic symptoms of diaphoresis and palpitations along with this patient's feeling of impending doom are strongly suggestive of panic disorder. Symptoms generally subside within one hour. Ref: Psych/Harrison's 16, p.2547-2548/ RMS-RAL
A 45-year old female previously diagnosed with panic disorder consistently misses her counseling sessions. When you reach her by phone, she absolutely refuses to leave her home to come to your office. What is the most likely diagnosis?

A) Arachnophobia
B) Xenophobia
C) Agoraphobia
D) Claustrophobia
E) Social phobia
The correct answer is: C

This patient has a fear of public places, agoraphobia, which prevents her from leaving her home. Agoraphobia may be a feature in some patients with panic disorder. Ref: Psych/Harrison's 16, p. 2548/ RMS-RAL
Which of the following medical disorders may present with symptoms suggestive of a primary anxiety disorder?

A) Hashimoto's thyroiditis
B) Meniere's disease
C) Graves disease
D) Crohn's disease
E) Addison's disease
The correct answer is: C

Hyperthyroidism, as in Grave's disease, may produce tachycardia, nervousness, and diaphoresis typical of panic disorder. Ref: Psych/Harrison's 16, p.2548/ RMS-RAL
Which of the following agents may be useful in the immediate management of panic disorder?

A) Sertraline (Zoloft) B) Clonazepam (Klonepin) C) Imipramine (Tofranil) D) Tranylcypromaine (ParnatE) E) None of the above
The correct answer is: B

Antidepressants of the selective serotonin reuptake inhibitor (SSRI) type (sertralinE) , tricyclics (TCA) (imipraminE) , and monoamine oxidase inhibitors (MAOI) (tranylcyprominE) may all be useful, but take several weeks to be effective. Early in treatment, benzodiazepines, such as clonazepam, may provide immediate relief. Ref: Psych/Harrison's 16, p. 2548/ RMS-RAL
A 20-year-old male presents with muscle aches, impaired concentration, restlessness, insomnia, and a prior history of sedative abuse. While in counseling he speaks of a history of excessive fears throughout childhood and social inhibition. He also states that his mother has a past history of "panic attacks". What is your diagnosis?

A) Bipolar depression
B) Unipolar depression
C) Borderline personality disorder
D) Schizophrenia
E) Generalized Anxiety Disorder (GAD)
The correct answer is: E

The historical data that supports the diagnosis of GAD is this patient's insomnia, history of sedative abuse, social inhibition, and fears throughout childhood. The fact that this patient has a first-degree relative who also suffers from an anxiety disorder further supports this diagnosis. Ref: Psych/Harrison's 16, p. 2549/ RMS-RAL
A 44-year-old female seeks medical attention after being stuck in an elevator two weeks ago. She states that she has had an exceedingly difficult time getting into her car, and once she gets home from work she insists that the windows be open. What is your diagnosis?

A) Claustrophobia
B) Agoraphobia
C) Xenophobia
D) Social phobia
E) Performance anxiety
The correct answer is: A

This patient's irrational fear of closed spaces, resulting in a serious disruption of her daily life, is consistent with the diagnosis of claustrophobia. Ref: Psych/Harrison's 16, p. 2550/ RMS-RAL
Which of the following modalities is NOT appropriate for the treatment of social phobia?

A) Monoamine oxidase inhibitors (MAOI's) B) Selective serotonin re-uptake inhibitors (SSRI's) C) Benzodiazepines
D) Behavioral psychotherapy
E) Beta blockers
The correct answer is: E

While beta blockers may alleviate the symptoms of performance anxiety, they are not indicated for the treatment of general social phobia. SSRI's, MAOI's, benzodiazepines, and behavioral therapy have all proven useful. Ref: Psych/Harrison's 16, p. 2550-2551/ RMS-RAL
A nine-year-old boy, who witnessed the suicide of his mother when he was seven, presents with the recent onset of difficulty sleeping, night terrors, flashbacks of the suicide, and a markedly increased startle response. What is the most likely diagnosis?

A) Post-traumatic stress disorder (PTSD) B) Generalized anxiety disorder (GAD) C) Unipolar depression
D) Histrionic personality disorder
E) Claustrophobia
The correct answer is: A

This child is experiencing a delayed stress disorder, PTSD, secondary to the extreme traumatic event he has experienced. His hypervigilance, difficulty sleeping, and flashbacks of the traumatizing incident support this diagnosis. Ref: Psych/Harrison's 16, p. 2551/ RMS-RAL
Which of the following is NOT a feature of post-traumatic stress disorder (PTSD) ?

A) Hypervigilance
B) Intrusion of thoughts with past traumas
C) Feelings of depersonalization
D) Alternating euphoria and depression
E) Disruptions in sleep cycles
The correct answer is: D

Alternating euphoria and depression are features of bipolar depression, while hypervigilance, feelings of depersonalization, difficulty sleeping, and intrusion of thoughts with past traumas characterize PTSD. Ref: Psych/Harrison's 16, p. 2552/ RMS-RAL
A 24-year-old male presents with complaints of repetitive behaviors, tics, and obsessional thoughts. His notes that he has to check exactly three times to see if his doors are locked every night. He also describes an exaggerated fear of germs on objects in public places, leading him to severely restrict his activities outside the home. What is the most likely diagnosis?

A) Borderline personality disorder
B) Antisocial personality disorder
C) Obsessive compulsive disorder
D) Dysthymic disorder
E) Minor depression
The correct answer is: C

The repetitive, obsessional nature of this patient's thoughts characterize obsessive compulsive disorder. Additional features include ritualistic behaviors and a deterioration of psychosocial functioning. Ref: Psych/Harrison's 16, p. 2552/ RMS-RAL
Reconditioning therapy in patients with obsessive-compulsive disorder (OCD) generally consists of:

A) gradual introduction of escalating doses of benzodiazopines.
B) forced exposure to the provocative stimulus.
C) exposure to the compulsive behavior in conjunction with an unpleasant physical stimulus.
D) gradual exposure to the provocative stimulus followed by prepared relaxation techniques.
E) gradually decreasing doses of fluoxetine.
The correct answer is: D

Reconditioning relies on exposure to the stimulus that provokes the obsessive behavior while substituting relaxation techniques for the usual pathologic response. Eventually the patient becomes desensitized. Ref: Psych/Goroll 4, p. 1151/ RAL
Which of the following is not a feature of unipolar depression?

A) Early morning awakening
B) Changes in sleep patterns
C) Difficulty concentrating
D) Changes in weight
E) Repetitive, obsessional thoughts
The correct answer is: E

Repetitive, obsessional thoughts are characteristic of obsessive compulsive disorder, not depression. Ref: Psych/Harrison's 16, p. 2553/ RMS-RAL
Which of the following statements is FALSE regarding unipolar depression?

A) Patients with depression should be questioned regarding suicidal ideation.
B) Patients may have brief episodes of mania.
C) Irritability and difficulty concentrating are symptoms.
D) Tricyclic antidepressants (TCA's) may be contraindicated in depressed patients with established cardiovascular disease.
E) Counseling is a key component of therapy.
The correct answer is: B

Manic episodes are indicative of bipolar, not unipolar, depression. Patients should be questioned regarding symptoms, suicidal ideation, as many may not offer this information. TCA's may cause cardiac arrhythmias, and may be contraindicated in patients with known cardiovascular disease or risk factors. Ref: Psych/Harrison's 16, p.2554-2555/ RMS-RAL
Which antidepressant may lower the seizure threshold, and is contraindicated in patients with a seizure disorder?

A) Buproprion (Wellbutrin) B) Trazodone (Desyrel) C) Amitriptyline (Elavil) D) Sertraline (Zoloft) E) Mirtazapine (Remeron)
The correct answer is: A

Buproprion should be avoided in patients with seizure disorder. Ref: Psych/Harrison's 16, p. 2555/ RAL
Over what period of time must cycles of mania and depression be present in order to establish a diagnosis of bipolar disorder?

A) 2 weeks
B) 6 weeks
C) 12 weeks
D) 6 months
E) 2 years
The correct answer is: E

Mood fluctuations must be present for 2 years before bipolar disorder can be diagnosed. Ref: Psych/Harrison's 16, p. 2556/ RAL
A 32-year old male presents complaining of general malaise, weakness, ataxia, and sexual dysfunction. He states that he underwent a divorce that ended after a long custody battle for his two year old daughter. A thorough medical evaluation reveals no abnormality. What is the most likely diagnosis?

A) Panic disorder
B) Conversion disorder
C) Unipolar depression
D) Agoraphobia
E) Huntington's disease
The correct answer is: B

This patient manifests the stress he has endured through vague somatic, and particularly, neuromuscular complaints. He has unconsciously "converted" his emotional stress into physical symptoms. Ref: Psych/Harrison's 16, p. 2558; Goroll 4, p. 1184/ RMS-RAL
A 44-year old female presents with a sprained ankle in the emergency room. She is brought in on a stretcher and is crying and writhing in pain. An exaggerated pain response is elicited with cutaneous stimulation of her affected ankle. She asks one of the emergency physicians for a work note. While alone in the examination room, one of the nurses sees her standing and bearing weight on the injured ankle. What is the most likely diagnosis?

A) Conversion disorder
B) Somatization disorder
C) Bipolar disorder
D) Schizoaffective disorder
E) Malingering
The correct answer is: E

Given this patient's initial response to light touch of her injured extremity and the fact that she requested a work note before pain medication, suspicions should be aroused for the possibility of malingering. The clinician must consider motives for this behavior and should proceed tactfully when gently confronting the malingering patient. Additional psychiatric diagnoses must also be ruled out when considering the malingering patient. Ref: Psych/Harrison's 16, p.2558/ RMS-RAL
A "difficult patient" in your practice habitually mistrusts those around him, including health care providers. He is suspicious to a degree that isolates him socially and impairs him in his occupation. Nevertheless, he is not completely out of touch with reality. The personality disorder that best describes this patient is:

A) Antisocial
B) Schizotypal
C) Borderline
D) Narcissistic
E) Paranoid
The correct answer is: E

Habitual mistrust and unjustified suspicion typify the paranoid personality disorder. Ref: Psych/Harrison's 16, p.2558/ RAL
All of the following are characteristics personality disorders EXCEPT:

A) Is often noncompliant with treatment plans
B) May be overly demanding
C) Has a limited emotional range
D) Depressed affect
E) Exhibits antisocial behavior
The correct answer is: D

Depressed affect would lead the clinician to consider a mood disorder. All other choices represent common findings in personality disorder. Ref: Psych/Harrison's 16, p. 2558/ RMS-RAL
Which of the following traits are common to avoidant, dependent, and obsessive-compulsive personality disorders?

A) Magical beliefs
B) Unusual perceptual experiences
C) Anxiety and fear
D) Results from substance abuse
E) Impulsive behavior
The correct answer is: C

So-called "Cluster C" personality disorders

avoidant, dependent, and obsessive-compulsive disorders

are characterized by anxiety and fear. Magical beliefs and odd perceptual experiences are typical of the schizotypal disorder. Impulsive behavior is more common in "Cluster B" disorders, such as the antisocial, borderline, histrionic or narcissistic disorders. Ref: Psych/Harrison's 16, p. 2558/ RAL
A 27-year old male presents with decreased motor activity, echolalia, and auditory and visual hallucinations. He has been living in an institutionalized setting since age 19. What is the most likely diagnosis?

A) Paranoid schizophrenia
B) Catatonic schizophrenia
C) Bipolar disorder
D) Obsessive compulsive disorder
E) None of the above
The correct answer is: B

The presentation of echolalia, decreased motor activity, and sensory hallucinations suggests the diagnosis of catatonic schizophrenia. Ref: Psych/Harrison's 16, p. 2559/ RMS-RAL
Which of the following symptoms would likely predominate in residual-type schizophrenia?

A) Delusions
B) Hallucinations
C) Motor disturbances
D) Decreased emotional expression
E) All are about equal
The correct answer is: D

Negative symptoms (anhedonia, loss of function and social engagement, decreased emotional expression, and impaired concentration) characterize residual-type schizophrenia. Ref: Psych/Harrison's 16, p. 2559/ RAL
Tardive dyskinesia is a serious side effect related to some antipsychotic agents. Which of the following statements is TRUE?

A) It is always reversible with discontinuation of the medication.
B) If it does not occur when therapy is initiated, it is not likely to occur.
C) It is characterized by involuntary movements of the tongue and lips.
D) It may be temporarily reversed by treatment with Vitamin E.
E) Clozapine is the agent most likely to cause tardive dyskinesia.
The correct answer is: C

Tardive dyskinesia, involuntary, repetitive movements of the face, mouth and tongue, generally occurs after chronic treatment with older antipsychotic agents. It may be irreversible. Clozapine and other newer agents appear to be less likely to cause this disorder. Ref: Psych/Harrison's 16, p. 2561; Tintinalli 9, p. 1046/ RAL
Which of the following statements regarding abused individuals is FALSE?

A) Individuals with a long history of abuse may present with post-traumatic stress disorder.
B) Providers should question suspected victims of abuse in the presence of the suspected abuser

to avoid fueling paranoid ideation and to force a therapeutic confrontation.
C) Abused women may present with concurrent anxiety, mood, or substance abuse disorders.
D) Abused individuals frequently express feelings of helplessness and social isolation.
E) The first priority is eliciting assurances from the perpetrator that violence will stop.
The correct answer is: B

Abusers and victims should be questioned separately. Ref: Psych/Harrison's 16, p.2561/ RMS-RAL
An 18-year-old motorcyclist has been involved in a collision with a truck. He arrives by helicopter, but has a mortal head injury and is subsequently pronounced dead. Which statement regarding the notification and support of the family is TRUE?

A) It is important to get permission for the autopsy within 15 minutes of notification.
B) Denial is a common response among relatives who have contributed to the demise of the patient.
C) Family members who wish, should be permitted to view the body of the deceased.
D) Normal grieving lasts 2 to 3 weeks.
E) Expressions of guilt among surviving family members should be noted and reported to the authorities.
The correct answer is: C

In the acute period of grief, the surviving family members are typically incapacitated by psychological pain and are unable to make any decisions. Denial may be a temporary protective mechanism, and is common. Like guilt, it is part of the normal grief response, which typically lasts for 6 to 8 months. Viewing the body may be comforting to family members and should be permitted if requested. Ref: Psych/Tintinalli 9, p. 1833/ RAL
Which drug or class of drugs is the best choice for the long-term management of panic disorder?

A) Alprazolam (Xanax) B) Selective serotonin reuptake inhibitors (SSRI's) C) Olanzapine (ZyprexA) D) Haloperidal (Haldol) E) Tricyclic antidepressants (TCA's)
The correct answer is: B

SSRI are the drugs of choice for long-term management, while alprazolam is effective for short-term relief of symptoms. Ref: Psych/Tintinalli 9, p.1829/ RAL
The acute phase of psychological reaction to sexual assault:

A) Typically lasts 3 to 6 months
B) Is consistently characterized by severe agitation
C) Typically begins when the victim presents to a health care facility complaining of sexual assault
D) May be masked by other emotional or physical complaints
E) All of the above
The correct answer is: D

The acute phase of the "rape-trauma" syndrome usually lasts for hours to a few days. A minority of victims complain of sexual assault; most present with other psychiatric or medical complaints. Agitation or unnatural calm may predominate. Ref: Psych/ DeCherney 9, p. 1090/ RAL
In the long-term or "reorganization" phase of the rape trauma syndrome:

A) Nearly all victims reestablish rewarding sexual and emotional relationships with their partners.
B) The process of reintegration and emotional recovery lasts for 3 to 6 months.
C) Post-traumatic stress disorder (PTSD) is a common sequela.
D) Suicide is rare.
E) None of the above
The correct answer is: C

PTSD is common. More than half of victims experience difficulty in their sexual or emotional relationship with their partner. The process or reorganization may take several years. 20% of victims not seeking treatment attempt suicide. Ref: Psych/ DeCherney 9, p. 1090/ RAL
Standard criteria for the diagnosis of "alcohol abuse" include all of the following EXCEPT:

A) A need to increase the amount of alcohol taken to achieve the desired level of intoxication (tolerancE) B) Legal problems related to drinking
C) Driving while intoxicated
D) Loss of employment due to intoxication
E) Social conflict resulting from intoxication
The correct answer is: A

According to DSM-IV, the occurrence of tolerance is a criterion of "alcohol dependence", a step up in severity of disease from "alcohol abuse". Ref: Psych/ Goroll, p. 1171/ RAL
All of the following conditions favor successful treatment of alcohol abuse or dependence except:

A) Early detection and treatment
B) Longer term treatment programs
C) Involvement of the patient's family members
D) Delay of treatment until total abstinence has been established
E) Involvement of the patient in planning treatment
The correct answer is: D

Prompt initiation of treatment is a determinant of success. The other choices favor successful outcomes. Ref: Psych/ Goroll 4, p. 11173-1174/ RAL
Patients with this condition may present with alternating mood swings of depression and mania.

A) Bipolar disorder
B) Minor depression
C) Post traumatic stress disorder (PTSD) D) Generalized anxiety disorder
E) Pheochromocytoma
The correct answer is: A

Bipolar depression frequently presents with alternating depression and mania. Ref: Psych/Harrison's 16, p.2554-2556/ RMS-RAL
Selective serotonin reuptake inhibitors (SSRI's) and tricyclic antidepressants (TCA's) are the current mainstay of treatment for this type of depression.

A) Bipolar depression
B) Unipolar depression
The correct answer is: B

Selective serotonin reuptake inhibitors (SSRI's) and tricyclic antidepressants (TCA's) are the current mainstay of treatment for unipolar depression. Ref: Psych/Harrison's 16, p. 2554/ RMS-RAL
The diagnosis for this type of depression consists mainly of depressed mood, difficulty concentrating, and changes in weight or sleep.

A) Bipolar depression
B) Unipolar depression
The correct answer is: B

Although difficulty concentrating and changes in sleep patterns may be indicative of the manic phase of bipolar depression, the fact that depressed mood is present leads to the diagnosis of unipolar depression. Ref: Psych/Harrison's 16, p. 2553-2554/ RMS-RAL
Decreased exposure to light may induce episodes of this type of depression.

A) Bipolar depression
B) Unipolar depression
The correct answer is: B

Studies have shown that light deprivation may induce a type of unipolar depression, seasonal affective disorder, in some patients, particularly women. Increased exposure to light may be therapeutic. Ref: Psych/Harrison's 16, p.2554/ RMS-RAL
Lithium carbonate remains a mainstay of the treatment for patients suffering from this type of depression.

A) Bipolar depression
B) Unipolar depression
The correct answer is: A

Lithium bicarbonate is the pharmacologic agent currently indicated for bipolar depression. Ref: Psych/Harrison's 16, p. 2557/ RMS-RAL
A 22-year-old sexually active female presents to the emergency department with burning on urination and urinary frequency. An abdominal exam reveals suprapubic tenderness. Urine cultures reveal a significant bacteriuria. What is your most likely diagnosis?

A) Pelvic inflammatory disease
B) Nephrolithiasis
C) Glomerulonephritis
D) Urinary retention
E) Acute cystitis
The correct answer is: E

The diagnosis of acute cystitis is based upon the clinical presentation of dysuria, urinary frequency, and suprapubic tenderness. Ref: GU/ Harrison's 16, p. 1717/ RMS-RAL
The most common bacterial pathogen causing cystitis is:

A) Proteus mirabilis
B) Klebsiella pneumoniae
C) Staph aureus
D) Escherichia coli
E) Neisseria gonorrhea
The correct answer is: D

Escherichia coli is the causative pathogen in 80% of urinary tract infections. Although the other pathogens may be implicated in this infectious process, E. coli is by far the most common cause. Ref: GU/Harrison's 16, p. 1715/ RMS-RAL
Which statement is TRUE regarding urinary tract infections and pregnancy?

A) Maternal UTI's during gestation do not affect the birthweight of the infant.
B) Decreased ureteral tone and peristalsis are contributory factors.
C) Bladder catheterization with delivery decreases the likelihood of infection.
D) Pregnant women enjoy a natural protection from pyelonephritis.
E) Urinary tract infections are uncommon throughout pregnancy.
The correct answer is: B

Decreased ureteral tone and peristalsis, in addition to vesicoureteral reflux, have been shown to contribute to the development of urinary tract infections. Urinary tract infections are a common occurrence throughout pregnancy, pyelonephritis is particularly common, and the incidence of low-birth-weight infants is increased. Ref: Renal/Harrison's 14, p. 1716/ RMS-RAL
Which of the following is TRUE regarding Gonococcal urethritis?

A) Gonococcal urethritis is caused by gram positive diplococci.
B) In women, a physical examination may reveal mucopurulent cervicitis.
C) Suprapubic pain may be prominent with palpation of the abdomen.
D) Gross hematuria is frequently present.
E) Sudden onset of illness favors a diagnosis of gonococcal urethritis.
The correct answer is: B

Mucopurulent cervicitis is present with Gonococcal urethritis. Hematuria and suprapubic pain are typically absent, and the onset is typically gradual. Neisseria gonorrhoeae is a gram-negative diplococci. Ref: GU/Harrison's 16, p. 1717/ RMS-RAL
Which of the following is true regarding nephrolithiasis?

A) Thiazide diuretics increase urine calcium.
B) A low calcium diet is protective.
C) Struvite stones are a byproduct of Klebsiella urinary tract infections.
D) Calcium stones are most commonly found.
E) Open surgery is currently preferred over shock wave lithotripsy for dissolution of stones.
The correct answer is: D

Calcium stones may be of hydroxyapatite, brushite, or oxalate subtypes. They are most commonly found in men over 30 years of age. Thiazide diuretics lower urine calcium. A low calcium diet does not treat or prevent stone disease. Struvite stone derive from the action of bacteria, usually Proteus species. Open surgery has largely been replaced by lithotripsy and percutaneous or ureteroscopic procedures. Ref: GU/ Harrison's 16, p. 1710-1713/ RMS-RAL
A 28-year old male presents with a one-day history of shaking, chills, diarrhea, nausea, and vomiting. Physical exam reveals a temperature of 103ŒÁF and a sinus tachycardia of 110 beats per minute. Labs reveal leukocytosis and pyuria. All of the following are TRUE regarding this patient EXCEPT:

A) This condition will respond to therapy within 48 to 72 hours.
B) The clinical picture is consistent with acute pyelonephritis.
C) Possible etiologies include nephrolithiasis, tuberculosis, or tumor.
D) Hematuria is not likely in this patient.
E) Leukocyte casts on microscopic examination of the urine confirm the diagnosis.
The correct answer is: D

Hematuria is frequently present during the acute phase of pyelonephritis. and should resolve with treatment. Aggressive antibiotic therapy results in the resolution of this condition within 48 to 72 hours. Ref: GUl/ Harrison's 16, p. 1717/ RMS-RAL
All of the following are likely causes of urinary retention EXCEPT:

A) Bladder cancer
B) Ureteral calculus
C) Diabetic neuropathy
D) Benign prostatic hyperplasia
E) Abdominal aortic aneurysm
The correct answer is: E

Possible causes of urinary retention include bladder cancer, ureteral calculus, diabetic neuropathy with neurogenic bladder, and benign prostatic hyperplasia. When managing urinary retention, it is important to discern whether the etiology is intrinsic or extrinsic, and to treat the underlying cause. Ref: GU/Harrison's 16, p. 1722-1724/RMS-RAL
All of the following are true regarding urinary tract infections associated with catheters except:

A) Sterile technique with placement of the catheter is key in avoiding infection.
B) Proteus, Klebsiella, and Pseudomonas are common causative pathogens.
C) Symptoms of infection usually resolve with removal of the catheter.
D) Bacteria may migrate through the urine in the catheter lumen.
E) Infection is unrelated to the length of time during which the catheter remains in-dwelling.
The correct answer is: E

Catheterization of greater than two weeks duration is associated with an increased risk of infection. Ref: GU/ Harrison's 16, p. 1717/RMS-RAL
All of the following are true regarding glomerulonephritis EXCEPT:

A) Red blood cells are frequently present in the urine sediment.
B) Protein excretion less than 3g per day is usual.
C) Red blood cell casts suggest another diagnosis.
D) Renal biopsy is the "gold standard" for diagnosis.
E) In acute glomerulonephritis, urine output of less than 400 ml per day is common.
The correct answer is: C

Red blood cell casts and red cells are commonly seen. 24-hour output of protein in the urine is less than 3 grams. Biopsy is the diagnostic standard. In ARF (acute renal failurE) due to glomerulonephritis, urine output falls to less than 400ml per day. Ref: GUl/Harrison's 16, p. 1674, 1679/RMS-RAL
A 68-year old male presents with hematuria. He also complains of sharp flank pain that radiates to the groin, beginning about 1 hour ago, and now severe. What is the most likely diagnosis?

A) Bladder Carcinoma
B) Nephrolithiasis
C) Prostatitis
D) Renal tuberculosis
E) Nephrotic syndrome
The correct answer is: B

Given this clinical picture, this patient most likely has nephrolithiasis. However, bladder carcinoma, prostatitis, and tuberculosis may also cause hematuria. Ref: Renal/Harrison's 16, p. 1710/ RMS-RAL
Which laboratory result is most characteristic of prerenal acute renal failure (ARF)?

A) Fractional excretion of sodium (FENA) greater than 1.0%
B) Fractional excretion of sodium less than 1.0%
C) Serum Creatinine less than 0.4
D) Urine Output less than 400 ml per 24 hours
E) Urine positive for myoglobin
The correct answer is: B

The kidneys conserve sodium in prerenal ARF to expand intravascular volume. The result is a FENa below 1.0%. Ref: GU/Harrison's 16, p.1649/ RMS-RAL
In chronic renal disease, what is the glomerular filtration rate (GFR) below which dialysis is generally required?

A) 90 ml/min per 1.73 m2
B) 60 ml/min per 1.73 m2
C) 30 ml/min per 1.73 m2
D) 15 ml/min per 1.73 m2
E) 0 ml/min per 1.73 m2
The correct answer is: D

GFR less than 15 ml/min per 1.73 m2 suggests end-stage, dialysis-dependent renal failure. Ref: GU/Harrison's 16, p. 1653/ RAL
Which of the following conditions is NOT a common complication of chronic renal failure?

A) Pruritus
B) Hypertension
C) Hypertriglyceridemia
D) Metabolic acidosis
E) Hypokalemia
The correct answer is: E

Hypokalemia is rare in most types of chronic renal failure; hyperkalemia is common. Also common are uremic pruritus, hypertension, elevation of serum triglycerides, and metabolic acidosis. Ref: GU/Harrison's 16, p. 1655/ RMS-RAL
A 5-year-old male presents with a six-week history of bedwetting, night terrors, and sleep disturbances. He has been previously potty trained, but has resumed nocturnal bed wetting since the birth of a younger sibling. What is the most likely diagnosis?

A) Stress incontinence
B) Primary enuresis
C) Secondary enuresis
D) Overflow incontinence
E) Conversion disorder
The correct answer is: C

Secondary enuresis typically follows a stressful event, such as the introduction of a younger sibling, in the life of a previously toilet-trained child. Ref: GUl/ Hay 7, p. 207-208/ RMS-RAL
Pharmacological therapy for enuresis includes which of the following?

A) Imipramine (Tofranil) B) Methylphenidate (Ritalin) C) Sertraline (Zoloft) D) Lorazapam (Ativan) E) All of the above
The correct answer is: A

Imipramine and intranasal DDAVP have been used successfully to treat enuresis. Ref: GU/ Hay 9, p. 208/ RMS-RAL
A 79-year old male presents with complaints of decrease in the force and caliber of his urinary stream, dribbling, and the feeling of urinary hesitancy. What is the most likely diagnosis?

A) Benign prostatic hyperplasia (BPH) B) Ureteral stricture
C) Meatal stenosis
D) Bladder carcinoma
E) None of the above
The correct answer is: A

The most likely diagnosis is benign prostatic hyperplasia. Post mortem epidemiological studies have shown that as many as 90% of males in their eighth decade have BPH. Ref: GU/ Harrison's 16, p. 543-544/ RMS-RAL
Which of the following tests or examinations is likely to be abnormal in men with benign prostatic hyperplasia (BPH)?

A) American Urological Association (AUA) symptom index
B) Measurement of post-void residual volume
C) Prostate specific antigen (PSA) D) Digital rectal examination (DRE) E) All of the above
The correct answer is: E

The AUA Symptom Index is a questionnaire used to quantitate voiding symptoms and would likely be abnormal in men with BPH. Likewise, PSA and post-void residual may be elevated, and DRE would reveal an enlarged gland. Ref: GU/ Harrison's 16, p. 544/ RMS-RAL
Which of the following is used to treat benign prostatic hyperplasia (BPH)?

A) Finasteride (Proscar) B) Terazosin (Hytrin) C) Radiotherapy
D) Both A and B
E) All of the above
The correct answer is: D

Terazosin and finasteride are both considered appropriate medical management of BPH. Radiotherapy is utilized in the treatment of prostatic carcinoma, not BPH. Ref: GU/ Harrison's 16, p. 545/ RMS-RAL
Which of the following statements about the management of symptomatic benign prostatic hyperplasia (BPH) is FALSE?

A) Transurethral resection of the prostate is the most common surgical intervention.
B) Open prostatectomy is the procedure of choice in younger patients with fewer medical comorbidities.
C) Finasteride works by blocking the conversion of testosterone to dihydrotestosterone.
D) Terazosin relaxes the smooth muscle of the bladder neck.
E) All of the above are true.
The correct answer is: B

Open prostatectomy has a higher morbidity and mortality than transurethral resection of the prostate (TURP) and is generally reserved for glands greater than 80 to 100 grams. Ref: GU/ Schwartz 8, p. 1530-1531/ RMS-RAL
At what age should prostate cancer screening be offered to men (according to the American Cancer Society)?

A) 35
B) 40
C) 50
D) 60
E) 65
The correct answer is: C

As issues regarding prostate cancer screening are complex, with false positive tests, unnecessary invasive procedures, and potential side effects of surgical and radiation treatment to be considered, it is recommended by the U.S Preventive Services Task Force that the decision be individualized by patient and provider. The American Cancer Society recommends that prostate cancer be offered to men at age 50. Ref: GU/ Harrison's 16, p. 27; Schwartz 8, p. 1536/ RMS-RAL
Which physical finding is most indicative of malignant prostate disease?

A) An enlarged, boggy, symmetrical gland
B) A gland that is markedly tender to palpation
C) Boggy nodules on the medial lobe of the prostate
D) Hard, irregular nodules on the postero-lateral lobes of the prostate
E) Generalized induration of the prostate
The correct answer is: D

Hard, irregular nodules on the postero-lateral lobes of the prostate are highly suggestive of prostatic carcinoma. Ref: GU/ Harrison's 16, p. 544/ RMS-RAL
Which of the following is FALSE regarding the prostate specific antigen (PSA) ?

A) A PSA above 4 nanograms per mL is considered abnormal.
B) The combination of the digital rectal examination (DRE) and the PSA is effective in detecting localized prostatic carcinoma.
C) Annual follow up is appropriate for PSA levels less than 4ng/ml.
D) PSA is useful in staging prostate cancer.
E) Annual screening with PSA decreases mortality from prostate cancer.
The correct answer is: E

No study has shown improvement in mortality with PSA screening. Ref: GU/ Harrison's 16, p. 546; Schwartz 8, p. 1535/ RMS-RAL
The best results in the management of prostate cancer localized to the prostate gland are obtained from:

A) Retropubic prostatectomy
B) Radiation therapy
C) Watchful waiting
D) Microwave ablation
E) None of the above
The correct answer is: E

Microwave ablation is a treatment for BPH with voiding symptoms. None of the other strategies is clearly superior based on clinical research, although all are acceptable approaches in some patients. Ref: GU/ Harrison's 16, p. 546/ RAL
Which of the following therapies is used for palliation of bone metastases secondary to carcinoma of the prostate?

A) Paclitaxel
B) Ochiectomy
C) External beam radiation
D) A and B only
E) All of the above
The correct answer is: E

All three modalities are used for palliation in patients with bone metastases. Ref: GU/ Harrison's 16, p. 548-549/ RMS-RAL
Which sexually transmitted disease is a leading cause of death for people 25-44 years of age?

A) Gonorrhea
B) Syphilis
C) HIV
D) HSV
E) Chlamydia
The correct answer is: C

Infection with HIV, once the leading cause of death in Americans between the ages of 25 to 44, is now sixth. Ref: GU/Harrison's 16, p. 8011084/ RMS-RAL
A 23-year old male presents with painful vesicles located on the shaft of the penis. When eliciting the history, he admits to anal sex with a partner who had perianal ulcerative lesions within the last 7 days. What is the most likely diagnosis?

A) Syphilis
B) HIV
C) Herpes simplex virus (HSV) D) Chlamydia
E) None of the above
The correct answer is: C

Although syphilis may cause ulcerations on the penis, they are typically painless and solitary. Also, the incubation time for syphiolis is 9 to 90 days, compared to 2 to 7 days for HSV. Ref: GU/ Harrison's 16, p. 772/ RMS-RAL
Which of the preparations below is specific for the detection of herpes simplex virus (HSV)?

A) Dark-field examination
B) Gram stain
C) Potassium hydroxide (KOH) preparation
D) Tzanck smear
E) None of the above
The correct answer is: D

The Tzanck smear is a preparation specific for HSV. As it requires specialized skills, most clinicians today rely on viral tissue cultures or detection of HSV DNA in specimens for diagnosis. Ref: GU/Harrison's 16, p. 1040/ RMS-RAL
A 27-year old sexually active male presents with painful unilateral inguinal lymphadenopathy. Two weeks ago he had a small painless lesion on his penis that healed without treatment. What is the most likely diagnosis?

A) Syphilis
B) Gonorrhea
C) HSV
D) Lymphogranuloma venereum
E) None of the above
The correct answer is: D

Painful unilateral inguinal lymphadenopathy two weeks after having a small painless lesion is consistent with the diagnosis of lymphogranuloma venereum. Ref: GU/ Harrison's 16, p. 1013-1014/ RMS-RAL
Which antibiotic regimen is most effective in treating lymphogranuloma venereum (LGV)?

A) Ceftriaxone 1gm IM, single dose
B) Azithromycin 1gm po, single dose
C) Penicillin 1 million units IM qd for 10 days
D) Gentamycin 1mg per kg IV every 8 hours for 7 days
E) Cefixime 400mg po bid for 10 days
The correct answer is: B

Azithromycin 1g po, single dose, is effective in treating C. trachomatis infection, the causative organism in LGV, and the ease of this regimen assures patient compliance. No other single dose therapy is effective. Ref: GU/Harrison's 16, p.1016/ RMS-RAL
A 32-year old pregnant female presents in her first trimester with a painless discharge and inflammation surrounding the endocervix. Microbiologic studies reveal C. trachomatis. What is your recommendation for antibiotic treatment in this patient?

A) Erythromycin 500mg po qid for 14 days
B) Doxycycline 100mg po bid for 10 days
C) Tetracycline 500mg po qid for 7 days
D) Ofloxacin 1gm po as a single dose
E) None of the above
The correct answer is: A

Although all of the above antibiotics are appropriate in treating this patient's mucopurulent cervicitis, only erythromycin is approved for treatment of pregnant patients. Doxycycline, ofloxacin, and tetracycline are teratogenic. Ref: GU/Harrison's 16, p. 1016/ RMS-RAL
A 28-year old male presents with a painless, solitary chancre on the shaft of his penis and nontender inguinal lymphadenopathy. What is the most likely diagnosis?

A) Chlamydia
B) Gonorrhea
C) Syphilis
D) Herpes simplex virus (HSV) E) Human papilloma virus (HPV )
The correct answer is: C

This clinical scenario is consistent with primary syphilis, which presents as a painless, solitary lesion. Ref: GU/Harrison's 16, p. 772 /RMS-RAL
A 29-year old female presents with fever, polyarthralgia, and multiple papular pustules on the distal extremities. You aspirate her knee joint and the culture reveals intracellular diplococci. What is the most likely diagnosis?

A) Human papilloma virus (HPV) B) Gonorrhea
C) Secondary syphilis
D) Herpes simplex virus (HSV) E) Human immunodeficiency virus (HIV )
The correct answer is: B

This patient presents with fever, polyarthralgia, and skin manifestations which is consistent with disseminated gonococcal infection. Morphologically, N. gonorrheae are intracellular diplococci. Ref: GU/ Harrison's 16, p. 859/ RMS-RAL
A 35-year old male presents with unilateral scrotal pain, fever, and epididymal tenderness. What is the most likely causative pathogen of this patient's epididymitis?

A) Neisseria gonorrhoeae
B) Treponema pallidum
C) Herpes simplex virus (HSV) D) Chlamydia trachomatis
E) None of the above
The correct answer is: D

The most likely pathogen causing epididymitis in this patient is C. trachomatis. Ref: GU/ Harrison's 16, p. 1015/ RMS
A 42-year old male presents with gray white papular lesions over the palms, soles, and intertriginous areas of his skin. He also complains of fever, headache, and weight loss. You note nontender lymphadenopathy and patchy alopecia on exam. What is the most likely diagnosis?

A) Primary syphilis
B) Secondary syphilis
C) Tertiary syphilis
D) Both A and B
E) None of the above
The correct answer is: B

This patient's constellation of fever, headache, weight loss, and papular rash (otherwise known as condyloma latA) lead us to the diagnosis of secondary syphilis. Ref: GU/ Harrison's 16, p. 979/ RMS-RAL
A 19-year old female with a past history of anal receptive intercourse presents with several painless cauliflower-shaped lesions. What is your diagnosis?

A) Herpes simplex virus (HSV) B) Human papillomavirus (HPV) C) Syphilis
D) Chlamydia
E) Gonorrhea
The correct answer is: B

Human papilloma virus is the causative agent that produces painless, cauliflower shaped lesions, called condylomata acuminata, in anal receptive intercourse. HPV is also been associated with cervical and vaginal lesions in females. Ref: GU /Harrison's 16, p. 1057/ RMS-RAL
A 21-year-old male presents with a painless, elevated area of red, friable granulation tissue along his inguinal region. He has had this lesion for about a month and it first began as a papule that subsequently ulcerated. What is the most likely diagnosis?

A) Granuloma inguinale
B) Tertiary syphilis
C) Secondary syphilis
D) Chlamydia
E) Herpes simplex virus (HSV )
The correct answer is: A

This patient's history of a papule that developed into friable granulation tissue is indicative of granuloma inguinale (also known as Donovanosis). When co-infected with anaerobes, a foul smelling discharge and pain ensues. Ref: GU/ Harrison's 16, p. 933/ RMS-RAL
Which of the following systems is not affected by tertiary Syphilis?

A) Nervous
B) Cardiovascular
C) Integumentary
D) Skeletal
E) Muscular
The correct answer is: E

The muscles are not involved in complications surrounding tertiary Syphilis. The nervous system may be affected with paresis, tabes dorsalis, and meningeal syphilis. The cardiovascular system may manifest with aortic regurgitation and saccular aneurysm. The integument and skeletal systems may be affected with gummas, or small granulomatous lesions. Ref: GU/ Harrison's 16, p. 980-981/RMS-RAL
Which is the most common malignant tumor of the kidney?

A) Renal cell carcinoma
B) Carcinoma of the renal pelvis
C) Ureteral carcinoma
D) Carcinoma of the bladder
E) Burkitt's lymphoma
The correct answer is: A

Renal cell carcinoma accounts for 90-95% of all kidney tumors. Carcinoma of the bladder, renal pelvis, and ureter less commonly affect the kidney. Burkitt's lymphoma is a tumor that commonly affects children, and has no renal involvement. Ref: GU/ Harrison's 16, p. 541/ RMS-RAL
Which of the following are risk factors for renal cell carcinoma?

A) Smoking
B) Obesity
C) Male gender
D) Von Hippel Lindau (VHL) syndrome
E) All of the above
The correct answer is: E

Smoking and obesity have a strong correlation with the development of renal cell carcinoma. The male:female ratio is 2:1. VHL is an inherited syndrome that predisposes to retinal hemanioma, hemangioblastoma of the spinal cord, and renal cell carcinoma. Ref: GU/Harrison's 16, p. 541/ RMS-RAL
An 81-year old male presents with hematuria, abdominal pain, and a flank mass. He has had a 10 pound weight loss within the past 2 months. Which is the most likely diagnosis?

A) Renal cell carcinoma
B) Bladder carcinoma
C) Lung carcinoma
D) Prostate carcinoma
E) None of the above
The correct answer is: A

The triad of hematuria, abdominal pain, and a flank mass, along with a history of weight loss is a classic presentation of renal cell carcinoma. This presentation is not indicative of either bladder, lung, or prostate carcinoma. Ref: GU/Harrison's 16, p. 542/ RMS-RAL
A 67-year old male presents with hematuria and ureteral obstruction. His past medical history is significant for a 50-pack year smoking history. He has a significant past occupational exposure to aniline dyes in a textile plant. What is the most likely diagnosis?

A) Renal cell carcinoma
B) Bladder carcinoma
C) Bone carcinoma
D) Leiomyoma
E) Osteosarcoma
The correct answer is: B

The combination of hematuria and ureteral obstruction coupled with the exposures to smoking and aniline dyes predispose this patient to bladder carcinoma. Ref: GU/ Harrison's 16, p. 539/ RMS-RAL
Which of the following imaging modalities is best for staging bladder carcinoma?

A) Computerized tomographic (CT ) scan
B) Supine and upright films of the abdomen
C) Cystoscopy
D) Cross-table lateral films of the abdomen
E) Intravenous pyelogram (IVP)
The correct answer is: A

The CT scan is useful in visualizing the perivesical fat and regional lymph nodes, which are areas to which the carcinoma may metastasize. MRI and ultrasound may also be useful in this regard. Ref: GU/ Harrison's 16, p. 539/ RMS-RAL
Which of the following is the standard therapy for invasive bladder carcinoma?

A) Combination chemotherapy
B) Radioactive implants
C) External beam radiotherapy
D) Resection of the bladder
E) None of the above
The correct answer is: D

Resection of the bladder (radical cystectomy) is indicated for invasive bladder carcinoma, while combination chemotherapy is indicated for metastatic disease. Ref: GU/ Harrison's 16, p. 540/ RMS-RAL
A 24-year old female presents with cervical motion tenderness, adnexal tenderness, and uterine fundal tenderness on bimanual examination. Visualization with the speculum reveals mucopurulent discharge surrounding the cervix. What is your diagnosis?

A) Fitz-Hugh

Curtis syndrome
B) Appendicitis
C) Human papillomvirus (HPV) infection
D) Pelvic Inflammatory Disease
E) None of the above
The correct answer is: D

Pelvic inflammatory disease typically presents with cervical motion tenderness, adnexal tenderness, and mucopurulent discharge. The examination is not consistent with Fitz-Hugh

Curtis syndrome (perihapatitis), appendicitis, or HPV. Ref: GU/ Harrison's16, p. 769-770/ RMS-RAL
All of the following statements regarding urinary tract infections are true except:

A) Voiding after intercourse increases the risk of cystitis.
B) Escherichia coli causes about 80% of clinically apparent infection.
C) Chlamydia trachomatis may be a causative agent in young sexually active females.
D) Staphylococcus aureus may be isolated in patients with a history of nephrolithiasis.
E) Cultures should be collected with a clean-catch midstream technique.
The correct answer is: A

Voiding after intercourse has actually been shown to reduce the risk of resultant infection. It is thought that urination has a flushing effect on the pathogens that may have been introduced with intercourse. E. coli causes 80% of infection. Ref: GU/ Harrison's 16, p. 1715-1716/ RMS-RAL
Following an episode of pelvic inflammatory disease (PID) , a woman complains of pleuritic right upper abdominal pain. Liver function tests and right upper quadrant ultrasound are normal. The most likely diagnosis is:

A) acalculous cholecystitis.
B) liver abscess.
C) perihepatitis.
D) appendicitis.
E) hepatitis B.
The correct answer is: C

3-10% of women with PID develop perihepatitis, with exertional or positional right upper quadrant pain, often referred to as Fitz-Hugh-Curtis syndrome. Chronic inflammation and dense fibrous adhesions produce the symptoms. Ultrasound and LFT's are typically normal. Ref: GU/ Harrison's 16, p. 770/ RAL
Which statement concerning pelvic inflammatory disease (PID) is FALSE?

A) Tubal sterilization increases the risk of salpingitis.
B) N. gonorrhoeae and C. trachomatis are the most common causative organisms.
C) Most patients will have tenderness on palpation of the uterine fundus.
D) Most patients will have an elevated white blood cell count.
E) A yellow endocervical discharge is usually seen on speculum examination.
The correct answer is: A

Tubal sterilization reduces the risk by preventing the upward spread of infection through the fallopian tubes. Neisseria gonorrhoeae and Chlamydia trachomatis are the most common organisms, although others have been implicated. Uterine and adnexal tenderness, leukocytosis, and endocervical discharge are typical. Ref: GU/ Harrison's 16, p. 769-770/ RAL
The main value of serum beta-human chorionic gonadotropin (beta hCG ) assay in patients with suspected pelvic inflammatory disease (PID) is to:

A) provide for proper notification of sex partners.
B) rule out ectopic pregnancy.
C) avoid ionizing radiation in patients who may be pregnant.
D) rule out adenexal torsion.
E) detect functional ovarian cysts.
The correct answer is: B

Although the presentation of PID may be similar to that of ectopic pregnancy, a negative pregnancy test will help to lead the clinician toward the correct diagnosis. Ref: GU/ Harrison's 16, p. 769-770/ RAL
What is the usual duration of antibiotic treatment for pelvic inflammatory disease?

A) A single intramuscular dose of cefoxitin is generally adequate.
B) A seven-day course of parenteral antibiotics is mandatory.
C) A ten-day course of oral antibiotics is standard.
D) A combined fourteen-day course of parenteral and oral antibiotics is recommended.
E) Four to six weeks of inpatient or outpatinet intravenous antibiotics is required.
The correct answer is: D

Patients may be treated on an inpatient or outpatient basis, depending on clinical and social variables, but the total duration of antibiotic treatment, oral or parenteral, should be fourteen days. Ref: GU/ Harrison's 16, p. 769-770/ RAL
Which intervention reduces the risk of subsequent pelvic inflammatory disease (PID) ?

A) Insertion of an intrauterine device (IUD) B) Screening at-risk women for chlamydial infection
C) Vaginal douching
D) Dilatation and curettage
E) All of the interventions help reduce the risk.
The correct answer is: B

Available evidence suggests that public health measures, such as risk-based screening for chlamydial infections, may reduce the risk of PID. Recent insertion of an IUD, vaginal douching, or dilatation and curettage increase the risk of PID. Ref: GU/ Harrison's 16, p. 769-771/ RAL
Varicella is infectious approximately 48 hours prior to appearance of the characteristic exanthem. When is it no longer communicable?

A) When the lesions have resolved
B) When the lesions are crusted
C) When the lesions have spread to the trunk
D) When the lesions have become infected
E) When the lesions appear
The correct answer is: B

Varicella is highly infectious approximately 48 hours prior to appearance of the characteristic exanthem, until all the lesions have crusted over. Ref: Derm/ Hay 9, p. 1150/ PS-RAL
The treatment of choice in varicella (chickenpox) is:

A) Acyclovir
B) Broad spectrum antibiotics
C) Glucocorticoids
D) Antiretrovirals
E) None of the above
The correct answer is: A

Acyclovir 800mg po 5 times daily may be given to adults or adolescents. Children may be dosed at 20mg/kg every 6 hours. Ref: Derm/ Harrison's 16, p. 1044/ PS-RAL
The patient presents with painless, confluent white plaques, adherent to the oral and pharyngeal mucosa, that reveal a raw, bleeding surface when scraped. What is the most likely diagnosis?

A) Leukoplakia
B) Kaposi's sarcoma
C) Peutz-Jeghers syndrome
D) Amalgam tattoo
E) Candidiasis
The correct answer is: E

Oral candida or thrush is a painless confluent white lesion that can be scraped off. Definitive diagnosis is made microscopically. There are oval budding cells, hyphae, and pseudohyphae. The lesions of leukoplakia are more firm, and may be fissured or ulceratede. The lesions of Kaposi's sarcoma are red or blue. The lesions of Peutz-Jeghers syndrome are dark brown. Amalgam tattoo of the gingiva and mucobuccal fold is a blue-black pigmented area associated with amalgam particles. Ref: Derm/ Harrison's 16, p. 198-199/ PS-RAL
Which of the following is effective treatment for oral thrush?

A) Ciprofloxacin
B) Nystatin suspension rinse
C) Miconazole cream
D) Viscous lidocaine
E) The disease is self-limiting and requires no treatment.
The correct answer is: B

A suspension of nystatin that is rinsed (swisheD) and either spit or swallowed is effective treatment. Other options are oral fluconazole and clotrimazole troches. Ref: Derm/ Harrison's 16, p. 1187/ PS-RAL
A patient who has recently completed a course of broad-spectrum antibiotics has developed a red, macerated intertriginous rash. What is the most likely diagnosis?

A) Staphylococcal infection
B) Vasculitis
C) Drug reaction
D) Cutaneous candidiasis
E) Acne vulgaris
The correct answer is: D

Candida thrives in warm moist environments such as the intertriginous areas. Treatment of cutaneous candidiasis is largely through meticulous hygiene and drying of the intertriginous areas and the application of topical anti-fungals, such as nystatin powder and clotrimazole or miconazole cream. Cutaneous candidiasis is a frequent complication of a course of broad-spectum antibiotics, which alters the normal skin flora. Ref: Derm/ Harrison's 16, p. 1185-1187/ PS-RAL
The classic manifestations of cellulitis include all of the following EXCEPT:

A) Erythema
B) Pain
C) Fluctuance
D) Heat
E) Swelling
The correct answer is: C

Redness, tenderness, heat, and swelling (rubor, dolor, calor, and tumor) are classically associated with cellulitis. Fluctuance generally indicates the presence of subcutaneous fluid, and is associated with abscess formation. Ref: Derm/ Harrison's 16, p. 743/ PS-RAL
Infection within the pulp of the distal finger is commonly referred to as which of the following?

A) Paronychia
B) Felon
C) Subungual abscess
D) Flexor tenosynovitis
E) Osteomyelitis
The correct answer is: B

A felon is an infection within the tightly enclosed fibrous compartments of the finger pulp, most tender on the palmar aspect of the distal involved digit. A paronychia is a superficial infection of the epithelium lateral to the nail. This infection may spread locally under the nail forming a subungual abscess. Flexor tenosynovitis is an infection of the flexor tendon sheath resulting in generalized swelling of the digit, exquisite tenderness over the flexor tendon sheath, and excruciating pain on extension of the involved finger. Immediate incision of the sheath with irrigation, elevation, and systemic antibiotics are generally indicated. Ref: Derm/ Goroll 4, p. 867/ PS-RAL
Which is the usual treatment for herpetic infection of the fingertip, or herpetic whitlow?

A) Incision and drainage
B) Topical miconozole
C) Broad-spectrum antibiotics
D) Ultraviolet light treatments
E) The disease is self-limiting and does not require specific treatment.
The correct answer is: E

Herpetic whitlow resolves without treatment in 3 to 4 weeks. It presents with pain and vesicle formation at the fingertip, and is commonly seen in hospital workers. Ref: Derm/ Schwartz 8, p. 1762/ RAL
A patient, whose general appearance is one of poor personal hygiene, presents with a group of erythematous macules with a honey-colored crust and surrounding erythema. What is the most likely diagnosis?

A) Erysipelas
B) Dermatophytosis
C) Varicella
D) Impetigo
E) Rubeola
The correct answer is: D

Impetigo is a bacterial skin infection seen in children and common in adults with poor hygiene. Varicella and Rubeola are viral infections. Dermatophytosis is a fungal infection. Ref: Derm/ Goroll 4, p. 1043/ PS-RAL
Which is the appropriate preparation for examining skin scrapings from a patient with a suspected dermatophyte infection?

A) Gram's stain
B) Potassium hydroxide preparation
C) Wright's stain
D) India ink stain
E) Dark-field examination
The correct answer is: B

The potassium hydroxide preparation clears the epithelial cells, allowing identification of the hyphae that are characteristic of fungal infections. Ref: Derm/ Goroll 4, p. 1045-1046/ PS-RAL
Risk factors for the development of dermatophytosis include all of the following EXCEPT:

A) Diabetes mellitus
B) Glucocorticoid therapy
C) Minor trauma
D) Exposure to ultraviolet light
E) Poor hygiene
The correct answer is: D

Minor trauma, poor hygiene, diabetes and glucocorticoid treatment favor the growth of dermatophytes. Ref: Derm/ Goroll 4, p. 1045; Harrison's 16, p. 1191/ PS-RAL
Which is the major risk factor for cryptococcal infections?

A) AIDS
B) Hepatitis-C
C) Hemophilia
D) Severe burns
E) Diabetes mellitus
The correct answer is: A

AIDS is the most likely predisposing factor. These patients are generally vulnerable to cryptococcal infections when their CD4+ cell counts are below 200. Ref: Derm/ Harrison's 16, p. 1183/ PS-RAL
The treatment of inflammatory acne may include all of the following EXCEPT:

A) Oral tetracycline
B) Oral erythromycin
C) Benzoyl peroxide
D) Topical ketoconozole
E) Topical metronidazole
The correct answer is: D

The oral and topical antibiotics listed are useful in the management of inflammatory acne. Ketoconozole is an antifungal agent not normally used to treat acne. Ref: Derm/ Goroll 4, p. 1028/ PS-RAL
Linear eruptions on the skin, marked by erythema, vesiculation, and severe pruritus are symptoms of allergic plant dermatitis (poison ivy, oak, and sumaC) . Which is the agent commonly used to treat these eruptions?

A) Broad-spectrum antibiotics
B) Antihistamines
C) Glucocorticoids
D) Methotrexate
E) NSAID's
The correct answer is: C

The treatment of allergic contact dermatitis is usually with high potency fluorinated topical glucocorticoids. If widespread or more severe, oral glucocorticoids are necessary. Treatment should be tapered over two to three weeks. If the treatment course is too short, skin lesions will recur. Ref: Derm/ Harrison's 16, p. 289/ PS-RAL
Erythema migrans (a large annular lesion, usually with a bright red outer border and partial clearing centrally) is associated with which of the following diseases?

A) Syphilis
B) HIV
C) Lyme disease
D) Herpes Simplex Virus
E) Psoriasis
The correct answer is: C

Lyme disease, caused by Borrelia burgdorferi, may begin with a characteristic expanding skin lesion, erythema marginatum, at the site of the tick bite. Smaller erythematous skin lesions may develop at distant sites. Ref: Derm/ Harrison's 16, p. 298/ PS-RAL
Intensely pruritic linear lesions mixed with erythematous papules, commonly found between the fingers, on the volar surface of the wrist, the elbows, and elsewhere, should give rise to the suspicion of which disease?

A) Scabies
B) Rickets
C) Lyme disease
D) Zoster
E) Salmonella
The correct answer is: A

Scabies is caused by the mite Sarcoptes scabiei hominis, and is transmitted by close bodily contact. The female measures 0.4 mm and burrows into the superficial epidermis and deposits two to three eggs per day. Diagnosis is by low-power microscopic examination of shavings from infected skin. Ref: Derm/ Goroll 4, p. 1060-1061/ PS-RAL
Which of the lesions is yellowish and associated with certain hyperlipidemias?

A) Xanthelasma
B) Tophi
C) Neurofibroma
D) Neuroma
E) Epidermoid inclusion cysts
The correct answer is: A

Xantalasma (lesions of the eyelids) and xanthomas are related to lipid disorders. Tophi may also be yellowish, but are related to gout and appear on the helix of the ear, elbow and knee. The other lesions are skin-colored. Ref: Derm/ Harrison's 16, p. 305-307/ PS-RAL
Which is the most common melanoma?

A) Lentigo maligna melanoma
B) Superficial spreading melanoma
C) Nodular melanoma
D) Acral lentiginous
E) None of the above
The correct answer is: B

Superficial spreading melanoma accounts for 70% of all melanomas. If excised while it remains superficial, 5-year survival rates approach 100%. Lentigo maligna melanoma is generally seen in elderly patients and has a good prognosis when treated surgically. Nodular melanoma and acral lentiginous melanoma have a poorer prognosis. Ref: Derm/ Goroll 4, p. 996-997/ PS-RAL
Which of the characteristics below is NOT a criterion for suspicion that a pigmented lesion is a melanoma?

A) Asymmetry
B) Color variegation
C) Irregular border
D) Erythema
E) Greater than 6mm in diameter
The correct answer is: D

The mnemonic "ABCD" for "Asymmetry", "Border irregularity", "Color variegation", and "Diameter greater than 6mm" is a way to remember the characteristics of a melanoma. Erythema is not a criterion. Ref: Derm/ Goroll 4, p. 999/ RAL
Intertrigo affects body folds, and is more likely in obese patients and in warm weather. The primary cause is:

A) Bacterial infection
B) Yeast infection
C) Autoimmunity
D) An error of lipid metabolism
E) None of the above
The correct answer is: E

The primary pathologic mechanism is mechanical, although secondary bacterial and yeast infections may occur. Heat and moisture irritate the skin in groin, axillary, and inframammary skin folds, causing erosion of the epidermis. Treatment consists of drying the affected areas, applying steroids topically, and treating bacterial and fugal secondary infections as needed. Ref: Derm/ Goroll 4, p. 1037/ RAL
A 60-year-old man develops right-sided chest wall tingling, itching, and eventually, severe pain. He treats himself with over-the-counter analgesics for three days, until finally presenting with vesicles on an erythematous base in a deramatomal distribution on the chest wall. The most likely diagnosis is:

A) Herpes simplex virus infection, type 1
B) Herpes simplex virus infection, type 2
C) Herpes zoster
D) Varicella
E) Tinea cruris
The correct answer is: C

The prodrome, the cutaneous erythema and vesicles, and the dermatomal distribution are characteristic of herpes zoster. The pathogenic organism is identical to the virus that causes varicella, or chickenpox, in younger individuals. Ref: Derm/ Goroll 4, p. 1054-1055/ RAL
Bilaterally symmetrical, pale red to pink, non-pruritic round macular and papular lesions, approximately 3 -10 mm in size frequently involving the palms and soles are characteristic of what disease?

A) Varicella-zoster
B) Secondary syphilis
C) Impetigo
D) Schistosomiasis
E) Contact dermatitis
The correct answer is: B

The lesions are characteristic cutaneous manifestations of secondary syphilis Ref: Derm/ Harrison's 16, p. 979/ PS-RAL
A sacral decubitus ulcer involving loss of the full thickness of skin, but not involving deeper tissues (muscle and bonE) would be classified as which of the following?

A) Stage I
B) Stage II
C) Stage III
D) Stage IV
E) Stage V
The correct answer is: C

Decubitus ulcers are graded according to the depth of involvement. Stage I ulcers are superficial and appear as erythema. Stage II ulcers are partial thickness. Stage III ulcers are full thickness. Stage IV ulcers expose underlying muscle and bone. The classification system does not include a Stage V. Ref: Derm/ Schwartz 8, p. 1825/ PS-RAL
Which of the following is NOT a common site of pressure (decubitus) ulcers in debilitated patients?

A) Ischium
B) Heel
C) Trochanter
D) Popliteal fossa
E) Sacrum
The correct answer is: D

Trochanteric, sacral, ischial, and heel decubitus ulcers represent anatomic points at which a bony prominence underlies the skin. The popliteal fossa is an unlikely place for a pressure ulcer. Ref: Derm/ Schwartz 8, p. 1825/ RAL
The most likely cause of excessive body hair (hirsutism) in women with amenorrhea is:

A) Abuse of anabolic steroids
B) Polycystic ovary disease
C) Cushing's syndrome
D) Adrenal hyperplasia
E) Chronic phenytoin (Dilantin) use
The correct answer is: B

Hirsutism may be caused by all of the above, but polycystic ovary disease is the most likely cause in women presenting with oligomenorrhea or amenorrhea. Ref: Derm/ Goroll 4, p. 619-620/ PS-RAL
Which is the most common anatomic location of psoriatic plaques?

A) The chest and back
B) Intertriginous folds
C) The palms and plantar surfaces of the feet
D) The mucocutaneous border
E) The extensor surfaces of the extremities
The correct answer is: E

The extensor surfaces are the most common, although any area of skin may be affected. Ref: Derm/ Goroll 4, p. 1033/ RAL
Which of the following systemic agents (or classes of agents) is NOT indicated for the treatment of psoriasis?

A) NSAIDS
B) Cyclosporine
C) Methotrexate
D) Acitretin (SoriatanE) E) All are indicated
The correct answer is: A

NSAID's, beta-blockers, and lithium can exacerbate psoriasis and should be eliminated or minimized. The other systemic agents are commonly used to treat severe disease. Ref: Derm/ Goroll 4, p. 1035-1036/ RAL
This is a skin infection characterized by brown, pink or red scaly patches on the chest back and shoulders. It may also present as hypopigmented lesions in this anatomical distribution.

A) Vitiligo
B) Tinea cruris
C) Tinea capitis
D) Tinea versicolor
E) Rosacea
The correct answer is: D

Tinea versicolor is a fungal infection characterized by the multicolored or hypopigmented lesions described. Vitiligo is an autoimmune hypopigmented lesion. Rosacea is an inflammatory lesion of the face. Tinea cruris and tinea capitis are fungal infections of intertriginous areas and the scalp, respectively. Ref: Derm/ Goroll 4, p. 1009, 1030, 1045/ RAL
Which is a first-line agent in the treatment of post-herpetic neuralgia?

A) Amitriptyline (Elavil) B) Oxycodone (Percocet, etc.) C) Phenytoin (Dilantin) D) Sertraline (Zoloft) E) Lorazapam (Ativan)
The correct answer is: A

The tricyclic antidepressant amitriptyline is a frequently used agent, which is effective in about 50% of patients. Gabapentin (Neurontin) and carbamazepine (Tegretal) may also be used. Narcotics may be required in pain resistant to other treatments. Ref: Derm/ Goroll 4, p. 1056/ RAL
Koplik's spots - small, red, irregular mucous membrane lesions with blue-white centers, appearing one to two days before the onset of maculopapular rash

are associated with which of the following viral infections?

A) Mumps
B) Rubella
C) Rubeola
D) Varicella
E) HIV
The correct answer is: C

The mucous membrane lesions (Koplik's spots) consist of vesicle formation and epithelial necrosis seen early in the course of the now-rare rubeola, or measles. 116 cases were reported in the United States in 2001. Ref: Derm/ Harrison's 16, p. 1148/ RAL
The congenital form of which of the following viral infections has been linked to heart malformations, cataracts, deafness, and intrauterine growth retardation?

A) Mumps
B) Rubella
C) Rubeola
D) Varicella
E) HIV
The correct answer is: B

Rubella, or German measles, is generally a self- limiting febrile exanthem lasting about three days. Congenital rubella, however, resulting from maternal infection during the first trimester of pregnancy, has more serious consequences. The syndrome of congenital rubella may consist of heart malformations, eye lesions, microcephaly, mental retardation, deafness and other manifestations. Ref: Derm/ Harrison's 16, p. 1153/ PS-RAL
What is the most common infectious complication of Varicella?

A) Bacterial superinfection of the skin
B) Aseptic meningitis
C) Varicella pneumonia
D) Hepatitis
E) Nephritis
The correct answer is: A

While all of the above may be associated with varicella, the most common is bacterial superinfection of the skin, usually caused by Streptococcus pyogenes or Staphylococcus aureus. This may result from excoriation of the skin lesions. Gram stain of skin lesions should help clarify the etiology of unusually erythematous and pustulated lesions. Ref: Derm/ Harrison's 16, p. 1043/ PS-RAL
The reactivation of which latent virus, from the dorsal root ganglion, may cause a unilateral vesicular eruption within a single dermatome?

A) Varicella - zoster virus
B) Epstein - Barr virus
C) Cytomegalovirus
D) HIV
E) RSV
The correct answer is: A

Varicella-zoster, commonly known as shingles, is a reactivation of the varicella virus from the dorsal root ganglion. Its skin manifestations generally consist of a unilateral vesicular eruption within a single dermatome. Ref: Derm/ Harrison's 16, p. 1043/ PS-RAL
Which of the following best represents the Parkland formula for the 24-hour resuscitation fluid requirement in burn patients?

A) 4ml / percent body surface area burned/ hour, given as normal saline
B) 2ml / percent body surface area burned/ hour, given as lactated Ringer's solution
C) 4ml / kg of body weight/ percent of body surface area burned/ 24 hours, given as lactated Ringer's solution
D) 1ml / kg of body weight/ hour, given as lactated Ringer's solution
E) 4ml / total body surface area in square meters/ 24 hours, given as normal saline
The correct answer is: C

The Parkland formula for burn resuscitation is: 4ml * weight in kilograms * the percent of the body surface area burned. In this example, the 24-hour fluid requirement is given as: (4ml)(70Kg)(40%), or 11.2 liters of IV fluid, half of which should be given in the first 8 hours. Adjustments are made according to patient response. Ref: Derm/ Schwartz 8, p. 198/ PS-RAL
Fluid resuscitation of burn victims with burns over greater than 20% total body surface area (TBSA) , during the first 24 hours, is approximated by the Parkland formula. Which of the following correctly represents the estimated 24-hour fluid requirement in a 70-kilogram male with 40% of the total body surface area burned?

A) 2.4 liters
B) 3.6 liters
C) 4.8 liters
D) 8.6 liters
E) 11.2 liters
The correct answer is: E

The Parkland formula for burn resuscitation is: 4ml * weight in kilograms * the percent of the body surface area burned. In this example, the 24-hour fluid requirement is given as: (4ml)(70Kg)(40%), or 11.2 liters of IV fluid, half of which should be given in the first 8 hours. Adjustments are made according to patient response. Ref: Derm/ Schwartz 8, p. 198/ PS-RAL
Which is the organism associated with cellulitis following a cat bite?

A) Aeromonas hydrophilia
B) Pasteurella multocida
C) Fusobacterium
D) Pseudomonas aeruginosa
E) Bacteroides species
The correct answer is: B

Pasteurella multocida is commonly seen in infections due to cat bites. It is usually sensitive to penicillin. Ref: Derm/ Harrison's 16, p. 743; Goroll 4, p. 867/ RAL
A patient presents with a troublesome folliculitis following a weekend at a resort spent, in part, relaxing in a hot tub. What organism should be suspected?

A) Streptococcal species
B) Pasteurella multocida
C) Bacteroides species
D) Pseudomonas aeruginosa
E) Aeromonas hydrophila
The correct answer is: D

Typical hot-tub folliculitis is caused by Pseudomona aeruginosa. A third generation cephalosporin or fluoroquinolone are generally effective in treating these infections. Ref: Derm/ Harrison's 16, p. 743/ RAL
A papule is a:

A) flat, colored lesion less than 2cm in diameter.
B) small, fluid filled lesion less than 5cm in diameter.
C) small, solid lesion less than 0.5cm in diameter.
D) vesicle filled with leukocytes.
E) translucent, fluid-filled lesion greater than 0.5cm in diameter.
The correct answer is: C

A papule is solid and less than 0.5cm in diameter. Answer A is a macule; B is a vesicle; D is a pustule; and E is a bulla. Ref: Derm/ Harrison's 16, p. 283/ RAL
The difference between a macule and a patch is that the:

A) macule is smaller.
B) macule is raised.
C) patch is different in color from the surrounding skin.
D) patch has an irregular border.
E) macule contains fluid.
The correct answer is: A

Both lesions are flat and different in color from the surrounding skin. The macule is less than 2cm in diameter, while the patch is greater than 2cm in diameter.
Which of the following is a life-threatening drug reaction characterized by widespread bullae, mucous membrane erosions, ocular lesions, and fever?

A) Stevens-Johnson syndrome
B) Pyogenic granuloma
C) Erythema nodosum
D) Lichen simplex chronicus
E) Rosacea
The correct answer is: A

Stevens-Johnson Syndrome (SJS), also known as toxic epidermal necrolysis, is a drug induced illness which may produce bullae on more than 30% of the body surface, including mucous membranes. Erythema multiforme is a similar syndrome, occurring after systemic infections, typically by herpes simplex virus. Treatment of SJS, which is generally nonspecific and supportive, requires hospitalization, ideally in a burn unit or ICU. Ref: Derm/ Harrison's 16, p. 321; Tintinalli 6, p. 1513/ RAL
A drug reaction characterized by the appearance of pruritic wheals, usually lasting no more than 24 hours is:

A) Epidermolysis
B) Desquamation
C) Urticaria
D) Vasculitis
E) Lichenification
The correct answer is: C

Urticaria, or hives, is one of the most common drug reactions. It may be immediate, occurring within minutes, or develop several days after exposure, but individual lesions rarely last longer than 24 hours. Ref: Derm/ Harrison's 16, p. 319-320/ RAL
This skin reaction, involving deep dermal and subcutaneous elements, may be part of a life-threatening anaphylactic reaction, resulting in airway obstruction and circulatory collapse.

A) Vasculitis
B) Desquamation
C) Angioedema
D) Epidermal necrolysis
E) Purpura
The correct answer is: C

Angioedema may involve mucous membranes and thus, cause airway obstruction. Drugs notorious for this type of reaction are ACE inhibitors, radiographic contrast, and NSAID's. Epinephrine and glucocorticoids are the mainstay of treatment. Ref: Derm/ Harrison's 16, p. 319-320/ RAL
This drug may cause a serious skin reaction characterized by hemorrhagic bullae and skin necrosis. Patients with protein C deficiency are at high risk.

A) Chlorpromazine (ThorazinE) B) Warfarin (Coumadin) C) Ibuprofen (Motrin) D) Phenytoin (Dilantin) E) Penicillin
The correct answer is: B

Warfarin causes a sudden drop in protein C, which may result in a transient hypercoagulable state in protein C-deficient patients. This, in turn, may result in thrombosis in the cutaneous microvasculature and skin necrosis. The reaction is prevented if patients with protein C deficiency are anticoagulated with heparin prior to the start of oral anticoagulation.
Which is the area most commonly affected by allergic contact dermatitis (ACD) ?

A) Hands
B) Neck and face
C) Groins and intertriginous folds
D) Trunk
E) Lower extremities
The correct answer is: A

The hands are most often affected, often due to occupational exposures. If it is not possible to identify or avoid the specific sensitizing agent, hand protection should be worn

preferably vinyl gloves, as latex may itself cause serious hypersensitivity reactions. Ref: Derm/ Harrison's 16, p. 289-290/ RAL
A middle age man presents with skin lesions on the dorsum of his hands and the pretibial areas. He reports that each one began as a small, edematous papule. These have now progressed to pruritic, oval "coinlike" lesions. A few have crusted. What is the most likely diagnosis?

A) Psoriasis
B) Seborrheic dermatitis
C) Impetigo
D) Tinea versicolor
E) Numular eczema
The correct answer is: E

The presentation is typical for numular eczema. The disease is most common in middle-aged men. Treatment is generally with topical steroids. Ref: Derm/ Harrison's 16, p. 290; Goroll 4, p. 1023/ RAL
Which statement concerning topical steroid preparations is TRUE?

A) Ointments should be used on dry, scaling lesions.
B) In general, facial lesions should be treated with higher potency formulations.
C) Systemic absorption and adrenal suppression is a frequent clinical problem.
D) Higher-potency formulations should be used in the groin and axilla.
E) Creams are more potent than ointments.
The correct answer is: A

In general, ointments are more potent; they are best used on thick, scaling lesions. High potency formulations should not be used in the groin, axilla (where they may cause striaE) , or on the face (where they may cause rosaceA) . Systemic absorption and adrenal suppression is rarely clinically significant. Ref: Derm/ Goroll 4, p. 1024/ RAL
This skin disorder is characterized by pruritic, erythematous macules that can coalesce to form patches or plaques. Lesions in adults are typically located in the popliteal or antecubital fossae. There is a strong familial predisposition, and most patients develop concurrent asthma or allergic rhinitis.

A) Psoriasis
B) Vitiligo
C) Atopic dermatitis
D) Actinic keratoses
E) Stasis dermatitis
The correct answer is: C

There is a clear genetic predisposition to this pruritic dermatitis, which may present as early as the first year of life. 80% of patients eventually develop allergic rhinitis or asthma. Ref: Derm/ Harrison's 16, p. 288-289/ RAL
Which is the most common site of involvement in stasis dermatitis?

A) Extensor surfaces of the arms
B) Groins
C) Medial aspect of the ankle
D) Palmar and plantar surfaces
E) Scalp
The correct answer is: C

Stasis dermatitis is characterized by erythema and scaling, usually in the region of the medial malleoli, and may progress to brawny discoloration or ulceration. It is caused by venous incompetence in the lower extremities. Treatment includes topical glucocorticoids, elastic support hose, and leg elevation. Ref: Derm/ Harrison's 16, p. 290-291/ RAL
An elderly male outpatient presents with skin lesions of the ears and nasolabial folds. The lesions consist of greasy scales overlying erythematous patches. What is the most likely diagnosis?

A) Candidiasis
B) Seborrheic dermatitis
C) Lichen Planus
D) Pityriasis rosea
E) Impetigo
The correct answer is: B

Seborrheic dermatitis commonly affects the scalp, eyebrows, nasolabial folds, and ears. The scaling lesions on an erythematous base usually respond to low-potency topical glucocorticoids. Ref: Derm/ Harrison's 16, p. 291/ RAL
This skin disorder is characterized by pruritic, polygonal violaceous papules. A network of gray lines, called Wickham's striae, is often seen on the surface of the lesions. The lesions are seen on the wrists, shins, lower back, genitalia, and may affect the scalp, where it can cause alopecia. What is the most likely diagnosis?

A) Dermatophytosis
B) Pityriasis rosea
C) Rosacea
D) Lichen planus
E) Tinea versicolor
The correct answer is: D

Lichen planus may also affect the buccal mucosa. The cause is unknown, although the eruptions respond to topical steroids. Ref: Derm/ Harrison's 16, p. 292/ RAL
This skin disorder is characterized by brown to red oval papules, mildly pruritic, that align themselves along skin folds on the trunk. It occurs most often in the spring or fall. The palms and soles are spared.

A) Secondary syphilis
B) Tinea versicolor
C) Erysipelas
D) Herpes zoster
E) Pityriasis rosea
The correct answer is: E

Pityriasis rosea may be differentiated from the skin manifestations of secondary syphilis by the sparing of the palms and soles. The disease may develop a few days to weeks after a herald patch 2-6cm in diameter is noted. Antihistamines, medium-potency topical glucocorticoids, and UV phototherapy have been used for treatment. Ref: Derm/ Harrison's 16, p. 292/ RAL
This skin disorder affects the central face and is generally seen in patients over 30 years of age. Women are more commonly affected, but men may manifest a particularly severe form of the disease. Patients have a pronounced tendency toward facial flushing from many stimuli, which may become permanent with the development of overlying telangiectasias. Papules and pustules may also be superimposed.

A) Acne vulgaris
B) Rosacea
C) Chronic Eczematous dermatitis
D) Lichen planus
E) Tinea capitis
The correct answer is: B

Rosacea is a chronic inflammatory condition of the central face, most commonly affecting individuals of Irish, Scottish, and English descent. A severe form of this disorder may cause lobulated overgrowth of connective tissue of the nose, rhinophyma. Topical metronidazole is used to treat the skin eruptions of rosacea. Ref: Derm/ Goroll 4, p. 1030-1031; Harrison's 16, p. 295/ RAL
Which of the following dermatoses is a fungal infection?

A) Erysipelas
B) Lichen planus
C) Tinea versicolor
D) Stasis dermatitis
E) Impetigo
The correct answer is: C

Tinea versicolor is a fungal skin infection characterized by brown, pink, red or white patches with scaling on the chest, back and shoulders. The diagnosis may be confirmed by viewing scrapings in a KOH prep, which will demonstrate hyphae and spores. The causal agent is Malassezia furfur. Erysipelas and impetigo are bacterial infections. Lichen planus is an inflammatory lesion that may be precipitated by various drug exposures. Stasis dermatitis is caused by venous insufficiency. Ref: Derm/ Goroll 4, p. 1045; Harrison's 16, p 293-294/ RAL
Terbinafine (Lamisil) is an oral agent used for the treatment of onychomycosis, or fungal nail infections. What laboratory monitoring is required for patients on this medication?

A) CBC and creatinine
B) CBC and liver enzymes
C) CBC and platelets
D) Serum potassium
E) Creatine kinase (CK)
The correct answer is: B

Terbinafine has the potential for bone marrow suppression and liver injury. Treatment for toenail infection is continued for 12 weeks. Treatment for fingernail involvement lasts for 6 weeks. Recurrences after treatment are common. Ref: Derm/ Goroll 4, p. 1047/ RAL
The etiologic agent in molluscum contagiosum is which of the following?

A) A poxvirus
B) Human papillomavirus
C) A budding yeast
D) A mycobacterium
E) A spirochete
The correct answer is: A

Molluscum contagiosum is caused by a poxvirus related to the agent that causes smallpox in humans. The characteristic lesion, a flesh-colored papule 2-5mm in diameter with a central dimple, may occur anywhere except the palms and soles. Transmission is by close contact, including sexual contact. There is no specific treatment, but the lesions tend to heal spontaneously after 3 to 4 months. Ref: Derm/ Harrison's 16, p. 1053/ RAL
The etiologic agent causing common warts is:

A) Human papillomavirus (HPV) B) Herpes simplex virus (HSV) C) Varicella-zoster virus (VZV) D) Cytomegalovirus (CMV) E) Epstein-Barr virus (EBV)
The correct answer is: A

Warts are caused by human papillomavirus. They are flesh-colored, generally painless papules. Warts on the weight-bearing surface of the foot, however, plantar warts, may be painful. Ref: Derm/ Goroll 4, p. 1056-1059/ RAL
Treatment for common warts (verruca vulgaris) includes all of the following EXCEPT:

A) Cryosurgery
B) Electrodessication
C) High-potency topical steroids
D) Topical antimetabolites
E) Topical nitric acid
The correct answer is: C

Treatment is by ablation of the lesion with various agents, including: liquid nitrogen, 5-fluorouracil (an antimetabolitE) , nitric acid, and electrical current. The goal is to abolish the lesion with a minimum of scarring. If untreated, about two-thirds of warts will resolve spontaneously. Ref: Derm/ Goroll 4, p. 1056-1059/ RAL
Which neoplasm has been linked to infection with human papillomavirus?

A) Malignant melanoma
B) Basal cell carcinoma
C) Cervical carcinoma
D) Endometrial carcinoma
E) Soft-tissue sarcomas
The correct answer is: C

Human papillomavirus has been implicated in the etiology of cervical carcinoma and in situ squamous cell carcinoma (Bowen's diseasE) . Ref: Derm/ Harrison's 16, p. 294; Goroll 4, p. 1056-1057/ RAL
A woman with symmetrical, frontoparietal hair loss at age 35 should be described as having:

A) Female-pattern baldness
B) Alopecia totalis
C) Alopecia areata
D) Patchy alopecia
E) Androgenetic alopecia
The correct answer is: E

Male-pattern baldness (androgenetic alopeciA) begins symmetrically in the frontoparietal scalp and is an inherited characteristic. Female-pattern baldness is diffuse and incomplete. Androgenetic alopecia in a female patient requires investigation. Ref: Derm/ Goroll 4, p. 1016/ RAL
Appropriate work-up for an adolescent woman with androgenetic alopecia would include which of the following investigations?

A) A detailed drug history
B) Serum testosterone levels
C) Urinary 17-ketosteroids
D) Pelvic ultrasound
E) All of the above
The correct answer is: E

Male-pattern baldness in women may be caused by performance-enhancing anabolic steroids, or by a virilizing tumor of the ovaries or adrenal glands. The blood and urine tests may detect virilizing hormones, and pelvic ultrasound or abdominal CT scan may demonstrate the anatomical location of a virilizing tumor. Ref: Derm/ Harrison's 16, p. 297-298; Goroll 4, p. 619-621/ RAL
Rapid hair loss from the scalp in round patches is known as:

A) Androgenetic alopecia
B) "Moth-eaten" alopecia
C) Alopecia areata
D) Female-pattern baldness
E) Telogen effluvium
The correct answer is: C

Alopecia areata refers to hair loss in circular patterns. Although the etiology is unknown, it is occasionally associated with hyperthyroidism, and it is reasonable to check thyroid function studies when it is encountered. Ref: Derm/ Tintinalli 6, p. 1525/ RAL
Nikolsky's sign is present in which of the following conditions?

A) Essential telangiectasia
B) Pemphigus vulgaris
C) Disseminated cryptococcosis
D) Vitiligo
E) Erythema marginatum
The correct answer is: B

Pemphigus vulgaris is an autoimmune disease characterized by the formation of multiple large bullae that may involve a significant portion of the total body surface area. It is a life-threatening disease. Nikolsky's sign is present when manual pressure on the skin results in separation of the epidermis from the dermis. It is positive in pemphigus vulgaris, but negative in bullous pephigoid, a similar, but much less life-threatening bullous disease. Ref: Derm/ Harrison's 16, p. 311-314; Tintinalli 6, p. 519-520/ RAL
An acute, generalized maculopapular rash, usually related to viral infection or drug reaction, is called:

A) Piebaldism
B) An exanthem
C) Eczema
D) Dermatitis
E) A dermatophytosis
The correct answer is: B

An exanthem is an acute generalized skin eruption. Typical viral exanthems of childhood are rubella and rubeola. Morbilliform is a term applied to an acute exanthem comprised of macules and papules. Ref: Derm, Harrison's 16, p. 304/ RAL
Which is a premalignant growth that may undergo malignant transformation to become a squamous cell carcinoma?

A) Actinic keratoses
B) Telangiectasias
C) Acanthosis
D) Scleroderma
E) Kaposi's sarcoma
The correct answer is: A

Approximately 1 out of 1000 actinic keratoses undergo malignant transformation to become squamous cell carcinomas. Lesions occur in sun-exposed areas and may be red-brown with adherent scale. Actinic keratoses and squamous cell carcinoma in situ (Bowen's diseasE) can be treated with cryotherapy. More advanced squamous cell carcinomas are generally treated with surgical excision. Ref: Derm/ Goroll 4, p. 996; Harrison's 16, p. 325/ RAL
Which of the following is characterized by chalk-white macules of the face, trunk, and extremities?

A) Psoriasis
B) Lichen planus
C) Vitiligo
D) Actinic keratosis
E) Atopic dermatitis
The correct answer is: C

Vitiligo is an autoimmune, depigmenting skin condition that may be associated with other autoimmune conditions, including: pernicious anemia, Hashimoto's thyroiditis, male hypogonadism, and diabetes mellitus. Ref: Derm/ Harrison's 16, p. 285; Goroll 4, p. 1009/ RAL
Which is a chronic dermatitis characterized by well-circumscribed plaques of thickened skin, scaling, and pruritus, typically occurring on the wrists, thighs, and occipital region?

A) Lichen simplex chronicus
B) Numular eczema
C) Contact dermatitis
D) Seborrheic dermatitis
E) Melasma
The correct answer is: A

Lichen simplex chronicus is a condition that may result from eczema or neurodermatitis in the locations described. Ref: Derm/ Goroll 4, p. 1023/ RAL
Which is first-line therapy for pediculosis capitis?

A) Lindane shampoo (Kwell) B) Domeboro compresses
C) Selenium sulfide (Selsun) shampoo
D) Nizoral shamoo
E) Permethrin (NIX) rinse
The correct answer is: E

Permethrin 1% is the agent of choice

it kills both the parasite and their eggs (nits). Ref: Derm/ Tintinalli 6, p. 1536/ RAL
The body louse (Pediculus corporis) is the insect vector for which of the following diseases?

A) Bubonic plague
B) Epidemic typhus fever
C) Hepatitis A
D) Equine encephalitis
E) Hansen's disease
The correct answer is: B

Typhus is caused by Rickettsia prowazekii and spread by Pediculus corporis. Ref: Derm/ Tintinalli 6, p. 1536/ RAL
Which of the following is appropriate treatment for the bite of the brown recluse spider, with signs of cellulitis, in a patient without systemic symptoms?

A) Immediate surgical debridement
B) Oral antibiotics
C) Oral glucocorticoids
D) Local infiltration of glucocorticoids
E) None of the above
The correct answer is: B

Oral antibiotics are appropriate if there are signs of infection. Surgical debridement should be delayed for 2-3 weeks to allow complete demarcation of the wound. Steroids are not useful. Ref: Tintinalli 6, p. 1195/ RAL
This hyperpigmented lesion, consisting of darkening of the skin folds of the neck, elbows, and interphalangeal spaces, may be associated with obesity, insulin resistance, or may represent a paraneoplastic syndrome.

A) Pigmented nevi
B) Seborrheic keratoses
C) Cafe au lait macule
D) Acanthosis nigricans
E) Ephelides
The correct answer is: D

Acanthosis nigicans is a velvety hyperpigmentation seen in flexural areas of skin. It may be associated with endocrine disorders or gastrointestinal malignancies. Ref: Derm/ Harrison's 16, p. 301-302, 427/ RAL
Which of the following conditions is described as symmetric hyperpigmented, brown spots on the cheeks, upper lid, and forehead?

A) Melasma
B) Urticaria pigmentosa
C) Pigmented actinic keratoses
D) Ephelides
E) Acanthosis nigricans
The correct answer is: A

Melasma has been described in women who use oral contraceptives, in pregnancy, and in patients taking the anticonvulsant phenytoin (Dilantin). Ref: Derm/ Harrison's 16, p. 302/ RAL
What is the usual pathogen in folliculitis?

A) Staphylococcus aureus
B) Beta-hemoloytic streptococcus
C) Proteus mirabilis
D) Propionibacterium acnes
E) Pseudomonas aeruginosa
The correct answer is: A

Staphylococcal species are the most common. P. acnes folliculitis occasionally affects the occipital area in men. Pseudomonas species may cause folliculitis in patients using hot tubs. Ref: Derm/ Harrison's 16, p. 817; Goroll 4, p. 1043/ RAL
The primary target of the HIV virus is the human immune system, causing profound immunodeficiency, which in turn leads to opportunistic infections. The specific target of the virus is:

A) Segmented Neutrophils
B) Antibody producing B lymphocytes
C) CD 4+ helper T lymphocytes
D) CD 8+ suppresser T lymphocyte
E) Vascular endothelium
The correct answer is: C

HIV specifically infects CD 4+ lymphocytes. CD 4+ cells are responsible for the secretion of cytokine messages to the immune system that are the regulatory messages of normal immune function. Ref: Heme/ Harrison's 16, p. 1077/WL-RAL
A 28-year-old male comes to the emergency room with complaints of shortness of breath, cough, and fever that has progressed over the last 4-5 days. He was diagnosed as HIV positive 2 months ago, but has not started any specific antiviral treatment. Chest X-ray shows a patchy alveolar infiltrate in the right lower lobe. The most likely diagnosis is:

A) Tuberculosis
B) Pneumocystis carinii pneumonia
C) Community acquired lobar pneumonia
D) Cytomegalovirus interstitial pneumonia
E) Haemophilus influenzae pneumonia
The correct answer is: B

The most common pneumonia in patients with HIV disease is pneumocystis pneumonia, although it is now better controlled with antiretroviral therapy and propylactic antibiotics. Ref: Hematol/ Harrison's 16, p. 1105/ WL-RAL
What percent of patients diagnosed with AIDS will have Kaposi's sarcoma?

A) Less than 1%
B) 10 to 20%
C) About 50%
D) 80-90-%
E) Nearly 100%
The correct answer is: A

Although 79% of patients diagnosed in 1981 had AIDS, by 1997 less than 1% of patients had the neoplastic skin disease. Ref: Heme/ Harrison's 16, p. 1120/ WL-RAL
A 24-year-old nulliparous female comes to your office with complaints of fatigue, shortness of breath with exertion, and occasional palpitations. Onset has been gradual, and she has noticed a craving for certain foods such as salad. Physical exam is normal, past medical history unremarkable, and she is not taking any medications. CBC is remarkable for a hematocrit of 30%. Blood smear shows hypochromic, microcytic red cells. Serum ferritin is low. The most likely diagnosis is:

A) Iron deficiency anemia
B) Folate deficiency
C) Pernicious anemia
D) Vitamin B12 deficiency
E) Acute hemorrhage
The correct answer is: A

Iron deficiency anemia is the most common anemia. Fatigue, exertional dyspnea, and occasionally, palpitations are characteristic of iron deficiency anemia. Red cells are classically microcytic and hypochromic. Serum ferritin is low. Folate deficiency, B12 deficiency, and pernicious anemia are all megaloblastic anemias. Ref: Heme/ Harrison's 16, p. 589/ WL_RAL
Which is NOT a typical finding in acute myeloid leukemia (AML)?

A) Fatigue
B) Abnormal bruising
C) Hypochromic, microcytic anemia
D) Thrombocytopenia
E) Splenomegaly
The correct answer is: C

The anemia is typically normochromic and normocyic. All other choices represent frequent presenting complaints, laboratory findings, or physical findings in AML. Ref: Heme/ Harrison's 16, p. 633/ WL-RAL
A 58-year-old post menopausal woman comes to your office with a constellation of complaints that includes significant and worsening lower back pain, fatigue, and frequent episodes of bacterial pharyngitis. Work up reveals anemia, compression fractures T12, L1, and L3, elevated serum creatinine and serum calcium. The most likely diagnosis is:

A) Post-menopausal osteoporosis
B) Amyloidosis
C) Multiple myeloma
D) Lupus Erythematosis
E) Primary hyperparathyroidism
The correct answer is: C

Multiple myeloma is a disease of older patients characterized by plasma cell proliferation with lytic bone lesions causing pathologic/compression fractures, anemia and hypercalcemia from bone breakdown. Renal insufficiency can be caused by plasma cell infiltration of the kidney and/or a function of hypercalcemia. Recurrent bacterial infections are common. Ref: Heme/ Harrison's 16, p. 657-658/ WL-RAL
The virus that causes mononucleosis is:

A) Cytomegalovirus
B) Herpes Zoster
C) Epstein-Barr virus
D) An adenovirus
E) A retrovirus
The correct answer is: C

Epstein-Barr virus causes infectious mononucleosis. Typical findings are fever, sore throat, lymphadenopathy, and atypical lymphocytosis. Ref: Heme/ Harrison's 16, p. 1046/ WL-RAL
A sixteen-year-old male comes to your office with a week-long history of progressive malaise, fever, and increasingly painful pharyngitis. On exam, you find cervical lymphadenopathy, and laboratory work-up is remarkable for a lymphocytosis on CBC and positive heterophile antibody test. The most likely diagnosis is:

A) Exudative streptococcal pharyngitis
B) Gonorrheal pharygitis
C) Herpetic pharyngitis
D) Infectious mononucleosis
E) Ludwig's angina
The correct answer is: D

The positive heterophile antibody test, with a titer of 40-fold or greater, is conclusive for infectious mononucleosis in a patient with symptoms compatible with the disease and atypical lymphocytes seen in a peripheral smear. Ref: Heme/ Harrison's 16, p. 1047/ WL-RAL
A patient is placed on Coumadin therapy for chronic atrial fibrillation. In order to monitor the effectiveness of the anticoagulation the most important test is:

A) Prothrombin Time (PT)/International Normalized Ratio (INR) B) Partial Thromboplastin Time (PTT) C) Bleeding Time
D) Platelet count
E) Hematocrit
The correct answer is: A

PT/INR is the study used to monitor oral anticoagulants, since these medications lower the levels of prothrombin and factors VII and X. Ref: Heme/ Harrison's 16, p. 690/ WL-RAL
The most frequent type of transfusion reaction is the febrile, nonhemolytic transfusion reaction (FNHTR). Which symptom would be LEAST LIKELY to occur in a patient having this reaction?

A) A 1ŒÁ C rise in temperature
B) Chills
C) Rigors
D) Hemaglobinuria
E) All are unlikely
The correct answer is: D

Hemoglobinemia and hemoglobinuria are seen only in hemolytic transfusion reactions. The other symptoms are typical of FHNTR. Ref: Heme/ Harrison's 16/ p. 665/ WL-RAL
In disseminated intravascular coagulation (DIC) the basic problem is:

A) Inappropriate formation of thrombus
B) Hemorrhage
C) Increased blood viscosity
D) Both A and B
E) None of the above
The correct answer is: D

In DIC there is inappropriate activation of the coagulation mechanism, with deposition of thrombi in the microvasculature, followed by a coagulopathy due to consumption of clotting factors and platelets, leading to hemorrhagic complications. Ref: Heme/ Harrison's 16, p. 683-684/ WL-RAL
Approximately 40% of HIV infected patients are diagnosed with some form of cancer. Which of the following malignancies is most commonly associated with HIV?

A) Lymphoma
B) Leukemia
C) Gastric cancer
D) Testicular cancer
E) Rectal carcinoma
The correct answer is: A

Patients with AIDS have about a 120-fold greater risk of developing lymphoma than the general population. This is by far the most common malignancy of the choices listed in AIDS patients. Ref: Heme/ Harrison's 16, p. 1120-1122/ WL-RAL
A 26-year-old female presents with complaints of progressive left upper quadrant abdominal pain unassociated with trauma. On exam, the spleen tip is palpable 8 centimeters below the left costal margin on deep inspiration. You should:

A) Refer the patient to a gastroenterologist, as she is most likely having some form of stomach problem that is displacing the spleen.
B) Tell the patient that an enlarged spleen is common with infection and start her on oral antibiotics.
C) Institute a full work up, as splenomegaly is almost always abnormal and may be the first sign of diseases such as chronic myeloid leukemia.
D) Give the patient an appointment in a week, as an isolated finding of splenomegaly in the absence of other symptoms should be followed serially before further work up.
E) This innocuous finding can be treated with reassurance only.
The correct answer is: C

Splenomegaly of this magnitude is highly associated with serious illnesses, such as non-Hodgkin's lymphoma, chronic lymphocytic leukemia, hairy cell leukemia, chronic myelogenous leukemia, myelofibrosis, myeloid metaplasia, and polycythemia vera. Evaluation, including abdominal imaging studies and CBC, are required in determining the significance of this physical finding. Ref: Heme/ Harrison's 16, p. 347/ WL-RAL
An AIDS patient with a declining CD 4+ T cell counts and recurrent, painful perirectal lesions, should be considered for prophylaxis with which of the following agents?

A) Penicillin
B) Trimethoprim-sulfamethoxazole (Bactrim) C) Acyclovir
D) Zidovudine
E) Stavudine
The correct answer is: C

A likely cause of the lesions is infection with or reactivation of herpes simplex virus (HSV). Future outbreaks may be lessened or prevented with acyclovir or famciclovir. Antibiotics are not effective against HSV. Antiretroviral agents zidovudine and stavudine would be helpful only indirectly, and could hardly be considered prophylaxis. Ref: Heme/ Harrison's 16, p. 1115/ WL-RAL
A 76-year-old man presents in your office with a new onset of a moderately painful rash. The rash is vesicular and is in a band-like distribution ranging from the spine around the left side of the chest without crossing the midline to the other side. The most likely diagnosis is:

A) Herpes Simplex 1 (HSV-1) dermal infection
B) Impetigo
C) Varicella zoster virus (VZV) reactivation
D) Roseola
E) Psoriasis
The correct answer is: C

A vesicular rash confined to dermatomal distribution that doesn't cross the midline is almost always Varicella zoster reactivation. It is common in immunocompromised patients as well as the elderly. Ref: Heme/ Harrison's 16, p. 1042-1043/ WL-RAL
Polycythemia is defined as a condition in which:

A) the total white blood cell count is greater than 12,000.
B) the total platelet count is greater than 1,000,000.
C) the hematocrit percentage is above the upper limit of normal.
D) the hemoglobin level is greater than 75% of the total hematocrit percentage.
E) there is a pathologic elevation of the red cell mass, causing thrombotic complications.
The correct answer is: C

Polycythemia is defined simply as an increase in red blood cells above normal. It may be real or represent an apparent increase due to a decrease in plasma volume. It may be a physiologic adaptation (to smoking or living at high altitudE) , a primary disorder, or secondary to an overproduction of erythropoietin. Ref: Heme/ Harrison's 16, p. 335-336/ WL-RAL
Which of the following laboratory tests would be most useful in evaluating a patient with polycythemia?

A) Prothrombin time (PT, INR) B) Partial thromboplastin time (PTT) C) Bleeding time
D) Thyroid-stimulating hormone (TSH) level
E) Erythropoietin (EPO) level
The correct answer is: E

An EPO level will help determine whether the polycythemia represents a physiologic response to tissue hypoxia or the result of the autonomous production of erythropoietin (if levels are high), or may represent a primary overproduction of red cells, polycythemia vera (if levels are low). Ref: Heme/ Harrison's 16, p. 336/ RAL
Fever can be one of the presenting signs of many disorders, the most likely of which are infections. Which of the following represents the most likely noninfectious cause of fever?

A) Trauma
B) Hypersensitivity disorders
C) Factitious fever
D) Malignancy
E) Drug fevers
The correct answer is: D

Of these choices, malignancy is the most likely noninfectious cause of fever. In some recent series, however, noninfectious inflammatory diseases, such as polymyalgia rheumatica, lupus, and sarcoidosis, were the second most likely source of fever. Ref: Heme/ Harrison's 16, p. 117/ WL-RAL
In septic shock:

A) Hypotension generally responds to intravenous volume repletion.
B) Peripheral vascular resistance is increased.
C) Cardiac output is decreased.
D) Gram-negative bacteria are always the cause.
E) Adult respiratory distress syndrome (ARDS) develops about 50% of the time.
The correct answer is: E

In septic shock cardiac output is increased, while peripheral vascular resistance is decreased. Hypotension generally does not respond to volume resuscitation. Gram-positive bacteria, Gram-negative bacteria, or fungi may be the cause. ARDS develops in about 50% of cases. Ref: Heme/ Harrison's 16, p. 1606-1609/ RAL
In the acute treatment of the patient in septic shock, the first priority is:

A) identifying the underlying organism to provide appropriate antimicrobial therapy.
B) sustaining the blood pressure with fluids and/or vasoactive drugs.
C) monitoring the cardiac rhythm.
D) inserting a catheter to monitor central pressures.
E) administering glucocorticoids.
The correct answer is: B

Only about one-third of patients will respond to fluid resuscitation; the remainder may require support with inotropes or vasopressors. Effective empirical antibiotic treatment and cardiovascular monitoring is also essential while supporting circulation. Adrenal insufficiency may be treated presumptively while awaiting the result of diagnostic laboratory tests. Ref: Heme/ Harrison's 16, p. 1610-1611/ WL-RAL
The volume replacement of initial choice in the treatment of septic shock is which of the following?

A) Whole blood
B) Dextran volume expanders
C) Isotonic saline
D) Heparinized saline
E) Fresh-frozen plasma
The correct answer is: C

Isotonic, or normal saline is easily available and can be infused very rapidly. Blood products are not usually as readily available and whole blood is almost never available. Colloids and other volume expanders can impair coagulation. Fresh-frozen plasma is useful in treating the coagulopathy of disseminated intravascular coagulation (DIC) , when it occurs, but would not otherwise be the volume expander of choice. Ref: Heme/ Harrison's 16, p. 1610/ WL-RAL
HIV infections are documented world-wide. The most seriously affected geographic area is:

A) North America.
B) Asia.
C) South and Central America.
D) Sub-Saharan Africa.
E) Eastern Europe.
The correct answer is: D

The adult prevalence of HIV/AIDS in Sub-Saharan Africa is about 8%. There are 26.6 million cases, or about two-thirds of all the world's 40 million cases in that region. By comparison, the adult prevalence of HIV/AIDS in North America is 0.6%. Ref: Heme/ Harrison's 16/ p. 1083-1084/ WL-RAL
The most common form of pulmonary opportunistic infection in HIV infected patients is Pneumocystis carinii. Which is the preferred prophylaxis for P. carinii infection?

A) Inhaled pentamidine weekly
B) Trimethoprim/ Sulfamethoxazole (TMP/SMX, Bactrim) C) Levofloxacin
D) Diflucan
E) Acyclovir
The correct answer is: B

TMP/SMX, one double-strength tablet daily, is effective prophylaxis for P. carinii and should be provided for any HIV/AIDS patient with a CD4+ T cell count of less than 200; with a previous history of P. carinii pneumonia; with unexplained fever for longer than 2 weeks; or with oral candidiasis. Ref: Heme/ Harrison's 16, p. 1105-1107/ WL-RAL
A 38-year-old male comes to you for an insurance physical exam. He states he feels well, is quite active, takes no medications, and has no significant medical history. His exam is benign, but on CBC he is noted to have a WBC of 110,000, made up primarily of mature neutrophils, and his platelet count is 375,000. The most likely explanation is:

A) a reactive leukocytosis.
B) chronic lymphocytic leukemia.
C) chronic myelogenous leukemia.
D) occult infection.
E) acute myeloid leukemia.
The correct answer is: C

Chronic myelogenous leukemia may well present in an innocuous manner. It is a condition that can go unnoticed for quite awhile. The patient's history argues against a reactive leukocytosis or occult infection and the differential of predominantly neutrophils rules out chronic lymphocytic leukemia. Patients with acute myeloid leukemia are likely to present with constitutional symptoms. Ref: Heme/ Harrison's 16, p. 633, 637/ WL-RAL
A 47-year-old Hispanic female comes to your office with clearly apparent exudative pharnygitis. She states that she is allergic to penicillin and her family has been treated with chloramphenicol in the past in her native country. You choose not to use this agent because you think:

A) it is not broad enough coverage for her pharyngitis.
B) there are more modern and expensive antibiotics to choose from.
C) chloramphenicol is not available on most formulates.
D) that chloramphenicol is associated with aplastic anemia.
E) that the incidence of pseudomembranous colitis is unacceptable.
The correct answer is: D

Chloramphenicol is widely available outside the United States without the need of a prescription. It has a broad pattern of coverage, but is clearly associated with drug induced irreversible aplastic anemia and should never be used if there is any other antibiotic alternative. Ref: Heme/ Harrison's 16, p. 802/ WL-RAL
A 57-year-old construction worker presents in your office with a two day history of streaking, painful erythema up his upper right arm into his axilla. He notes no trauma to the area but does remember removing a splinter from the area a few days ago. Exam is notable for tender right axillary adenopathy. Your tentative diagnosis is which of the following?

A) Septic thrombophlebitis
B) Superficial cellulitis
C) Impetigo
D) Lymphadenitis
E) Lymphoma
The correct answer is: D

Erythematous streaking (lymphangitis) arising from a wound toward regional lymph nodes that are swollen and tender is the hallmark of lymphadenitis. This is usually caused by Staphylococcal or Streptococcal bacteria and needs to be treated aggressively with antibiotics to prevent systemic sepsis. Superficial cellulitis rarely involves regional lymph nodes, and thrombophlebitis is rare in the upper extremities, usually doesn't involve lymph nodes, and doesn't arise from a wound. Ref: Heme/ Goroll 4, p. 64/ WL-RAL
Opportunistic infections can be a deadly consequence of HIV infection. Which of the following is NOT a common opportunistic infection associated with HIV?

A) Mycoplasma avium intercellular pneumonia
B) Cytomegalovirus retinitis
C) Pneumocystis carinii pneumonia
D) Kaposi's sarcoma
E) Cryptosporidial gastrointestinal infections
The correct answer is: D

Kaposi's sarcoma is a malignant neoplasm seen in AIDS patients, not an infection. Ref: Heme/ Harrison's 16, p. 1120/ WL-RAL
Which of the following studies in HIV infected patients most closely predict the risk of opportunistic infection?

A) CBC with differential
B) Absolute CD 4+ T cell count
C) HIV viral load
D) P24 antigen positivity
E) ELISA
The correct answer is: B

HIV preferentially infects CD4+ T cells, which are essential for both humoral and cell-mediated immune function. Therefore, absolute CD4+ T cell count, especially when less than 200 cells/εL, correlates highly with risk of infection. Ref: Heme/ Harrison's 16, p. 1087/ WL-RAL
What is the vector for Lyme disease?

A) Ticks
B) Mosquitoes
C) House flies
D) Fleas
E) Rats
The correct answer is: A

Lyme disease is spread by the bite of ticks of the Ixodes genus that harbor Borrelia burgodorferi, a spirochete, the pathogenic organism causing the disease. Ref: Heme/ Harrison's 16, p. 995/ WL-RAL
Pernicious anemia is associated with a deficiency in:

A) Folate
B) Iron
C) Vitamin C
D) Vitamin B12
E) Vitamin B6
The correct answer is: D

Pernicious anemia is a disorder generally caused by the inability to absorb cobalamin or Vitamin B12. Poor absorption of natural cobalamin may be caused by inadequate gastric acid secretion (achlorhydriA) or inadequate secretion of intrinsic factor (IF), produced by parietal cells in the stomach. Pernicious anemia often follows total gastrectomy. In the intact GI tract, the cobalamin-IF complex is eventually absorbed in the terminal ileum. Ref: Heme/ Harrison's 16, p. 602-604/ WL-RAL
Patients who have undergone splenectomy are at higher risk for infection. The most common pathogenic organism infecting splenectomized patients is:

A) Streptococcus pneumoniae
B) Pseudomonas aeruginosa
C) Streptococcus viridans
D) Escherichia coli
E) Escherichia coli
The correct answer is: A

Splenectomized patients are particularly susceptible to infections caused by S. pneumoniae and when infected, may develop overwhelming sepsis. Patients who are undergoing elective splenectomy should receive pneumococcal vaccine 2 weeks preoperatively. Those undergoing emergency splenectomy should receive the vaccine postoperatively. Ref: Heme/ Harrison's 16, p. 348/ WL-RAL
The acute painful incidents of "sickle crisis" in sickle cell anemia are caused by:

A) Microembolization from thrombi on the mitral and aortic valves
B) Release of thrombotic engendering cytokines
C) Vaso-occlusion causing clustering of sickled red cells in the microvasculature
D) Septic events caused by inadequate tissue perfusion
E) Necrosis of peripheral nerves
The correct answer is: C

Sickle cell crisis is caused by vaso-occlusive events, which may be precipitated by hypoxia, infection, vigorous exercise, or changes in temperature. With the clustering of sickled red cells in the microvasculature, painful infarcts may occur in nearly any tissue. Ref: Heme/ Harrison's 16, p. 595-596/ WL-RAL
A six-year-old boy presents in your clinic with a week-long history of increasing fatigue, pallor, fevers, petechiae and bone pain in the pelvis and spine. Fever, stomatitis, and purpura are evident on exam. WBC is elevated at 125,000 with anemia and thrombocytopenia also present. Your working diagnosis is:

A) Acute Lymphoblastic Leukemia
B) Acute Myeloid Leukemia
C) Leukomoid reaction to sepsis
D) Hodgkin's Disease
E) Thalassemia major
The correct answer is: A

Acute Lymphoblastic Leukemia (ALL) accounts for 25% of pediatric malignancies. The peak age of onset is 4 years old. Patients present, as in this vignette, with fever, weakness and bone pain. WBC count may be normal or elevated to as high as 300,000. Anemia and thrombocytopenia are usual. Acute Myeloid Leukemia is relatively rare in children. Leukocytosis, secondary to infection, is not usually accompanied by decreases in other lines. Ref: Heme/ Hay 17, p. 911-912/ WL-RAL
Non-Hodgkin's Lymphoma consists of a diverse group of malignant lymphomas that is one of the fastest growing categories of malignancy in the United States. In order to make the diagnosis, it is essential that the patient:

A) agrees to undergo cyclic chemotherapy and/or radiation.
B) has a constellation of symptoms including swollen lymph nodes.
C) undergoes an invasive biopsy procedure to prove the diagnosis.
D) presents with an acute illness representing progressive lymphoma.
E) agrees to surgical resection of all affected tissue.
The correct answer is: C

Tissue diagnosis is mandatory in order to proceed with the rigorous chemotherapy and/or radiation necessary to treat lymphoma. Ref: Heme/ Harrison's, p. 643-649/ WL-RAL
An immunocompromised patient is due for immunization. You would choose which of the following?

A) Vaccines made from live viruses
B) Deferral of vaccination until such time as the immunocompromise resolves
C) Vaccination only against viral illnesses
D) Vaccines made from killed viruses
E) A course of prophylactic antiviral medication
The correct answer is: D

Live vaccines in an immunocompromised host may proliferate and cause actual disease. Live attenuated vaccines are contraindicated in immunocompromised patients, with certain exceptions. Ref: Heme/ Harrison's 16, p. 721/ WL-RAL
Retinal infections in HIV positive patients can be a significant complication. The most common infectious agent causing retinitis in this population is:

A) Herpes Simplex
B) Trichomonas
C) Candida Albicans
D) Cytomegalovirus
E) Pneumocystis carinii
The correct answer is: D

Cytomegalovirus (CMV) can cause many infectious complications in HIV patients including pneumonia, enteritis, and retinitis. CMV retinitis presents as progressive, painless loss of vision. Patients with a CD4+ T cell count below 100 should undergo ophthalmological examination every 3 to 6 months. Treatment consists of intravenous gancyclovir, or oral valganciclovir, among other options. Ref: Heme/ Harrison's 16, p. 1119/ WL-RAL
A patient diagnosed with a deep venous thrombosis is discharged home from the hospital on low molecular weight heparin administered by subcutaneous injection two times a day. You would monitor the efficacy of this therapy by:

A) PTT
B) PT
C) Both PT and PTT
D) Clinical observation
E) Bleeding time
The correct answer is: D

Dosing of low molecular weight heparin is based on body weight. Efficacy is determined by clinical observation rather than laboratory anticoagulation parameters. Ref: Heme/ Harrison's 16, p. 1492/ WL-RAL
Which is the most consistent laboratory finding in hemolytic anemia?

A) Positive Combs test
B) Increased reticulocyte count
C) Heinz bodies on peripheral smear
D) Target cells on peripheral smear
E) Elevated lactate dehydrogenase (LDH)
The correct answer is: B

Although other laboratory findings are present in specific types of hemolytic anemia, an increased reticulocyte count would be expected in all types. Ref: Hematol/Harrison's 16, p. 607/ RAL
A decrease in mean red cell volume (MCV) associated with high total iron-binding capacity (TIBC) and low serum ferritin, is typical of which type of anemia?

A) Iron-deficiency anemia
B) Anemia due to Vitamin B12 or Folate deficiency
C) Aplastic anemia
D) Acute hemorrhage
The correct answer is: A

Iron-deficiency anemia is typically microcytic and hypochromic. Ref: Heme/ Harrison's 16, p. 334/ RAL
An increased MCV is typical of which type of anemia?

A) Iron-deficiency anemia
B) Anemia due to Vitamin B12 or Folate deficiency
C) Aplastic anemia
The correct answer is: B

Folate or B12 deficiency results in impaired DNA synthesis and a megaloblastic (enlarged red cell) anemia. Ref: Heme/ Harrison's 16, p. 601/ RAL
Normal or elevated MCV with a decreased reticulocyte count is typical of which type of anemia?

A) Iron-deficiency anemia
B) Anemia due to Vitamin B12 or Folate deficiency
C) Aplastic anemia
The correct answer is: C

Anemias due to bone marrow failure, such as aplastic anemia, are characterized by few reticulocytes and increased MCV in the peripheral blood smear. Ref: Hematol/Harrison's 16, p. 620/ RAL
Which finding is least likely to be a manifestation of bleeding due to a platelet disorder?

A) Bleeding localized to the skin or mucous membranes
B) Bleeding easily controlled by local pressure
C) Spontaneous bleeding into the joints, retroperitoneum, or other body cavity
D) Bleeding occurring immediately after trauma or surgery
The correct answer is: C

Answers A, B, and D are common characteristics of hemorrhage associated with platelet disorders. Spontaneous bleeding into body cavities, delayed bleeding occurring several days after surgery or trauma, and bleeding difficult to control with local pressure suggest underlying plasma coagulation disorders. Ref: Heme/ Harrison's 16, p. 673, 681/ RAL
Which of the following diseases has not been completely or nearly eliminated by vaccination programs in the United States?

A) Diptheria
B) Lyme disease
C) Measles
D) Poliomyelitis
E) Congenital rubella syndrome
The correct answer is: B

The vaccine for Lyme disease has been withdrawn from the market. The number of incident cases has now increased to about 15,000 per year in the United States. In each of the remaining choices, the morbidity of the disease has been reduced by at least 99% compared with baseline levels, largely due to vaccination programs. Ref: ID/ Harrison's 16, p.995, 999/ RAL
Which of the following diseases have not been linked to an infectious etiologic agent?

A) Kaposi's sarcoma
B) Peptic ulcer disease
C) Cervical carcinoma
D) Lymphoma
E) All are associated with an infectious agent.
The correct answer is: E

Human herpes virus has been linked to Kaposi's sarcoma. Helicobacter pylori causes peptic ulcer disease. Human papillomavirus infection is an important cause of cervical carcinoma. Epstein-Barr virus has been implicated in the etiology of certain lymphomas and Hodgkin's disease. Ref: ID/ Harrison's 16, p. 695/ RAL
In the microscopic examination of a sputum specimen, the presence of squamous epithelial cells in the specimen suggests:

A) aspiration pneumonia.
B) impaired host defense mechanisms.
C) contamination of the specimen with normal oral flora.
D) upper airway infection related to oropharyngeal malignancy.
E) mixed nosocomial pneumonia.
The correct answer is: C

The presence of multiple organisms and more than 10 epithelial cells per low-power field suggest a specimen derived from the oropharynx and contaminated with the multiple species of bacteria that are normally found there. Ref: ID/ Harrison's 16, p. 1506/ RAL
By the age of 18 months, a child in the United States should have received all of the following vaccines except:

A) Hep B (hepatitis B) B) DTaP (diptheria, tetanus, acellular pertussis) C) Var (varicella-Zoster) D) Typhoid
E) MMR (measles/mumps/rubellA)
The correct answer is: D

All choices are recommended for routine administration to children before the age of 18 months, except the typhoid vaccine, which is normally reserved for those with travel or occupational exposure to the disease. Ref: ID/ Harrison's 16, p. 717-718/ RAL
Roseola is a childhood exanthem characterized by a maculopapular rash and a high fever. It affects children from 6 months to 3 years old. The causative agent is:

A) human herpesvirus (HHV).
B) Epstein-Barr virus (EBV).
C) a parvovirus.
D) an adenovirus.
E) a rhinovirus.
The correct answer is: A

Roseola is a self-limited disease caused by human herpes virus type 6. It is characterized by a 3 to 4 day febrile prodrome followed by the characteristic rash, which lasts for an additional 1 to 2 days. There is no vaccine or specific treatment. Ref: ID/ Hoekelman, p. 1198-1200; Harrison's 16, p. 1052/ RAL
Which of the following is NOT a risk factor for oral candidiasis?

A) HIV infection
B) Cigarette smoking
C) Diabetes mellitus
D) Concurrent cancer chemotherapy
E) Treatment with inhaled steroids
The correct answer is: B

The other choices are known risk factors for oral thrush, or candidiasis. Ref: ID/ W.Wilson, p. 734/ RAL
Which statement is true regarding oral candidiasis?

A) Topical nystatin or clotrimazole may be effective.
B) Culture of oral scrapings for Candida species establishes the diagnosis.
C) Candida leukoplakia is a benign condition of primarily cosmetic importance.
D) Oral candidiasis is an important precursor of systemic candidiasis in the immunocompetent host.
E) Infection is always confined to the tongue and palate.
The correct answer is: A

Both nystatin and clotrimazole may be used to treat oral candidiasis. Cultures are not specific, as Candida species are normally found in the oral cavity, and are not generally useful. Candida leukoplakia is subject to malignant transformation. Oral candidiasis does not cause systemic candidiasis. Infection may spread to the corners of the mouth, where it is termed angular cheilitis. Ref: ID/ W.Wilson, p. 737; Harrison's 16, p. 199/ RAL
Effective agents for the treatment of vaginal candidiasis include all of the following EXCEPT:

A) Oral fluconazole
B) Topical clotrimazole
C) Topical nystatin
D) Oral griseofulvin
E) Topical miconazole
The correct answer is: D

All are effective except griseofulvin, which is used primarily to treat fungal infection caused by Trichophyton and Microsporum species, but which is not active against Candida. Ref: ID/ W.Wilson, p. 739; Harrison's 16, p. 1176/ RAL
Which statement is true regarding the management of recurrent episodes of vaginal candidiasis?

A) Treatment of the sexual partner is imperative.
B) The use of oral contraceptives may be helpful.
C) Testing to rule out diabetes should be considered.
D) Daily vaginal douching is the treatment of choice.
E) All of the above
The correct answer is: C

As diabetes is a known risk factor, a fasting glucose to rule it out is reasonable. Oral contraceptive use is another predisposing factor for vaginal candidiasis, and discontinuing treatment may be helpful. Treatment of the sexual partner and douching are not recommended. Ref: ID/ W.Wilson, p. 740.
Which statement is true regarding management of the patient with candidemia documented by positive blood cultures?

A) Treatment is indicated only if signs of sepsis are present.
B) Strict isolation precautions must be maintained.
C) Treatment, once initiated, should be continued until a negative blood culture is obtained.
D) Elimination of Candida species from the environment is the preferred method of prevention.
E) Fluconazole and amphotericin B are both commonly used systemic agents to treat candidemia.
The correct answer is: E

Both agents are currently in use. All patients with candidemia should be treated until 2 weeks after negative blood cultures are obtained. Candida is ubiquitous; eliminating it from the environment is not possible and strict isolation is not generally helpful. Ref: ID/ W.Wilson, p. 740-742; Harrison's 16, p. 1187/ RAL
In pulmonary Cryptococcus infection of HIV-positive patients, the usual course of treatment with fluconazole is:

A) 2 weeks.
B) 4 to 6 weeks.
C) until clinical improvement is noted.
D) 6 to 12 months.
E) lifelong.
The correct answer is: E

In HIV-positive patients, the aim of treatment is usually lifelong suppression. Cryptococcosis is the most common serious fungal infection of patients with AIDS. Treatment of HIV-negative patients extends for a period of 6 to 12 months. Ref: ID/ W.Wilson, p. 748-749; Harrison's 16, p. 1184-1185/ RAL
Acute pulmonary histoplasmosis:

A) Is caused by the fungus Histoplasma.
B) Is generally a self-limited disease requiring no treatment.
C) May produce X-ray findings of fibronodular apical infiltrates
D) Is most common in the southeastern and mid-Atlantic regions of the United States.
E) All of the above
The correct answer is: E

Although acute histoplasma pneumonitis may be asymptomatic, or produce only mild symptoms, chronic pulmonary histoplasmosis and acute disseminated histoplasmosis are serious illnesses requiring aggressive treatment with antifungal agents, usually itraconazole or amphotericin B. Ref: ID/ Harrison's 16, p. 1179-1180/ RAL
Which statement concerning Pneumocystis carinii pneumonia is false?

A) Risk factors include HIV infection and history of organ transplantation.
B) Typical X-ray findings include bilateral diffuse infiltrates.
C) Bronchoalveolar lavage is a useful diagnostic technique.
D) The causative organism is an acid-fast bacillus.
E) Typical presenting symptoms include fever and nonproductive cough.
The correct answer is: D

Pneumocystis carinii is a fungus. Immunosuppressed status is a predisposing factor. Histopathologic staining for the causative organism, performed on specimens obtained by fiberoptic bronchoscopy and bronchoalveolar lavage, is a standard technique for establishing the diagnosis. Ref: ID/ Harrison's 16, p. 1194-1195/ RAL
Which of the following agents would not likely be effective in a patient with P. carinii pneumonia?

A) Trimethoprim-sulfamethoxazole (TMP-SMX) B) Acyclovir
C) Dapsone
D) Clindamycin
E) Pentamidine
The correct answer is: B

TMP-SMX is now the drug of choice for P. carinii infections. Regimens involving dapsone, clindamycin and pentamidine are alternatives. Acyclovir is an antiviral agent. Ref: ID/ Harrison's 16, p. 1195/ RAL
Which statement concerning botulism is TRUE?

A) The etiologic agent, Clostridium botulinum, is an aerobic gram-negative rod.
B) The disease occurs primarily among the old and debilitated.
C) C. Botulinum spores can be destroyed by boiling contaminated food in water for at least 10 minutes.
D) High, spiking fever is typical.
E) Diplopia, dysarthria, and dysphagia are early symptoms.
The correct answer is: E

Illness is caused by the toxin elaborated by C. botulinum, which typically affects the cranial nerves first (causing diplopia, dysarthria, and dysphagiA) and then other peripheral nerves. The central nervous system is spared. C. Botulinum, like other species of Clostridia, is an anaerobic gram-positive rod. Boiling for 10 minutes inactivates the toxin, but treatment at higher temperatures, as in a pressure cooker, is required to kill the spores. Patients typically are afebrile. Ref: ID/ Harrison's 16, p. 842-843/ RAL
Management of botulism in the adult may include all of the following except:

A) Broad-spectrum antibiotics
B) Administration of antitoxin
C) Mechanical ventilation
D) Cathartics and enemas
E) Gastric lavage
The correct answer is: A

Antibiotics have not been shown to be of use. An equine antitoxin is available. Respiration should be monitored, and mechanical ventilation may be required. If ingestion of contaminated food is recent, gastric lavage, emetics, cathertics or enemas may be useful. Ref: ID/ Harrison's 16, p. 844/ RAL
Which of the following clinical syndromes has (havE) been attributed to infection by Chlamydia species?

A) Pneumonia
B) Urethritis
C) Conjunctivitis
D) Upper respiratory tract infection
E) All of the above
The correct answer is: E

The three pathogenic species of Chlamydia (C. trachomatis, C. psittaci, and C. pneumoniae] cause upper and lower respiratory disease, STD's (including urethritis), and trachoma conjunctivitis, a common cause of blindness in infants and adults in the underdeveloped world. Ref: ID/ W.Wilson, p. 694-695/ RAL
Which of the following antibiotics is NOT effective in treating Chlamydia trachomatis genital infections?

A) Erythromycin
B) Ofloxacin
C) Ceftriaxone
D) Azithromycin
E) Doxycycline
The correct answer is: C

Cephalosporins are not effective against C. trachomatis. Ref: ID/ W.Wilson, p. 698/ RAL
Which of the following statements concerning Chlamydia trachomatis genital infections is TRUE?

A) Lymphogranuloma venereum usually presents as non-tender, bilateral inguinal lymphadenopathy.
B) A single dose of azithromycin is usually curative.
C) The treatment of choice in pregnant patients is ciprofloxacin.
D) C. Trachomatis is the most common cause of epididymitis in men over 35 years old.
E) Instillation of silver nitrate ophthalmic solution is the treatment of choice for C. trachomatis ocular infections in infants.
The correct answer is: B

Azithromycin 1g by mouth is usually effective. Lymphogranuloma venereum usually presents as tender unilateral inguinal or femoral lymphadenopathy. The treatment of choice in pregnant women is erythromycin. Erythromycin is also the treatment of choice in ocular infections in infants. C. Trachomatis is the most common cause of epididymitis in men under 35 years old; in men over 35, E. coli and P. aeruginosa are more likely. Ref: ID/ W.Wilson, p. 696-699; Harrison's 16, p. 1011-1016/ RAL
The most important aspect of the medical management of cholera is which of the following?

A) Identification of household contacts
B) Vigorous replacement of fluid and electrolytes
C) Replacement of gastrointestinal blood loss
D) Maintenance of a patent airway and ventilatory support
E) Lowering core body temperature
The correct answer is: B

Vibrio cholerae, produces an enterotoxin which causes a profuse, watery diarrhea. Victims die due to dehydration and electrolyte abnormalities. Replacing these losses is essential to the management of the disease. Treatment with oral or intravenous ampicillin, tetracycline, or trimethoprim-sulfamethoxazole has been shown to shorten the course of the illness. Ref: ID/ W.Wilson, p. 603-605/ RAL
A gray pharyngeal "pseudomembrane" with areas of necrosis, low-grade temperature, and cervical lymphadenopathy are most typical of which of the following?

A) Streptococcal pharyngitis
B) Gonococcal pharyngitis
C) Viral upper respiratory tract infection
D) Pertussis
E) Respiratory diphtheria
The correct answer is: E

Corynebacterium diphtheriae produces a toxin that causes cell necrosis in pharynx, resulting in the characteristic "pseudomembrane", which is differentiated from exudate in that it cannot be removed without causing bleeding of the underlying tissue. Ref: ID/ W.Wilson, p. 834; Harrison's 16, p. 833/ RAL
Appropriate management of a patient with respiratory diphtheria includes all of the following EXCEPT:

A) Begin oral tetracycline and schedule return office visit in 48 hours.
B) Administer antitoxin after skin testing to assess sensitivity.
C) Obtain throat cultures from close contacts, even if asymptomatic.
D) Isolate patient and institute cardiac monitoring.
E) Notify public health officials.
The correct answer is: A

The drug of choice for respiratory diphtheria is erythromycin. Penicillin is an alternative. Clindamycin and rifampin are also effective. Patients should be hospitalized and isolated. Cardiac monitoring should be instituted, due to the risk of myocarditis and various arrhythmias. Diphtheria antitoxin is available and may be lifesaving. Ref: ID/ W.Wilson, p. 534-536; Harrison's 16, p. 834-835/ RAL
A sexually-active 22-year-old male presents with a complaint of dysuria and urethral discharge. Which statement is TRUE regarding the management of this patient?

A) Gram-positive cocci on Gram's stain of the urethral discharge suggest gonococcal infection.
B) Thayer-Martin medium is appropriate for cultures of the urethral discharge.
C) The presence of a mucoid or purulent discharge makes Chlamydia trachomatis infection unlikely.
D) The interval between exposure and the development of clinical signs of gonococcal urethritis is 6 to 8 weeks.
E) All of the above statements are true.
The correct answer is: B

Thayer-Martin is an appropriate gonococcal-selective culture medium. Neisseria gonorrhoeae appears as a gram-negative intracellular diplococcus on Gram's stain. Gonorrhea cannot be differentiated from nongonococcal urethritis, including that caused by C. trachomatis, by the character of the urethral discharge. Clinical signs of gonococcal urethritis generally develop between 2 and 7 days after exposure. Ref: ID/ Harrison's 16, p. 857-860/ RAL
In treating uncomplicated gonococcal infections of the urethra, cervix, pharynx or rectum:

A) The minimum duration of oral therapy is 7 days.
B) Azithromycin or doxycycline should be added to cover Chlamydia trachomatis.
C) A single IM dose of penicillin is the treatment of choice in noncompliant populations.
D) Repeat cultures should be obtained in 2 weeks to document effective treatment.
E) HIV-positive patients require parenteral antibiotics.
The correct answer is: B

Concurrent infection with Chlamydia trachomatis is common in patients with gonococcal infections; the Centers for Disease Control and Prevention recommend treatment of this pathogen when treating gonorrhea. C. Trachomatis is the most common cause of nongonococcal urethritis. There are a number of 1-dose oral and parenteral regimens for N. gonorrhoeae, including third generation cephalosporins, quinolones, and spectinomycin. Penicillin is no longer effective due to the emergence of penicillin-resistant strains. Follow-up cultures are generally not indicated. Therapy is not altered in HIV-positive patients. Ref: ID/ Harrison's 16, p. 860-861/ RAL
The 2 most common causes of pelvic inflammatory disease are:

A) N. gonorrhoeae and C. trachomatis.
B) N. gonorrhoeae and Haemophilus species.
C) C. trachomatis and herpes simplex virus.
D) C. trachomatis and Candida species
E) C. trachomatis and Trichomonas vaginalis.
The correct answer is: A

Neisseria gonorrhoeae and Chlamydia trachomatis are the most common organisms causing pelvic inflammatory disease, although many other organisms have been cultured. Ref: ID/ W.Wilson p.208; Harrison's 16, p. 769/ RAL
What percent of women with gonococcal cervicitis will be asymptomatic?

A) 100%
B) 75%
C) 50%
D) 25%
E) 0%
The correct answer is: C

Typical symptoms include vaginal discharge, dysmenorrhea, dyspareunia, and dysuria-all of which are nonspecific. Ref: ID/ W.Wilson, p. 207/ RAL
Bacterial vaginosis (BV):

A) is most commonly caused by lactobacilli.
B) generally produces intense pruritus.
C) is characterized by petechial lesions of the cervix.
D) typically produces a thin gray-white, malodorous discharge and a vaginal Ph greater than 4.5.
E) typically produces an odorless, whitish, clumped ("cottage cheese") discharge.
The correct answer is: D

Other useful clinical signs of BV include the presence of "clue cells" in the discharge and a positive "whiff test" (fishy odor on mixing the discharge with 10% KOH). Unlike in vaginal candidiasis, which produces a clumped whitish discharge, pruritus and inflammation are minimal. Petechia of the cervix are characteristic of the vaginitis produced by Trichomonas vaginalis. Lactobacilli are the predominant normal flora of the vagina. Ref: ID/ W.Wilson, p. 208-210/ RAL
Which of the following is NOT a recommended agent for the treatment of vulvovaginal candidiasis?

A) Clotrimazole
B) Miconazole
C) Fluconazole
D) Metronidazole
E) Ketoconazole
The correct answer is: D

Oral metronidazole is an effective agent for the treatment of both trichomoniasis and bacterial vaginosis. The others are oral and topical fungal agents useful in the treatment of vaginal candidiasis. Ref: ID/ W.Wilson, p. 211/ RAL
Which statement concerning typhoid fever is FALSE?

A) The causative agent is a Salmonella species.
B) The disease remains a major cause of mortality in the underdeveloped world.
C) Diffuse abdominal pain is the most common presenting symptom.
D) Effective oral treatment is possible with ciprofloxacin.
E) Most cases in the United States are related to international travel.
The correct answer is: C

Abdominal pain is present in only 20-40% of cases. Prolonged fever is the most common presenting complaint. The disease may be caused by Salmonella typhi or Salmonella paratyphi, and accounts for about 600,000 deaths per year worldwide. The disease is rare in the United States, and 70% of cases are related to recent international travel. Despite the emergence of multidrug-resistant strains, quinolones, such as ciprofloxacin, constitute effective oral treatment. Ref: ID/ Harrison's 16, p. 898-899/ RAL
Nontyphoidal Salmonella enteritis:

A) is generally spread by direct contact with an infected individual.
B) should be treated aggressively with IV antibiotics.
C) generally resolves within 24 hours.
D) is a food-borne infection most commonly spread by infected chicken eggs and poultry.
E) has been eradicated in the United States by effective sanitation and other preventive measures.
The correct answer is: D

There are over 2 million cases of salmonellosis per year in the United States. It is spread by contaminated food, most commonly chicken eggs. Symptoms are diarrhea, which usually resolves within a week, and fever, which may last up to 72 hours. Antibiotic treatment is usually reserved for infants, the elderly, and the immunocompromised. Third-generation cephalosporins and quinolones are the usual agents. Ref: ID/ Harrison's 16, p. 900-902/ RAL
The most common causative organism in bacterial dysentery is:

A) Shigella species.
B) Salmonella species.
C) Campylobacter species.
D) Escherichia coli.
E) Yersinia enterocolitica.
The correct answer is: A

Worldwide, there are 200 million cases of shigellosis annually with 650,000 deaths. The other organisms are less common causes of dysentery. Ref: ID/ Harrison's 16, p. 903/ RAL
Which antibiotic is an appropriate treatment for Shigella enteritis with severe bloody diarrhea and dehydration?

A) Cephalexin
B) Streptomycin
C) Chloramphenicol
D) Tetracycline
E) Ciprofloxacin
The correct answer is: E

Ciprofloxacin is highly effective against all strains. Cephalexin has not been shown to influence the course of the disease. Resistance to chloramphenicol, streptomycin, and tetracycline is common. Ref: ID/ Harrison's 16, p. 905-906/ RAL
Which of the following is NOT a clinical feature of generalized tetanus?

A) Intense inflammatory reaction surrounding a puncture wound or laceration
B) Trismus
C) Abdominal rigidity
D) Opisthotonos
E) Increased deep tendon reflexes
The correct answer is: A

Clostridium tetani produces little inflammation at the portal of entry, which may not even be apparent. Generalized tetanus is characterized by rigidity of the facial muscles, producing trismus ("lockjaw"), and spasm of the central muscles of the trunk, producing abdominal rigidity and arching of the back (opisthotonos). The distal muscles of the extremities are usually spared. Deep tendon reflexes are typically increased. Ref: ID/ Harrison's 16, p. 840-841/ RAL
Which of the following interventions is NOT likely to be useful in a patient with generalized tetanus?

A) Wound debridement
B) Administration of intravenous diazepam
C) Administration of tetanus immune globulin (TIG)
D) Lumbar puncture
E) Administration of intravenous penicillin
The correct answer is: D

The cerebrospinal fluid is normal in tetanus, and lumbar puncture is generally not useful in establishing the diagnosis. Removal of organisms actively producing neurotoxins, by wound debridement, is beneficial. Likewise, circulating neurotoxin may be inactivated by the administration of tetanus immune globulin (TIG). Penicillin is effective against Clostridium tetani infection, as are metronidazole, erythromycin, and clindamycin. IV benzodiazepines, such as diazepam, are useful in controlling spasms. Ref: ID/ Harrison's 16, p. 841/ RAL
A 45-year-old male farm worker presents 1 hour after suffering a severe, contaminated puncture wound. He is known to have completed a primary series of three tetanus vaccinations in childhood and medical records are available indicating a "booster" vaccination for tetanus 8 years ago. Appropriate prophylaxis now consists of:

A) Tetanus immune globulin (TIG) only.
B) TIG and a single dose of combined tetanus diphtheria toxoid (TD) .
C) a 3-dose course of Td.
D) a single dose of Td only.
E) No prophylaxis is required.
The correct answer is: D

A "booster" dose of tetanus toxoid, usually given as Td in adults, should be given to individuals with tetanus-prone injuries who have had their last "booster" injection more than 5 years ago. In patients with clean, less tetanus-prone wounds, Td is given if the last dose was given more than 10 years ago. TIG is only given if the patient has not received the initial series of vaccinations, or if the history is unknown. Ref: ID/ Harrison's 16, p. 842/ RAL
A skin test for tuberculosis (PPD) is administered to an adult with no known risk factors. The test is considered positive if there is an area:

A) of erythema greater than 5mm in diameter after 48 hours.
B) of erythema greater than 10mm in diameter after 72 hours.
C) of induration greater than 5mm in diameter after 48 hours.
D) of induration greater than 5mm in diameter after 72 hours.
E) of induration greater than 15mm in diameter after 48 hours.
The correct answer is: E

The PPD is read 48 to 72 hours after intracutaneous injection. The area of induration, not erythema, is measured. Induration greater than 15mm is positive in a low-risk individual. Greater than 5mm of induration is positive in high-risk individuals (those with AIDS, close contacts of known cases, etc.). Greater than 10mm is positive in intermediate risk individuals (patients with diabetes, residents of correctional facilities or nursing homes, Asian immigrants, etC) . Ref: ID, W.Wilson, p. 646-647; Harrison's 16, p. 964/ RAL
Possible causes of a false-negative PPD (tuberculosis skin test) include all of the following EXCEPT:

A) Previous vaccination with Bacillus Calmette-Guerin (BCG) B) HIV infection
C) Malnutrition
D) Renal failure
E) Concurrent treatment with corticosteroids
The correct answer is: A

Previous BCG vaccination (rare in the United States) is a cause of false-positive reactions. The other choices may cause a false-negative result due to an incompetent immune system. Ref: ID/ W.Wilson, p. 647/ RAL.
The recommended duration of treatment with isoniazid for latent tuberculosis (positive PPD, no active disease evident) in a US-born patient not living in an area of reported drug-resistant disease is:

A) 2 weeks.
B) 4 weeks.
C) 9 months.
D) 2 years.
E) lifelong.
The correct answer is: C

The recommended duration of treatment is 9 months. Isoniazid is potentially hepatotoxic and should not be given to patients with liver disease. Patients at risk for liver disease (eg., alcoholics) should have liver enzymes checked monthly during treatment. Ref: ID/ Harrison's 16, p. 965/ RAL
Which statement concerning postprimary, or reactivation, pulmonary tuberculosis is TRUE?

A) It is primarily a pediatric disease.
B) Infection is usually localized to the upper lobes.
C) The causative organism thrives in an environment of low oxygen saturation.
D) A diffuse reticulonodular pattern on chest X-ray is typical.
E) Polycythemia is the most common hematologic finding.
The correct answer is: B

Postprimary, reactivation, or secondary pulmonary tuberculosis (the terms are synonymous) is primarily a disease of adults. Infection is usually localized to the upper lobes, where oxygen saturation is higher. Reticulonodular infiltrates are more typical of miliary or disseminated tuberculosis. Anemia and leukocytosis are the most common hematologic findings. Ref: ID/ Harrison's 16, p. 957/ RAL
A 28-year-old health care worker undergoing periodic surveillance coverts to a positive Mantoux test (PPD 5tu injected intradermally). She has no clinical evidence of active tuberculosis. An appropriate intervention at this time would be which of the following?

A) No treatment, continue annual surveillance
B) No treatment, repeat PPD at 25 tu
C) Begin oral isoniazid 5mg/kg daily for 9 months
D) Begin oral streptomycin 15mg/kg three times per week for 6 months
E) Begin oral ethambutol 15/mg per kg twice weekly for 2 months
The correct answer is: C

For treatment of latent tuberculosis (positive PPD, no clinical evidence of diseasE) in an otherwise healthy individual, the recommended regimen is isoniazid 5mg/kg daily for 9 months. Alternative regimens, including twice weekly treatment with isoniazid, a 2-month course of rifampin plus pyrazinamide, or a 4-month course of daily rifampin, are acceptable. Ref: ID/ Harrison's 16, p. 965/ RAL
Which of the following is the most common pathogenic Mycobacterium species in the United States?

A) Mycobacterium avium complex (MAC) B) Mycobacterium leprae
C) Mycobacterium tuberculosis
D) Mycobacterium xenopi
E) Mycobacterium fortuitum
The correct answer is: A

MAC is the most common mycobacterial pulmonary pathogen in the United States, surpassing M tuberculosis. Ref: ID/ Harrison's 16, p. 974/ RAL
The most common symptom in individuals with Entamoeba histolytica infections of the colon is which of the following?

A) Fever
B) Abdominal pain
C) Diarrhea
D) Severe dehydration
E) Most individuals are asymptomatic.
The correct answer is: E

90% of infected individuals are asymptomatic carriers. Among the remaining 10%, the majority of patients will complain of lower abdominal pain and watery diarrhea, sometimes with gross blood. Fever and severe dehydration are rare, although somewhat more common in children. Ref: ID/ W.Wilson, p. 818/ RAL
A patient with a history of amebic colitis presents with fever, right upper quadrant, and right shoulder pain. The most likely diagnosis is:

A) Recurrent Entamoeba histolytica colitis.
B) Concurrent viral hepatitis.
C) Septic arthritis of the right shoulder.
D) Liver abscess.
E) Endocarditis with multiple septic emboli.
The correct answer is: D

The most common site of E. histolytica infection, after the colon, is the liver. Typically, the organism spreads from the colon, through the portal system, to the liver, producing right upper quadrant abdominal pain referred to the right shoulder, fever, and cough. Ref: ID/ W.Wilson, p. 819/ RAL
The agent of choice for symptomatic Entamoeba histolytica infection is:

A) Metronidazole.
B) Diloxanide.
C) Iodoquinol.
D) Chlorquine.
E) Quinine sulfate.
The correct answer is: A

Metronidazole is effective in both acute amebic colitis and E. histolytica liver abscess. Diloxanide or Iodoquinol are intraluminal agents for the elimination of E. histolytica cysts from the intestinal wall after treatment with metronidazole. Chloroquine and quinine sulfate are antimalarial agents. Ref: ID/ W.Wilson, p. 820-821; Harrison's 16, p. 1217/ RAL
The usual route of infection for Necator americanus and Ancylostoma duodenale (hookworm) is:

A) fecal contamination of food.
B) percutaneous invasion.
C) rodent vector.
D) household contacts.
E) contaminated drinking water.
The correct answer is: B

Hookworm larvae hatch from eggs in the soil and penetrate the skin of the human host. They are carried by the circulation to the lungs, where they develop, and eventually enter the gastrointestinal tract by being coughed up and swallowed. The adult organism attaches itself to the small bowel mucosa, where it ingests the blood of the host and sheds eggs in the feces. It is estimated that one-fourth of the world's population is infected. Ref: ID/ Harrison's 16, p. 1257/ RAL
The most important clinical manifestation of hookworm disease is which of the following?

A) Profuse bloody diarrhea.
B) Postprandial abdominal pain.
C) Anemia.
D) Nocturnal abdominal pain.
E) Oxygen desaturation.
The correct answer is: C

The most clinically significant manifestation is iron deficiency anemia. Although most hookworm infections cause no symptoms, postprandial abdominal pain, transient cough, diarrhea, and pruritic inflammation at the site of skin penetration (usually the foot) may also be noted. Ref: ID/ Harrison's 16, p. 1257 RAL
Which species of Plasmodium is responsible for most clinical cases of malaria?

A) P. vivax
B) P. ovale
C) P. malariae
D) P. falciparum
E) None of the above
The correct answer is: A

P. vivax is the cause of 80% of malaria, worldwide. Ref: ID/ W.Wilson, p. 794/ RAL
Which species of Plasmodium typically causes the most severe disease, characterized by shaking chills and spiking fevers in an irregular pattern?

A) P. vivax
B) P. ovale
C) P. malariae
D) P. falciparum
E) None of the above
The correct answer is: D

Falciparum malaria typically produces the most severe symptoms. It is a medical emergency, with microvascular thrombosis in a variety of organs giving rise to many possible life-threatening complications. One clue to the diagnosis is the pattern of shaking chills and spiking fever. In malaria caused by P. vivax, P. ovale, and P. malariae, these symptoms occur on a regular 48- or 72-hour cycle. In P. falciparum malaria, fever and chills typically occur in an irregular pattern. Ref: ID/ W.Wilson, p. 798/ RAL
Which is NOT a typical clinical manifestation of severe falciparum malaria?

A) Anemia
B) Jaundice
C) Meningismus
D) Coma
E) Hypoglycemia
The correct answer is: C

Although changes in mental status, up to and including coma, are commonly seen in severe falciparum malaria, meningeal signs are rare. The direct action of the parasite on erythrocytes causes a (typically) normochromic, normocytic anemia. Hemolytic anemia, in turn causes jaundice, which may be worsened by liver dysfunction. The glucose consumption of the parasite, and decreased gluconeogenesis in the host, also due to liver dysfunction, frequently result in hypoglycemia. Ref: ID/ Harrison's 16, p. 1222-1223/ RAL
Which drug is most likely to be of benefit in the management of severe falciparum malaria?

A) Prednisone
B) Heparin
C) Chloroquine
D) Quinidine
E) Dextran
The correct answer is: D

Quinidine gluconate is most commonly used to treat severe malaria in the United States, although patients must be carefully monitored for arrhythmias, hypotension and other side effects. Chloroquine is not generally used due to widespread drug resistance. Steroids, heparin, and dextran are not generally useful. Ref: ID/ Harrison's 16, p. 1226/ RAL
Travelers visiting endemic areas may receive prophylaxis for malaria with any of the following agents EXCEPT:

A) Quinine
B) Chloroquine
C) Mefloquine
D) Doxycycline
E) Primaquine
The correct answer is: A

Quinine may be used to treat active infection, but is not used in prevention. Mefloquine is the prophylaxis of choice in many areas. Chloroquine can be used in areas where widespread resistance to it has not been reported. Doxycycline and primaquine are alternatives. Ref: ID/ Harrison's 16, p. 1230-1232/ RAL
Immunocompetent adults infected with the protozoan Toxoplasma gondii typically:

A) are asymptomatic.
B) develop fever, chills, and spiking fevers.
C) develop lymphadenopathy and malaise.
D) complain of impaired vision.
E) are pregnant females.
The correct answer is: A

Toxoplasmosis is asymptomatic in 90% of immunocompetent adults. In most of the remaining 10%, the disease presents as malaise and lymphadenopathy without fever. Rarely, chorioretinitis, myocarditis, polymyositis, and other systemic symptoms may be noted. Ref: ID/ W.Wilson, p. 809/ RAL
Likely clinical manifestations of toxoplasma encephalitis in the immunocompromised adult include all of the following EXCEPT:

A) Hemiparesis
B) Dysphasia
C) Meningismus
D) Seizure
E) Alteration in mental status
The correct answer is: C

Toxoplasmosis is a life-threatening infection in the immunocompromised host. The brain is the organ most frequently affected. All of the manifestations listed are common with the exception of meningismus; the infection usually spares the meninges and does not produce meningeal irritation. Ref: ID/ W.Wilson, p. 810/ RAL
Congenital toxoplasmosis is usually acquired:

A) at conception.
B) by reactivation in the immunocompromised fetus.
C) in utero.
D) in the birth canal, during delivery.
E) due to chronic infection in the mother.
The correct answer is: C

The fetus acquires the disease in utero when the mother is infected during, or shortly before, gestation. Although about 85% of infected neonates will appear clinically normal at birth, most will go on to develop neurological, ophthalmological, and other serious systemic manifestations. Ref: ID, W.Wilson, p. 809-810; Harrison's 16, p. 1243-1246/ RAL
Useful measures for the control of toxoplasmosis include all of the following EXCEPT:

A) Cats in the household of seropositive pregnant women should be euthanized and tested for the disease.
B) All meat should be thoroughly cooked.
C) Fruit should be washed before eating.
D) Pregnant women should not handle cat litter.
E) Hands should be thoroughly washed after gardening.
The correct answer is: A

Cats can live in the household of pregnant women, but someone else should handle the kitty litter. If this is not possible, thorough hand washing after changing the litter is essential. Trimethoprim/sulfamethoxazole is used for primary prophylaxis in AIDS patients. The main reservoir of the parasite is in the small intestine of the cat; therefore, food or soil potentially contaminated with cat feces represents a hazard. Lamb, pork, and less commonly beef may contain the encysted parasite and should be thoroughly cooked. Ref: ID/ W.Wilson, 807, 814-816; Harrison's 16, p. 1248/ RAL
In the United States, the most common illness transmitted to humans by an insect vector is:

A) Tularemia.
B) Lyme disease.
C) Malaria.
D) Rocky Mountain spotted fever.
E) West Nile virus.
The correct answer is: B

Lyme disease may be transmitted to humans by the bite of several species of tick in the genus Ixodes. There are about 15,000 cases per year in the United States. Tularemia and Rocky Mountain spotted fever are less common tick-borne diseases. Malaria and West Nile virus are transmitted by mosquito vectors and are also less common than Lyme disease in the United States. Ref: ID/ W.Wilson, p.682-683; Harrison's 16, p. 995/ RAL
Borrelia burgdorferi, the causative agent in Lyme disease, is a(n):

A) Aerobic diphtheroid
B) Gram negative bacillus
C) Retrovirus
D) Spirochete
E) Protozoan
The correct answer is: D

B. burgdorferi is a microaerophilic spirochete. It can be cultured from biopsy specimens of skin lesions in patients with Lyme disease, but not usually from other tissues

regardless of clinical manifestations. Ref: ID/Harrison's 16, p. 995/ RAL
Rocky Mountain spotted fever:

A) is a self-limiting viral illness.
B) should be suspected in North America west of the Mississippi River only.
C) typically produces a confluent, truncal rash.
D) typically produces thrombocytopenia.
E) should be treated only after positive serological tests are obtained.
The correct answer is: D

Most patients develop thrombocytopenia. Rocky Mountain spotted fever is caused by Rickettsia rickettsii, a bacterial pathogen. Untreated cases are usually fatal. It is equally common in the eastern and western United States. It initially produces a maculopapular rash at the ankles and wrists, which later becomes generalized and petechial. Serologic testing does not become positive early enough to guide treatment decisions. Ref: ID/ W.Wilson, p. 708-710; Harrison's 16, p. 1000-1001/ RAL
The tissue most affected by R. rickettsii is which of the following?

A) Vascular endothelium
B) Bone marrow
C) Circulating erythrocytes
D) Circulating lymphocytes
E) Epithelial cells at various locations
The correct answer is: A

The intracellular pathogen affects vascular endothelium, causing increased permeability in the microcirculation of many organ systems. Noncardiac pulmonary edema, renal failure, and profound neurological symptoms may result. Ref: ID/ Harrison's 16, p. 1000/ RAL
The drug of choice in Rocky Mountain spotted fever is:

A) Doxycycline.
B) Gentamycin.
C) Erythromycin.
D) Prednisone.
E) Trimethoprim/sulfamethoxazole.
The correct answer is: A

Doxycycline 100mg orally or intravenously bid is the drug of choice in children and adults. Gentamycin, erythromycin, and steroids are of no value. Sulfa drugs may worsen the patient's condition. Ref: ID/ Harrison's 16, p. 1001/ RAL
The median incubation period of primary syphilis is:

A) 3 days.
B) 7 days.
C) 3 weeks.
D) 10 weeks.
E) 6 months.
The correct answer is: C

The period between inoculation with Treponema pallidum, the causative agent, and the development of the syphilitic chancre is about 21 days, but may vary from 10 to 90 days, depending, in part, on the number of organisms inoculated. Ref: ID/ W.Wilson, p. 670; Harrison's 16, p. 978/ RAL
Which of the following is NOT likely to be a manifestation of primary syphilis?

A) A single, painless, indurated papule of the penis
B) A painless chancre of the cervix
C) Painless bilateral inguinal lymphadenopathy
D) Spiking fevers
E) Complete healing of the primary lesion in untreated patients
The correct answer is: D

Spiking fevers are not usually a part of the clinical presentation of syphilis, and are more commonly associated with other entities producing genital lesions, such as primary genital herpes. The primary lesion in syphilis may begin as a painless nodule and progress to a characteristic ulcer or chancre. The lesion may occur on the penis, labia, cervix, rectum, or mouth, with non-tender, symmetrical regional lymphadenopathy. With or without treatment, the primary lesion usually heals in 4-6 weeks. Ref: ID/ Harrison's 16, p. 978-979/ RAL
Which of the following is NOT a symptom of secondary syphilis?

A) Saccular aneurysm of the aorta
B) Condyloma lata
C) Rash involving the palmar and plantar surfaces
D) Faint truncal macular rash
E) Patchy alopecia involving the scalp, eyebrows, and facial hair
The correct answer is: A

Saccular aortic aneurysm is a manifestation of late (or tertiary) syphilis, usually appearing 10 to 40 years following the initial infection. The other choices are manifestations of secondary syphilis, which appear about 6 weeks after the resolution of the primary phase. Ref: ID/ W.Wilson, p. 672-673; Harrison's 16, p. 979-981/ RAL
Which of the following conditions may cause a false-positive fluorescent treponemal antibody test?

A) HIV infection
B) Pregnancy
C) Lyme disease
D) Infectious mononucleosis
E) Tuberculosis
The correct answer is: C

While the less-specific VDRL may yield a false positive result due to any of the conditions listed, the more-specific fluorescent treponemal antibody test may be falsely positive for syphilis only in patients exposed to other spirochetal illnesses, such as Lyme borreliosis. Ref: ID/ W.Wilson, p. 675/ RAL
The treatment of choice for primary syphilis is:

A) Tetracycline 500mg po bid for 28 days
B) Ciprofloxacin 500mg po bid for 7 days
C) One dose of Benzathine penicillin G 2.4 million units IM
D) Erythromycin 500mg po qid X 14 days
E) Penicillin G 4 million units IV q4 hours for 14 days
The correct answer is: C

A single intramuscular dose of penicillin is the treatment of choice for primary syphilis. Oral doxycycline, tetracycline, or erythromycin are alternatives, particularly in penicillin-allergic patients. Ref: ID/ W.Wilson, p. 677; Harrison's 16, p. 983/ RAL
Which statement is TRUE concerning cytomegalovirus (CMV) infection?

A) Clinical disease in immune-competent adults produces a mild flu-like syndrome.
B) Infection of the newborn occurs exclusively in the birth canal at the time of delivery.
C) Infect neonates are generally identifiable by neurological screening at birth.
D) It is an important cause of blindness in HIV patients
E) Pulmonary involvement in HIV-infected adults causes a characteristic lobar infiltrate.
The correct answer is: D

Clinical disease in the newborn usually results from primary infection of the mother during pregnancy and transplacental infection of the fetus. Infection during birth and from maternal milk is possible, but less likely to cause significant disease. Neonates are usually asymptomatic. It causes a mononucleosis-like syndrome in normal adults with severe malaise, fever, and arthralgias, lasting 2 to 6 weeks. HIV patients may develop blindness due to ocular infection. Pulmonary manifestations in immunocompromised adults include diffuse reticulonodular infiltrates. Ref: ID/ Harrison's 16, p. 1050-1051; W.Wilson, p. 407/ RAL
Which statement is TRUE concerning adult acquired cytomegalovirus (CMV) infection in immunocompetent hosts?

A) It is most commonly acquired among the elderly.
B) It is not spread by sexual contact.
C) Heterophile antibody testing is typically positive.
D) The presence of more than 10% atypical lymphocytes is a common finding on peripheral blood smear.
E) It is typically transmitted by casual contact.
The correct answer is: D

Atypical lymphocytes are the most common laboratory finding. Among adults, it is most common among the young and can be sexually transmitted. Transmission rarely occurs via casual contact. The disease produced is a mononucleosis-like syndrome, with fever, chills and fatigue, but with negative heterophile antibody test

unlike true mononucleosis. Ref: ID/ Harrison's 16, p. 1050-1051/ RAL
Which of the following agents would be the best first choice in an immunocompromised patient with active CMV retinitis?

A) Acyclovir
B) Ganciclovir
C) Foscarnet
D) Cidofovir
E) Tetracycline
The correct answer is: B

In several studies, 70-90% of AIDS patients with active CMV infections responded to ganciclovir. Acyclovir may have some value in prophylaxis, but is not useful in treating clinical CMV infections. Foscarnet and cidofovir may be effective, but their usefulness is limited by toxic effects of the drugs. Tetracycline is not useful. Ref: ID/ Harrison's 16, p. 1051-1052/ RAL
An 18-year-old male patient presents with a one-week history of fever, sore throat, and severe malaise. On examination you note tender posterior cervical lymphadenopathy and splenomegaly. Despite some tonsillar hypertrophy his airway is widely patent. Peripheral blood smear reveals more than 10% atypical lymphocytes. In addition, you send a specimen, which comes back positive for heterophile antibody. You advise the patient that he has infectious mononucleosis, and you make which of the following interventions?

A) Begin prednisone taper.
B) Begin acyclovir.
C) Advise the patient to avoid contact sports.
D) Instruct the patient to practice strict household isolation.
E) Begin prophylactic amoxicillin.
The correct answer is: C

Due to the presence of splenomegaly in infectious mononucleosis, there is a significant risk of splenic rupture. Steroids are generally used when there is threat of compromise of the airway. Acyclovir and antibiotics are of no benefit. Since the virus is ubiquitous, isolation precautions are not generally practiced. Ref: ID/ Harrison's 16, p.1048; W.Wilson, p.410/ RAL
The causative organism in erythema infectiosum (or fifth diseasE) is which of the following?

A) Parvovirus
B) Retrovirus
C) Human papillomavirus
D) Rhinovirus
E) Spirochete
The correct answer is: A

Erythema infectiosum is a self-limiting disease of children characterized by a low-grade fever and facial rash, sometimes spreading to the trunk and extremities. It is caused by a parvovirus known as B19. The same pathogen causes an acute arthritis in adults, also usually resolving spontaneously without specific treatment. The most serious manifestation of parvovirus B19 infection is aplastic anemia produced by the virus in patients with an underlying hemolytic disease. Ref: ID/ Harrison's 16, p. 1054-1055/ RAL
Genital herpes simplex virus (HSV) infection:

A) is always caused by HSV-2.
B) is only transmissible to sex partners when symptomatic lesions are present.
C) may be accompanied by pain, dysuria fever and malaise.
D) produces painless genital lesions.
E) cannot be transmitted to the neonate.
The correct answer is: C

First genital infection with HSV-1 or HSV-2 may be accompanied by local and constitutional symptoms, or may be asymptomatic. The disease can be transmitted regardless of the presence of symptoms. Genital infections are more frequently caused by HSV-2, but 30% are caused by HSV-1 in the United States. Genital lesions, when present, are painful. Neonatal infection, usually acquired by contact with infected secretions in the birth canal, carries a 65% mortality rate without treatment. Ref: ID/ Harrison's 16, p. 1037-1038; W.Wilson, p. 402-403/ RAL
The characteristic finding on light microscopy noted in herpes simplex (HSV) virus infection is which of the following?

A) Disruption of the cell membrane
B) Multinucleated giant cells
C) Intracytoplasmic inclusion bodies
D) Clue cells
E) Gram-negative diplococci
The correct answer is: B

Unroofed vesicles prepared with Giemsa's stain ("Tzanck test") may reveal characteristic multinucleated giant cells. Intracytoplasmic inclusions are associated with chlamydial infection. Clue cells are vaginal epithelial cells containing coccobacilli, seen on wet mount preparations, associated with bacterial vaginosis, most commonly Gardnerella vaginalis. Gram-negative diplococci are most commonly associated with gonorrheal infections. Ref: ID/ Harrison's 16, p. 1040; Goroll 4, p. 1051, 1055; W.Wilson, p. 289, 291, 293/ RAL
Which statement regarding antiviral treatment in herpes simplex virus infection is TRUE?

A) Ganciclovir is the agent of choice.
B) The risk of transmission of genital herpes can be eliminated by the chronic administration of acyclovir.
C) Acyclovir is the agent of choice in children and adults.
D) Topical acyclovir ointment is useful in recurrent oral-labial HSV.
E) Over-the-counter preparations, such as Anbesol, may shorten the course of symptomatic oral-labial recurrences.
The correct answer is: C

Acyclovir is considered the agent of first choice in all age groups. Valacyclovir and famciclovir are also effective. Ganciclovir has anti-HSV activity, but is more toxic and not commonly used. Ref: ID/ Harrison's 16, p. 1040; Goroll 4, p. 1052-1053; W.Wilson, p. 403/ RAL
Herpes simplex virus (HSV) poses the greatest risk to the neonate when the mother:

A) acquires the primary infection in the first trimester.
B) acquires the primary infection in the third trimester.
C) has a recurrence in the first trimester.
D) has a recurrence in the third trimester.
E) has clinically unsuspected asymptomatic chronic infection.
The correct answer is: B

The greatest risk to the neonate occurs when the maternal primary infection takes place late in pregnancy. The newborn, in this scenario, has not yet acquired maternal antibodies and is at high risk of infection, with the subsequent development of severe disease, during birth. Ref: ID/ Harrison's 16, p. 1038; W.Wilson, p. 403/ RAL
In general, in an asymptomatic HIV-positive patient, at what CD4+ T cell count should the initiation of antiretroviral therapy be considered?

A) Less than 500 cells/εL
B) Less than 250 cells/εL
C) Less than 100 cells/εL
D) Less than 55 cells/εL
E) Undetectable levels
The correct answer is: B

Although therapy must be individualized, therapy should generally begin when CD4+ T cell count is below 250 cells/εL. Ref: ID/ Harrison's 16, p. 1135/ RAL
Which is an indication for the initiation of antiretroviral therapy in an HIV-infected individual?

A) A commitment on the part of the patient to compliance with medication regimens
B) Any symptomatic disease
C) A willingness to enroll in experimental protocols
D) Infection with a virulent strain of the virus
E) Behavior suggesting risk of transmission of the disease
The correct answer is: B

Although indications for treatment are controversial, U.S. Public Health Service Guidelines suggest antiretroviral treatment for any symptomatic disease. Ref: ID/ Harrison's 16, p. 1135/ RAL
The risk of contracting HIV infection from a single unit of donated blood is approximately:

A) 1 in 1,200
B) 1 in 60,000
C) 1 in 120,000
D) 1 in 350,000
E) 1 in 725,000
The correct answer is: E

The risk has been estimated at 1 in 725,000 to 1 in 835,000. Ref: ID/ Harrison's 16, p. 1138/ RAL
Which statement concerning the transmission of HIV from mother to child is FALSE?

A) The disease may be transmitted to the infant during breast-feeding.
B) Antiretroviral treatment of an HIV-infected mother while breast feeding is contraindicated.
C) Antiretroviral treatment of an HIV-infected mother during pregnancy is contraindicated.
D) To be effective in reducing transmission of HIV to the child, antiretroviral therapies must take place for the entire period of gestation.
E) Infection may be transmitted by vaginal secretions during delivery.
The correct answer is: D

The disease may be transmitted in utero, but the fetus is most commonly infected during delivery. The child may be infected by breastfeeding from an infected mother, and breastfeeding is absolutely contraindicated in industrialized countries, where a reasonable alternative is available. Antiretroviral treatment is commonly provided during pregnancy and during breastfeeding in industrialized countries, but even treatment of brief duration (eg., during labor) significantly decreases the risk of transmission to the infant. Ref: ID/ Harrison's 16, p. 1082, 1137/ RAL
Which type of exposure has the lowest risk of HIV transmission?

A) Receiving one unit of banked blood
B) Exposure of intact skin to HIV-contaminated blood or secretions
C) Mucous membrane exposure to HIV-contaminated blood or secretions
D) Skin puncture with an HIV contaminated needle
E) Receiving treatment by an HIV-positive health care worker
The correct answer is: B

Transmission of HIV through intact skin has never been documented. The risk of infection by a needle-stick injury is 0.3%. The risk from a mucous-membrane exposure is 0.09%. The risk from one unit of banked blood is 1/725,000 (0.0000013%). There have been three cases of likely transmission of HIV from a practitioner to a patient. Ref: ID/ Harrison's 16, p. 1081/ RAL
You are monitoring an asymptomatic patient with known HIV infection, and you have noted a gradual decline in his CD4+ T cell count over a course of several years. At what CD4 count would it be most appropriate to begin prophylaxis for Pneumocystis carinii pneumonia?

A) Less than 600 cells per εl
B) Less than 400 cells per εl
C) Less than 200 cells per εl
D) Less than 100 cells per εl
E) Less than 50 cells per εl
The correct answer is: C

In an asymptomatic patient, P. carinii prophylaxis is usually begun when the CD4+ T cell count falls below 200 cells per εl. The agent of choice is trimethoprim/sulfamethoxazole, 1 DS tablet qd, although there are alternative regimens. Ref: ID/ Harrison's 16, p. 1105/ RAL
Which statement concerning tuberculosis (TB) in an HIV-infected patient is FALSE?

A) The goal of treatment is cure of the disease (TB) .
B) The treatment of an HIV-positive individual is the same as the treatment of an HIV-negative individual.
C) Asymptomatic HIV patients whose PPD shows greater than 5mm of induration should be treated.
D) TB usually develops in the final stages of HIV/AIDS.
E) HIV patients who are close household contacts of persons with active TB should be treated.
The correct answer is: D

Close household contacts should be treated with a 1- or 2-drug regimen. Active TB frequently develops early in the course of HIV disease. The cure of TB is possible in HIV infected patients, whose treatment is essentially identical to that of individuals not HIV-positive. Ref: ID/ Harrison's 16, p. 1107-1108/ RAL
The routine initial management of the asymptomatic HIV-positive patients should include all of the following EXCEPT:

A) VDRL test
B) PPD skin test
C) Serological testing for hepatitis A, B, and C
D) Immunization with pneumococcal vaccine
E) Sputum collection for acid-fast bacilli
The correct answer is: E

In addition to measures A through D, anti-Toxoplasma antibody testing, CD4+ T cell counts, and HIV RNA (viral loaD) levels are routine. Patients who are seronegative for hepatitis A or B should be vaccinated. Ref: ID/ Harrison's 16, p. 1124/ RAL
A health care worker injured by a needle contaminated with the blood of an HIV-positive patient should generally receive:

A) trimethoprim/sulfamethoxazole orally for 7 days.
B) oral retroviral therapy with at least 3 agents for one year.
C) oral retroviral therapy with at least 2 or 3 agents for 4 weeks.
D) isoniazid orally for 9 months.
E) ganciclovir orally for 7 days.
The correct answer is: C

Recommendations for postexposure prophylaxis include either 2- or 3-drug antiretroviral regimens for a 4-week course of treatment. Ref: ID/ Harrison's 16, p. 1137/ RAL
Patients with measles (rubeolA) may transmit the disease until:

A) the prodrome develops.
B) Koplik's spots are noted.
C) the rash develops.
D) four days after the rash develops.
E) all skin lesions fade.
The correct answer is: D

Patients remain contagious until four days after the skin rash develops. Koplik's spots are the characteristic blue-white spots that appear on the buccal mucosa before the skin rash is visible. Ref: ID/ Harrison's 16, p. 1149/ RAL
Appropriate health maintenance for a 55-year-old man without a documented history of measles or completion of the 2-dose immunization routine should include which of the following?

A) Administration of measles vaccine
B) Administration of standard immune globulin
C) Counseling to avoid household contacts who may have had recent exposure to measles
D) Counseling to avoid travel to endemic areas
E) None of the above
The correct answer is: E

Adults born before 1957 do not require immunization. Those born after 1957 who have not received vaccine or had measles should be vaccinated. Ref: ID/ Harrison's 16, p. 1151/ RAL
Human papilloma virus (HPV) is associated with which of the following malignancies?

A) Lymphoma
B) Basal cell carcinoma
C) Kaposi's sarcoma
D) Squamous cell carcinoma
E) Cervical carcinoma
The correct answer is: E

HPV is highly correlated with carcinoma of the uterine cervix. Ref: ID/ Harrison's 16, p. 1057/ RAL
Which of the following groups should not receive influenza vaccine?

A) Persons greater than 65 years old
B) Breast feeding mothers
C) Children less than 6 months old
D) Children with asthma
E) Adults with chronic renal failure
The correct answer is: C

Influenza vaccination is contraindicated in children less than 6 months old, due to the high risk that the vaccine will induce a febrile illness in this population. The elderly, and adults or children with serious chronic respiratory or other chronic diseases, are among the target population for immunization. Breast-feeding is not a contraindication. Ref: ID/ W.Wilson, p. 386-387/ RAL
The ideal timing for the administration of influenza vaccine is:

A) from September to mid-November.
B) from August to September.
C) from November to mid-December.
D) during late summer.
E) after the first frost.
The correct answer is: A

September to mid-November is the recommended period. It takes about 2 weeks after immunization for the full antibody response to take place. Influenza outbreaks usually begin in the late fall or early winter. Ref: ID/ W.Wilson, p. 380, 383, 385/ RAL
You suggest to an elderly patient with chronic obstructive pulmonary disease that he receive influenza vaccine. He objects, citing a risk of Guillain-Barrí© syndrome (GBS). Your response should be which of the following?

A) The increased risk of GBS is no more than 1 in 1000 and is justified by the risk to his life posed by influenza.
B) The risk of GBS is no more than 1 in 10,000 and the disease primarily affects children.
C) The risk of life-threatening GBS is greatest in the elderly. You thank him for the reminder and withdraw your recommendation.
D) The risk is nonexistent, and he should accept the vaccine.
E) The increased risk is probably no more than 1 in 1,000,000 much less than the risk to his life posed by influenza.
The correct answer is: E

The 1976 swine flu vaccination campaign was associated with a risk of slightly less than one additional case of GBS per 100,000 vaccinated individuals. More recent studies have shown a risk of about 1 case of GBS per 1,000,000 vaccinations. Ref: ID/ W.Wilson, p. 387/ RAL
A 26-year-old female surgical house officer presents to your employee health clinic during an outbreak of influenza A. As you recommended, she was vaccinated in October. Now she complains of sore throat, headache, severe malaise, and a fever up to 103ŒÁF. Your best course of action is which of the following?

A) Reassure her that she cannot have influenza, as she was vaccinated, and is probably coming down with a cold.
B) Reassure her that influenza is rarely fatal in healthy young individuals and recommend that she go to bed until she feels better.
C) Administer a second, booster dose of vaccine.
D) Administer a daily dose of azithromycin for 5 days and warn her of the symptoms of secondary bacterial infection.
E) Administer rimantadine daily for 7 days.
The correct answer is: E

Both antiviral drugs amantadine and rimantadine have been shown to be effective in shortening the duration of illness in young, otherwise healthy individuals with influenza A. Rimantadine is less likely to be associated with CNS side effects, such as anxiety and insomnia. The vaccine has been shown to reduce the duration of the illness by about 50%. Ref: ID/ W.Wilson, p. 385; Harrison's 16, p. 1069/ RAL
A child presents after having been bitten by a stray dog. The animal is captured and killed, and examination reveals that it was rabid. Appropriate treatment for the child now consists of all of the following EXCEPT:

A) Give rabies immune globulin 20 IU per kg, half to be infused into the area surrounding the wound.
B) Thorough cleansing and debridement of the wound
C) Administer passive immunization for rabies in 5 doses over 28 days.
D) Hospitalize and isolate the child for 28 days.
E) All the above are appropriate.
The correct answer is: D

Treatment of an individual bitten by a rabid animal consists of local wound care as well as passive and active immunization. Isolation is not necessary. Ref: ID/ W.Wilson, p. 458-459/ RAL
Which of the following statements concerning mumps is FALSE?

A) Mumps during pregnancy may cause severe birth defects.
B) Adolescent males who have not been vaccinated or had mumps should receive two doses of mumps vaccine.
C) Mumps vaccine is 95% effective in preventing the disease.
D) Mumps orchitis is bilateral in 15% of cases.
E) The first dose of mumps vaccine is routinely given at 12 to 15 months old.
The correct answer is: A

Mumps in pregnant women do not cause defects in the fetus. Children and adults who have not received vaccine, usually given at 12 to 15 months and at 4 to 12 years, should be vaccinated. This is especially true for adolescent males, because of the risk of sterility resulting from mumps orchitis. Fortunately, orchitis is bilateral in only 15% of cases. Two doses of the vaccine are 95% effective. Ref: ID/ Harrison's 16, p. 1154-1155/ RAL
The most common complication of varicella is:

A) Varicella pneumonia
B) Aseptic meningitis
C) Bacterial skin infections
D) Hepatitis
E) Myocarditis
The correct answer is: C

Secondary bacterial infection of skin lesions, usually with staphylococcal and streptococcal species, is the most frequent complication of chickenpox. Varicella pneumonia, meningitis, hepatitis, and myocarditis occur, but are much less common. Ref: ID/ Harrison's 16, p. 1043/ RAL
Varicella vaccine should be administered to all of the following EXCEPT:

A) Women of childbearing age
B) Immunocompromised adults
C) Male military personnel
D) Children
E) Health care workers
The correct answer is: B

Immunocompromised patients should not receive this live attenuated vaccine. It is also contraindicated in pregnant women or in women who intend to become pregnant within the next 3 months. Ref: ID/ Goroll 4, p. 32; Harrison's 16, p. 721/ RAL
A renal transplant recipient with a functioning graft, on full immunosuppressive therapy, presents with a 24-hour history of a painful vesicular rash in a typical dermatomal distribution. You strongly suspect herpes zoster. Your best treatment option is which of the following?

A) Admit the patient and begin intravenous acyclovir 10mg per kg every 8 hours for 7 days.
B) Begin high-titer varicella immune globulin.
C) Begin oral acyclovir 800mg 5 times daily for 7 days and follow closely as an outpatient.
D) Begin oral famciclovir 500mg 3 times daily for 7 days and follow up within 48 hours.
E) Begin oral valacyclovir 1g 3 times daily for 5 days and follow up within 48 hours.
The correct answer is: A

Immunocompromised patients with herpes zoster require intravenous therapy with acyclovir. Oral treatment of immunocompetent individuals with herpes zoster, with any of the 3 agents named, has been shown to reduce the pain and duration of the illness. Immune globulin is not useful once clinical manifestations of the disease are present. Ref: ID/ Harrison's 16, p. 1045; Goroll 4, p. 1055; W.Wilson, p. 405/ RAL
An otherwise-healthy 60-year old man presents to clinic with a 48-hour history of a painful vesicular rash in a typical dermatomal distribution. You begin oral acyclovir. What other oral agent, initiated at this point in the patient's illness is likely to reduce the incidence and severity of postherpetic neuralgia?

A) Amitriptyline
B) Prednisone
C) Gabapentin
D) Carbamazepine
E) Fluphenazine hydrochloride
The correct answer is: B

A tapering 3-week course of prednisone, begun with an antiviral agent early in the course of the disease, has been shown to be of value in preventing postherpetic neuralgia, particularly among older patients in whom the risk of this complication is the greatest. The other agents named have all been used to treat postherpetic neuralgia after symptoms of this complication develop. Ref: ID/ Harrison's 16, p. 1045; Goroll 4, p. 1056/ RAL
A previously healthy 24 year old man presents at a clinic in Southern New England ten days after a weekend camping trip. He reports the appearance of a painless skin lesion on his right thigh five days previously. He now notes malaise, low-grade fever, and migrating joint pain of 48 hours duration. Careful questioning fails to elicit any recent history of an insect bite. The lesion on the thigh had begun to fade, but you recognize an area of central clearing with an erythematous margin. Further examination reveals a second, similar lesion near the right axilla. Question: Based on the information in the scenario, the most likely diagnosis is:

A) Tinea corporis
B) Lyme disease
C) Streptococcal cellulitis
D) Erythema multiforme
E) Nummular eczema
The correct answer is: B

Although all the entities listed can cause lesions similar to erythema migrans, the lesion typical of Lyme disease, the systemic symptoms and exposure in an endemic area make Lyme borreliosis the most likely diagnosis. The skin lesion typically develops between 3 and 30 days after the tick bite occurs. Systemic symptoms may develop within a few weeks, with secondary skin lesions. Most patients with Lyme disease cannot recall having been bitten by a tick. Ref: ID/ W.Wilson, p.685; Harrison's 16, p. 996/ RAL
A previously healthy 24 year old man presents at a clinic in Southern New England ten days after a weekend camping trip. He reports the appearance of a painless skin lesion on his right thigh five days previously. He now notes malaise, low-grade fever, and migrating joint pain of 48 hours duration. Careful questioning fails to elicit any recent history of an insect bite. The lesion on the thigh had begun to fade, but you recognize an area of central clearing with an erythematous margin. Further examination reveals a second, similar lesion near the right axilla. Question: You determine that the patient referred to in the scenario has no known drug allergies. The most appropriate management at this time would be:

A) Begin doxycycline 100mg po bid for 7 days.
B) Begin amoxicillin 500mg po tid for 20 days.
C) Obtain a blood specimen for serology and make a follow-up appointment in 7 days.
D) Begin erythromycin 250mg po qid for 20 days.
E) Begin doxycycline 100mg po bid for 21 days.
The correct answer is: E

Doxycycline is the drug of first choice in an adult male. Amoxicillin is preferred in children and pregnant women. Erythromycin, clarithromycin, or azithromycin are alternatives for penicillin-allergic patients. Twenty-one days of treatment is preferred in this patient with early disseminated disease. Ref: ID/ W.Wilson, p.685-686; Harrison's 16, p. 998/ RAL
A previously healthy 24 year old man presents at a clinic in Southern New England ten days after a weekend camping trip. He reports the appearance of a painless skin lesion on his right thigh five days previously. He now notes malaise, low-grade fever, and migrating joint pain of 48 hours duration. Careful questioning fails to elicit any recent history of an insect bite. The lesion on the thigh had begun to fade, but you recognize an area of central clearing with an erythematous margin. Further examination reveals a second, similar lesion near the right axilla. Question: The patient referred to in the scenario returns three days later complaining of worsening weakness and lightheadedness. His pulse is 42 and his pressure in the supine position is 90/40. You quickly obtain an EKG, which reveals third degree heart block. The best management at this point would include:

A) Add oral cefuroxime 500mg po bid to his antibiotic regimen.
B) Stop all antibiotics immediately and administer Benadryl 50mg intramuscularly stat.
C) Administer atropine 1mg intravenously, observe the patient for 4 hours and discharge if he remains stable.
D) Admit the patient for cardiac monitoring and begin intravenous ceftriaxone 2g per day.
E) Stop all antibiotics immediately and admit the patient for electrophysiological studies.
The correct answer is: D

About 8% of patients with Lyme disease will develop cardiac involvement, usually some form of atrioventricular block. Patients with third degree heart block should be treated with intravenous ceftriaxone until the heart block resolves and then complete a 30-day course of oral antibiotics. The development of neurological symptoms also requires treatment with intravenous antibiotics. Ref: ID/ Harrison's 16, p. 996, 998/ RAL
A previously healthy 24 year old man presents at a clinic in Southern New England ten days after a weekend camping trip. He reports the appearance of a painless skin lesion on his right thigh five days previously. He now notes malaise, low-grade fever, and migrating joint pain of 48 hours duration. Careful questioning fails to elicit any recent history of an insect bite. The lesion on the thigh had begun to fade, but you recognize an area of central clearing with an erythematous margin. Further examination reveals a second, similar lesion near the right axilla. Question: The patient referred to in the scenario recovers completely. His love of the outdoors, however, is unabated, and he informs you that he intends to continue regular outdoor activities in known tick habitats. Your best advice would include which of the following?

A) Reassurance that lifelong immunity to Lyme borreliosis is conferred by a single episode of clinical disease
B) Recommendation of prophylactic doxycycline during the summer months
C) Recommendation that he refrain from outdoor activities in endemic areas for life
D) Recommendation that he undergo a series of three injections of Lyme vaccine
E) Recommendation that he carefully inspect himself for ticks on a weekly basis during the summer months and resume antibiotic treatment if any are found
The correct answer is: C

Reinfection is possible. The previously available Lyme vaccine has been withdrawn by the manufacturer. Prophylactic antibiotics, if given within 72 hours after the tick bite, will prevent the development of the disease

but the tick bite is often not noted. The best advice is to avoid areas of tick infestation. Ref: ID/ W.Wilson, p. 686; Harrison's 16, p. 998-999/ RAL